Sie sind auf Seite 1von 388

ˆ_Œ

SMLE
KSAU-HS
Question Bank
1st Edition
Internal Medicine Questions

This is an accumulative effort from King Saud bin Abdulaziz University for Health Sciences (2016-17/
Batch 9) interns to organize and answer what have been collected previously from SMLE Q Bank
2015-16

We would like to acknowledge:


- King Saud bin Abdulaziz University for Health Sciences (2016-17/Batch 9) interns for their huge efforts in
accomplishing this project
- SMLE Q Bank Group
‫ مدونة طالب طب سعودي‬-

‫جهد بشري قابل للخطأ والصواب‬


For any comments, kindly contact us at
SMLE2016.17@gmail.com

Highlighted in yellow are queried questions


Highlighted in red are some repeated questions

2
Internal medicine

Table of Contents

Cardiology ................................................................................................................................................... 4
Pulmonology ............................................................................................................................................. 61
Nephrology ............................................................................................................................................... 91
Hematology ..............................................................................................................................................118
Neurology ................................................................................................................................................174
Infectious disease .....................................................................................................................................204
Rheumatology ..........................................................................................................................................281
Endocrinology ..........................................................................................................................................305
Gastroenterology .....................................................................................................................................342
Immunology & Allergy ..............................................................................................................................376
Bullet points .............................................................................................................................................382

3
Cardiology

4
1. Best initial screening test for pt suspected with coarctation of aorta?
A. Echo/doppler
B. CT cardiac
C. MRI cardiac
D. Cardiac angio

Answer: A
No specific laboratory tests are necessary for coarctation of the aorta. Imaging studies that may be helpful include the following:
Chest radiography (Findings vary with the clinical presentation of the patient), Barium esophagography (Classic “E sign,”
representing compression from the dilated left subclavian artery and poststenotic dilatation of the descending aorta)
Echocardiography (2-dimensional echocardiography, pulsed-wave Doppler, and color flow mapping). In another reference once
suspected, an echocardiogram is the most commonly used test to confirm the diagnosis. It is more accurate. Cardiac
catheterization helps to confirm the diagnosis when echocardiography findings are not completely clear.
Http://www.cdc.gov/ncbddd/heartdefects/coarctationofaorta.html
Http://emedicine.medscape.com/article/150369-overview

2. Pt had urti for 2 weeks later developed orthopnea, severe pulmonary edema, what is the dx?
A. Infective endocarditis
B. Acute epicarditis
C. Acute myocarditis
D. Acute bronchitis

Answer: c
Patients with myocarditis have a clinical history of acute decompensation of heart failure, (e.g. Tachycardia, gallop, mitral
regurgitation, edema). In viral myocarditis, patients may present with a history of recent (within 1-2 wk) flulike syndrome of fevers,
arthralgia, and malaise or pharyngitis, tonsillitis, or upper respiratory tract infection.
Http://emedicine.medscape.com/article/156330-clinical#b1

3. Mid diastolic murmur in left sternum?


A. Mitral stenosis
B. Mitral regurge
C. Aortic stenosis
D. Aortic regurge

Answer: a
Ms: mid diastolic to late diastolic , rumbling
Mr: holosyatolic ‘pansystolic “, blowing radiate to left axilla
As: midsystic , harsh radiated to carotid artery
Ar: early diastolic

4. Best initial screening test for pt suspected with coarctation of aorta?


A. Echo/doppler
B. CT cardiac
C. MRI cardiac
D. Cardiac angio
5
Answer: chest xray (best initial test according to master the boards, first aid, kaplan)
Uptodate: “ in children with coarctation, echocardiography often provides adequate anatomic and hemodynamic information for
the surgeon or interventional cardiologist without the need for a further imaging study. However, MRI or ct is generally used as a
complementary diagnostic tool in adolescent and adult patients, and provides important anatomic data prior to intervention.
In adults, cranial mr angiography (or CT angiography) is also appropriate to search for intracranial aneurysms

5. Patient has history of mi suddenly he became breathless and harsh systolic murmur heard what the cause?
A. PE
B. Rupture aortic cusp
C. Tricu regurge
D. Rupture

Answer: interventricular septum rupture


''Rupture of the ventricular septum is often accompanied by a new harsh holosystolic murmur best heard at the left lower sternal
border''.
Reference: http://www.clevelandclinicmeded.com/medicalpubs/diseasemanagement/cardiology/complications-of-acute-
myocardial-infarction/

6. Mid diastolic murmur in left sternum?


A. Mitral stenosis
B. Mitral regurgitation
C. Aortic stenosis
D. Aortic regurgitation

Answer: a
Causes of mid diastolic murmur :
Mitral stenosis is best heard at apex.
Tricuspid stenosis is best heard along the left sternal border.
Atrial myxoma.
Reference : uptodate

7. With thyrotoxicosis, what is the most common arrhythmia?


A. SVT
B. Vt
C. Vfib
D. Wpw

Answer: SVT (toronto note mentions AFib as most common)


According to medscape, arrhythmias in thyroid storm are usually supraventricular, with atrial fibrillation being the most common.
Atrial flutter and paroxysmal atrial tachycardia are uncommon, whereas ventricular premature contractions and ventricular
fibrillation are rare.
They are due to the direct augmenting effect of the thyroid hormones on the β-adrenergic receptor's sensitivity to catecholamines
and increasing myocardial excitability, and on the intrinsic sinoatrial electrophysiologic function, thus decreasing conduction time
and resulting in nodal blockade in longstanding disease
Image bellow is from toronto notes.
6
8. The best initial screening test for patient suspected with coarctation of aorta
A. Echo/doppler
B. CT cardiac
C. MRI cardiac
D. Cardiac angio

Answer: a
Reference: http://cursoenarm.net/uptodate/contents/mobipreview.htm?17/11/17584#h17

9. Patient has history of mi suddenly he became breathlessness and harsh systolic murmur heard what the cause:
A. Pe
B. Rupture aortic cusp
C. Tricuspid regurgitation
D. Rupture septum

Answer: d
Acute sob with high pitched systolic murmur post mi is usually due to acute mitral regurgitation with or without injury to the
papillary muscle chordae tendineae
Reference: merck manual

10. Old lady with sharp chest pain and fever diagnosed with pericarditis what will you do to dx the case, most accurate test is :
A. Acid fact stain
B. Pericardial biopsy (my answer)

7
C. Pleural aspiration
D. Forget the last choice( the answers look like they are talking about tb as the cause )

11. Hypertensive patient on ACEI but not controlled Blood pressure , what to add?
A. Thiazide
B. Beta blocker
C. Furosemide
D. Nefidipine (calcium channel blocker)

Answer: d
In nonblack hypertensive patients, begin treatment with either a thiazide-type diuretic, CCB , ACE inhibitor, or arb
If a patient's goal Blood pressure is not achieved within 1 month of treatment, increase the dose of the initial agent or add an agent
from another of the recommended drug classes; if 2-drug therapy is unsuccessful for reaching the target Blood pressure , add a third
agent from the recommended drug classes
Http://emedicine.medscape.com/article/241381-treatment

12. Mitral stenosis cause enlargement in which chamber :


A. Left atrium
B. Right atrium
C. Left ventricle
D. Right ventricle

Answer: a
Mitral stenosis (ms) is characterized by obstruction to left ventricular inflow at the level of mitral valve due to structural abnormality
of the mitral valve apparatus. The most common cause of mitral stenosis is rheumatic fever. The normal mitral valve orifice area is
approximately 4-6 cm2. Patients will not experience valve-related symptoms until the valve area is 2-2.5 cm2 or less, at which point
moderate exercise or tachycardia may result in exertional dyspnea.
Http://emedicine.medscape.com/article/155724-overview#a5

13. The same scenario as the above question, how to treat? DVT case
A. Aspirin
B. Heparin
C. Lmwh
D. Fracion heparin
Answer c
According to medscape: regular unfractionated heparin was the standard of care until the introduction of lmwh products.
Reference: http://emedicine.medscape.com/article/1911303-treatment#d10

14. Most specific for coronary artery disease


A. Cholesterol
B. Triglycerides
C. Hdl
D. Ldl
Answer: d
Reference: http://emedicine.medscape.com/article/2087735-overview#a2
8
15. What types of hypersensitive rapid onset of ? I did not understand the q
Fast acting anti-hypertensives:
A. Captopril
B. Clonidine
C. Labetalol
D. Nifedipine
E. Sodium nitroprusside

16. Cardiac notch is at which intercostal level?


A. 3
B. 4
C. 5
D. 6
Answer:???
The cardiac notch is an angular notch that lies along the fifth and sixth intercostals space, where the
pericardium is exposed.
Http://www.anatomyexpert.com/app/structure/10604/1786/

17. Which is more at risk for cad


A. 25 male, smoker, sedentary life
B. 35 female, type 1dm, obese
C. 55 male, hypertension, high cholesterol
D. 50 male, obese, dm
Answer: c or d, age over 45, HTN hypercholesterolemia, dm, and obesity are all major risk factors but i think c because the older age.
Http://emedicine.medscape.com/article/164163-overview#a1

18. A case of rheumatic fever he had a severe arthritis and mitral regurgitation what is your short management?
A. Steroid and high dose aspirin
B. Daily steroid and aspirin
C. Im penicillin
D. Monthly
Answer: d
Anti-inflammatory agents are used to control the arthritis, fever, and other acute symptoms. Salicylates are the preferred agents,
although other nonsteroidal agents are probably equally efficacious.
Http://emedicine.medscape.com/article/236582-medication#2

19. Newly diagnosed hypertensive patient came to the primary clinic complaining of dry cough and shortness of breath. Which
medication he used?
A. Perindopril
9
B. Valsartan
C. Atenolol
D. Thiazide
Answer: a- perindopril (dry cough is one of the sides effect of ACEIs) drugs.com
Https://www.drugs.com/cdi/perindopril.html

20. Which murmur is associated with mitral stenosis?


A. Mid systolic
B. Holo systolic
C. Mid diastolic
D. Early systolic
Answer: c- mid diastolic. Easyauscultation.com
Http://www.easyauscultation.com/cases?Coursecaseorder=14&courseid=31

21. What condition should to take more attention regarding prevention of coronary artery disease
A. 25 yo with dm1 and htn
B. 55 yo with DM and htn
C. 55 (or 50) yo with DM and high cholesterol
D. 25 yo with DM and something
Answer: b
Http://emedicine.medscape.com/article/164163-overview

22. Which of the following is a minor criterion in duke’s criteria for rheumatic fever?
A. Fever
B. Lymphocytosis
C. Carditis
D. Arthritis
Answer: a
Major criteria:
carditis†
• clinical and/or subclinical
arthritis
• polyarthritis only
chorea
erythema marginatum
subcutaneous nodules
Minor criteria
polyarthralgia
fever (≥38.5°c)

esr ≥60 mm in the first hour and/or crp ≥3.0 mg/dl§


prolonged pr interval, after accounting for age variability (unless carditis is a major
criterion)

10
23. 60 years old male, c/c chest pain, he was brought to er of tertiary center. Ecg showed st elevation in leads ii, v1, v2, v3, after
initial management in the er, next step is?
A. Heparin
B. Streptokinase
C. Tpa
D. Stenting of coronary artery
Answer: d
Read more in: http://circ.ahajournals.org/content/110/5/588.full

24. 3 yo known case of cardiac disease. What will you do?


A. Eco
B. Ecg
C. Catheterization
D. Observation
Answer: a, incomplete

25. Pt c/o sever uncontrolled htn, “renography” not sure”, showed l renal artery stenosis, next step is to?
A. Venography
B. Ivp
C. CT angiograpgy
D. Renography
Answer: c
Reference: http://emedicine.medscape.com/article/245023-workup#c7

26. Man with chest pain while exercises, hx of chest pain in rest and exercise. Ejection systolic murmur at left sternal, not
radiating, ecg > left atrial enlargement with no specific st changes. Dx?
A. Aortic stenosis
B. Pulmonary stenosis,
C. Restrictive cardiomyopathy,
D. Hypertrophic cardiomyopathy.

Answer: a?

27. 50 y\o male presented to er with severe respiratory distress, no other history obtained, investigations were done to him cxr:
pulmonary edema with infiltrate ecg: he was having atrial fibrillation, his pulse rate was 125 what investigation you will do to
him:
A. Echocardiography
B. Spiral ct
C. V\q match
D. Arteriogram

Answer : a
11
Reference: stepup to medicine +
Kumar and clarck’s clinical medicine

28. Newly diagnosed hypertensive patient came to the primary clinic complaining of dry cough and shortness of breath. Which
medication he used?
A. Pernidopril
B. Valsartan
C. Atenolol
D. Thiazide
Answer: a
Http://www.rxlist.com/ace_inhibitors-page2/drugs-condition.htm

29. 60 years old patient has only htn, what’s the best drug to start with?
A. ARB
B. ACEI
C. BB
D. Diuretics
Answer: d

12
13
30. Which murmur is associated with mitral stenosis?
A. Mid systolic
B. Holo systolic
C. Mid diastolic
D. Early systolic
Answer: c
Http://www.merckmanuals.com/professional/cardiovascular-disorders/valvular-disorders/mitral-stenosis
Http://www.wilkes.med.ucla.edu/msmain.htm

31. Elderly patient presented with chest pain he is a smoker with positive family history of coronary artery disease and he is
dyslipedmic he is also over weight and can not tolerate excersise his ecg is normal what will do:
A. Stress echo
B. Stress test
C. Resting perfusion scan

14
D. Angiography

Answer: c
Http://www.hopkinsmedicine.org/healthlibrary/test_procedures/cardiovascular/myocardial_perfusion_scan_resting_92,p07978
/

32. Patient with high level of cholesterol, what to avoid?


A. Organ meat
B. Avocado
C. Chicken
D. White egg
Answer: A

33. Most specific for coronary artery disease


A. Cholesterol
B. Triglycerides
C. Hdl
D. Ldl
Answer: d
Http://www.medscape.com/viewarticle/545401_2

34. Which of the following is most likely to be the presentation of a patient with early stemi?
A. Troponin of 0.12 with t inversion in v1-v4
B. Pathological q wave with subsided chest pain
C. Presence of chest pain with 0.3 mm elevation in st segment in leads 2,3, avf
D. St depression in 2,3, avf

Answer: c?
Normal ecg prior to mi
Hyperacute t wave changes - increased t wave amplitude and width; may also see st elevation
Marked st elevation with hyperacute t wave changes (transmural injury)
Pathologic q waves, less st elevation, terminal t wave inversion (necrosis)
(pathologic q waves are usually defined as duration ≥ 0.04 s or ≥ 25% of r-wave amplitude)
Pathologic q waves, t wave inversion (necrosis and fibrosis)
Pathologic q waves, upright t waves (fibrosis)

35. Pt with past history valve heart disease. 2 weeks ago he had tooth procedure & now present with symptoms of infective
endocarditis. Which of the following is the most likely organism?
A. Staph aureus
B. Strep pneumonia
C. Strep viridans
D. Moraxella catarrhalis
Answer: a?
Toothbrushing and chewing can cause bacteremia (usually due to viridans streptococci) in patients with gingivitis

15
Reference: merck manual

36. Ecg with t inverted in some leads otherwise normal.


Labs: tropi high ck high what dx?
A. Low risk unstable angina.
B. High risk unstable angina.
C. Nstemi.
D. Acute st elevation mi.
Answer

37. The best for lifestyle tt for HTN patient:


A. Na restriction less than 6
B. Give to him k
C. Run every day 1.5 km
D. Do something 4 time per week

•reduce sodium intake to no more than 100 mmol/d (2.4 g sodium or 6 g sodium chloride; range of approximate sbp reduction, 2-8
mm hg)
•engage in aerobic exercise at least 30 minutes daily for most days (range of approximate sbp reduction, 4-9 mm hg)

38. Ecg finding of ostium secundum atrial septal defect.


A. Lt axis deviation
B. Rbbb
C. Lvh
D. Delta wave
Answer: b
The most frequent ecg abnormalities in asd are right bundle branch block (rbbb).
Source: http://www.metealpaslan.com/ecg/asden.htm

39. Most important risk factor for cardiac?


A. Ldl
B. Total cholesterol
C. Blood pressure
D. Smoking
Answer: c?

40. Which of the following is side effect of atropine:


A. Vasoconstriction
B. Decrease iop
C. Decrease urine output
D. Dry of mouth
16
Answer: d
Most of the side effects are directly related to its antimuscarinic action. Dryness of the mouth, blurred
vision, photophobia and tachycardia commonly occur with chronic administration of therapeutic doses. Anhidrosis also may occur
and produce heat intolerance or impair temperature regulation in persons living in a hot environment. Constipation and difficulty
in micturition may occur in elderly patients.
[http://www.rxlist.com/atropine-drug/side-effects-interactions.htm]

41. Which in lipid profile is most important risk for coronary heart disease ?
A. Ldl,
B. Hdl,
C. Triglyceride ,
D. Total cholesterol
Answer a

42. Mitral stenosis cause enlargement in which chamber:


A. Left atrium
B. Right atrium
C. Left ventricle
D. Right ventricle
Answer a
Chest radiographic findings suggestive of mitral stenosis include left atrial enlargement (eg, double shadow in the cardiac silhouette,
straightening of left cardiac border due to the large left atrial appendage

43. Patient has history of mi suddenly he became breathless and a harsh systolic murmur was heard:
A. Pe
B. Rupture aortic cusp
C. Tricuspid regurge
D. Rupture .......
Answer: if answer d is ruptured papillary muscle then it’s true. If not go with b

44. Post mi, patient develops atrial fibrillation. Which medication is required to prevent stroke?
A. Warfarin
B. Heparin
C. Aspirin
D. Dabigatran

Answer: a
Taking an anticoagulant (blood thinner) can reduce the risk of having a stroke by approximately 50 to 70 percent.
Reference:
Http://www.uptodate.com/contents/atrial-fibrillation-beyond-the-basics#h17

45. Long scenario asking how amlodipine cause ll edema


17
A. Increase capillary oncotic pressure ?
B. Decrease plasma ...
C. Decrease cardiac output
D. Increased hydrostatic pressure

Answer: d
Reference: medscape: http://www.medscape.com/viewarticle/460070_1

46. Mitral stenosis cause enlargement in which chamber


A. Left atrium
B. Right atrium
C. Left ventricle
D. Right ventricle
Answer: a
Medscape

47. Ecg finding of ostium secundum atrial septal defect


A. Left axis deviation
B. Rbbb
C. LVH
D. Delta wave

Answer: b
toronto: ecg: rad, mild rvh, rbbb

48. Phytosterol mode of action;


A. Decrease cholesterol level
B. Decrease cholesterol synthesis
C. Decrease triglyceride synthesis
D. Decrease de no vo cholesterol synthesis
Answer:
Decreases cholesterol absorption from small intestine

18
Https://en.wikipedia.org/wiki/phytosterol

49. Ecg with AFib pattern. A patient present with irregular cardiac palpitation some duration in the past for which he was taking
a drug, lab values given demonstrating normal upper limit of aptt, normal pt, normal inr, normal platelet, low hemoglobin,
what is the mechanism of action of the drug?
A. Anti-thrombin iii
B. Decrease factor viii
C. Inhibit platelets aggregation
D. Decrease vitamin k dependent factors

50. Acute mi patient will present with?


A. High troponin i with st elevation in v2-v6
B. Ongoing chest pain with high st elevation in lead ii - iii and avf
C. ST elevation with no chest pain
D. ST depression with .....

Answer: missing information

51. Case presented with st elevations mi for 6 hours no neurological symptoms after CPR the patient died in autopsy you well
find:
A. Brain absess
B. Interventcular hemorrhage
C. Red cells in the hypocample area
D. Necrosis in the area associated with midcerbral infarct
Answer: d

52. Which medication will delay the surgery for chronic aortic regurgitation?
A. Digoxin
B. Verapamil
C. Nifedipine
D. Enalapril

Answer: C
o Nifedipine is the best evidence-based treatment in this indication.
o ACE inhibitors are particularly useful for hypertensive patients with AR.
o Beta blockers may be indicated to slow the rate of aortic dilatation and delay the need for surgery in patients with AR associated
with aortic root disease. Furthermore, they may improve cardiac performance by reducing cardiac volume and LV mass in
patients with impaired LV function after AVR for AR.
Reference: 3rd Edition UQU>Cardiology> Q3
In severe aortic valve insufficiency, the excess in afterload increases burden on the left side of the heart. Theoretically, any
medication that can reduce afterload could be expected to improve left ventricular function and decrease regurgitant backflow from
the aorta. This would provide a temporizing measure by which surgical intervention can be postponed. One study showed that the
use of nifedipine in asymptomatic patients with severe aortic regurgitation who had normal LV function could delay the need for
surgery by 2-3 years. This result may also be expected with the use of similar vasodilating agents.
19
Reference: Medscape.

53. (long scenario) man with chest pain and abnormal EKG. Which one of the following will be elevated?
A. ESR.
B. M2 Protein.
C. CRP.
D. Creatinine.

Answer: C
References:
Http://circ.ahajournals.org/content/113/6/e72.full
Http://circ.ahajournals.org/content/123/10/1092.full
Http://circ.ahajournals.org/content/107/3/499.figures-only

54. Patient with decreascendo-increasendo murmure, in midsternal not radiating, high pitched first sound?
A. Ejection diastol
B. Ejection systolic
C. Physiologic
D. Innocent
Answer: b
Systolic ejection (not sure 100%) maybe some points missed in the questions because answer overlapping. Systolic ejection murmurs
may be functional or organic.The most common functional systolic ejection murmur in adults is probably a variant of Still's murmur,
the so-called innocent murmur of childhood. It is a short, buzzing, pure, medium-pitched, nonradiating, midsystolic murmur heard
best along the upper left sternal border.

55. What valve lesion you'll find in acute infective endocarditis?


A. Mitral stenosis
B. Mitral regurge
C. Aorta stenosis
D. Aorta regurge.

Answer: b or d
The most common affected valve is mitral valve, aortic valve, tricuspid valve then pulmonary valve. However, in left side vegetation,
the most common underlying lesions being mitral valve prolapse and degenerative mitral and aortic regurgitation. In mitral valve the
is some degree of regurgitation.
References:
Http://patient.info/doctor/infective-endocarditis-pro
Http://www.ncbi.nlm.nih.gov/books/nbk2208/
Http://emedicine.medscape.com/article/216650-overview#a3
Http://emedicine.medscape.com/article/216650-clinical#b2
Http://emedicine.medscape.com/article/150490-overview
Http://www.texasheart.org/hic/heartdoctor/answer_1428.cfm
Http://www.wilkes.med.ucla.edu/mvpmain.htm

20
56. Patient has history of MI, suddenly he became breathlessness and harsh systolic murmur heard what is the cause?
A. PE
B. Rupture aortic cusp
C. Tricu regurge
D. Rupture .......

Answer: Rupture of papillary muscle cause Mitral Regurgitation (pansystolic), ventricular septal defect cause harsh pansystolic
murmur and Left Ventricular Outflow Tract Obstruction cause systolic murmur
Https://books.google.com.sa/books?Id=anbzecmvdyic&pg=PA107&lpg=PA107&dq=myocardial+infarction+harsh+systolic&source=bl
&ots=Zc4qWUx2t6&sig=6Vaup9OPqC3FhVYwI7ElgI7b2bA&hl=en&sa=X&redir_esc=y#v=onepage&q&f=true
Http://www.clevelandclinicmeded.com/medicalpubs/diseasemanagement/cardiology/complications-of-acute-myocardial-
infarction/

57. What is diagnosis?

A. SVT
B. AF
C. VT
D. VF
Answer: A
“same ECG that shows SVT in UQU cardiology section” << couldn’t find it, so I uploaded another ECG strip.
Read about other arrhythmias.

58. Patient with chest pain. A picture of an ECG (NO ST ELEVATION). Lab results: HIGH
TROPONIN, HIGH LDH, HIGH ASPARTAT.
A. A HIGH RISK UNSTBLE ANGINA
B. Low risk
C. NSTEMI
D. STEMI

Answer: NSTEMI
Increase LDH, AST, and Troponin in skeletal muscle injury and MI.
Since there are no ECG changes (ST elevation), the answer would be NSTEMI.
Reference: http://medicinembbs.blogspot.com/2011/06/what-is-difference-between-nstemi-and.html
NSTEMI indicate myocardial necrosis marked by elevation in troponin I and CK-MP without ST-segment elevation on the ECG.
Reference: First Aid USMLE STEP 2 CK

59. Obese male pt presented to family physician complaining of chest pain for 2 days, ecg normal, what to do next?
A. Treadmill
21
B. Coronary angio
C. 24 hrs monitoring

Answer: a
Chest x-ray, exercise stress test, ecg and angiography as definitive diagnostic test are considered in patient with chest pain
suspecting angina.
Reference: http://emedicine.medscape.com/article/150215-overview

60. Patient with chest pain and st changes, you will find elevation in:
A. ALT
B. AST
C. Troponin

Answer: c

61. Hypertensive patient on ACEI but not controlled Blood pressure , what to add?
A. Furosemide
B. Thiazide
C. Beta blocker

Answer: b (CCB if no option> thiazide)

62. Smoker and obese patient can't exercise, with family history of mi, came with vague chest pain. Ecg is normal. What’s the
next step?
A. 24 hours ecg
B. Exercise with ecg
C. Perfusion cardiac scan

Answer: c
According to medscape, if the patient cannot tolerate stress tests, one should proceed to myocardial perfusion scans

22
63. A patient with sudden chest pain, diaphoresis, what’s the best test to order?
A. Echo
B. ECG
C. Cardiac enzymes

Answer: b
Ecg is the first investigation in chest pain.
Reference: master the boards

64. Urti after 1 week complaining of severe shortness of breath and orthopnea with no fever
A. Acute myocarditis
B. Acute pericarditis
C. Ie

Answer is a. Patients with myocarditis have a clinical history of acute decompensation of heart failure,( e.g. Tachycardia, gallop,
mitral regurgitation, edema ).
In viral myocarditis, patients may present with a history of recent (within 1-2 wk) flulike syndrome of fevers, arthralgias, and malaise
or pharyngitis, tonsillitis, or upper respiratory tract infection.
Http://emedicine.medscape.com/article/156330-clinical#b1

65. Case of mi with ecg showing t wave inversion in v3 - v6 with elevated troponin and ck, what's the diagnosis:
A. Unstable angina
B. NSTEMI
C. STEMI
Answer: b

66. Patient come with friction rub what is your next action
A. Echo and cardiology consultation
B. X – ray
C. Pericardiocentesis
Answer: a
Pericarditis:
Ecg: initially diffuse elevated st segments ± depressed pr segment, the elevation in the st segment is concave upwardsg2-5 d later st
isoelectric with t wave flattening and inversion
• cxr: normal heart size, pulmonary infiltrates
• echo: performed to assess for pericardial effusion

67. Pt came with cough, dyspnea, distended neck vein, peripheral edema & ascites. By examination there is bilateral rales. What
is the underlying cause
A. Right heart failure
B. Left heart failure
C. Aortic valve regurge others

Answer: should be congestive heart failure

23
68. Patient presented with chest pain ecg showing st segment elevation in lead i ,avl,v5andv6 ?
A. Lateral ischemia
B. Ant ischemia
C. Post ischemia
Answer: a
Http://lifeinthefastlane.com/ecg-library/lateral-stemi/
Answer: a

69. What is the most common cause of secondary hypertension?


A. Kidney
B. Liver
C. Spleen
Answer: a

70. Patient has proximal nocturnal dyspnea orthopnea and dyspnea with a history of mitral stenosis ... What is the most likely
diagnosis:
A. Acute respiratory distress syndrome
B. Left heart failure
C. Right heart failure
Answer: c

71. Trauma patient with hypotension, distended jugular veins, good bilateral equal air entry, diagnosis;
A. Tension pneumothorax
B. Hemothorax
C. Cardiac tamponade
Answer: c
Good bilateral air entry makes you avoid a &b. Also hypotension and distended jvp are directing to it also.
Http://emedicine.medscape.com/article/152083-clinical

72. Patient came with irregularly irregular pulse:


A. Atrial fibrillation
B. Supraventricular tachycardia
C. Heart block

Answer: a
Kaplan step 2ck pediatrics 2014

73. Black man e high Blood pressure , +ve fx hx of HTN . Best med to give?' (cardio)
A. B blocker
B. ACEI
C. CCB
Answer: c thiazide diuretics and calcium channel blockers are preferred in black patients.
Reference: http://www.aafp.org/afp/2015/0201/p172.html

24
74. Which of the following clinically suggests an early onset st elevation mi ?
A. Elevated troponin1 and long inverted t wave in lead 1,2 avf
B. St elevation of .3 mm in lead v1-v6 with chest pain
C. St elevation without chest pain

The answer is b
T wave inversion is considered to be a late finding. The changes should be corresponding to a side of the heart. St elevation without
chest pain is atypical presentation.
Reference: step up to medicine

75. Ecg picture


A. AFib
B. Vt
C. SVT ?
Reference :for more information about SVT http://www.practicalclinicalskills.com/supraventricular-tachycardia

76. On physical examination, patient shows friction rib. What is your next step?
A. Echo and cardiology consultant
B. X – ray
C. Pericardiocentesis

Answer: a
Pericardial friction rub is specific for pericarditis. Echo is part of its initial approach. If the patient is unstable, pericardiocentesis will
be needed.
X- ray doesn't have a strong rule in diagnosing pericarditis
Extra note: echocardiography is recommended in all cases of pericarditis. Any form of pericardial inflammation can induce
pericardial effusion.
- Http://emedicine.medscape.com/article/156951-overview

77. Elderly patient presented with right leg swelling, pitting edema and history of knee swelling and pain. What will you doe
next?
A. Echocardiogram
B. Doppler
C. Chest x-ray
Answer: b
Dvt diagnosis is typically by ultrasonography with doppler flow studies (duplex ultrasonography).

78. A medication that will decrease cardiac output and decrease peripheral resistance?
A. Carvedilol
B. Hydralazine
C. CCB
Answer: ?? labetolol

25
79. 22 year old male patient presents with chest pain increase when he is lying flat, ecg shows diffuse st-elevation in all leads
what is the diagnosis?
A. Myocardial infarction
B. Pericarditis
C. Infective endocarditis
Answer: b, in mi, st elevation in leads that correspond to the cardiac muscle damage.
Http://www.usmleforum.com/files/forum/2011/2/594549.php

80. Very obese smoker patient complain of chest pain how to test for myocardial perfusion:
A. Exercise stress test
B. 24 hrs holter monitor
C. Dobutamine stress test
Answer: c, pharmacologic stress testing is usually used when patients cannot walk on a treadmill long enough to reach their target
heart rate because of deconditioning, musculoskeletal disorders, obesity, peripheral arterial disease, or other disorders.
- holter is for arrhythmias.
Http://www.merckmanuals.com/professional/cardiovascular-disorders/cardiovascular-tests-and-procedures/stress-testing

81. Newborn is irritable and sweating, chest is clear, vitals was provided, hr 300 beat/min, what's your action:
A. Cardiac dextroversion
B. Vagal massage
C. Digoxin

Answer: b vagal massage. Maneuvers that stimulate the vagus delay av conduction and thus block the reentry mechanism: the
valsalva maneuver, carotid sinus massage, breath holding, and head immersion in cold water (or placing an ice bag to the face)
Acute treatment: pharmacologic therapy
• IV adenosine—agent of choice due to short duration of action and effective- ness in terminating svts; works by decreasing
sinoatrial and av nodal activity.
• IV verapamil (calcium channel blocker) and IV esmolol (β-blocker) or digoxin
Are alternatives in patients with preserved left ventricular function.
• dc cardioversion if drugs are not effective or if unstable; almost always suc-
Cessful.
R
step up to medicine

82. Aspirin side effect:


A. Diarrhea
B. Constipation
C. Bleeding
Answer: c
Reference: http://reference.medscape.com/drug/zorprin-bayer-buffered-aspirin-343279#4

83. A 68-year-old diabetic, started 10 days ago on amlodipine 10mg for hypertension, now he is complaining of gross ankle
edema, on examination, jvp was not raised, by auscultation the chest no base crepitations
labs: na, k and ca all within normal range
what is the most likely cause of his edema?
A. Na and water retention
B. Increased capillary hydrostatic pressure
26
C. Decreased oncotic pressure
Answer: b
Its etiology relates to a decrease in arteriolar resistance that goes unmatched in the venous circulation. T this disproportionate
change in resistance increases hydrostatic pressures in the precapillary circulation and permits fluid shifts into the interstitial
compartment.
Link: http://www.medscape.com/viewarticle/460070_1

84. Which of following related to rheumatic heart disease?


A. Mitral stenosis
B. Mitral regurgitation
C. Aortic stenosis
Answer: a
Chronic rheumatic heart disease remains the leading cause of mitral valve stenosis and valve replacement in adults in the united
states.

85. Which heart disease is common in down syndrome:


A. VSD
B. Atrioventricular septal defect
C. Coartication of aorta
Answer: b
AVSD is the most common cardiac defect in trisomy 21.
[https://www.pediatricheartspecialists.com/blog/55-down-syndrome-and-congenital-heart-disease]

86. Patient with murmur in left sternal border change with stand or sitting what is dx:
A. Pulmonary stenosis
B. Aortic stenosis
C. Cardiomyopathy
Answer: a
Ps murmur is a left upper sternal border murmur that is affected by standing.

87. Smokers obese patient can't exercise with family history of mi came with
Vague chest pain . But ecg is normal next step ?
A. 24 hours ecg
B. Exercise with ecg
C. Perfusion cardiac scan
Answer : c
From master the boards: perfusion cardiac scan, when the patient can’t exercise . Abnormality will be detected by seeing decreased
thallium uptake.
Nb: if the question is describing a scenario in the er, acute chest pain do ecg and cardiac enzymes .
If the question is describing a scenario in the clinic or chest pain etiology is not clear or the ecg is not diagnostic then do exercise
tolerance test.

88. A patient with renal failure and mitral regurgitation, valve was 0.7 cm. Other findings in aortic valve? What is the proper
management?
A. Total valve replacement
B. Valvoplasty

27
C. Balloon
Answer: http://www.uptodate.com/contents/mitral-regurgitation-beyond-the-basics

89. Athletic come for check-up all thing normal except xanthelasma on achllis tendon and cholesterol?
A. Ldl receptor
B. Apo ll
C. Apo c

Answer: a
Familial hypercholesterolemia (fh) is an autosomal dominant disorder that causes severe elevations in total cholesterol and low-
density lipoprotein cholesterol (ldlc).
Xanthomas are noted commonly on the achilles tendons and metacarpal phalangeal extensor tendons of the hands of patients with
untreated fh.

90. Exercise for cardiac patients:


A. Yoga
B. Isometric
C. Isotonic
Answer: c
The usual mode of exercise recommended is dynamic, aerobic or isotonic
Exercise, including walking, running, cycling, swimming, aerobic dancing, cross-country skiing, and elliptical machines. Isometric
exercise does not improve cardiac performance. Http://www.ncbi.nlm.nih.gov/pmc/articles/pmc3116747/

91. ECG Picture?


A. AF
B. VT
C. SVT

92. Treatment of acute myocarditis?


A. Abx.
B. Immunoglobulin
C. Steroid
Answer: c
Medscape:
Avoid: NSAID, BB (unstable)
Standard treatment of clinically significant disease includes the detection of dysrhythmia with cardiac monitoring, the
administration of supplemental oxygen, and the management of fluid status.
Left ventricular dysfunction developing from myocarditis should be approached in much the same manner as other causes
of congestive heart failure (chf), with some exceptions. In general, sympathomimetic drugs should be avoided, because the
increase the extent of myocardial necrosis and mortality.
Intensive immunosuppressants “e.g. Steroids” has been shown to have some benefits only in small scale clinical trials, and
has not been validated in a larger scale. At this time, it is not recommended until a clear evidence is available.
28
93. Mid diastolic rumble with opening snap and dysphagia. Where is the lesion?
A. Left atrium
B. Aortic arch
C. Left ventricle
Answer: a

This is an auscultation finding of mitral stenosis. In mitral stenosis, blood pools in the left atrium causing it to enlarge. Left atrium is
located in front of esophagus so any enlargement of left atrium will compress on the esophagus causing dysphagia.
Usmle step 1 anatomy kaplan lecture notes

94. What lower blood pressure the most ??


A. Weight loss
B. Salt restricted diet
C. Aerobic exercise
Answer: a

Weight loss helps to prevent hypertension (range of approximate systolic Blood pressure reduction [sbp], 5-20 mm Hg per
10 kg).
Reduce sodium intake to no more than 100 mmol/d (2.4 g sodium or 6 g sodium chloride; range of approximate sbp
reduction, 2-8 mm hg)
Engage in aerobic exercise at least 30 minutes daily for most days (range of approximate sbp reduction, 4-9 mm hg)
Reference: medscape: http://emedicine.medscape.com/article/241381-treatment#d9

95. Patient with heart failure and af, you added digoxin, what is the benefit:
A. Decrease heart failure
B. Slow ventricular rate
C. Decrease ventricular efficacy
Answer: b

Reference: uptodate

29
96. Which murmur is a crescendo-decrescendo murmur?
A. Systolic ejection (aortic stenosis)
B. Physiologic murmur
C. Innocent murmur

Answer: a

Classic crescendo-decrescendo systolic murmur of aortic stenosis begins shortly after the first heart sound.

Reference: medscape: http://emedicine.medscape.com/article/150638-overview

97. Patient with mitral valve disease and murmur radiating to the axilla, right ventricle is enlarged. How will you manage?
A. Mitral valve replacement
B. Medical treatment
C. Pci
Answer: a
It’s mitral regurgitation.valve replacement is indicated when the heart starts to dilate. Do not wait for left ventricular end systolic
diameter (lvesd) to become too large because the damage will be irreversible.(many references)

98. Pt with mi he is on nitroglycerin and morphine. After 20 minutes he worse with raise jvp and no lung finding. Bp drop.
A. Ruptured
B. Arrhythmia
C. Rv infarction
Answer: c

99. Patient with mi. What is the drug that decrease the mortality rate?
A. Captopril
B. Digoxin
C. No b-blocker in the choices
Answer: a

100.High pitched? Murmur crescendo decrescendo, midsternal


A. Ejection systolic
B. Ejection regurgitate
C. Innocent

Answer: a

101.Which one of these can cause LBBB?


A. Aortic stenosis
B. PE
C. Cardiomyopathy

30
Answer: A (Most common of these choices)
Arteriosclerotic coronary artery disease is the most common adult cardiovascular disease. As a result, it is the most common cause
of left bundle branch block. Hypertension, aortic valve disease and cardiomyopathies continue to be important but less common
etiologic disorders.
Reference: Journal of the American College of Cardiology.

102.Patient with chest pain and ST changes, you will find elevation in:
A. ALT
B. AST
C. Troponin

Answer: C

103.Prophylactic antiarrhythmic therapy?


A. Procainamide
B. Lidocaine
C. Amiodarone

Answer: C
Don’t give the patient any anti-arrhythmic medication unless he has an arrhythmia.
Reference: Step up to Medicine.
C is the best answer compared to A and B (if not post MI)
The question might be similar to this (Post MI):

In this case the answer will be Metoprolol, which is a beta blocker (Prophylaxis for arrhythmias after MI: Beta blocker).
Reference: Lippincott Illustrated Reviews Pharmacology.

104.According to the modified criteria of rheumatic fever, which is considered as a minor criteria?
A. Carditis
B. Fever
C. Arthritis

Answer: B

31
105.(long scenario) case of endocarditis with negative bacterial culture, there is diastolic murmur radiate to the left axilla. What is
the most likely diagnosis?
A. SLE
B. Rheumatoid arthritis.
C. Acute myocarditis.

Answer: A
Libman-Sacks endocarditis (otherwise known as verrucous, marantic, or nonbacterial thrombotic endocarditis) is the most
characteristic cardiac manifestation of the autoimmune disease systemic lupus erythematosus.
One cohort study reported that pure mitral regurgitation was the most common valvular abnormality, followed by aortic
regurgitation, combined mitral stenosis and regurgitation, and combined aortic stenosis and regurgitation.
Reference: http://emedicine.medscape.com/article/155230-overview#showall
Blood culture-negative infective endocarditis (IE):
Http://www.uptodate.com/contents/epidemiology-microbiology-and-diagnosis-of-culture-negative-endocarditis

106.A male patient known case of HTN on ACEI but with poor control, which drug you will add?
A. Thiazide
B. Furosemide
C. Vasodilator

Answer: A

107.Hypertensive patient on ACEI but not controlled BLOOD PRESSURE , what to add?
32
A. Furosemide
B. Thiazide
C. Beta blocker

Answer: B
Step 1 antihypertensive treatment with an angiotensin-converting enzyme (ACE) inhibitor or a low-cost angiotensin-II receptor
blocker (ARB). If diuretic treatment is to be initiated or changed, offer a thiazide-like diuretic, such as chlortalidone. A combination
of ACE inhibitors and diuretics instead of ACE inhibitors alone is recommended for preventing recurrence of stroke based on findings
of (PROGRESS).
Http://www.clevelandclinicmeded.com/medicalpubs/diseasemanagement/nephrology/arterial-hypertension/
Https://www.nice.org.uk/guidance/cg127/chapter/1-guidance?UNLID=8546402482015836347
Http://www.pharmacology2000.com/Cardio/antihyper/antihype.htm

108.Pt with retrosternal chest pain for 3 days increase in the last 24 hrs relieved by sublingual nitro Dx?
A. Unstable angina
B. Pericardidits
C. MI

Answer: ???
The duration of chest pain here is more than 3 days it is possible to be cardiac cause but it is unlikely. Suggestion of Non Cardiac
cause eg. Esophageal spasm that will relieved by nitro.
Http://www.health.harvard.edu/heart-health/chest-pain-a-heart-attack-or-something-else
Http://emedicine.medscape.com/article/159383-clinical
Http://emedicine.medscape.com/article/174975-clinical
Http://patient.info/health/heart-attack-myocardial-infarction-leaflet
Http://www.merckmanuals.com/professional/cardiovascular-disorders/coronary-artery-disease/acute-coronary-syndromes-acs

109.Tx of acute myocrditis?! caused commonly by viral infection e.g Parovirus B19
A. Ab
B. Immunoglobulin
C. Steroids
Answer:

33
th
Reference: short text book of medical diagnosis & management , danish 11 edition pg127

110.Down fix spilt s2 +harsh systolic murmur + biventricular hypertrophy +.... Dx?
A. ASD
B. VSD
C. Atrioventrical septal defect

111.Academic professor present with headache sometimes resolve with 2 tablets of panadol. His Blood pressure is 170/100,
what is the type of his htn?
A. Essential
B. Secondary
C. Pain causing
Answer: a
Primary or essential hypertension accounts for 90-95% of adult cases, and a small percentage of patients (2-10%) have a secondary
cause.
Source: http://emedicine.medscape.com/article/241381-overview#a4

112.What is the best to dx coarctation of aorta?


A. Cardiac mri
B. Echo

Answer: ??
No specific laboratory tests are necessary for coarctation of the aorta. Imaging studies that may be helpful include the following:
chest radiography (findings vary with the clinical presentation of the patient), barium esophagography (classic “e sign,” representing
compression from the dilated left subclavian artery and poststenotic dilatation of the descending aorta) echocardiography (2-
dimensional echocardiography, pulsed-wave doppler, and color flow mapping). In another reference once suspected,

34
an echocardiogram is the most commonly used test to confirm the diagnosis. It is more accurate. Cardiac catheterization helps to
confirm the diagnosis when echocardiography findings are not completely clear.
References: http://www.cdc.gov/ncbddd/heartdefects/coarctationofaorta.html
Http://emedicine.medscape.com/article/150369-overview

113.Mi patient within 6 hours what is the most expected complication?


A. Pe
B. Arrhythmia

Answer: c
Arrhythmia is the most expected complication of mi within 6 hours. About 90% of patients who have an acute myocardial infarction
(ami) develop some form of cardiac arrhythmia during or immediately after the event.
References: fa step 1, http://emedicine.medscape.com/article/164924-overview#a2

114.Young pt with HTN discrepancy, what is the tx?


A. Trans-aortic stenting
B. Thiazides

Answer: b

115.What is the commonest cause of HTN in adolescent?


A. Idiopathic
B. Renal
Answer: ?

116.Patient with acute rheumatic fever show acute cardiac symptoms, what is the treatment?
A. IV penicillin
B. Im steroid

Answer: b
Treatment of arf consists of anti-inflammatory therapy, antibiotic therapy, and heart failure management.
Patients with severe carditis (significant cardiomegaly, congestive heart failure, and/or third-degree heart block) should be treated
with conventional therapy for heart failure.
Corticosteroids should be reserved for the treatment of severe carditis.
Reference: http://emedicine.medscape.com/article/333103-medication#4

117.What is the commonest cause of HTN in adolescent?


A. Idiopathic
B. Renal

Answer: a
Reference: http://www.stanfordchildrens.org/en/topic/default?Id=high-blood-pressure-in-children-and-adolescents-90-p01609

35
118.Ischemic heart patient with lab result of high lipid. What is the next order?
A. TFT
B. LFT

Answer: a
It is recommend treating all patients with cvd with high-dose statin therapy.
In 2012, the us food and drug administration revised its labeling information on statins to only recommend liver function testing
prior to initiation of statin therapy and to only repeat such testing for clinical indications.
Reference: uptodate

119.An ecg with st elevation in v1-v5:


A. Extensive anterior mi
B. High lateral mi

Answer: anterior or anteroseptal mi


In extensive mi there has to be st elevation of 6 or more leads.
Extensive anterior mi includes st elevation in anterior leads (v3, v4) + septal leads (v1, v2), and lateral leads (i, avl, v5, v6).
Reference: toronto notes

120.What is the commonest cause of HTN in adolescents:


A. Idiopathic
B. Renal

Answer: a
Essential HTN applies to more than 95% of cases of htn. 

Renal artery stenosis (most common cause of 
secondary htn), 
 birth control pills are the most common secondary cause of HTN in
young women.
Reference: step-up of medicine

36
121.HTN patient with decrease gfr ;
A. Bilateral renal artery stenosis
B. DM nephropathy
Answer:

122.Patient with chest pain, ischemic, presented after 6 h what to give him?
A. Aspirin
B. Thrombolytic
Answer: a

123.A case of myocardial infarction .. What is the complications if he didn't managed after six hour?
A. Myocardial rupture
B. Arrhythmias
Answer: b!
Pericarditis:
post infarction pericarditis usually begins several days after the infarct, due to an inflammatory exudate in the pericardium.
Ventricular septal rupture:
acute ventricular septal rupture can occur usually several days following the acute infarction, due to softening of the necrotic
portion of the septum.
Complications may occur due to ischemic or injured tissue and therefore may begin within 20 minutes of the onset of m.i., when
myocardial tissue injury begins. These complications include arrhythmias and heart block (due to injured or ischemic conduction
system tissue), and hypotension and congestive heart failure (due to ischemic or injured muscle tissue, resulting in abnormal filling
{"diastolic dysfunction"} or abnormal emptying {"systolic dysfunction"}).
Referance:
http://www.brown.edu/courses/bio_281-cardio/cardio/handout4.htm

124.If you give the patient ace inhibitors for hypertension but there was no effect ,what is the drug you are going to add ?
A. Thiazide diuretic
B. Beta blocker

Answer: a
Http://www.ccmdweb.org/dsl/middle.aspx?Slideid=3607&catid=1047
Read jnc8 guidelines

125.Mitral stenosis ecg :


A. Bifid p wave left axis deviation
B. Elevated p wave right axis deviation

Answer: b
Explanation: the ecg in mitral stenosis is often normal early in disease. The most common finding is left atrial enlargement (p-
mitrale), however this finding disappears if the patient enters atrial fibrillation. Right heart strain may produce findings of right axis
deviation and right ventricular hypertrophy on ecg. In pure mitral stenosis, left ventricular hypertrophy would be absent.
Reference http://www.healio.com/cardiology/learn-the-heart/cardiology-review/mitral-stenosis/introduction-and-etiology
37
126.Traveling women 18hr in flight developed ll edema what will you do:
A. Compression ultrasonography
B. Angiography

Answer: a
According to bmj best practice long flight is strong risk factor and lower limb swelling is a common symptom of dvt." first-line test in
all high-probability patients (wells' score of 2 or more) or in low-probability patients (wells' score <2) with an elevated d-dimer level
is venous duplex ultrasound (dus)."
Reference: http://bestpractice.bmj.com/best-practice/monograph/70/diagnosis/tests.html

127.Mi patient presented to er after resuscitation he developed coma and then died, what postmortem change you will find
A. Inter ventricular hemorrhage
B. Brown colored area supplied by middle meningeal artery
Answer: b
Reference: https://annalsofintensivecare.springeropen.com/articles/10.1186/2110-5820-1-45

38
128.In patients with an ostium seccundum defect, electrocardiogram (ecg) results usually demonstrate the following:
A. Left -axis deviation
B. Right ventricular hypertrophy
Answer b
Atrial septal defect (asd) is one of the most common congenital cardiac abnormalities identified in adults. There are 3 types of asd:
ostium secundum, ostium primum, and sinus venosus. In patients with an ostium secundum defect, electrocardiogram (ecg) results
usually demonstrate the following:
Right-axis deviation
Right ventricular hypertrophy
Rsr' pattern in the right precordial leads with a normal qrs duration
Http://emedicine.medscape.com/article/348121-overview

129.A patient with left bundle branch block will go for dental procedure, regarding endocarditis prophylaxis:
A. Amoxicillin before procedure
B. No need
Answer: b
antibiotic prophylaxis is indicated for the following high-risk cardiac conditions: prosthetic cardiac valve, history of infective
endocarditis, congenital heart disease, cardiac transplantation recipients with cardiac valvular disease.
Http://emedicine.medscape.com/article/1672902-overview#a2

130.Patient with mi, blood pressure 80/65 ecg shows bradycardia (i think complete heart block) what is your action.
A. Dopamin/ norepinephrin
B. IV fluid bolus / percutaneous pacemaker
Answer: b in case of severe bradycardia or hemodynamic unstably: atropine (if inf. Mi) and placement of pacemaker
step up to medicine
http://www.clevelandclinicmeded.com/medicalpubs/diseasemanagement/cardiology/complications-of-acute-myocardial-
infarction/

131.Which of the following is major criteria of infective endocarditis?


A. Positive blood culture
B. Echo shows valvular …
Answer: b we need positive blood cultures from 2 separate blood cultures or 2 positive cultures from samples drawn > 12 hrs.
Http://reference.medscape.com/calculator/endocarditis-diagnostic-criteria-duke

132.A patient complains of chest pain that radiates to left shoulder and jaw what’s best analgesics?
A. All answers are painkillers medications
B. Morphine
Answer: b
Morphine should be given for the relief of chest pain or anxiety. This also decrease work demand on cardiac muscle. The use of
other analgesic agents, such as nonsteroidal anti-inflammatory drugs (NSAIDs) should be avoided if at all possible, as the use of
these agents has been associated with adverse cardiovascular events.
Http://emedicine.medscape.com/article/155919-treatment#showall
Http://cursoenarm.net/uptodate/contents/mobipreview.htm?39/40/40576#h6
39
133.Patient post delivery by one week presented with orthopnea and increase jvp symptoms of heart failure the x-ray will show:
A. Bilateral infiltration
B. Thoracocardiac ratio <50%
Answer: not complete options.
Chest x ray heart failure findings include: pleural effusions, cardiomegaly (enlargement of the cardiac silhouette cardio-thoracic ratio
> 50%), kerley b lines (horizontal lines in the periphery of the lower posterior lung fields), upper lobe pulmonary venous congestion
and interstitial edema.

134.Most common cause of 2dry hypertension


A. Kidneys disease
B. Cons syndrome
Answer: a
Link: http://www.aafp.org/afp/2010/1215/p1471.html

135.Digoxin antidote
A. Lidocaine
B. Immune fab
Answer: b
In the case of severe digoxin intoxication, an antidote digoxin immune fab (digibind) is available. Digibind binds and inactivates
digoxin.

136.Pain management in mi:


A. Aspirin
B. Morphine
Answer: b

137.Indication of defibrillator (pace maker) is injury in?


A. Sa node
B. Av node
This question is incorrect since defibrillators and pacemakers are different devices and have totally different indications
Defibrillators:
Electrical cardioversion and defibrillation have become routine procedures in the management of patients with cardiac arrhythmias.
Cardioversion is the delivery of energy that is synchronized to the qrs complex, while defibrillation is the nonsynchronized delivery of
a shock randomly during the cardiac cycle.
Most defibrillators are energy-based, meaning that the devices charge a capacitor to a selected voltage and then deliver a
prespecified amount of energy in joules. The amount of energy which arrives at the myocardium is dependent upon the selected
voltage and the transthoracic impedance (which varies by patient).

Pacemakers
Cardiac pacemakers are effective treatments for a variety of bradyarrhythmias. By providing an appropriate heart rate and heart
rate response, cardiac pacing can reestablish effective circulation and more normal hemodynamics that are compromised by a slow
heart rate.

40
138.Most common risk factor for cad:
A. Smoker and 50 y m
B. Dyslipidemia and obesity
Answer: b
Link:http://www.heart.org/heartorg/conditions/more/myheartandstrokenews/coronary-artery-disease---coronary-heart-
disease_ucm_436416_article.jsp#.v2fir8dbfq0

139.Young patient with HTN discrepancy, what is the tx ?


A. Trans-aortic stenting
B. Thiazides

Answer : a
If there is a discrepancy in the blood pressure between two limbs this points to coarctation of the aorta which can be treated with
stenting.
Reference: Medscape

140.Patient with acute rheumatic fever show acute cardiac symptoms, rx?
A. IV penicillin.
B. Im steroid.

Answer: a

141.50 years old patient with third reading of persistent hypertension wasn't started on medication yet, lab shows high na of 147
and low k of 3 other parameters were normal what's most likely the diagnosis?
A. Essential hypertension
B. Hyperaldosteronism

Answer: b

Liddle syndrome is a rare autosomal dominant disorder characterized by severe hypertension and hypokalemia. Liddle syndrome is
caused by unrestrained sodium reabsorption in the distal nephron due to mutations found in genes encoding for epithelial sodium
channel subunits. Inappropriately high reabsorption of sodium results in both hypertension and renal potassium wasting.
Reference: toronto note, merck manual.

142.Patient with cad angio done for him showed left coronary artery 90% stenosis and the right is 40%, what is the management
A. Left coronary bypass
B. Bypass all vessel
C. Stent
answer: a
Check the reference for the classification and indication for cabg done by the american college of cardiology (acc) and the american
heart association (aha)
reference: http://emedicine.medscape.com/article/164682-overview#a2

143.Patient HTN and hyperlipidemia with chest pain and when he is coming to hospital take drug relieve his symptoms which
drug:
41
A. Nitric oxide
B. Digoxin
Answer: a.
Nitroglycerin is a nitrate. It works by relaxing blood vessels which allows blood to flow more easily. This reduces the heart's workload
and the amount of oxygen needed by the heart.

144.Patient with chest pain for 6 hours what to give?:


A. Tpa
B. Aspirin
Answer:b.
Management:
o Immediate assessment with cardiac monitoring
o Early therapy (e.g., intravenous access, oxygen, aspirin, nitroglycerin, morphine, unfractionated heparin or low-molecular-
weight heparin [lmwh], nitrates, beta-blockers, p2y12 inhibitors, glycoprotein iib/iia receptor antagonist)
o Initial management according to the american heart association advanced cardiac life support guideline for those with
compromised vital signs
o Percutaneous coronary intervention or coronary artery bypass graft if indicated
o Thrombolytic
Https://www.guideline.gov/content.aspx?Id=39320

145.Academic professor complaining of headache relieved by 2 tablets of panadol which type of htn:
A. Malignant
B. Essential
Answer: b

146.Ischemic heart pt with lap result of high lipid .. What the next order ?
A. TFT
B. LFT
Answer: a

147.Female with leg pain when she walk 300 m. Relief by rest:
A. Claudication
B. Dvt
Answer: a
Reference: toronto notes
claudication
Pain with exertion: usually in calves or any exercising muscle group relieved by short rest: 2-5 min, and no postural
changes necessary reproducible: same distance to elicit pain, same location of pain, same amount of rest to relieve pain

148.What can cause Coarctation of aorta?


A. Coronary artery disease
B. Aortic dissection

Answer: B

42
Complications of untreated patient include: HTN, stroke, aortic aneurysm, aortic dissection, premature coronary artery disease, HF,
brain aneurysm or hemorrhage. The most common complication in adult is HTN and CAD.

149.A patient with renal function test abnormalities. Tests show beads on string appearance. What is the diagnosis?
A. Renal artery disease
B. Fibromuscular dysplasia

Answer: B
The string-of-beads appearance is considered pathognomonic for medial fibroplasia on diagnostic angiography.
Reference: http://emedicine.medscape.com/article/417771-overview#a

150.A Patient had an MI and was treated for that, after that he developed chest pain that worsen with movement and taking
deep breath. On examination there was distant heart sounds and pericardial rub. What is the most ECG changes associated
with this condition?
A. ST changes
B. PR prolongation

Answer: A
A case of pericarditis.
ECG: initially diffuse elevated ST segments ± depressed and shorten PR segment, the elevation in the ST segment is concave upwards
>> 2-5 d later ST isoelectric with T wave flattening and inversion.
Reference: Toronto Notes.

151.A male patient came to the ER complaining of palpitations, tachycardia ... ECG shows
deep S wave in lead (?) And tall R wave in lead (?) , Dx ?
A. LBBB
B. RBBB

Answer:?
LBBB:
QRS duration >120 msec
V1 and V2: W pattern and wide deep slurred S wave
V5 and V6: wide QRS complex with M pattern or rabbit ear pattern

RBBB: the opposite of LBBB


QRS duration >120 msec
V1 and V2: wide QRS ( more than 3 small squares) with ear rabbit pattern or M shape
pattern
V5 and V6: wide and deep/slurred S

LVH:
S inv1 + R in V5 or V6 > 35mm above age 40, (>40 mm for age 31-40, > 45 mm for age 21-30)
R in avl >11mm
R in I + S in III >25mm

RVH:
Right axis deviation
43
R/S ratio > 1or qr in lead V 1
RV strain pattern: ST segment depression and T wave inversion in leads V1-2
Reference: Toronto Notes

152.What is the commonest cause of HTN in adolescents?


A. Idiopathic
B. Renal

Answer:A
Secondary hypertension is more common in preadolescent children, with most cases caused by renal disease. Primary or essential
hypertension is more common in adolescents and has multiple risk factors, including obesity and a family history of hypertension.
Http://emedicine.medscape.com/article/889877-overview#a4
Http://www.aafp.org/afp/2006/0501/p1558.html

153.A patient comes to you for routine check up, his dad died of hear disease aged 66 what is the best screening test:
A. Lipid profile
B. Ecg
Answer: a http://www.lifelinescreening.com/what-we-do/what-we-screen-for/heart-disease

154.60 Year-old has only HTN the best drug for him?!!
A. Diuretics
B. CCB
Answer: i think the answer is diuretics!
Uptodate:-in general, three classes of drugs are considered first-line therapy for the treatment of hypertension in elderly patients:
low-dose thiazide diuretics (eg, 12.5 to 25 mg/day of chlorthalidone), long-acting calcium channel blockers (most often
dihydropyridines), and ace inhibitors or angiotensin ii receptor blockers. A long-acting dihydropyridine or a thiazide diuretic is
generally preferred in elderly patients because of increased efficacy in blood pressure lowering.
Among elderly patients in whom there is a reasonable likelihood of requiring a second drug (eg, more than 10/5 mmHg above goal),
some physicians who practice according to the results of the accomplish trial would prefer initial therapy with a long-acting
dihydropyridine calcium channel blocker. This is because, if additional therapy is required, a long-acting ace inhibitor/arb can be
added to achieve the desired combination regimen.
Medscape: on the basis of Blood pressure -lowering efficacy and outcomes data, CCB s are acceptable alternatives to diuretics for
first-line treatment of hypertension in the elderly and may offer advantages in some patient groups, eg, those with the metabolic
syndrome.

American family physician:


Low-dose thiazide diuretics remain first-line therapy for older patients. Beta blockers, angiotensin-converting enzyme inhibitors,
angiotensin-receptor blockers, and calcium channel blockers are second-line medications that should be selected based on
comorbidities and risk factors.

Refrence: http://emedicine.medscape.com/article/241381-treatment
Http://www.aafp.org/afp/2005/0201/p469.html

44
155.What is the initial management of hypotension induced lumber puncture ?
A. IV fluid
B. Bed rest Intracranial hypotension caused by LP, treated by: bed rest, blood patch, IV caffiene, theophylline.
Answer:?

156.Early detection of st segment elevation mi :


A. Typical chest pain with … some ecg changes
B. Atypical chest pain with some ecg changes

Answer: a (question not clear, and choices incomplete)


Explanation: presents with central chest pain that is classically heavy in nature, like a sensation of pressure or squeezing. St-
elevation mi (stemi) is suspected when a patient presents with persistent st-segment elevation in 2 or more anatomically contiguous
ecg leads in the context of a consistent clinical history. Cardiac enzymes confirm diagnosis. Treatment should, however, be started
immediately in patients with a typical history and ecg changes, without waiting for laboratory results.
Reference: http://bestpractice.bmj.com/best-practice/monograph/150/highlights/summary.html

157.Ecg showed wide complex tachycardia, jvp: a cannon wave, diagnosis?


A. Vt
Answer: a
This patient has ventricular tachycardia based on the presence of a wide complex tachycardia and cannon "a" waves in the jugular
veins. Cannon "a" waves are due to the unsynchronized contraction of the ventricles and the atria. This results in a retrograde flow
of blood back to the jugular veins with atrial systole. The variation of the intensity of s1 is caused by the ventricle contracting at
times when the av valves are open and at other times when they are closed. Procainamide, amiodarone, and lidocaine are the most
effective treatments for a hemodynamically stable patient.

158. Mi patient has big thrombus in left coronary artery what is drug that cause thrombolytic action?
Thrombolytic therapy is indicated in patients with evidence of st-segment elevation mi (stemi) or presumably new left bundle-
branch block (lbbb) presenting within 12 hours of the onset of symptoms if there are no contraindications to fibrinolysis.
Http://emedicine.medscape.com/article/811234-overview#a3

159.Patient presented with chest pain was diagnosed with stem. What is the other finding you can detect on his ecg?
Answer: inverted t wave
Other ecg findings: reciprocal st segment depression, q wave indicating old infarct.
Source: step-up to medicine

160.A patient presented with inferior mi. Which artery is affected?


Answer: right coronary artery (rca)
Inferior mi present in ecg with st elevation in ii, iii and avf. Rca supply the right atrium and ventricle, which lie on the diaphragm and
constitute the inferior wall of the heart muscle.

45
161.What is most common heart lesion in down syndrome?
Answer: avd
Http://downsyndrome.nacd.org/heart_disease.php

162.How calcium channel blocker leads to edema?


Answer: increase hydrostatic pressure
Its etiology relates to a decrease in arteriolar resistance that goes unmatched in the venous is disproportionate change in resistance
increases hydrostatic pressures in the pre-capillary circulation and permits fluid shifts into the interstitial compartment.
Http://www.medscape.com/viewarticle/460070_1

163.Most common cause HTN in adult?


Hypertension may be primary, which may develop as a result of environmental or genetic causes, or secondary, which has multiple
etiologies, including renal, vascular, and endocrine causes. Primary or essential hypertension accounts for 90-95% of adult cases, and
secondary hypertension accounts for 2-10% of cases.
Http://emedicine.medscape.com/article/241381-overview

164.SE of nitroglycerin:
A. Throbbing headache

165.Mitral stenosis ecg:


A. Bifid p wave left axis deviation
B. Elevated p wave right axis deviation

Answer ; ?
46
In mitral stenosis the ecg is often normal. Pathological changes in the ecg include:
Left atrial hypertrophy:
o Causes bifid p wave
o Increased voltage in the later part of the p wave gives it a large, bifid appearance in leads ii, iii and avf
o The second half of the p wave is negative in v1
o Atrial fibrillation is present in 60 to 70%
Right ventricular hypertrophy:
o A dominant r wave in v1 and v2 indicates pulmonary hypertension
Right axis deviation

There may be digitalis effects

166.Heart sound that supports pulmonary htn.


o Loud pulmonic component to the second heart sound.
o A right ventricular fourth heart sound the murmur of tricuspid regurgitation, which is holosystolic, located at the le lower
sternal border, and augments with inspiration, is common in patients with moderate to severe pulmonary hypertension.
o Other findings on auscultation may include an early systolic click and the murmur of pulmonic regurgitation. A right
ventricular third heart sound o en signifies advanced disease and right heart failure.

167.A man lost his consciousness during playing football i.e. Syncopal attack. He had history of aortic stenosis. How can you
explain this case: (no low cardiac output not mentioned in the choices!)
A. Cardiac arrhythmia.
Answer: a might be right because aortic stenosis is common cause of left bundle branch block

168.Which of the following is a thrombolytic?


Answer: streptokinase
Streptokinase (sk) is an enzyme secreted by several species of streptococci that can bind and activate human plasminogen. Sk is used
as an effective and inexpensive thrombolysis medication in some cases of myocardial infarction (heart attack) and pulmonary
embolism.
Belongs to a group of medications known as fibrinolytics.
Https://en.m.wikipedia.org/wiki/streptokinase

47
169.Patient come with pericardial pain, ecg st segment elevation, patient given aspirin and nitrate, but no relieve of pain what
next step you will do?
Answer: give morphine iv, refractory or severe pain should be treated symptomatically with IV morphine.
Http://emedicine.medscape.com/article/155919-treatment#d6

170.Child with episodic of cyanotic lip with cold extremities, what is the diagnostic investigation: echocardiography to prevent
cyanosis in chd?
A. Prostaglndin e

171.St elevation at lead 2, 3, and avf , that represent which cardiac parts? Inferior mi

172.?? Year old male came with history of leg pain after walking, improved after resting, he notices loss of hair in the shaft of his
leg and become shiny?
A. Chronic limb ischemia

173.Acs which enzyme to order after 1 hour

Cardiac markers
Marker Initial elevation Peak elevation Return to baseline
Myoglobin 1-4 h 6-7 h 18-24 h

Ck-mb 4-12 h 10-24 h 48-72 h


Cardiac trop i 3-12 h 10-24 h 3-10 d
Cardiac trop t 3-12 h 12-48 h 5-14

174.Ischemic heart disease first test to diagnose?

48
bmjhttp://bestpractice.bmj.com/best-practice/monograph/152.html

175.Atrial septal defect changes on ecg: notched R wave


Not diagnostic: normal; or tall p waves, large r waves in v1, crochetage pattern in inferior limb leads. Bmj
http://bestpractice.bmj.com/best-practice/monograph/1099/diagnosis/tests.html

176.Asd: . Clinical features mcq.


o Mild systolic ejection murmur at pulmonary area secondary to increased pulmonary blood flow
o Fixed split s2 ( important)
o Diastolic flow “rumble” murmur across tricuspid valve area secondary to increased blood flow
o In advanced disease, signs of rvf may be seen
Step up to medicine

177.Signs of chronic stable angina?


Chest pain or substernal pressure sensation that lasts less than 10 to 15 minute, gradual in onset, brought on by exertion or
emotion, relieved with rest or nitroglycerin, pain does not change with breathing nor with body position. Stepuptomedicine

178.Which one is criteria of TOF?


Pulmonary outflow stenosis

Tetralogy of fallot involves four heart defects:


- A large ventricular septal defect (vsd)
- Pulmonary stenosis
- Right ventricular hypertrophy
- An overriding aorta

179.Cardiac defect communication between aortic arch and pulmonary vessels


Answer:PDA

49
180.HTN on treatment now has gout what drug?
Answer: hydroxythiazide
Thiazide side effects: hyperglycemia, hyperlipidemia, hyperuricemia, hypercalcemia, hypokalemia, hyponatremia, hypomagnesemia,
hypocalciuria.

181.Patient present with typical symptoms of angina relieve by using sublingual drug what is the mechanism of action of this
drug?
Answer: relax smooth muscle and dilation of vein and artery and decrease resistance.
Http://www.uptodate.com/contents/nitrates-in-the-management-of-stable-angina-pectoris#h2

182.Description of congenital heart disease there is decrease vascularization of the lungs on x-ray, cardiac shadow (they
described boot shaped but not in a direct way)
A. Teratology of fallot was my answer
Answer: a
Link: http://radiopaedia.org/articles/boot-shaped-heart-1

183.Old woman has ht it’s not controlled even with multi drugs. She sleeps afternoon a lot and feels fatigue most of the time,
what is the cause of her resistance ht?
A. Obstructive sleep apnea
Answer: a
Severe obstructive sleep apnea may interfere with blood pressure (Blood pressure )–lowering treatment in patients at high
cardiovascular disease risk or with established cardiovascular disease, results of a multicenter clinical trial suggest.
Link: http://www.medscape.com/viewarticle/832716

184.Patient has history of endocarditis and underwent to extract his teeth the doctor what will do before extraction:
Answer: 2 g amoxicillin before or 2 g clindamycin before

185.Treatment of acute myocarditis?


Treatment of myocarditis includes supportive therapy for symptoms of acute heart failure with use of diuretics,
nitroglycerin/nitroprusside, and angiotensin-converting enzyme (ace) inhibitors. Inotropic drugs (eg, dobutamine, milrinone) may be
necessary for severe decompensation, although they are highly arrhythmogenic.

186.Patient with moderate mitral stenosis, syncope, what is the cause of syncope?
Most likely is by causing afib

soft, less than 3, no thrill, systolic or continues ( both systolic and diastolic), change with postion (
187.2 yo ejection systolic murmur cause? increase with standing )
Http://emedicine.medscape.com/article/1894036-overview#a8

50
188.Cause of delayed arterial radiofemoral pulse:
Answer: coarctation of aorta

189.Heart defect result from malrotation and truncus arteriosus


A. Transposition of great arteries
Answer: a

190.Management of the acute attack


A single dose of benzyl penicillin (1.2 million u im) or oral phenoxymethylpenicillin (250 mg 4 times daily for 10 days) should be given
on diagnosis to eliminate any residual streptococcal infection. If the patient is penicillin-allergic, erythromycin or a cephalosporin can
be used. Treatment is then directed towards limiting cardiac damage and relieving symptoms.

Reference: davidsone

191.Patient elderly k/c of DM and HTN with hx of previous tia and ecg showed atrial fibrillation controlled on digoxin, what’s next
for him?
A. Anticoagulate with warfarin.
Answer: a

192.Most common cause of HTN in saudi arabia.


Answer: essential htn.

193.What is first thing to do in mi?


A. Ecg

Answer: a
The first goal for healthcare professionals is to diagnose in a very rapid manner whether the patient is having an stemi or
nstemi because therapy differs between the 2 types of myocardial infarction.

Reference: medscape: http://emedicine.medscape.com/article/155919-treatment

194.Pt with decreased lower limbs pulses and intercostal notching in x-ray. What is the diagnosis?
A. Coarctation of aorta
Answer: a
Coarctation of the aorta
HTN in upper extremities with hypotension in lower extremities
Cxr
Notching of the ribs

51
“figure 3” appearance due to indentation of the aorta at site of coarctation, with dilation before and after the stenosis

Reference: step up to medicine

195.69 Year-old female on antiarrhythmic and she developed hyperpigmentation. Which drug cause that?
Answer: amiodarone
1. Amiodarone has been used both in the treatment of acute life-threatening arrhythmias as well as the chronic suppression
of arrhythmias. Long-term administration of amiodarone (usually more than eighteen months) is associated with a light-
sensitive blue-grey discoloration of the skin; such patients should avoid exposure to the sun and use sunscreen that
protects against ultraviolet-a and -b. The discoloration will slowly improve upon cessation of the drug, however, the skin
colour may not return completely
Reference: wikipedia: https://en.wikipedia.org/wiki/amiodarone

196.Lipid profile of a patient shows high level and patient is on simvastatin, what to add?
Answer:
Reference: davidsons 22

197.Mi patient within 6 hours what is the most expected complication? :


Answer: arrhythmias mostly vf

Complication of mi:

o Electrical dysfunction (arrhythmias) the commonly causes mortality in the first 72 h


o Mechanical dysfunction (heart failure, myocardial rupture or aneurysm, papillary muscle dysfunction)
o Thrombotic complications (recurrent coronary ischemia, mural thrombosis)
o Inflammatory complications (pericarditis, dressler syndrome)
52
Reference: merck manual

198.HTN patient develops gout, which drug side effect


Answer: thiazide
Hyperuricemia is a common finding in patient treated with thiazide due to decrease renal execration of uric acid and may lead to
gouty arthritis
Reference: uptodate

199.Patient had mi, he was given sublingual nitrate and analgesia, the moa of the analgesic that was given ?
Answer: morphine bind full agonist to mu receptor
Reference: rxlist.

200.Scenario of a patient with chest pain that relieved when lying down and increase when lying forward with chest x-ray show
the heart (globular like a ball) what is the dx?
A. Pericardial effusion
Answer: a

201.Drug causing malignant syndrome.


haloperidol and chlorpromazine

202.This is a classical presentation of aortic stenosis, high pitched, crescendo-decrescendo, midsystolic murmur located at the
aortic area
Http://www.healio.com/cardiology/learn-the-heart/cardiology-review/heart-murmurs

203.SVT scenario what most important to test?


A. TSH
Answer
Electrolyte levels - should be checked because electrolyte abnormalities can contribute to paroxysmal supraventricular tachycardia
(paroxysmal svt)
CBC help to assess anemia
TSH to assess hyperthyroidism

204.Ecg showing ventricular tachycardia the patient is unstable what is the management?
Answer:
Hemodynamic compromise: electrical cardioversion.
No hemodynamic compromise: electrical cardioversion, lidocaine, amiodarone, type ia agents (procainamide, quinidine).
Source: toronto notes 2014 c20

53
205.Side effect of nitroglycerin (ttt of stable angina ) :
A. Headache
Common se of the drug are severe headache, severe hypotension, and, in certain cases, and bradycardia.
Https://en.wikipedia.org/wiki/nitroglycerin_(drug)#adverse_effects

206.Patient came with history of heart failure & pulmonary edema. Treatment?
A. Diuretics.
Answer: a
Acute treatment of pulmonary edema.
Treat acute precipitating factors (e.g. Ischemia, arrhythmias). Mnemonic: lmnop.
o L: lasix® (furosemide) 40-500 mg IV
o M: morphine 2-4 mg iv: decreases anxiety and preload (vasodilation)
o N: nitroglycerin: topical/iv/sl
o O: oxygen: in hypoxemic patients
o P: positive airway pressure (cpap/bipap): decreases preload and need for ventilation when appropriate
o P: position: sit patient up with legs hanging down unless patient is hypotensive

207.Right dominant coronary artery, branch of it?


A. Marginal
Answer: a. Toronto: LAD: Digonal and spetal perforates
RCA: Acute marginal, PDA, Posterior lateral
Right coronary artery (rca) branches:
Circumflex: obtuse marginals.
o Acute marginal branches.
o Atrioventricular (av) nodal artery.
o Posterior interventricular artery (piv) = posterior descending artery (pda).

208.Black man with high Blood pressure -, +ve fx h-x of -htn


A. CCB
Or thiazide or combination (jnc8).

209.Which type of the angina present when go to bed?


A. Prinzmetal (variant) angina
Typically, occur between midnight and 8 am (toronto 2015)

210.Patient has hypertension in upper limb and hypotension in lower limb , what is the most likely diagnosis ?
Answer: coarctation of aorta
Characterised by Blood pressure differential between upper and lower extremities (upper >lower).

211.Aspirin in high dose cause hyperthermia through which mechanism:


A. Aspirin and hyperthermia
Uncoupling of oxidative phosphorylation: electron transport continues but without the coupled atp synthesis. Result is heat
generation.
Http://www.acmt.net/ijmt/ijmt/4_5/case1c/tsld009.htm
Salicylate poisoning is manifested clinically by disturbances of several organ systems, including the central nervous system (cns)
and the cardiovascular, pulmonary, hepatic, renal, and metabolic systems. Salicylates directly or indirectly affect most organ
54
systems in the body by uncoupling oxidative phosphorylation, inhibiting krebs cycle enzymes, and inhibiting amino acid
synthesis.
Http://emedicine.medscape.com/article/1009987-overview#a5

212.Patient use anti-hyperlipidemia, liver enzymes normal, creatin kinase high:


A. Statins
Reference: http://www.medsafe.govt.nz/profs/puarticles/statinmyop.htm

213.27 Year-old female , recurrent palpitations ecg :


A. Supraventricular tachycardia
Http://www.practicalclinicalskills.com/mobile/ekg/supraventricular-tachycardia.aspx

214.Tetralogy of fallot presents with a vsd.


A. Tetralogy of fallot: http://emedicine.medscape.com/article/2035949-overview

215.Case of cardio change in ecg show , inverted t and st change , what is cause ?Ischemia - ( mi - infarction )

216.Cardiac case ..his brother died when walk dx?


A. Cardiomyopathy..

217.Which type of the angina present when go to bed


A. Variant prinzmetal angina.
Vasospasm in diseased or normal artery and during the episode patient will have st elevation not depression.

218.Side effect of atropine?


Frequent effects include xerostomia (dry mouth), dry skin, blurred vision, cycloplegia, mydriasis, photophobia, anhidrosis, urinary
hesitancy and retention, tachycardia, palpitation, xerophthalmia, and constipation, which may appear at therapeutic or sub-
therapeutic doses.
Http://www.drugs.com/sfx/atropine-side-effects.html

122- pt with STEMI, what you will find in the ECG?


Answer:
Https://www.heart.org/idc/groups/heart-public/@wcm/@mwa/documents/downloadable/ucm_467056.pdf

219.Cardiac patient on Aspirin, no new complaints. He is having low platelet (less than 10) for the last 6 months. What is your
management?
Answer: ?
The diagnosis is rare and called aspirin induced thrombocytopenia
Stop aspirin and find the cause (Answered by a senior cardiologist consultant)

55
220.Which of these anti HTN medications decrease afterload and preload?
A. ACEI
Answer: A

221.A patient presented with frothy red sputum, flushed cheeks, etc (long scenario). What is the diagnosis?
A. Mitral stenosis

Answer: A
Reference: Master the boards + Medical diagnosis and Management by Mohammad Inam Danish

222.A patient known case of aortic stenosis had syncope. What is the cause?
A. Systemic hypotension

Answer: A
Syncope from aortic stenosis often occurs upon exertion when systemic vasodilatation in the presence of a fixed forward stroke
volume causes the arterial systolic blood pressure to decline. It also may be caused by atrial or ventricular tachyarrhythmias.
Syncope at rest may be due to transient ventricular tachycardia, atrial fibrillation, or (if calcification of the valve extends into the
conduction system) atrioventricular block. Another cause of syncope is abnormal vasodepressor reflexes due to increased LV
intracavitary pressure (vasodepressor syncope).
Reference: http://emedicine.medscape.com/article/150638-clinical

223.A patient presented with STEMI. What is the first priority to do?
A. ECG
Answer: A

224.The most important risk factor for stroke ?


Hypertension, is the single most important treatable risk factor for stroke
Reference: uptodate.com

225.X-ray of pericardial effusion ( repeated )

Answer: Plain film: chest radiograph


Globular enlargement of the cardiac shadow giving a water bottle configuration. Lateral CXR may show a vertical opaque line
(pericardial fluid) separating a vertical lucent line directly behind sternum (epicardial fat) anteriorly from a similar lucent vertical
lucent line (pericardial fat) posteriorly; this is known as the Oreo cookie sign. Widening of the subcarinal angle without other
evidence of left atrial enlargement may be an indirect clue.
Echocardiography is the imaging modality of choice for the diagnosis of pericardial effusion.
Http://radiopaedia.org/articles/pericardial-effusion
Http://emedicine.medscape.com/article/157325-workup#c10

226.Anomaly between aortic arch and pulmonary trunk?


A. Patent ductus arteriosus .
Answer: A

56
227.Long scenario I don't remember exactly but there is upper limb hypertension and low or absent lower limb pulse, What is Dx?
A. Coarctation of aorta
Answer : A

159- MI patient within 6 hours what is the most expected complication? :


Answer: Arrhythmia

Reference:
Http://www.brown.edu/Courses/Bio_281-cardio/cardio/handout4.htm
&
Http://emedicine.medscape.com/article/164924-overview#a2
&
Consultant cardiologist

228.X-ray of pericardial effusion


Answer

57
229.What’s the acute management in rheumatic fever?
Answer:
o Antibiotic treatment for GAS infection eradication: either oral penicillin OR amoxicillin OR single dose intramuscular benzathine
penicillin
o Aspirin for fever, arthritis, and/or MILD carditis: 15 to 25 mg/kg PO QID (to a maximum daily dose of 4 to 6 g) for 2 to 4 weeks
and then taper the dose over another 4 weeks.
o Prednisone for MODERATE/SEVERE carditis: 1 mg/kg po bid (up to 60 mg/day)
o 10 years to life-long prophylaxis to recurrent GAS infection.
READ MORE DETAILS HERE: http://www.merckmanuals.com/professional/pediatrics/miscellaneous-bacterial-infections-in-infants-
and-children/rheumatic-fever#v1092890

230.Infective endocarditis (migratory arthritis)

Answer:
o Duke criteria for diagnosis of infective endocarditis
o The presence of 2 major OR 1 major + 3 minor OR 5 minor is DIAGNOSTIC.

58
231.Wide complex ECG

Answer: see picture

59
232.Enzyme that get elevated in MI?
A. Creatine phosphokinase (Note: no troponin I in the options)
Answer: A. Creatine Kinase-MB
Serial measurements of CK-MB isoenzyme levels were previously the standard criterion for the diagnosis of myocardial
infarction. However, sensitivity and specificity are not as high as they are for troponin levels. The trend has favored using
troponins and they are now considered to be the criterion standard for defining and diagnosing myocardial infarction,
according to the American College of Cardiology.
Reference: http://emedicine.medscape.com/article/155919-workup#c9

233.Long scenario I don't remember exactly but there is upper limb hypertension and low or absent lower limb pulse What is Dx?

A. Coarctation of aorta
Answer: A
Keys to diagnosis Coarctation of aorta include blood pressure (BLOOD PRESSURE ) discrepancies between the upper and lower
extremities and reduced or absent lower extremity pulses to palpation.
Reference: http://emedicine.medscape.com/article/895502-clinical#b4

234.Pt HTN came with uric acid 200 you prescribe antihypertensive drug for him after 1 weak uric acid 400 what is the drug ?
Answer: Thiazide diuretics

60
Pulmonology

61
1. Calcified lesion in the upper lung?
A. Bronchoscopy biopsy
B. Percutaneous biopsy
C. Thoracotomy
D. Follow up with serial x-ray
Answer: d
In asymptomatic pts with calcified nodule, we follow the pt with serial x ray for about 3 months, if it is not changed without sx
development, the calcification is mostly benign

2. A 40 year old patient has right upper lobe nodule, no symptom, other information, what will you do?
A. Transcutaneous biopsy
B. Bronchoscopy
C. Thoracotomy
D. Reassurance

Http://www.ncbi.nlm.nih.gov/pmc/articles/pmc3507065/

3. Lung tumor, size: 3.5 cm, enlarged ipsilateral bronchopulmonary ln, what is the tnm stage?(med)
A. T1 n0 m0
B. T2 n1 m0
C. T3 n2 m0
D. T4 n3 m0

Answer: b
Lung cancer staging
0 1 2 3 4

T No evidence of 3 cm or less 3-7 cm 7 cm or more A tumor of any


primary tumor size that
invades

62
adjacent
structures

N No regional Ipsilateral Ipsilateral Contralateral


lymph node peribronchial mediastinal mediastinal
and\or hilar and \or Contralateral
lymph node subcarinal hilar
and Ipsi or
intrapulmonar contralateral
y node scalene or
supracavicular

M No distant Distant
metastasis metastasis

Reference:http://cancerstaging.org/references-tools/quickreferences/documents/lungmedium.pdf

4. Asthma medication question. Patient on saba prn, symptoms not controlled. What to add?:
A. Inhaled corticosteroid
B. Long acting beta agonist
C. Iatropium bromide
D. Steorid oral

Answer: a
Reference:
Http://www.nhlbi.nih.gov/health-pro/guidelines/current/asthma-guidelines/quick-reference-html

5. Lung curve for cardiac at what level of costal cartilage:


A. 3
B. 4
C. 5
D. 6
Answer b
The anterior border of the left lung curves away laterally from the line of pleural reflection, beginning at the level of the 4th costal
cartilage (then curves back down behind the sixth costal cartilage). This produces a cardiac notch
(https://web.duke.edu/anatomy/lab04/lab5_prelab.html)

6. High pco2, low ph ;


A. Metabolic acidosis
B. Metabolic alkalosis
C. Respiratory acidosis
D. Respiratory alkalosis
Answer: c

63
Http://emedicine.medscape.com/article/301574-workup#c8

7. Patient admitted to you with diarrhea and dry cough after that he presented with tachypnea and during p/e there was
rhonchi on the base of the lung cxr showing opacification hyperdense area on the lower parts of the lung , so what is the
most likely organism ?
A. Pasteurella multocida
B. Mannheimia haemolytica
C. Staphylococcus aureus
D. Legionella pneumophila

Answer: d
S.aureus pneumonia presents with multiple nodular infiltrates, and bilateral pleural effusions.
Both b&c should have history of animal contact
Diarrhea is more with legionella pneumophila (legionnaires' disease)
Https://www.nlm.nih.gov/medlineplus/ency/article/000616.htm

8. Which of the following lung cancer contraindication of doing surgery ?


A. Small cell carcinoma
B. Squamous cell carcinoma
C. Adenocarcinoma
D. Fibroma related to something(ican not remember) i think this is the answer*

Answer:

9. Which of the lung tumor can be treated by ant- epithelium?


A. Adeno
B. Small lung
C. Squamous
D. Carcinoid
Answer: a?

10. Calcified lesion in the upper lung?

A. Bronchoscope biopsy
B. Percutaneous biopsy
C. Thoracotomy
D. Follow up with serial x-ray

Answer: a (follow up, mostly it’s benign) ??

If there is hrct it is the right answer


Http://www.clevelandclinicmeded.com/medicalpubs/diseasemanagement/hematology-oncology/pulmonary-nodules/

11. In acute pe with no signs of dvt; what will you give?


A. Im warfarin
64
B. Im heparin
C. Warfarin + heparin
D. Streptomycin
ِanswer : c

12. Upper limb edema, intercostal vein engorgement, lesion in right lung, compression in which side?
A. Ant mediastinum
B. Post mediastinum
C. Rt hilum
D. Median mediastinum

Answer: d
(d) .. The tumor in the right lung is compressing the svc in the middle mediastinum causing these symptoms.

(a) .. Would obstruct only a small portion of the thoracic wall drainage and would not produce the neck and upper limb effects.

(b) .. Obstruction of the azygos vein would not influence the neck and upper limb drainage because of collateral connections with
the anterior intercostal veins and would not produce the neck and upper limb effects.

(c) .. Compression of the hilum would obstruct pulmonary functions would not produce the neck and upper limb effects.

Reference: lippincott. Uptodate

13. Long scenario about tb and what is the antibiotic to start:


A. Rifampicin, inh, ethambutol and pyrazinamide
B. Rifampicin, inh and ethambutol
C. Rifampicin and inh
D. Inh

Answer: a
A. For initial empiric treatment of tb, start patients on a 4-drug regimen: isoniazid, rifampin, pyrazinamide, and either
ethambutol or streptomycin.
Reference: medscape: http://emedicine.medscape.com/article/230802-treatment

65
14. A middle-aged man presents with a cough and fever lasting several weeks. Posteroanterior chest radiograph shows a
prominent paratracheal area on the right, lymphadenopathy, a cavitary opacity in the right upper lobe, and a focal
consolidation in the middle lung zone on the right. Cxr shown below. What is the dx?
A. Copd
B. Ba
C. Pneumonia
D. Tb

Answer: d
Common findings include segmental or lobar airspace consolidation, ipsilateral hilar and mediastinal lymphadenopathy,
and/or pleural effusion. Atelectasis may occur in primary pulmonary tuberculosis, often as a consequence of tuberculous
airway involvement.
Reference: medscape: http://emedicine.medscape.com/article/358610-overview#a2

15. Which lobe is most commonly affected in lobar pneumonia?


A. Right mid
B. Right upper
C. Right lower
D. Left upper
Answer: c
Reference: https://goo.gl/ciwlo8

16. Which of the following decrease the recurrence of asthma?


A. Salbutamol
B. Aminophylline
C. Ipratropium
D. Montilukust
Answer d?
Leukotriene receptor antagonists are one of the controller medications for asthma

66
17. Asthmatic with 3/week of frequency on short acting and last sever attack was 3 months ago. What the appropriate
management?
A. Short
B. Long
C. Ipratropim
D. Dexamethas

Answer: d

18. In emphysema, which part is affected in alfa one antitrypsin deficiency?


A. Interstitial
B. Centroacinar
C. Peripheral
D. Subseptal or something like this
Answer:
Generally, pulmonary emphysema is classified into three types related to the lobular anatomy: centrilobular emphysema, panlobular
emphysema, and paraseptal emphysema. Panlobular (panacinar) emphysema: seen in patients with α1-antitrypsin deficiency.
Destruction involves both proximal and distal acini with predilection for lung bases. *step up to medicine.

19. Upper limb edema, intercostal vein engorgement, lesion in right lung, compression in which side:
A. Ant mediastinum
B. Post mediastinum
C. Rt hilum
D. Median mediastinum
Answer: d

20. Smoker + hemoptysis what to do (not specified first or best)?


A. Chest x-ray
B. Chest ct
C. Ppd
D. Coagulation profile
Answer: a
Dd for hemoptysis & tobacco use: acute bronchitis, chronic bronchitis, lung cancer & pneumonia. After a careful history and
examination, a chest radiograph should be obtained. If a diagnosis remains unclear, further imaging with chest computed
tomography (ct) or direct visualization with bronchoscopy often is indicated.
Http://www.aafp.org/afp/2005/1001/p1253.html

21. 40 years old present with 1.5 cm lung nodule what is the most useful to do?
A. Transthoracic biopsy.
B. Thoracic biopsy.
C. Evaluate by multiple x-ray .
D. Look at prior x rays.
Answer: d. Look at prior x-rays. Diagnostic evaluation of a lung nodule includes : cinical features and radiographic features: assess
size – larger lesions are more likely to be malignant than smaller lesions ,border, calcification, density ,and growth – review of
available prior imaging studies is a critical part of the diagnostic evaluation. Uptodate: diagnostic evaluation and management of the
67
solitary pulmonary nodule.
Https://yhdp.vn/uptodate/contents/mobipreview.htm?43/31/44537

22. Asymptomatic Patient. Chest X ray shows a unilateral calcified nodule on the upper zone of his lung?
A. Adenoma
B. Granuloma
C. Hamartoma
D. SCC

Answer: c
The most common cause of nodule calcification is granuloma formation, usually in the response to healed infection.
Reference: http://radiopaedia.org/articles/calcified-pulmonary-nodules

23. Most specific test for TB:


A. PCR
B. Chest X ray
C. Sputum culture
D. PPD

Answer: C
Reference: http://www.cdc.gov/tb/publications/factsheets/testing/diagnosis.htm

24. An asthmatic who needs daily short acting beta 2 inhalers, oral steroids and daily spirometry monitoring of pfts. What is his
asthma stage?
A. Mild intermittent
B. Mild persistent
C. Moderate
D. Severe

Answer: D
Once the patient on oral steroid he/she classified as a severe asthma.
Reference: http://www.med.umich.edu/1info/FHP/practiceguides/asthma/EPR-3_pocket_guide.pdf

25. Which of the lung tumar can tt by anti epithelium


68
A. Adeno
B. Small lung
C. Seqoums
D. Carcinoid

26. Biopsy of lung found some material in the macrophage ( i cant remebre the material )
A. (pcp )pneocyteic jnera
B. CMV
C. Bacterial
D. Hf

27. Acute onset chest pain + image coronary angio not so clear ..
A. Tpa
B. Pci
C. Heparin
D. The q not clear

28. Most common cause of acute bronchiolitis?


A. Rsv
B. Adeno
C. Parainfluenza

Answer: a
References: http://emedicine.medscape.com/article/961963-overview
Http://www.ncbi.nlm.nih.gov/books/nbk11786/

29. Case of pneumonia, what is the finding on auscultation?


A. Dispred crackles
B. Bronchial breath sound
C. Absence of vesicular breath sound

Answer: b
(short textbook of medical diagnosis and management, inam danish, page 344) (step up to medicine)

30. Most common cause of chronic cough in adult?


A. Gerd
B. Postnasal drip
C. Asthma

Answer: b
Http://www.uptodate.com/contents/chronic-cough-in-adults-beyond-the-
basics?Source=outline_link&view=text&anchor=h3#h3

31. Case of pneumonia, what is your finding on the auscultation?


A. Dispred crackles
B. Bronchial breath sound
C. Absence of vesicular breath sound
69
Answer: b
Reference: http://emedicine.medscape.com/article/234240-overview#a8

32. Patient with asthma exacerbation. Which drug will decrease the mucous secretion more than bronchodilation?
A. Oral steroids
B. Ipratropium
C. Leukotriene
Answer a
Corticosteroids reduce the mucus secretion by inhibiting the release of secretagogue from macrophages.
Reference: http://www.ncbi.nlm.nih.gov/pubmed/3026210

33. Smoking withdrawal symptoms peak at ?


A. 1-3days (48hours)
B. 4-7 days
C. 2 weeks
Answer: a
Http://www.webmd.com/smoking-cessation/understanding-nicotine-withdrawal-basics

34. Patient present with pleuritic chest pain, examination shows decrease air entry in the right lower lung, attached chest x-ray
showing radiolucent right cost-phrenic angle, what is next?
A. Ventilation perfusion scan
B. Needle decompression
C. Chest tube
Answer: b
Following treatment recommendations in first episodes of primary spontaneous pneumothorax: simple observation in the case of
clinically stable patients with small pneumothoraces; lung reexpansion with a small-bore catheter or placement of a chest tube
attached to either a heimlich valve or a water seal device in the case of clinically stable patients with large pneumothoraces; and
lung reexpansion with a chest drain or small bore catheter attached to a water-seal device or applying suction of unstable patients
with large pneumothoraces
read more: http://www.atsjournals.org/doi/full/10.1164/rccm.200111-078oc#.v1_flvkdbbc

35. Old patient with small cell lung cancer treated by chemotherapy on examination there is crepitation on the lung no ll swelling
lab result showed hyponatremia what is your advice ?
A. IV furosemide
B. Fluid restriction
C. Desmopressin

The answer is b
This is a case of paraneoplastic syndrome (sidah ).to treat the hyponatremia : treat the underlying cause and restrict free water
intake.
Reference: toronto notes

70
36. Asthmatic boy on muntelukast present to er, he has symptoms everyday exacerbated by exercise what to give for
maintenance
A. Oral steroid daily with long acting when needed
B. B- inhaled steroid twice with short acting when needed
C. Muntelukast with long acting
Answer:

Step 3 - moderate persistent asthma the preferred controller medication is either a low-dose inhaled corticosteroid plus a long-acting beta-agonist
[78]
(laba) (combination medication preferred choice to improve compliance) or an inhaled medium-dose corticosteroid. Alternatives include an inhaled
low-dose ics plus either a leukotriene receptor antagonist (montelukast) or low-dose theophylline.
Reference:http://emedicine.medscape.com/article/296301-treatment#d8

37. After delivery shortness of breath at night. What findings in the x-ray support diagnosis?
A. Increase in mediastinal width.
B. Increase shadowing
C. Cardiothoracic increase.
Answer:

71
38. Asthma on leukotriene inhibitor >>increase cough & on exercise >> admission on short acting β blocker then discharge >>
what is the drug on discharge?
A. Long acting β agonist and short acting β agonist as need-
B. Oral steroid once + short acting β agonist as need-
C. Inhaler steroid 2 time + short acting β agonist as need

Answer: a
[1]
The basis of treatment is with pre-exercise short-acting β2 -agonist administration. a role also exists for long-acting β2 -agonists and
mast cell stabilizers. Anti-leukotriene drugs have been shown to be effective as well
Http://emedicine.medscape.com/article/1938228-treatment

39. Patient had 1.5 cm calcified lesion in the routine chest x-ray. He's symptomless. Next action will be.
Answer:
A. Observation
B. Percutaneous biopsy
C. Transbronchial biopsy
Answer: a
No symptoms + calcified lesion less than 3cm is almost always benign.
Http://www.clevelandclinicmeded.com/medicalpubs/diseasemanagement/hematology-oncology/pulmonary-nodules/

40. Rx of patient with lung Castage iiib came with sudden lower back pain?
A. MRI only
B. MRI with steroid
C. Radiotherapy
Answer: c or a
Http://www.uptodate.com/contents/non-small-cell-lung-cancer-treatment-stage-iv-cancer-beyond-the-
basics?Source=outline_link&view=text&anchor=h10#h10

72
41. Drug that cause white patches in mouth. What is the inhaler?
A. Ipratropium
B. Short acting beta2 agonist
C. Steroid
Answer: c

Children on inhaled steroids also have increased incidence of oral candidiasis.

Reference: medscape: http://emedicine.medscape.com/article/969147-overview

42. Case of pneumonia, what your finding on the auscultation?


A. Dispred crackles
B. Bronchial breath sound
C. Absence of vesicular breath sound
Answer: b
On physical examination, approximately 80 percent are febrile, although this finding is frequently absent in older patients and
temperature may be deceptively low in the morning. A respiratory rate above 24 breaths/minute is noted in 45 to 70 percent of
patients and may be the most sensitive sign in older adult patients; tachycardia is also common. Chest examination reveals audible
crackles in most patients, while approximately one-third have evidence of consolidation. However, no clear constellation of
symptoms and signs has been found to accurately predict whether or not the patient has pneumonia.

43. A patient with positive ppd test for the first time and negative chest x-ray. There are no signs or symptoms of tb. What’s the
appropriate approach?
A. Reassure

B. Inz for 6 months.
C. Others
Answer:
Patients with positive ppd or interferon gamma release assay (igra) with no evidence of active disease (asymptomatic and normal x-
ray) should receive a 9-month therapy of isoniazid (inh) and vitamin b6.
Kaplan internal medicine 2013.
Http://www.uptodate.com/contents/treatment-of-latent-tuberculosis-infection-in-hiv-uninfected- adults?Source=see_link

44. Smoker with hilar mass, suspect what


A. Lymph node
B. Squamous cell ca
C. Adenocarcinoma

Answer: b
It is impossible to differentiate between them radiologicaly but squamous cell carcinoma usually caused by heavy smoking
reference:http://radiopaedia.org/articles/squamous-cell-carcinoma-of-the-lung

45. Scenario about tb and the pt take 4 medication for tb he developed numbness in hand and feet
A. Rifmpacin
73
B. Inh
C. Ethambutol

Answer: b

46. Which of the following medication decrease mucus production in a patient with emphysema?
A. Cromolyn sodium
B. Steroids
C. Anticholinergics
Answer: c
Anticholinergic agents will dilate bronchi and decrease secretions. They are
Very effective in copd.
Ref : http://err.ersjournals.com/content/19/116/127

47. Hemosiderin deposition in macrophage in lung in


A. CMV
B. Pneumocystic jervoci
C. Chronic lung infection
Answer c
The presence of hemosiderin laden macrophages indicates there was blood in the alveolar space which canbe caused by infection.
Http://www.ncbi.nlm.nih.gov/pmc/articles/pmc2610638/

48. Case of copd what will you see in bronchus


A. Hyper plastic mucous gland
B. Thinning in the basement membrane
C. C-dilated sac.

Answer: a

49. . Which one of these lung cancers is associated with anti-epithelial cell receptor therapy?
A. Adenocarcinoma
B. Squamous cell carcinoma
C. Small cell carcinoma
Answer: a
80% of lung adenocarcinomas are associated with egfr mutation, in which cetuximab, a monoclonal antibody that binds the egfr
receptor can be used.
Http://emedicine.medscape.com/article/279960-treatment#d12

50. 3.5 cm lung tumor with paratracheal lymph nodes?


A. T1 n 0
B. T2 n1
C. T3 n2
Answer: b (not sure n2)
Lung ca
- t2a, n1, m0: the cancer has 1 or more of the following features:
74
~>
the main tumor is larger than 3 cm across but not larger than 5 cm.
The tumor has grown into a main bronchus, but is not within 2 cm of the carina (and it is not larger than 5 cm).
The tumor has grown into the visceral pleura (the membranes surrounding the lungs) and is not larger than 5 cm.
The tumor is partially clogging the airways (and is not larger than 5 cm).
The cancer has also spread to lymph nodes within the lung and/or around the area where the bronchus enters the lung (hilar lymph
nodes). These lymph nodes are on the same side as the cancer. It has not spread to distant sites.

- but here lymph nodes are in para trachea so n2


n2 = the cancer has also spread to lymph nodes around the carina (the point where the windpipe splits into the left and right
bronchi) or in the space between the lungs (mediastinum). Affected lymph nodes are on the same side as the main lung tumor. It has
not spread to distant sites.

Http://www.cancercenter.com/lung-cancer/stages/
http://www.cancer.org/cancer/lungcancer-non-smallcell/detailedguide/non-small-cell-lung-cancer-staging

51. Pt with acute asthma given drug that works by inhibition of phophodiestrase enzyme which drug was given?
A. Salmeterol
B. Beclomethasone
C. Aminophylline
Answer: c
Http://www.drugbank.ca/drugs/db01223

52. Pt with chest infection was treated with oral ab for 4 weeks later came complain from rt lung effusion what dx?
A. Parapneumonic effusion empyema
B. Tb
C. Lung ca
Answer: a
Definition:
Pus in pleural space or an effusion with organisms seen on a gram stain or culture (e.g. Pleural fluid is grossly purulent)
Positive culture is not required for diagnosis

Etiology:
Contiguous spread from lung infection (most commonly anaerobes) or infection through chest wall (e.g. Trauma, surgery)
Signs and symptoms:
Fever, pleuritic chest pain
Treatment:
Antibiotic therapy for at least 4-6 wk (rarely effective alone)
Complete pleural drainage with chest tube
If loculated, more difficult to drain – may require surgical drainage with video-assisted thorascopic surgery (vats

53. Patient presented to the er with cough hemoptysis night sweats and malaise. What is most appropriate initial step in the
management?
A. Isolation in negative pressure room
B. Start anti tb
C. Give opd appointment after 2 weeks
75
Answer: a

54. Drug addicted, unconscious came with no gag reflex. What would you do?
A. Intubation
B. Gastric lavage
C. Give naloxone
Answer: a
Start with the abcs

55. Surgery is contraindicated for which lung ca?


A. Scc
B. Scl
C. Adenocarcinoma
Answer: b (small - oat - cell)

Sclc
Treatment — sclc is a disseminated disease in most patients at presentation and is very responsive to chemotherapy. Thus systemic
chemotherapy is an integral part of the initial treatment.
Patients with limited stage disease are primarily treated with a combination of chemotherapy and radiation therapy, since the
addition of radiation therapy has been shown to prolong survival compared with chemotherapy therapy alone
For patients with extensive stage sclc, chemotherapy alone is used as the initial therapy

Nsclc
Treatment — surgical resection offers the best opportunity for long-term survival and cure in patients with resectable nsclc. The
appropriateness of surgical resection of candidates with known or suspected nsclc includes preoperative staging and an assessment
of performance status with concurrent comorbidities and pulmonary function to allow prediction of postoperative function.

56. Bronchial asthma daily cs, daily pft type of ba ?


A. Mild persistent
B. Moderate
C. Severe

Answer: c

57. Pediatric patient complains of cough he controls it by leukotrienes when he visited the primary physician 6 month ago. Now
he develops cough for 4 days after exercise they give him albuterol the symptoms were relieved what you going to give him:
A. Leukotriene + short act b-2 agonist
B. Oral steroid + short act b-2 agonist
C. Long act b-2 agonist + short act b-2 agonist
Answer: a

58. What is the treatment for female pt with uncontrolled asthma (wake her up from sleeping at night) she is on short beta 2
agonist?
A. Long acting beta 2
76
B. Steroid
C. Theophylline
Answer: b next step management inhaled
Http://getasthmahelp.org/documents/gist-stepwise-approach.pdf

59. Most common cause of cough-in-adults-beyond-adult?


A. GERD
B. Postnasal drip
C. Asthma

Answer: B
The most common causes of chronic cough are postnasal drip, asthma, and acid reflux from the stomach.
Http://www.uptodate.com/contents/chronic-cough-in-adults-beyond-the-basics?Source=outline_link&view=text&anchor=H2#H2

60. Female non smoker with nodule by CT found calcium and fat
A. Hamartona
B. Mystheoma
C. No adeno

Answer: A
Pulmonary hamartomas, the most common benign tumors of the lung, are the third most common cause of solitary pulmonary
nodules. It is composed of cartilage, connective tissue, muscle, fat, and bone. CT is far superior than x-ray in detecting intralesional
fat and calcification.
Http://emedicine.medscape.com/article/356271-overview
Http://radiopaedia.org/articles/pulmonary-hamartoma-1

pancost: arm pain, horner, hand muscle atrophy


61. Smoker with hilar mass what suspect?
A. Lymph node squamous: smoker, hypercalcemia ( PTH LIKE ) , pancost:
B. Squamous cell Ca Small: paraneoplastic SIADH
C. Adenocarcinoma

Answer: B
Central tumors are generally squamous cell carcinomas (sccs) and most peripheral tumors are adenocarcinomas or large cell
carcinomas which can cause pleural effusion
Http://emedicine.medscape.com/article/279960-overview
Http://radiopaedia.org/articles/squamous-cell-carcinoma-of-the-lung

62. Pt present with pleuritic chest pain , examination shows decrease air entry in the right lower lung, attached chest x-ray
showing radiolucent right costophrenic angle, what is next ?
A. Ventilation perfusion scan
B. Needle decompression
C. Chest tube
Answer: isn’t this pneumothorax?

63. Anion gap of corrected na 138:


77
A. 26
B. 34
C. 44
Answer:?

64. Anioin gap in mmol/l ?


A. 3
B. 95
C. 123
Answer: question incomplete (if question is which is a normal value, answer is a)
Explanation: the anion gap is the difference between primary measured cations (sodium na+ and potassium k+) and the primary
measured anions (chloride cl- and bicarbonate hco3-) in serum. The reference range of the anion gap is 3-11 meq/l
Reference: http://emedicine.medscape.com/article/2087291-overview

65. Bronchial cancer mets to sypathatic plexus what is the sign?


A. Ptosis
B. Dilated pupil
answer: a
the answer is horner’s syndrome ptosis ,anhydrosis,miosis
Toronto notes neoplasm (r28)
http://www.uptodate.com/contents/superior-pulmonary-sulcus-pancoast-tumors

66. Cancer of lung with high keratin?


A. Lung adenocarcinoma
B. Squamous cell carcinoma

Answer: b
Reference: http://www.ncbi.nlm.nih.gov/pmc/articles/pmc3155291/

67. Lung cancer 3cm rt mediastinal ln (paratracheal) stage:


A. T1n1m
B. T1n2m
Answer:b
N1=ipsilatral peribronchial n2=ipsilateral mediastinal n3=contralateral mediastinal
http://www.radiologyassistant.nl/en/p42459cff38f02/lung-cancer-new-tnm.html

68. Patient with small cell lung cancer c/o fatigue for 1month lab result show hyponatremia and urine hypo-osmolarity ? How u
will manage?
A- normal saline
B- fluid restriction

Answer: b

78
Explanation: patient most likely suffer from syndrome of inappropriate antidiuretic hormone as a paraneoplastic syndrome from his
small lung cancer for which the first line treatment is fluid restriction, however, there probably is a mistake in the question instead
urine hyposmolarity there is urine hyperosmolarity. Patients with saidh usually present with hyponatremia, serum hyposmolality,
and urine hyperosmolality.
References: http://emedicine.medscape.com/article/246650-overview
Http://emedicine.medscape.com/article/280104-clinical

69. Lung cancer 3cm rt mediastinal ln (paratracheal) stage:


A. T1n1m
B. B- t1n2m << note: (n1=ipsilatral peribronchial n2=ipsilateral mediastinal n3=contralateral mediastina
Answer: b
Reference: http://emedicine.medscape.com/article/2007813-overview

70. Old male with recurrent episodes of cough with sputum and hemoptysis
A. Bronchiectasis
B. Tb
Answer: a
Recurrence is a feature of bronchiectasis that classically manifests as cough and the daily production of mucopurulent and tenacious
sputum lasting months to years that presents with sputum and episodes of hemoptysis.
Http://cursoenarm.net/uptodate/contents/mobipreview.htm?9/24/9601

71. What is most cause of cough in adult?


A. Post nasal drip
B. Gerd
Answer: a
The most common causes of chronic cough are postnasal drip, asthma, and acid reflux from the stomach. These three causes are
responsible for up to 90 percent of all cases of chronic cough. Less common causes include infections, medications, and lung
diseases.
Http://www.uptodate.com/contents/chronic-cough-in-adults-beyond-the-basics

72. Tb case, what is the next appropriate step to get a definitive dx?
A. Sputum smear under microscope
B. Sputum culture

Answer: b

The presence of acid-fast-bacilli (afb) on a sputum smear or other specimen often indicates tb disease. Acid-fast microscopy is easy
and quick, but it does not confirm a diagnosis of tb because some acid-fast-bacilli are not m. Tuberculosis . Therefore, a culture is
done on all initial samples to confirm the diagnosis. (however, a positive culture is not always necessary to begin or continue
treatment for tb.) A positive culture for m. Tuberculosis confirms the diagnosis of tb disease.

Http://www.cdc.gov/tb/publications/factsheets/testing/diagnosis.htm

73. Patient came for routine checkup cxr shows a unilateral calcified nodule on the upper zone of his lung:
79
A. Scc
B. Hamartoma

Answer: b
Pulmonary hamartomas, the most common benign tumors of the lung, are the third most common cause of solitary pulmonary
nodules.
Reference: http://emedicine.medscape.com/article/356271-overview

74. Pt with lung nodule and high Ca& parathyroid


A. Hyperparathyroidism
B. Lung ca
Answer: b
Squamous cell carcinoma of the lung secreting parathyroid hormone–related peptide causing hypercalcemia.
Reference: http://emedicine.medscape.com/article/279960-workup?Src=refgatesrc1#c5

75. Positive ppd test but normal x-ray, what will you do?
A. Isoniazid for 6 months
B. Rifampin for 6 months

Answer: a

1. For a positive tb exposure and a positive ppd test (but no active disease), treatment is inh (isoniazid) only.

Reference: step up to medicine

76. Patient with small lung carcinoma (slc) undergoing chemotherapy. He developed …… lab values : low blood na, low urine
osmolality. The treatment?
A. Desmopressin
B. Nothing
Answer: a
Decreased urine osmolality in hyponatremic patients is observed in some patients with reset osmostat syndrome, when water intake
reduces the serum osmolality below the new threshold for adh release. Reset osmostat syndrome is a variant of the syndrome of
inappropriate adh secretion (siadh) which is a common paraneoplastic syndrome in slc. Siadh is managed by desmopressin.
Http://www.ncbi.nlm.nih.gov/pmc/articles/pmc100882/

77. Obstructive sleep apnea. Most effective treatment


A. Bmi less than 30
B. Cpap
Answer: b
Cpap is the standard treatment option for osa and generally can reverse this condition quickly with the appropriate titration of
devices.
Reference: http://emedicine.medscape.com/article/295807-treatment

78. Hemosiderin laden in alveolar lavage?

80
A. Due to pcp(pnemocytits ji..
B. CMV

These hemosiderin-laden alveolar macrophages are characteristically found in bal fluid or lung biopsy from patients with diffuse
alveolar hemorrhage. Classically, diffuse alveolar hemorrhages are classified based on histologic appearance as capillaritis (wegener
granulomatosis, systemic lupus erythematosus, and propylthiouracil related), bland hemorrhage (coagulopathies, congestive heart
failure, and rapamycin related), diffuse alveolar damage (ards and crack cocaine inhalation), and miscellaneous conditions
(lymphangioleiomyomatosis and pulmonary capillary hemangiomatosis).

79. Pt. With moderate persistent bronchial asthma on beta agonist prn and low dose inhaled steroid comes with uncontrolled ba.
What you will add to the steroid?
A. Long acting beta agonist
B. Theophylline
Answer: a http://getasthmahelp.org/documents/gist-stepwise-approach.pdf

80. 20 year snoring ... He has enlarged tonsils treatment of obstructive sleep apnea.
A. Cpap.
B. Reduced weight.
Answer: missing information. But osa is an indication for tonsillectomy if enlarged.

81. According to the new classification of lung cancer, which of the following is considered carcinoma in-situ?
A. Adenocarcinoma less than 2 cm.
B. Atypical hyperplasia

Answer: A
Adenocarcinoma in situ (AIS) with no invasive features is a localized, small (≤3 cm) adenocarcinoma with growth restricted to a
noninvasive lepidic pattern and an absence of papillary or micropapillary patterns or intraalveolar tumor cells.
Reference: uptodate.

82. Most common cause of excessive sleepiness in the daytime is?


A. Narcolepsy
B. Obstructive Sleep Apnea (OSA)

Answer: B
The most common causes of excessive daytime sleepiness are sleep deprivation, obstructive sleep apnea, and sedating medications.
Reference: American Family Physician Journals.
If attack its narcolepsy

83. A male patient who is a smoker, developed symptoms? Cais High. CXR showed solitary nodule. What is the most likely
diagnosis?
A. Squamous Cell Carcinoma SCC
B. Adenocarcinoma

Answer: A
81
High Ca> Paraneoplastic of SCC.

84. Common cause of bronchiolitis?


A. Para influenza virus
B. RSV

Answer: B
Bronchiolitis is an acute inflammatory injury of the bronchioles that is usually caused by a viral infection (most commonly respiratory
syncytial virus and human metapneumovirus) Medscape

85. Old male with recurrent episodes of cough with sputum and hemoptysis:
A. Bronchiectasis(my answer)
B. Tb

86. Lung tumor with keratinization, what type of cells?


A. Epithelial
B. Squamous
Answer: b
Explanation: squamous cell carcinoma. Loose cohesive sheet of tumor cells and single tumor “strap cells” cells with keratinization
are present.
Link: http://www.ncbi.nlm.nih.gov/pmc/articles/pmc2872744/

87. Case of asthma sever, cough every week , he took nebulizer steroid , what is the next step of management ?
A. Add long acting b agonst
B. Ibrapritom

88. Copd patient was on oral steroid and there was improvment 17%in breathing , which medication will u put him on :
A. Theophthylin
B. Amitriptalin inhaler or oral .
Answer: choices incomplete and answer most likely missing
Explanation: systemic corticosteroids should be initiated after the first treatment of short-acting inhaled bronchodilators; beta-2
agonists (albuterol) are typically favoured as first-line as they function more rapidly than anticholinergic bronchodilators
(ipratropium).
Reference; http://bestpractice.bmj.com/best-practice/monograph/8/treatment/step-by-step.html

89. Cancer of lung with high keratin?


A. Non small cell cancer
Answer: sq c c
For sure squamous cell and adenocarcinoma contain keratin
Reference: https://www.iarc.fr/en/publications/pdfs-online/pat-gen/bb10/bb10-chap1.pdf

82
90. Old patient with squamous cell carcinoma of the lung present with lethargy for 2 months. Lab showed hyponatremia (na:
121) (case of siadh), treatment?
A. Fluid restriction (since it is asymptomatic hyponatremia- no neurological manifestation)
Answer: severe symptoms (altered mental status, seizure, and coma) or sodium <125 mmol/l (<125 meq/l), divided into 2:
Acute onset 48hrs or less:
o IV hypertonic saline
o In the er, start with 50 ml 3% saline iv, followed by 200 ml IV infusion over 4 to 6 hours
o There is a risk of central pontine myelinolysis (osmotic demyelination syndrome) in case of rapid correction
o Fluid restriction
o Treat underlying cause
o Furosemide: used in addition to hypertonic saline, especially if the patient is at risk for volume overload. It
helps to correct hyponatremia by increasing free water excretion.
Chronic onset more than 48hrs or unknown
IV hypertonic saline (same as above)
IV vasopressin receptor antagonist → either conivaptan or tolvaptan
Conivaptan (a non-selective vasopressin receptor antagonist) + monitoring the patient for hypotension is
a must + common se is skin reaction at the site of iv
Tolvaptan (a selective v2 receptor antagonist)+ serum sodium should be checked at baseline and 8 hours
after the first dose + the drug should be discontinued immediately in patients with signs or symptoms of
liver injury (e.g., fatigue, anorexia, right upper abdominal discomfort, dark urine, jaundice, elevated LFTs).
Http://bestpractice.bmj.com/best-practice/monograph/196/treatment/details.html

91. An asthmatic patient is on inhaled corticosteroids. His asthma becomes more severe, what should you add?
Answer: laba
Https://www.nhlbi.nih.gov/files/docs/guidelines/asthma_qrg.pdf

92. Patient with chronic bronchitis (scenario write symptom productive cough for last 3 month per year through last 2 year) and
what is find in microscopy under histopathology?
Answer: chronic bronchitis is defined as a cough that occurs every day with sputum production that lasts for at least 3 months, 2
years in a row.

93. Low ph, hco3, co2?


A. Compensated metabolic acidosis.
83
Answer: you need the exact numbers to determine if it is compensated or not

Reference: toronto note, pocket of medicine

94. Scenario of atypical pneumonia, causative organism ?


A. Mycoplasma pneumonea
Source: medscape

95. Cystic fibrosis mode of inheritance?

Autosomal recessive condition predominantly affecting caucasians

Reference: step up to medicine

96. Pneumonia bilateral infiltration ?


A. Ciprofloxacine
Answer :?!

Bilateral infiltration usually caused by staph aureus

84
97. 7month present with respirtory sx ?? Bronchlitis ++
Answer: not clear

98. Pneumocyte type 1 features?


Answer: 95% of the alveolar surface
Type i alveolar cells:
Type i alveolar cells are squamous (giving more surface area to each cell) and cover approximately 90–95% of the alveolar surface.
Type i cells are involved in the process of gas exchange between the alveoli and blood. These cells are extremely thin (sometimes
only 25 nm) – the electron microscope was needed to prove that all alveoli are covered with an epithelial lining. These cells need to
be so thin to be readily permeable for enabling an easy gas exchange between the alveoli and the blood.
Type ii alveolar cells:
Type ii alveolar cells cover a small fraction of the alveolar surface area. Their function is of major importance in the secretion of
pulmonary surfactant, which decreases the surface tension within the alveoli.

99. The question is missing a lot of information. Asthma with daily symptoms and frequent night time symptoms is classified as
severe.
Http://emedicine.medscape.com/article/296301-treatment#d8

85
100.Pt develop cough during exercise: which medication want to give her before exercise?
Answer: b2 agonist

101.Patient taking high dose of salicylate what to find?


A. Respiratory alkalosis & metabolic acidosis.
Answer: a
Medscape:

102.DM patient with abg values show ph 7.2 . Patient is hyperventilated. Why this happen?!
A. To get rid of co2
Answer: a
Since co2 is acidic, and the ph is acidic.. Getting rid of the co2 would increase the ph.

103.Status asthmaticus on drug inhibit cholinesterase what is the drug?


A. Ipratropium

104.Scenario about horner syndrome asking about the site of tumor:


A. On the lung apical .
Explanation: the common lesions that cause horner syndrome interfere with preganglionic fibers as they course through the upper
thorax. Pancoast tumor (tumor in the apex of the lung, most commonly squamous cell carcinoma) can cause horner syndrome.
Reference: http://emedicine.medscape.com/article/1220091-overview#a4

105.Pt came e hx of hf & pulmonary edema. Tx?!


A. Diuretics
Answer: a. Treat the cause and give o2 and IV furosemide and/or nitrates as needed; try noninvasive ventilatory assistance initially
but use tracheal intubation and assisted ventilation if necessary. (step up to medicine)

86
106.Easy q about cystic fibrosis "sweat test" ?
Sweat test is a diagnostic test of cf ( confirm ).
Sweat test and genetic test , it's caused by mutation in both copies of gene for the cystic fibrosis transmembrane conductance
regulator (cftr) protein .

107.Patient develop cough during exercise : which medication want to give her before exercise .

108.Asthmatic controlled on albuterol prn, now she got pregnant,and she started to have daily symptoms & night ... ?
Answer: inhaled steroid + laba
Pregnant women manage asthma the same way nonpregnant women do. Like all people with asthma, pregnant women need to
have an asthma action plan to help them control inflammation and prevent and control asthma attacks.
Http://www.webmd.com/asthma/tc/asthma-during-pregnancy-topic-overview

109.Abg case of patient presented with vomiting?


Answer: metabolic alkalosis
Metabolic alkalosis is a primary increase in serum bicarbonate (hco3-) concentration. This occurs as a consequence of a loss of h+
from the body or a gain in hco3-. In its pure form, it manifests as alkalemia (ph >7.40).
Http://emedicine.medscape.com/article/243160-overview

110.Case of pt took nebulizer steroid , develop white patch on tongue ( moth) what is digenesis ?
A. Oral thrush

111.Lung cancer which stage:


A. Stage 1
Answer:a
Reference:http://cancerstaging.org/references-tools/quickreferences/documents/lungmedium.pdf

112.Non small cell lung cancer has 4 risk factors which are stage of the disease , condition of the patient and male sex
Question is not clear and choices incomplete, but male sex is a risk factor for non-small cell lung cancer
Reference: http://bestpractice.bmj.com/best-practice/monograph/1082/diagnosis/history-and-examination.html

113.Heart sound that supports pulmonary htn.


Answer: loud pulmonic component of the second heart sound (p 2) and sudden lift of the sterum. If pulmonary regurgatation occurs,
graham steell murmur will be noted (a high pitched early diastolic murmur). Right sided s 3 gallop.
Reference: http://emedicine.medscape.com/article/301450-clinical#b2

114.Acid base disorder calculation?


Reference: http://fitsweb.uchc.edu/student/selectives/timurgraham/stepwise_approach.html

115.Patient with metabolic acidosis with high anion gap (aspirin toxicity)
Answer:
87
Activated charcoal and alkaline diuresis with extra kcl
Source: http://www.merckmanuals.com/professional/injuries;-poisoning/poisoning/aspirin-and-other-salicylate-poisoning

116.Regarding the previous case, what should be given ?


A. Misprostol
Answer: a
atropine plus pralidoxime (2-PAM)
Muscrinic effect: SLUDGE (salivation, lacrimation, urination, diarrhea, GI upset, emesis)
117.Case of surfactant deficiency? and DUMBELS (diaphoresis and diarrhea; urination; miosis; bradycardia, bronchospasm,
bronchorrhea; emesis; excess lacrimation; and salivation).
118.Typical case of organophosphorus poisoning dx
Nicotinic signs and symptoms include muscle fasciculations, cramping, weakness, and
119.Typical case of monoxide poisoning dx diaphragmatic failure. Autonomic nicotinic effects include hypertension, tachycardia,
mydriasis, and pallor.
120.Case of hypernatremia and hypokalemia and ask for how to replace them ?

121.RSV ttt
Http://www.mayoclinic.org/diseases-conditions/respiratory-syncytial-virus/basics/treatment/con-20022497

122. Pt with obstructive lung disease (FEV1), what is your best advice for him?
A. Stop smoking.
Answer:
All patients with COPD should be advised to quit smoking, educated about COPD, and given a yearly influenza vaccination, In
addition to pneumococcal polysaccharide vaccine.
Pharmacological management:
- For all patients with COPD, short-acting bronchodilator (eg, beta-agonist, anticholinergic agent) be prescribed for use as-needed for
relief of intermittent increases in dyspnea
- for patient insufficient to control symptoms or who have other high-risk predictors, including severe or very severe airflow
obstruction or two or more exacerbations in the previous year,
We prefer the long-acting inhaled anticholinergic agent to the twice daily long-acting beta agonists.
- For patients who continue to have symptoms or have repeated exacerbations despite an optimal long-acting inhaled
bronchodilator regimen,
We suggest adding an inhaled glucocorticoid .
- For symptomatic patients with GOLD Stage II, III, or IV COPD, we recommend pulmonary rehabilitation.
- long-term oxygen therapy in all patients with COPD who have chronic hypoxemia (Grade 1A).
Reference:
Http://www.uptodate.com/contents/management-of-stable-chronic-obstructive-pulmonary-disease#H40

123.Lung disease that causes clubbing?


Answer: A. Bronchiectasis (The only option mentioned)
Clubbing usually begins in the thumb and index fingers and is most often associated with pulmonary or cardiovascular diseases,
including lung cancer, interstitial pulmonary fibrosis, lung abscess, pulmonary tuberculosis, pulmonary lymphoma, congestive heart
failure, infective endocarditis, and cyanotic congenital heart disease. Less frequently, digital clubbing may occur in patients with
extrathoracic disease, including inflammatory bowel disease, liver cirrhosis, and gastrointestinal neoplasms.
Reference: Uptodate

124.A patient with lung cancer. Lab results: low PTH and High Calcium. What is the reason?
88
A. PTH related peptide for lung ca
Answer: A
Reference: kumar and clark’s

125.Patient with bronchiectasis. What else beside medical treatment can benefit this patient?
A. Chest physiotherapy
Answer: A
Reference: Toronto Notes

126.Best drug to decrease bronchial secretion in COPD?


A. Ipratropium
Answer: A
Reference: Master the board.

127.Most common cause of pneumonia? 2 times


Answer: Streptococcus pneumonia is by far the most common cause of typical bacterial pneumonia.
Reference: http://emedicine.medscape.com/article/300157-overview#a4

128.Pt have lesion in right upper lung look like calcium how to treat?
Answer:

The first step in the evaluation of a pulmonary nodule is to look for a prior x-ray. Finding the same pulmonary
nodule on an x-ray done years ago may save you from doing any further workup. If no prior x-ray is available,
then consider whether this patient is high or low risk for lung cancer.

In low-risk patients, <35 years of age and nonsmokers with calcified nodules, you may follow
the patient with chest x-rays or chest CT every 3 months for 2 years. Stop the follow-up if after 2 years
there is no growth.

High-risk patients >50 years of age with a smoking history and a nodule are likely to have bron-chogenic
cancer. The best diagnostic procedure is open-lung biopsy and removal of the nodule at the same time.

Reference : Kaplan Internal Medicine 2013

129.Asthmatic patient on monteleukast and bronchodilator, has dry cough every day came to ICU, what to give for long term?
Answer: high-dose inhaled corticosteroid plus a leukotriene receptor antagonist plus an oral corticosteroid. Consider omalizumab
for patients who have allergies.
Reference: Medscape

89
130.Bilateral pneumonia treatment
Answer: ??? Nothing specific found about bilateral pneumonia
Outpatient:
Previously healthy, no antibiotic for 3 months – macrolide (azithromycin, clarithromycin) or doxycycline
Comorbidity, use of antibiotic in 3 months – respiratory fluoroquine (levofloxacine, moxifloxacine)
Inpatient: respiratory fluoroquine (levofloxacine, moxifloxacine) or ceftriaxone + azithromycine
Reference: master the boards USMLE step 2, Toronto notes

131.Most common cause of obstructive sleep apnea ?


Answer:
Most patients have OSA because of a small upper airway. As the bones of the face and skull develop, some people develop a small
lower face, a small mouth, and a tongue that seems too large for the mouth. These features are genetically determined, which
explains why OSA tends to cluster in families. Obesity is another major factor. Tonsil enlargement can be an important cause,
especially in children.
Reference: Uptodate

132.Patient with cough and SOB. X-Ray shows consolidation in right upper lob what’s the treatment?

Answer: Anti-TB drugs.


TB may be found in any part of the lung, but upper lobe involvement is most common.
Reference: http://emedicine.medscape.com/article/230802-workup#c12

90
Nephrology

91
1. High pco2, low ph ; (medicine)
A. Metabolic acidosis
B. Metabolic alkalosis
C. Respiratory acidosis
D. Respiratory alkalosis
Answer: c

2. Ph= 7.1, hco3= 18, co2= 25 dx:


A. Metabolic alkalosis
B. Metabolic acidosis
C. Respiratory alkalosis
D. Respuratory acidosis
Answer: b
Reference: http://lifeinthefastlane.com/investigations/acid-base/

3. Pt had pyelography showing rt kidney 7 cm and left kidney 16 or 14 cm, arteriography shows rt renal stenosis, next
management ?
A. Arteriography
B. Lt kidney percutaneous biopsy
C. CT angiography
D. CT abdomen
Answer : c ( not sure )

4. Urine incontinence, bladder palpable during examination, what type of urine does the patient has?
A. Stress.
B. Overflow.
C. Reflux.
D. Urgency
Answer : b

5. Characteristic of nephritic syndrome?

92
A. Edema
B. Htn
C. Hypoalbuminemia
D. Hyperlipidemia
Answer: b
Nephritic syndrome:
- Hematuria: +++ blood – microscopic or macroscopic hematuria, red cell casts – distinguishing feature, form in nephrons &
indicate glomerular damage
- Haematuria occurs due to podocytes developing large pores which allows blood and protein to escape into the urine.
- 2 - proteinuria: ++ protein (small amount)
- 3 - hypertension: usually only mild
- 4 - low urine volume <300ml/day, due to reduced renal function.
Http://geekymedics.com/nephrotic-vs-nephritic-syndrome/

6. First symptom of hypomagnesaemia?


A. Muscle paralysis?
B. Hypotension
C. Loss of deep muscle reflex
D. Respiratory depression
Answer: a
- Neuromuscular manifestations: muscular weakness, apathy, tremors, paraesthesia, tetany, vertical nystagmus and positive
chvostek and trousseau signs.
- Severe effects such as seizures, drowsiness, confusion and coma occur at magnesium concentrations <0·4 mm
- Cardiovascular manifestations: hypomagnesaemia has been associated with both atrial and ventricular arrhythmias
- Ecg: flattened t-waves, u-waves, prolonged qt interval and widened qrs complexes
- Metabolic effects: 1) hypokalemia is associated with hypomagnesaemia in up to 60% of cases. Due to underlying common
etiologies that cause magnesium and potassium losses and partly due to a specific disorder of renal potassium wasting as a
result of hypomagnesaemia. 2) hypocalcaemia is also commonly associated with hypomagnesaemia; hypomagnesaemia
suppresses the release of pth and also induces end-organ resistance to pth
- Http://www.medscape.com/viewarticle/753881_4

7. What is the most common cause of secondary hypertension?


A. Renal disease
B. Cushing
C. Conn
D. Pheochromocytoma
Answer: a
Depending on the age http://www.aafp.org/afp/2010/1215/p1471.html

8. Case with: ph 7.2 \ pco2: decreased below normal range \ bicarbonate decreased below normal range . Dx :
A. Compensated metabolic acidosis
B. Uncompensated metabolic acidosis
C. Compensated respiratory acidosis
D. Uncompensated metabolic acidosis
Answer: b? Partially abnormal ma?

93
If ph is normal, paco2 and hco3 are both abnormal = compensated
if ph is abnormal, paco2 and hco3 are both abnormal = partially compensated
if ph is abnormal, paco2 or hco3 is abnormal = uncompensated
Http://goo.gl/3gsaqp

9. Long scenario with urine analysis only which shows: normal urine ph high chloride bicarbonate low And others within normal
limit. What is your diagnosis?
A. Metabolic acidosis
B. Metabolic alkalosis
C. Respiratory alkalosis
D. Respiratory acidosis
Answer: most likely b

10. Characteristic signs of nephritic syndrome:


A. Hypertension
B. Hyperlipidemia
C. Hypoalbuminia
D. Edema
Answer: a
Nephritic syndrome symptoms: hematuria, proteinuria, hypertension, blurred vision, azotemia, oliguria.

11. 68 years old male patient came with weight loss, hematuria, flank pain and flank mass. Which of the following imaging study
will confirm the diagnosis?
A. Us
B. Ct
C. Mri
D. Radionucliotide
Answer: b
Http://radiopaedia.org/articles/renal-cell-carcinoma-1

12. Renal or small cell cancer stage iii with bone pain what is the immediate action
A. MRI only
B. Radiotherapy
C. IV steroid and mri
D. No immediate action

Answer: a
The most sensitive way of detecting bone metastases is by isotope bone scan.
The main goals of management are: pain relief, preservation and restoration of function, skeletal stabilization, and local
tumour control (e.g. Relief of tumour 
impingement on normal structure).

94

 reference: davidsons 22

13. Filling defect seen with acoustic shadow in the renal us dense echo:
A. Tumor
B. Uric acid stone
C. Blood clot
D. Sloughed papilla
Answer: b
Acoustic shadowing occurs when the sound wave encounters a very echo dense structure, nearly all of the sound is reflected,
resulting in an acoustic shadow such as stones. Http://emedicine.medscape.com/article/381993-overview

14. Urge incontinence principle management:


A. Medical
B. Surgical
C. Medical and surgical
D. Bladder training and physio..
Answer: a
Mainly lifestyle modification, physiotherapy and medications. Rarely surgical.
Merck manual

15. Renin secreted from ?


A. Renal afferent
B. Renal efferent
C. Distal tubules
D. Proximal tubule
Answer: a ( by juxtaglomerular cells in afferent arteriole )
Reference: usmle step 1

16. 50+ old patient ē HTN and proteinuria 4+, pyelography showed right kidney 14 cm, left kidney 7cm, arteriography showed left
renal artery stenosis. What to do?
A. CT angio
B. CT abdomen
C. Radiolucent something
D. Percutaneous renal biopsy

Answer: a
A. Renal arteriogram is the gold standard in diagnosing ras.

95
Reference: step up to medicine. Uptodate.

17. Uti case patient resistant to b lactams, sensitive to fluoroquinolones, chloramphenicol, aminoglycosides which drug is
contraindicated:
A. Gentamicin
B. Azithromycin
C. Lovcloxacillin
D. Chloramphenicol

Answer: a

Not a clear question and options!

18. To prevent recurrence of uti , what you prefer of the following circumstances ?
A. Decreased ph ,increased urea or urea(i did not remember),decreased urine osmolarity (i think)*
B. Decreased ph ,increase urea ,increased urine osmolarity
C. Increased ph ,increased urea or urea(i did not remember),decreased urine osmolarity
D. Increased ph ,decreased urea or urea(i did not remember),increased urine osmolarity

Answer: b

19. Patient on diuretics developed palpitation. Due to the disturbance of which of these electrolytes lead to his presentation?
A. Na
B. K
C. Cl
D. Ca

Answer: B
Hyperkalemia: usually asymptomatic but may develop nausea, palpitations, muscle weakness, muscle stiffness, paresthesias,
areflexia, ascending paralysis, and hypoventilation. Can be caused by K+-sparing diuretics such as Spironolactone, Amiloride and
Triamterene. Reference: Toronto Notes.

20. (long scenario) 55 year old known diabetic patient came for checkup. What is the earliest effect of Diabetes Mellitus on the
kidneys?
A. Hydronephrosis with ↑ protein excretion.
96
B. Hydronephrosis with ↓ protein excretion.
C. Sclerosis with ↑ protein excretion.
D. Sclerosis with ↓ protein excretion.

Answer: ?
Hyperfiltration with normoalbuminuria or ↑ protein excretion.
(I think they meant by “hydronephrosis” = hyperfiltration).

References:Toronto Notes + http://www.pathophys.org/ckd/ + http://emedicine.medscape.com/article/238946-overview#showall

21. Calculate anion gap


o Na: 135
o Cl: 100
o Hco3: 12
A. 23
B. 10
C. 6

Answer: a
+ +
The anion gap is the difference between primary measured cations (sodium na and potassium k ) and the primary measured anions
- -
(chloride cl and bicarbonate hco3 ) in serum.
Ag = na - (cl + hco3) or (na +k) – (cl + hco3)
The normal range value for the serum anion gap is 8-16 meq/l
Reference: http://emedicine.medscape.com/article/2087291-overview
Https://en.wikipedia.org/wiki/anion_gap

22. Most common sign in renal cancer.


A. Cachexia
B. Hematuria
C. Abdomen mass

Answer: b
Renal cell carcinoma (rcc) may remain clinically occult for most of its course. The classic triad of flank pain, hematuria, and flank
mass is uncommon (10%) and is indicative of advanced disease. Twenty-five to thirty percent of patients are asymptomatic, and
their renal cell carcinomas are found on incidental radiologic study.
The frequency of the individual components of the classic triad is as follows:
Hematuria – 40%
97
Flank pain – 40%
A palpable mass in the flank or abdomen –25%
Reference: Medscape

23. Patient with pheochromocytoma and high catecholamine in urine. What’s the initial medical management?
A. ACEI
B. Aldosterone blocker
C. Phenoxybenzamine

Answer c
First goal in management is to control the blood pressure.
Phenoxybenzamine (alpha blockade) is the first to control blood pressure. Without alpha blockade, patients’ blood pressure can
significantly rise intra-operatively.
Reference: master the boards

24. Patient with pheochromocytoma and high catecholamine in urine initial medical management:
A. ACEI
B. Aldosterone blocker
C. Alpha blockers

Answer: c
All patients with pheochromocytoma need to undergo preoperative alpha-adrenergic blockade as the first line drug. After alpha
blocker is given we give beta blocker (control heart rate) for 2 – 3 days but never start beta blocker before the alpha blocker. If beta
blockade is started prematurely, unopposed alpha stimulation could precipitate a hypertensive crisis. Administer the last doses of
oral alpha and beta blockers on the morning of surgery. Surgical resection with early ligation of venous drainage is the treatment of
choice in pheochromocytoma
References: uptodate, Medscape

25. Patient with HTN taking medication c/o painful big toe what's the drug?!
A. Thiazide.
B. Mannitol.
C. Spironolactone.

Answer: a
Lippincott pharmacology:
Thiazide can precipitate a gouty attack by increasing uric acid in predisposed individuals.
Loop diuretics can have this effect too.

26. 50 yrs man diabetic, well controlled had colon cancer surgery ( coloectomy) they kept him on insulin and dextrose, after
surgery by 2 days he became irritable, in shock, his electrolyte ( na 129 ) ( k 3.2 ) urine and serum osmolality normal, what's
the dx:
A. Fluid overload.
B. Addison disease
C. Siadh

Answer: a
98
27. Patient drink antifreeze which contain ethylene glycol. What is the most likely complication he will have?
A. Rapid progression glomrernephrithis
B. Pyelonephritis
C. Atn
Answer: c
Ethylene glycol will produce oxalate crystals which will cause atn (kaplan internal medicine)

•siadh: hyponatremia, low serum osmolality, normal k+


• addison: hyponatremia, hyperkalemia
- fluid overload:

28. Patient with polyuria polydipsia on water deprivation test, low osmolality of urine ,,, not respond to desmopressin ,what is
the diagnosis ?
A. Central di
B. Nephrogenic di
C. Psychogenic polydipsia
Answer: b

29. Breast cancer metastasis to the lung before mastectomy pt is depressed , increase urination , thirsty all the time , lab
provided na low , urine osmo low?
A. Psych polydypsia
B. Siadh
C. Di

Answer: c
diabetes insipidus is defined as the passage of large volumes of dilute urine in 2 forms
Central: characterized by decreased secretion of antidiuretic hormone
Nephrogenic: characterized by decreased ability to concentrate urine because of adh resistance.
Reference: medscape http://emedicine.medscape.com/article/117648-overview

30. Anion gap mml/


99
A. 13
B. 28
C. 99
Answer: a

31. Case of a guy who has high phosphate. Question phosphate is elevated in which organ failure?
A. Liver
B. Kidney
C. Lung
Answer: b
Reference: http://emedicine.medscape.com/article/241185-overview#a4

32. Patient with vomiting and diarrhoea. What type of electrolyte imbalance?
A. Hypernatremia.
B. Hyperglycemia.
C. Hyperkalemia.
Answer: a
Ref.http://www.uptodate.com/contents/image?Imagekey=neph%2f69879&topickey=neph%2f2376&source=see_link&utdpopup=tr
ue

33. A patient with malar rash, arthritis, …… what is the diagnosis?


A. ITP
B. Lupus nephritis
C. Ra
Answer: b
The classic presentation of a triad of fever, joint pain, and rash in a woman of childbearing age should prompt investigation into the
diagnosis of sle. Patients may present with any of the following manifestations:
Constitutional (eg, fatigue, fever, arthralgia, weight changes)
Musculoskeletal (eg, arthralgia, arthropathy, myalgia, frank arthritis, avascular necrosis)
Dermatologic (eg, malar rash, photosensitivity, discoid lupus)
Renal (eg, acute or chronic renal failure, acute nephritic disease)
Neuropsychiatric (eg, seizure, psychosis)
Pulmonary (eg, pleurisy, pleural effusion, pneumonitis, pulmonary hypertension, interstitial lung disease)
Gastrointestinal (eg, nausea, dyspepsia, abdominal pain)
Cardiac (eg, pericarditis, myocarditis)
Hematologic (eg, cytopenias such as leukopenia, lymphopenia, anemia, or thrombocytopenia)
Http://emedicine.medscape.com/article/332244-overview

34. What is true about polycystic kidney disease?


A. Autosomal recessive
B. X-linked
C. Autosomal dominant.
Answer: c
Autosomal dominant polycystic kidney disease (adpkd) is a multisystemic and progressive disorder characterized by cyst formation
and enlargement in the kidney (see the image below) and other organs (eg, liver, pancreas, spleen). Up to 50% of patients with
adpkd require renal replacement therapy by 60 years of age.
100
Http://emedicine.medscape.com/article/244907-overview

35. Which cells forming the filtration layer in the kidney:


A. Mesangial
B. Podocytes
C. Partial
Podocytes play an important role in glomerular function. Together with endothelial cells of the glomerular capillary loop and the
glomerular basement membrane they form a filtration barrier.

36. Treatment of urge incontinence ?


A. Medical
B. Surgical
C. Medical + surgical
Answer: a
Http://www.webmd.com/urinary-incontinence-oab/america-asks-11/urge

37. Patient with vomiting and diarrhea. What type of electrolyte imbalance?
A. Hypernatremia.
B. Hyperglycemia.
C. Hyperkalemia.

Answer: a
Explanation: both upper and lower gastrointestinal losses can result in hypernatremia when water intake is limited. Loss of gastric
secretions (due to vomiting or drainage) and upper gastrointestinal losses, which contain both gastric and small intestinal secretions,
have a sodium plus potassium concentration well below that in the plasma and will therefore promote the development of
hypernatremia. Similar considerations apply to osmotic diarrheas but not to secretory diarrheas, which have a sodium plus
potassium concentration similar to that in the plasma, the loss of which will not directly affect the serum sodium concentration
Reference http://cursoenarm.net/uptodate/contents/mobipreview.htm?32/2/32800#h6

38. Abdominal solid mass (renal i guess but not sure ) confirmed by
A. CT
B. MRI
C. US

Answer: a.
Contrast-enhanced CT scanning has become the imaging procedure of choice for diagnosis and staging of renal cell cancer and has
virtually replaced excretory urography and renal ultrasonography. Http://emedicine.medscape.com/article/281340-workup

39. Treatment of streptococcus glomerulonephritis in children with edema and htn?


A. High dose of antibiotic
B. Diuretic for edema
C. Diuretic for htn
Answer: b
Uptodate:
101
There is no specific therapy for psgn. Management is supportive and is focused on treating the clinical manifestations of the disease,
particularly complications due to volume overload. These include HTN and less commonly pulmonary edema. General measures
include na and water restriction and loop diuretics. Loop diuretics generally provide prompt diuresis with reduction of Blood
pressure and edema.

40. Patient presented with soda-color urine since one week, during examination there are congestion of throat with cervical
lymphadenopathy with fever. What's the cause?
A. Acute glomureties
B. IgAnephropathy
C. Acute cystitis
Answer: a
IgAnephropathy ~> gross hematuria usually appears simultaneously or within the first 48-72 hours after the infection begins;
persists less than 3 days; and, in about a third of patients, is accompanied by loin pain, presumably due to renal capsular swelling.
Acute gn ~> symptom onset is usually abrupt. In the setting of acute postinfectious glomerulonephritis (gn), a latent period of up
to 3 weeks occurs before onset of symptoms. However, the latent period may vary; it is typically 1-2 weeks for postpharyngitis
cases and 2-4 weeks for cases of postdermal infection (ie, pyoderma). The onset of nephritis within 1-4 days of streptococcal
infection suggests preexisting renal disease.
Http://emedicine.medscape.com/article/239927-clinical#b1
http://emedicine.medscape.com/article/239278-clinical#b1

41. A scenario of a patient with HTN came with headache and anxiety and have 3 previous vistas of high blood pressure and
treated for it then we need to stop because of the patient had a hypotension what you will do:
A. 24 urine metamephrin (phenchromocytoma )
B. TSH
C. Acth
Answer: a

42. Patient drinks some material made from methanol used for freezing what is the complication?
A. Rapid progression glomerulonephritis
B. Pyelonephritis
C. Atn
Answer: c

43. A case of pyelonephritis, what is the next step?


A. Admit and give antibiotics
B. Do investigations
C. Give him antibiotics at home
Answer: treatment with fluids and oral antibiotics may be given on an outpatient basis if children are not vomiting and not markedly
ill so it depends on the case. It is prudent to order urinalysis (and urine culture in those with abnormal findings) in all febrile boys
younger than 6 months and febrile girls younger than 24 months with fever lasting more than 48 hours. Medscape

44. Pt with rt kidney 14 cm and left kidney 7 cm, arteriography: renal artery stenosis
What to do?
A. CT angio (same as arteriography)
B. CT abdomen
102
C. Biopsy
Answer: treat http://www.uptodate.com/contents/treatment-of-unilateral-atherosclerotic-renal-artery-stenosis

45. Anion gap mml/


A. 13
B. 28
C. 99

46. Nephritic syndrome?


A. HTN
B. Hypobilirubinemia
C. Edema

Answer: A
Nephritic syndrome (PHAROH): Proteinurea, Hemtourea, Azotemia, RBC cast, Oligurea, Hypertension
Nephrotic syndrome (HELP): hypoalbuminemia, edema, lipid abnormality, proteinurea
Source: Toronto Notes page NP22 and NP23
Http://geekymedics.com/nephrotic-vs-nephritic-syndrome/

47. Child with nephrotic syndrome on steroid for 3-6wks or 6 months have vaccine ?
A. Give the vaccine
B. Stop the steroids
C. 3month and give appointment

48. Action of cytotoxic?


A. Il6
B. Il10
C. Tnf gamma

49. Pt drink some material made from ethanol use for freezing what the complication
A. Rapid proggesion glomrernephrithis
B. Pyeleyno nephritis
C. Atn
There is no clear or conclusive explanation of he direct effect of ethanol on kidney but in that q the ethanol used for freezing which
can induce vasoconstriction and cause ischemic atn

50. Patient with pheocromocytoma and high catecholamine in urin. What is the initial medical management
A. ACEI
B. Aldosteron blocker

Answer: phenoxybenzamine
(alpha blocker - best initial therapy - master the boards & uptodate)

103
51. Pt with long hx of uncontrolled HTN , he presented to you with headache and 160/90 Blood pressure , what you will see in
his kidneys?
A. Decrease sclerosis
B. Increase hyalination of arterioles

Answer: b
Intimal thickening and luminal narrowing of the large and small renal arteries and the glomerular arterioles, and
Glomerulosclerosis: both focal global (involving the entire glomerulus) and focal segmental sclerosis
Reference: http://www.uptodate.com/contents/clinical-features-diagnosis-and-treatment-of-hypertensive-
nephrosclerosis?Source=preview&search=%2fcontents%2fsearch&anchor=h11268824#h2

52. Patient with long hx of uncontrolled htn, he presented to you with headache and 160/90 Blood pressure , what you will see
in his kidneys:
A. Decrease sclerosis
B. Increase hyalinization of arterioles

Answer: b
Benign hypertensive arteriolar nephrosclerosis is characterized histologically by arteriolar hyalinosis caused by insudation of plasma
proteins and medial thickening caused by both hypertrophy and hyperplasia of vascular smooth muscle cells, tubular
atrophy, interstitial fibrosis, periglomerular fibrosis, hyaline casts, protein and blood in urine
Reference: merck manual, wikipedia

53. Patient had throat infection 2 weeks ago was developed hematuria how to treat:
A. Corticosteroids
B. Thiazide
Answer: c
The diagnosis is post strep glomerulonephritis
During the acute phase of the disease, restrict salt and water. If significant edema or hypertension develops, administer diuretics.
Loop diuretics increase urinary output and consequently improve cardiovascular congestion and hypertension.
For hypertension not controlled by diuretics, usually calcium channel blockers or angiotensin-converting enzyme inhibitors are
useful. For malignant hypertension, intravenous nitroprusside or other parenteral agents are used.
Other features of therapy are as follows:
indications for dialysis include life-threatening hyperkalemia and clinical manifestations of uremia
restricting physical activity is appropriate in the first few days of the illness but is unnecessary once the patient feels well
steroids, immunosuppressive agents, and plasmapheresis are not generally indicated
Reference:
Http://emedicine.medscape.com/article/240337-treatment#d8

54. Effect of HTN on kidney:


A. Decrease glummer
B. Hyaline arteriolosclerosis
Answer: b

104
N benign hypertension, hyaline (pink, amorphous, homogeneous material) accumulates in the wall of small arteries and arterioles,
producing the thickening of their walls and the narrowing of the lumens. Http://www.pathologyatlas.ro/hyaline-arteriolosclerosis-
cardiovascular-pathology.php

55. Patient on diuretics, diarrhea. What type of electrolyte imbalance?


A- Hypokalemia.
B- Hyperkalemia.

Answer: a
Explanation: hypokalemia is caused by many mechanisms one of which is increased excretion. Increases excretion could be by non-
renal losses such as diarrhea or by renal losses by administration of diuretic such as loop and thiazide diuretics.
Reference http://www.clevelandclinicmeded.com/medicalpubs/diseasemanagement/nephrology/hypokalemia-and-hyperkalemia/

56. Effect of HTN on kidney:


A. Decrease glomerular…..
B. Hydraulic of vessels
Answer:b

57. Patient on diuretics, diarrhea. What type of electrolyte imbalance?


A. Hypokalemia.
B. Hyperkaliemia.
Answer: a
Ref.:http://www.clevelandclinicmeded.com/medicalpubs/diseasemanagement/nephrology/hypokalemia-and-hyperkalemia/

58. Patient missed his insulin injection. Most likely to be found in urine analysis?
A. Ketones.
B. Proteins.
Answer: a

59. Urology pt had papillary cancer removed biopsy shows removal of all tumor till muscle layer, what next?
A. Follow up with cystoscopy
B. Given something medication i think chemo and other two options
Answer:
For patients at low risk of recurrence following turbt, we recommend an immediate, single postoperative dose of chemotherapy.
This is considered sufficient in itself without the need for additional therapy, and bacillus calmette-guerin (bcg) is never given in
this setting. The most extensive data for intravesical chemotherapy are with mitomycin. Repeat urine cytology (particularly for high
grade cases) and cystoscopyare generally advised at three to six month intervals, depending on the number of tumor recurrences,
for the first four years and annually thereafter in the absence of tumor recurrence.
Ref. Uptodate

105
60. Post streptococcal glomerulonephritis. What is the treatment:
A. Steroid
B. Antibiotics
Answer: b
- Treatment is largely supportive, with management of fluid overload and hypertension with diuretics
- Most of cases resolve spontaneously (this is why biopsy is rarely needed)
- Antibioics should be given to eradicate the organism from the pharynx
- Kaplan usmle step2

61. Patient on diuretics developed diarrhea. What type of electrolyte imbalance?


A. Hypokalaemia.
B. Hyperkalaemia.
Answer: a

62. According to henoch schonlein purpura, which of the following carries bad prognosis :-
A. Renal failure
B. Hepatic failure
Answer: a
In adults, kidney involvement progresses to end-stage renal disease (esrd) more often than in children

63. Most common presentation of nephritic syndrome?


A. Edema
B. Hypertension
(no mention of hematuria)
Answer: both but i think of edema more because it's a clear symptom,
nephritic syndrome is defined by hematuria, proteinuria, dysmorphic RBCs, RBC casts on microscopic examination of urinary
sediment. Often ≥ 1 of the following elements are present: edema, hypertension, elevated serum creatinine, and oliguria.
- glomerular filtration rate is reduced stimulating the retention of salt and water culminating in hypertension and edema.
- edema (peripheral or periorbital) - this is reported in approximately 85% of pediatric patients in acute glomerulonephritis
merck manual + http://emedicine.medscape.com/article/239278-overview

64. Female patient swallow 1 liter of car anti-freeze water (ethylene glycol) what is going to happen?
A. Interstitial nephritis
B. End stage renal disease
Answer: can be b not sure if a is correct. Calcium oxalate crystals may form and accumulate in the renal cortex results in decreased
glomerular filtration and renal insufficiency. Oliguric or anuric renal failure is the result in the most severe cases and, although
permanent renal failure is rare, recovery of renal function may take up to two months.
Http://www.atsdr.cdc.gov/csem/csem.asp?Csem=12&po=10
http://www.aafp.org/afp/2002/0901/p807.html

65. Patient complaining of hematuria with wbc in urine and the culture is negative. Cystoscopy, revealed submucosal
hemorrhage what is the cause:
106
A. Cystolithiasis
B. Interstetial cystitis

Answer: b
Interstitial cystitis is a clinical syndrome characterized by daytime and nighttime urinary frequency, urgency, and pelvic pain of
unknown etiology. Interstitial cystitis has no clear etiology or pathophysiology.
Http://emedicine.medscape.com/article/2055505-overview

66. Long scenario about patient presented dry cough after being diagnosed with HTN what is the cause:
A. Furosemide
B. ACEI (they mentioned the drug name )

Answer: b
Angiotensin converting enzyme (ace) inhibitors are the treatment of choice in patients with hypertension, chronic kidney disease,
and proteinuria. Ace inhibitors reduce morbidity and mortality rates in patients with heart failure, patients with recent myocardial
infarctions, and patients with proteinuric renal disease. Ace inhibitors appear to act primarily through suppression of the renin-
angiotensin-aldosterone system.
Http://emedicine.medscape.com/article/241381-medication#5

67. Psgn treatment


A. Antibiotics
B. Supportive

Answer: b. Poststreptococcal gn: therapy is primarly supportive: antihypertensives, loop diuretics for edema; the use of antibiotics is
rd
controversial. Steroids ma be helpful in severe cases. Reference : step up to medicine 3 edition page 285

68. The most common cause of renal failure is:


A. Diabetes mellitus
B. Hypertension
Answer: a
adults with diabetes or high blood pressure, or both have a higher risk of developing ckd than those without these diseases.
Approximately 1 of 3 adults with diabetes and 1 of 5 adults with high blood pressure has ckd.
Http://www.cdc.gov/diabetes/pubs/pdf/kidney_factsheet.pdf

69. Differential diagnosis of renal filling defects:


A. Tumor
B. Stones (show acoustic shadow on us, may show calcification)
Clot (history of severe hematuria, resorted in follow up

70. A patient with dyspnea, pallor, edema of legs and itching. What is the diagnosis?
A. Scabies
B. Renal failure

Answer: B

107
71. Case of DKA with metabolic acidosis. What is the early mechanism to restore blood ph?
A. Excretion of CO2 through lungs.
B. Excretion of lactic acid through kidneys.

Answer: A
Reference: https://en.wikipedia.org/wiki/Acid%E2%80%93base_homeostasis
Http://fitsweb.uchc.edu/student/selectives/timurgraham/Compensatory_responses_metabolic_acidosis.html

72. HTN pt with decrease gfr ;


A. Bilateral renal artery stenosis
B. DM nephropathy
I didn’t understand the question. Do they mean that bilateral renal artery stenosis causes HTN and decrease gfr?

73. Patient with pheocromocytoma And high catecholamine in urine Initial medical management :
A. ACEI
B. Aldosterone blocker
Answer : from Medscape: the definitive treatment for pheocromocytoma is surgical resection of the tumor, and usually cures the
hypertension. The initial medical treatment is only as preoperative treatment with Alpha and beta blockers.
Start alpha blockade with phenoxybenzamine 7-10 days preoperatively
Provide volume expansion with isotonic sodium chloride solution
Encourage liberal salt intake
Initiate a beta blocker only after adequate alpha blockade, to avoid precipitating a hypertensive crisis from unopposed
alpha stimulation
Administer the last doses of oral alpha and beta blockers on the morning of surgery

74. A patient with signs and symptoms of renal and respiratory involvement. What is the diagnosis?
A. Glomerulonephritis
B. Wegener's granulomatosis

Answer: B?
Incomplete question but you should to keep one thing in your mind if they mentioned there is a Hx. Of sinusitis the diagnosis with be
100% Wegners
Reference: First Aid step 1
Reference: Master the boards

75. 40 year-old was missing. 3 days later, his colleagues found him in his home. He was thirsty and vomiting. High Ca>>>what is
the treatment?
A. Hydration

76. Ph: 7.2 , hco3 : 25 , co2: 60:


Answer: metabolic acidosis

108
77. Patient has HTN control on his medications, developed albuminuria, what you should add to his HTN medications:
A. ACEI
Angiotensin converting enzyme (ace) inhibitors are the treatment of choice in patients with hypertension and chronic kidney disease
or proteinuria.
Source: http://emedicine.medscape.com/article/241381-medication
Http://emedicine.medscape.com/article/238158-treatment#d9

78. Ph.: 7.2 , hco3 : 25 , co2 : 60:


A. Metabolic acidosis

Step 1

Step 2→ anion gap= 2 x serum na – cl + bicarbonate. Normal{8-12} ~ 10


Step 3 → third component i.e. Compensatory mechanism
∆ 𝐴𝑛𝑖𝑜𝑛 𝐺𝑎𝑝
= [> 2→ metabolic alkalosis/ 1-2 → metabolic acidosis/<1 → non- anion gap]
∆ 𝐵𝑖𝑐𝑎𝑟𝑏𝑜𝑛𝑎𝑡𝑒

79. High k (hyperkalemia), ecg showed wide qrs, how to treat:


A. Give Cagluconate
Answer: a
- Cagluconate: reverse ecg changes through membrane stabilization (most emergent treatment)
- Sodium bicarbonate: shifts k ions into cells
- Insulin: drives k ions intracellular (takes 30-60 min)
- Glucose: to prevent hypoglycemia
- Beta agonist: shifting potassium into the intracellular compartment
- Loop diuretics (frusemide): enhances renal potassium excretion and thus lower serum levels
- Http://emedicine.medscape.com/article/240903-medication#1

109
80. A 55 years old female has htn, fluid overload and azotemia what is the diagnosis:
A. Tubular acidosis (was one of the choices)??
Answer: acute glomerulonephritis:
Acute gn is defined as the sudden onset of hematuria, proteinuria, and red blood cell (RBC) casts. This clinical picture is often
accompanied by hypertension, edema, azotemia (ie, decreased glomerular filtration rate [gfr]), and renal salt and water retention.
Http://emedicine.medscape.com/article/239278-overview

81. What’s the most common organism in uti?


Answer: no choices
The most common organism of uncomplicated cystitis is e. Coli.
Sources: https://yhdp.vn/uptodate/contents/mobipreview.htm?13/15/13561#h899949156

82. Patients with HTN developed s hypercalcimea. What is the treatment?


Answer: furosemide
Loop diuretics act by competing for the chloride site on the na-k-2cl cotransporter. Inhibiting sodium chloride reabsorption also
inhibits the backleak of potassium and the generation of the lumen-positive potential. As a result, calcium excretion rises, an effect
that may be exploited in the treatment of hypercalcemia in selected patients.
Http://www.uptodate.com/contents/diuretics-and-calcium-balance
Factors that can aggravate hypercalcemia, including thiazide diuretic and lithium carbonate therapy, volume depletion, prolonged
bed rest or inactivity, and a high calcium diet (>1000 mg/day).
Http://www.uptodate.com/contents/treatment-of-hypercalcemia?Source=see_link&sectionname=saline+hydration&anchor=h4#h4

83. Non opaque stone on radiography. What type of stone?


Answer: uric acid

84. Early sign of hypermagnesemia:


A. Loss of deep tendon reflex, symptoms and signs include hyporeflexia, hypotension, respiratory depression, and cardiac
arrest. Merck manual

85. Renal failure with azotemia?


A. Bilateral renal artery stenosis.
Answer: missing info

86. Diagnostic tests for pheochromocytoma include the following:


- plasma metanephrine testing: 96% sensitivity, 85% specificity.
- 24-hour urinary collection for catecholamines and metanephrines: 87.5% sensitivity, 99.7% specificity.
Http://emedicine.medscape.com/article/124059-overview

110
87. Why is inulin used to measure gfr:
Freely filtered by glomeruli; inulin is the most accurate substance to measure because it is a small, inert polysaccharide molecule
that readily passes through the glomeruli into the urine without being reabsorbed by the renal tubules. Britannica
http://global.britannica.com/topic/inulin-clearance

88. Drug induced hyperuricemia:


Hydrochlorothiazide and furosemide. Medscape. Http://www.medscape.com/viewarticle/823994_8

89. Dm, HTN developed microalbominuria what to give to prevent renal failure
Ace inhibitors and angiotensin receptor blockers (arbs). Bmj http://bestpractice.bmj.com/best-
practice/monograph/530/prevention/screening.html

90. Gfr 80 in this stage what happen?


Mesangial proliferation and thinking of basement membrane. Not complete question.

91. Pt had hx of infection 2 week later present with hematuria.


Answer: post strephematuria.

92. Minimal change nephrotic syndrome, histo-pathalogical finding:


No histologic abnormalities on light microscopy; fusion of foot processes on electron microscopy.

93. Boy hematuria +snhl his father has end stage renal disease and snhl wt diagnosis?
Answer: alport syndrome
Alport syndrome encompasses a group of inherited, heterogeneous disorders involving the basement membranes of the kidney and
frequently affecting the cochlea and eye as well.

94. Patient with bilateral flank pain for 6 months and there is gene 16 mutation , what is the disease
A. Adult polycystic kidney disease

Answer : a

o Adpkd (adult) type 1 on the short arm of chromosome 16, type 2 on the long arm of chromosome 4, type 3 no genomic locus is
assigned
o Arpkd (pedia) on the short arm of chromosome 6.

Reference: medscape.

95. Early sign of hypomagnesemia?


111
A. Loss of deep tendon reflex
Answer: the reflexes in hypomagnesemia are hyperactive

Neuromascular Cardiovascular Metabolic


A. Muscular weaknes
Non specific t wave changes.
B. Tremor ,seizure,
Prolonged qt and qu interval.
parasthesia, and
Repolarization alternans.
tetani
Premature ventricular Hypokalemia
C. +ve chvostek and
contraction and monomorphic Hypclacemia
trousseau signs
ventricular tachychardia
D. Vertical and
horizontal Torsade de point
nastygmus Vfib, enhance digitalis toxicity
Reference: medscape

96. Why is inulin used to measure gfr?


A. Freely filtered by glomeruli

Answer: a (similar to q)
A. Recall that all of the plasma that is filtered and only the plasma filtered is cleared of inulin so that if one were to measure
the clearance of inulin, it would equal the amount of plasma filtered in a minute, the glomerular filtration rate. Therefore, the
clearance of inulin is equal to the glomerular filtration rate, the volume of plasma filtered in one minute.
Reference: http://www.austincc.edu/emeyerth/clearancehtm.htm

97. Calculate anion gap with corrected na 138


Answer:
Anion gap = na - ( cl + hco3 ) or (na + k) – (cl + hco3) normal range are varies depend on references (4- 14)
The na at the anion gap equation should be the measured na not corrected due to it will lead to false elevated calculation
Reference: pocket of medicine, medscape, clinical background

98. Polycystic kidney inheritance?


A. Dominant
Answer: autosomal dominant polycystic kidney disease (adpkd) is a multisystemic and progressive disorder characterized by cyst
formation and enlargement in the kidney and other organs (eg, liver, pancreas, spleen).
(http://emedicine.medscape.com/article/244907-overview)

99. Hydrochlothizaied fursemide what is the type of that something that is bind to ca in psudogout ?
- Pyrophosphate.
It is called calcium pyrophosphate.

100.Patient c/o abdominal pain & HTN , there's abnormality in chromosome 16 . What’s the diagnosis?
A. Polycystic kidney disease

112
Answer: a. Toronto

101.Case of congenital adrenal hyperplasia?


A. Give steroid
Treatment of congenital adrenal hyperplasia: correct electrolyte & give glucocorticoids and mineralocorticoids if needed ( toronto 15
p13)

102.Hydrochlothizaied fursmide what is the type of that something that is bind to ca in psudogout ?
A. Purophosphate

103.Patient with bilateral flank swelling dull move with shifting ?


A-ascitis
Answer:a
Reference: https://depts.washington.edu/physdx/liver/tech.html

104.Case of hyperosmolar state.


Article on hyperosmolar hyperglycemic state: http://emedicine.medscape.com/article/1914705-overview

105.Something about anion gap (causes)?


Mnemonic is karmel.
K — ketoacidosis
A — aspirin
R — renal failure
M — methanol
E — ethylene glycol
L — lactic acidosis
Further about anion gap:
Http://emedicine.medscape.com/article/2087291-overview#a2

106.Case about degree of dehydration?

113
Http://www.merckmanuals.com/professional/pediatrics/dehydration-and-fluid-therapy-in-children/dehydration-in-
children

107.Patient diagnosed with papillary renal carcinoma >> ttt.


Papillary renal cell carcinoma (prcc) is one of the subtypes of renal cancer. This sub type may account 13-20% of all renal
cell cancer. There is slightly increased male predilection. As with other types of renal cell cancer, most are asymptomatic
and incidentally discovered.
Http://radiopaedia.org/articles/papillary-renal-cell-carcinoma

108.Anion gap if corrected na 138 >> choose the closest number to 20

109.Post infection the abx was formed (low c3)


mutations in any one of the five components of the (NADPH) oxidase in
phagocytes.
This enzyme generates superoxide and is essential for intracellular killing
110.Male had +ve semen abx , what is the cause: of
exposure of ag of semen to the blood stream -as in trauma – pathogens by phagocytes.
The earliest manifestations often involve the skin.
Recurrent pyodermas are common, and they often appear as perianal,
111.About chronic granulomatous disease!
axillary, or scalp abscesses.
dx: (NBT) dye test

112.Questions about bph tx "medication and surgical, and one about se!

113.Tx of sle pt with urti

114.Wbc given, pale pt > iron deficiency anemia

115.Boy referral due to having recurrent chest infections & has brother die at 6yrs as same chest infection sister normal all
immunoglobulins low, t cell function is good ?
A. X-linked agammaglobulinemia

116.Child given heparin blood +ftp the dz is >>> dic with thromboses

117.Case of renal artery stenosis (investigation)

118.Pt c/o hematuria and cough with saddle nose


A. Wegener granulomatosi . Medscape http://emedicine.medscape.com/article/332622-overview

119.Why inulin is used in estimation of GFR?


Answer: ?
The perfect filtration marker: is not protein bound, is freely filtered by the glomerulus, is without any tubular secretion, is not
metabolised by the kidneys, and is physiologically inert. Very few substances fulfil these criteria: the gold standard has been a plant
polysaccharide called inulin. Reference: http://www.ncbi.nlm.nih.gov/pmc/articles/PMC3100284/
114
120.A patient with glomerulonephritis developed hemoptysis. What is the most likely diagnosis?
A. Goodpasture syndrome.

Answer: A
Frank hemoptysis suggests Goodpasture syndrome (glomerulonephritis and pulmonary hemorrhage associated with anti GBM
antibody) but this also can be a prominent feature of systemic vasculitis. Reference: Diseases of the Kidney and Urinary Tract
textbook edited by Robert W. Schrier.
NB. Wegner’s was not included in the choices.

121.A patient presented with hemoptysis and signs of nephropathy. Biopsy of the lung showed presence of anti-GBM antibodies.
What is the most likely diagnosis?
A. Goodpasture syndrome.

Answer: A
Reference: http://emedicine.medscape.com/article/981258-overview

122.Histopathology of minimal change nephrotic syndrome?

The glomeruli appear normal on light microscopy in patients with and there are no complement or immunoglobulin deposits on
immunofluorescence microscopy. Glomerular size is usually normal by standard methods of light microscopy, although enlarged
glomeruli may be observed

The characteristic histologic lesion in MCD is diffuse effacement (also called "fusion") of the epithelial foot processes on electron
microscopy. More specifically, there is retraction, widening, and shortening of the foot processes. The spaces between flattened
podocyte foot processes are reduced in number and support the burden of plasma filtration, which may play a role in the excess
albumin load into the urinary space. The degree of effacement does not correlate with the degree of proteinuria. Foot processes
regain a normal appearance with remission of proteinuria

Reference: http://www.uptodate.com/contents/etiology-clinical-features-and-diagnosis-of-minimal-change-disease-in-
adults?Source=machinelearning&search=minimal+change+nephrotic+syndrome&selectedtitle=2~104&sectionrank=5&anchor=H2#H
58052618

123.Pt c/o of loin pain & hematuria and inherited autosomal dominant what the diagnosis?
A. Polycystic kidney
Answer: polycystic kidney disease.
Http://www.uptodate.com/contents/polycystic-kidney-disease-beyond-the-
basics?Source=outline_link&view=text&anchor=H1#H1
Http://emedicine.medscape.com/article/244907-overview
Http://emedicine.medscape.com/article/1958746-overview flank pain & hematuria differentials

124.Low ph, bicarb, co2?


A. Compensated metabolic acidosis

Answer: A
This image for revision

115
125.In addition to anti- HTN , what to advice ?
Answer: Restrict Na to 9mg ,Walking ( such meters)
Not sure about it since we do not have the other choices.
''Control of hypokalemia and hypertension in IHA can be achieved with sodium restriction (to < 2 g/day) and administration of
spironolactone or amiloride, but additional antihypertensives are often needed to achieve good control in this patient group''

126.Best investigation to measure GFR


Answer: 24 urine creatinine collection
Other options were irrelevant
''Inulin is seldom used in clinical testing, although it is used in research'', So the answer will be mostly 24 urine creatinine
measurement.

127.Young adult having episodic palpitation and fear and tightness. Btw the attack she feel fatigue? What investigation should
you order?
a. Urine catecholamine test

Answer: ?
Pheochromocytoma is a nonmalignant lesion of the adrenal medulla autonomously overproducing catecholamine’s despite
a high blood pressure.
The classic history of a patient with a pheochromocytoma includes spells characterized by headaches, palpitations, and
diaphoresis in association with severe hypertension. These 4 characteristics together are strongly suggestive of a
pheochromocytoma. In the absence of these 3 symptoms and hypertension, the diagnosis may be excluded.
Best initial tests:
o High plasma and urinary catecholamine
o Plasma-free metanephrine and VMA
Most accurate test:
o CT or MRI of the adrenal glands
o Metastatic disease is detected with an MIBG scan
Reference: Master the Boards & Medscape

116
117
Hematology

118
1. A patient presented with fatigue, palpitation, sob and pallor, hgb 9. Shown is the peripheral film. What is the type of anemia?

A. Megaloblastic anemia
B. Hypochromic microcytic
C. Sickle cell
D. G6pd

Answer: a
Note that RBCs are as large as the neutrophil and lymphocyte. Heinz bodies is in g6pd
Reference: https://labtestsonline.org/understanding/analytes/blood-smear/details/

2. Long standing trip, swelled ll, no pain, high d-dimer management?


A. Aspirin
B. Lmwh
C. Warfarin
D. Unfractioned heparin with warfarin

Answer: d (best combined???)


Admitted patients may be treated with a lmwh, fondaparinux, or unfractionated heparin (ufh). Warfarin 5 mg po daily is initiated
and overlapped for about 5 days until the international normalized ratio (inr) is therapeutic >2 for at least 24 hours.
Patients treated with lmwh or fondaparinux do not require monitoring of the aptt.
Platelets should be monitored. Heparin or lmwh should be discontinued if the platelet count falls below 75,000. Fondaparinux is not
associated with heparin-induced thrombocytopenia (hit).
Warfarin is long-term anticoagulant.
References: http://emedicine.medscape.com/article/1911303-treatment
Http://bestpractice.bmj.com/best-practice/monograph/70/treatment/step-by-step.html
Http://www.aafp.org/afp/2007/0401/p1031.html

3. Post streptococcal infection generalized petechia and plt =15 management?


A. Splenectomy
B. Cyclo
C. Viii
D. :ivig

Answer: d
119
9
It is a case of immune thrombocytopenic purpura (ITP). In adults, treatment is recommended for a platelet count <30×10 /l. The ash
recommends that if treatment is needed and corticosteroids are given, longer courses (eg, prednisone 1 mg/kg orally for 21 days
then tapered) are preferred over shorter courses of corticosteroids or ivig as first-line treatment. Ivig be used with corticosteroids in
patients who require a more rapid increase in platelet count. If corticosteroids are contraindicated, either ivig (initially, 1 g/kg in a
single dose) or IV rhig (in appropriate patients) may be used as a first-line treatment.
Reference: http://emedicine.medscape.com/article/202158-treatment

4. Lab result show high aptt and bleeding time .. What factor deficiency can cause that?
A. 8
B. 7
C. 9
D. 10
Answer : a
Vwd associated with factor 8 deficiency

5. Lab results show high aptt and bleeding time. Factor deficiency?
A. 8
B. 7
C. 9
D. 10

Answer: a
Aptt is prolonged in both hemophilia a (factor 8 deficiency) and hemophilia b (factor 9 deficiency) and bleeding time should be
normal in both. In vwf disease (factor 8 + vwf factors deficiency) both aptt and bleeding time are prolonged. However vwd is not
one of the options.
Reference: step up to medicine

6. Which type of anemia associated with hyposplenism?


A. Sickle cell anemia
B. Thalassmia
C. Spherocitosis
D. B12

Answer: a

7. A patient was referred from cardiac unit due to severe decrease in platelets which were < 10,000. Patients is known to be
using heparin: what is the treatment:
A. Platelets transfusion.
B. Argatroban
C. Ivig
D. Steroid.

Answer: b
st
Explanation: there are two typer of heparin induces thrombocytopenia: type 1 which occurs in the 1 2 days of therapy with
platelets normalizing with continuing therapy, and type 2 which is immune mediated and occurs 4-10 days of initiating therapy. Hit
9
should be suspected when platelets count decrease 50% of baseline even of nadir is still above 150x10
120
Management of hit type 2 is by first discontinuing heparin, then giving the patient argatroban, a direct thrombin inhibtor. Platelet
tranfusion should be avoided as it may increase the thrombogenic effect.
Reference: http://emedicine.medscape.com/article/1357846-treatment

8. Rheumatoid arthritis patient on treatment for 15y now came with anemia , the pt wasn't on any NSAIDs or asa what is the
type of anemia :
A. Macrocytic hyperchromic
B. Microcytic hypochromic
C. Normocytic normochromic
D. Macrocytic hypochromic

Answer: c
Ra causes anemia of chronic inflammation. This type of anemia is usually normocytic and normochromic unless if it was severe it
could be microcytic hypochromic
Reference: toronto notes

9. Long case about patient eat rice only with loss of coordination between upper and lower extremity with tongue and lip rash ,
vitamin loss suspected what is this Vitamin ?
A. B1
B. B3
C. B6
D. B12

Answer: a
thiamin deficiency is seen:
As beriberi is seen with patients who consume only polished rice, in chronic alcohol-dependent patients and in starved patients.
Patients with berberi present with a symmetrical polyneuropathy. The initial symptoms
are heaviness and stiffness of the legs, followed by weakness, numbness, and pins and needles. The ankle jerk reflexes are lost and
eventually all the signs of polyneuropathy that may involve the trunk and arms are found. Cerebral involvement occurs, producing
the picture of the wernicke–korsakoff syndrome.
Reference: kumar and clark’s clinical medicine

10. Boy with elevated wbcs very high ?? Low hemoglobin low mcv low reticulocytes what is the diagnosis?
A. Iron deficiency
B. B thalassemia trait
C. Sickle
D. Anemia of chronic disease
Answer: b?

11. Scenario of a patient with malaria but no history of bleeding and give a peripheral smear of ring form of malaria what is the
type of malaria?
A. Plasmodium malariae
B. Plasmodium ovale
C. Plasmodium vivax
D. Plasmodium falciparum
Answer: d
121
Reference: http://www.malariasite.com/microscopic-tests/
Http://parasitologyillustrated.com/classes_of_parasites/protozoa/sporozoa/p_falciparum.html

12. Most abundant cells in peripheral smear?


A. Lymphocyte
B. Basophils
C. Neutrophil
D. Eosinophils
Answer: c
** neutrophils are the most abundant white blood cell, constituting 60-70% of the circulating leukocytes.
** basophils are the least abundant leukocyte.
Reference: http://www.histology-world.com/testbank/blood2a.htm

13. A vegetarian sle patient complaining of fatigue and tiredness. What will you find ?
A. Low iron low tibc
B. Low iron high tibc
C. High iron high tibc
D. High iron low tibc
Answer: a, if it mean anemia of chronic disease i think
Anemia is a common clinical finding in patients with systemic lupus erythematosus (sle).1-3 the most common form of anemia in
these patients is that of chronic disease (acd),1 however autoimmune hemolytic anemia (aha), iron deficiency anemia (ida), drug
induced myelotoxicity, and anemia of chronic renal failure are probably not uncommon.
Http://ard.bmj.com/content/59/3/217.full

14. What will you see in malaria rapid diagnostic test (rdt)?
A. Malaria antigens
B. Malaria antibodies
C. Malaria pigments
D. Parasite sexual
Answer: a
- Immunochromatographic tests detect the presence of malaria antigen or enzyme
- The main advantage of rdts is that they provide a means for rapid diagnosis, especially in health resource-limited areas where
microscopy is not available or reliable
- Http://bestpractice.bmj.com/best-practice/monograph/161/diagnosis/tests.html

15. A young male came back from visiting his family in sudan 2 weeks ago, now he presented in er complaining of severe
headache, fever and vomiting. Which of the following you will do immediately? Repeated
A. Blood culture
B. Stool culture
C. Peripheral blood smear
D. Lumbar puncture
Answer: c
- Malarial infection is suspected. Individuals are generally asymptomatic for 12 to 35 days but can commence symptoms as early
as 7 days (depending on parasite species) in most cases, the incubation period for p. Falciparum infection is about 12 to 14 days
(range 7 to 30 days); most infections due to p. Falciparum become clinically apparent within one month after exposure.

122
- Detection of parasites on giemsa-stained blood smears by light microscopy is the standard tool for diagnosis of malaria and
remains the most common onsite diagnostic method
o Source: https://yhdp.vn/uptodate/contents/mobipreview.htm?10/0/10248

16. Cml associated with which translocation :


A. T (14; 18)
B. T ( 11;14)
C. T (8; 14)
D. T (9; 22)
Answer: d

17. Chronic myelogenous leukemia is associated with which translocation :


A. T (14; 18)
B. T ( 11;14)
C. T (8; 14)
D. T (9; 22).
Answer: d
Explanation: cml is associated with the fusion of two genes: bcr (on chromosome 22) and abl1 (on chromosome 9) resulting in the
bcr-abl1 fusion gene. This abnormal fusion typically results from a reciprocal translocation between chromosomes 9 and 22, t(9;22)
that gives rise to an abnormal chromosome 22 called the philadelphia (ph) chromosome.
Reference: http://cursoenarm.net/uptodate/contents/mobipreview.htm?33/25/34192 \

18. Malaria fast diagnostic test


A. Malaria antigens
B. Malaria antibodies
C. Malaria pigments
D. Parasite sexual ...
Answer: a
Malaria rapid diagnostic tests (rdts) assist in the diagnosis of malaria by detecting evidence of malaria parasites (antigens) in
human blood. Http://cursoenarm.net/uptodate/contents/mobipreview.htm?3/28/3520

19. Case of pt with hemochromatosis what will be increased?


A. Ferttin
B. Cureloplasmin
C. Irrelevant
D. Ferittin
Answer: d

20. Long scenario with paragraph about patient who has low hemoglobin and macrocytic anemia. He treated now what does
brown line mean if the blue is hemoglobin
A. Reticulocyte
B. Hematocrit
C. RBCs
D. MCHc
Answer: look like picture in attachment. There’s no picture?
123
21. Patient with fatigue and pallor with blood values that show slightly low hemoglobin, wbc and platelets. Mcv and normal
reticulocyte are normal. Diagnosis?
A. Iron deficiency
B. Folate deficiency
C. Aplastic
D. Hypoplastic
Answer: c
Iron deficiency anemia will show low mcv, and folate deficiency will show high mcvc. Aplastic anemia shows normal mcv with a
decrease in all cell line but the reticulocyte count will be low
Toronto notes 2015

22. Patient with prostatic ca has 2 dvt how to prevent further dvt?
A. Short term lmwh followed by warfarin
B. Lmwh for 6 months
C. I think aspirin
D. Another long answer
Answer: b http://jco.ascopubs.org/content/25/34/5490.full

23. 75 years old male, asymptomatic, bm report: increased lymphocytes, immunohistochemestry: positive cd19, cd56. What is
the treatment?
A. No treatment
B. Rituximab + cvb
C. Rituximab + prednisolone
D. Cyclophosphomide
Answer:

24. Which of the following is associated with burkett’s lymphoma?


A. EBV
B. Hiv
C. Cocxacki
D. HBV
Answer: a- EBV. Uptodate
Https://yhdp.vn/uptodate/contents/search.htm?Search=burkitts+lymphoma&menu=0

25. 24 years old female, has 3 lan at the same side of diaphragm with no distant metastasis, diagnosed with hodgkin’s
lymphoma. What's the stage?
A. 4
B. 3
C. 2
D. 1
Answer: c

124
Text

26. Pt with petechia and bruises labs normal except prolonged ptt due to which factor?
A. V intrinisic ( TEN 8) : 12 , 11, 9, 8
B. Vii exterinsic 7
C. Viii MIX: 10, 5, 2, 1 Intrinsic: ptt
extrinisc: pt
D. X

Answer: c
Aptt measures deficiencies in the intrinsic pathway
Http://emedicine.medscape.com/article/2085837-overview#a2

27. How to stop bleeding in vwd?


A. Fresh frozen plasma ❌
B. Vit. K
C. Platelets transfusion
D. Something irrelevant !!
Answer:
• desmopressin (ddavp®) is treatment of choice for type 1 vwd ƒ causes release of vwf and factor viii from endothelial cells ƒ
variable efficacy depending on disease type; tachyphylaxis occurs ƒ
• tranexamic acid (cyklokapron®, antifibrinolytic) to stabilize clot formation
• high-purity factor viii concentrate containing vwf (hemate p®) in select cases and type ƒ
• frozen plasma (fp) and cryopreciptate ~> avoid
• conjugated estrogens (increase vwf levels)
• platelet transfusions may be helpful in some patients with vwd (eg, type 3) to control bleeding that is refractory to other
therapies.
Answer : neither desmopressin nor recombinant vwf concentrate were in the choices
Http://emedicine.medscape.com/article/206996-treatment

28. Which of the following is associated with burkitt’s lymphoma?


A. EBV
B. Hiv

125
C. Coxsackie-virus
D. HBV
Answer: a
Burkitt lymphoma (bl) is an aggressive b‐cell malignancy with endemic, sporadic and immunodeficiency‐associated variants. It has
been known for many years that the fundamental transforming event in bl is the translocation of the myc gene, and the events that
bring about this translocation and those that allow cells to survive with the constitutive expression of myc have been the subject of
intense investigation. Epstein–barr virus (EBV) infection, malaria, immunodeficiency and spontaneous, somatic mutation can all
contribute to the origin and maintenance of this cancer and their mechanisms are the subject of this review.
Reference: http://www.ncbi.nlm.nih.gov/pmc/articles/pmc2095571/

29. Lab result show high aptt and bt (bleeding time), factor deficiency?
A. 8
B. 7
C. 9
D. 10
Answer: a

Reference: step-up of medicine.

30. Young sickler patient had hx of tiredness and fatigue within 10 hrs, drop in hb and palpable liver and spleen 6 cm below the
costal margin, had 3 previous similar episodes. What you will do:
A. Splenectomy
B. Start hydroxyurea
C. Reticulocyte count
D. Regular blood transfusions**********
Answer: a or d
If hemoglobin drop > than 2 or showing s/s of anemia: transfusion
If recurrence >3 times: splenectomy

31. Boy with elevated wbcs very high ?? Low hemoglobin low mcv low reticulocytes what is the diagnosis?
A. Iron deficiency

126
B. B thalassemia trait
C. Sickle
D. Anemia of chronic disease
Answer: d
If the wbc are high due to inflammation and chronic. Http://www.aafp.org/afp/2010/1101/p1117.html

32. Thalassemia hb f 40 hb a2 20
A. Alfa thalassemia
B. Beta thalassemia minor
C. Beta thalassemia major
D. Thalassemia intermediate
answer: b

Reference : https://yhdp.vn/uptodate/contents/mobipreview.htm?41/24/42380&utdpopup=true

33. Theoretically which of the following cancer will be prevented by vaccination?


A. All
B. Cml
C. Adult t cell leukemia
D. Mycosis fungoides
Answer: c
Atl/l is more common in adults who have been exposed to the human t-cell leukemia/lymphoma virus, type 1 (htlv-1). That’s why it
can theoretically be prevented by vaccination.
Refrence: http://www.cancer.ca/en/cancer-information/cancer-type/non-hodgkin-lymphoma/non-hodgkin-lymphoma/types-of-
nhl/adult-t-cell-leukemia-lymphoma/?Region=on

34. Constant defect in von willbrand :


A. Pt
127
B. B- ptt
C. Prolonged bleeding
D. Factor viii.
Answer: factor viii activity
reference: cecil medicine table in hemorrhagic dz.

35. Long case of boy bleeding epistaxis and ecchymosis with long lab results showing anemia thrombocytopenia and leukopenia,
what is the diagnosis:
A. Ida
B. Aplastic
C. Hypoplastic
D. Hemolytic

Answer: b
Aplastic anemia
A. Symptoms of anemia, thrombocytopenia, and/or infection.
B. Investigation (cbc): anemia or neutropenia or thrombocytopenia (any combination) ± pancytopenia. Decreased
reticulocytes (<1% of the total RBC count)
Reference: toronto notes; (p552)

36. Blood film attached, asking for diagnosis:


A. Leishmaniasis
B. Malaria
C. Lymphoma
D. Leukemia

Answer: according to the slide

37. How to monitor the response to iron treatment?


A. Ferritin
B. Hct
C. RBC
D. Reticulocyte count

Answer: D
Monitoring response:
a. Reticulocyte count will begin to increase after one week
➢ Hb normalizes by 10 g/L per week (if no blood loss)
● Iron supplementation required for 4-6 mo to replenish stores.
Reference: Toronto notes

38. Asymptomatic patient, known case of chronic gastritis, has positive occult blood stool and his Hb=9. You will manage him by:
A. IM iron
B. Oral iron
C. Erythropoietin
128
D. Blood transfusion

Answer: A (IV iron would be more appropriate answer)

39. Elderly man on NSAIDs developed dyspepsia. Endoscopy showed gastritis. Labs showed iron deficiency anemia with Hb= 9.
What is the treatment?
A. IV iron
B. IM iron
C. Erythropoietin
D. Oral Iron
Answer: A
Oral ferrous sulfate associated with a significantly higher risk of GI side effects than IV iron. Acquired malabsorption for iron with
autoimmune atrophic gastritis or Helicobacter pylori infection
Reference: http://www.uptodate.com/contents/treatment-of-the-adult-with-iron-deficiency-anemia

40. A male patient presented with symptoms. Labs showed 80% blasts with 20% Aurer rodes. What is the diagnosis?
A. CML
B. AML
C. CLL
D. ALL

Answer: B
Auer rods are a hallmark of acute myeloid leukemia.

41. Old guy with lymph node enlargement and B-symptoms. What is the treatment?
A. CHOP-R
B. ABVD
C. Rituximab
D. CHOP

Answer: ? (It is not clear from the question whether it is HL or NHL)


B symptoms: fever, night sweats, weight loss
● NHL:
Local disease (stage Ia and Ila): local radiation and small dose/course of chemotherapy
Advanced disease (stage III and IV, any "B" symptoms): combination chemotherapy with CHOP and rituximab, an antibody
against CD20 (CHOP-R)
HL:
Stage Ia and Ila: local radiation with a small course of chemotherapy
Stage III and IV or anyone with "B" symptoms: ABVD
*A = adriamycin (doxorubicin)
B = bleomycin V = vinblastine D = dacarbazine
Reference: Master The Board USMLE Step 2 CK

129
42. A patient with leukemia, his peripheral blood smear showed blasts more than 80 %, and Auer Rod bodies, what is the type of
leukemia?
A. AML
B. ALL
C. Hairy Cell leukemia
D. CML

Answer: A
Reference: FA for the USMLE step 2.

43. Which of the following can be found on smear in sickle cell disease?
A. Bite cells
B. Howell-Jolly bodies
C. Acanthocyte
D. Spherocyte

Answer: B
Reference: Master the Boards.

44. Most comman cause of death in sicke cell anima :


A. Aplastic crises
B. Seqseration crises
C. Acute chest syndrome
D. Parovirus b19

45. Sickle cell anemia false positive test because?


A. High protein level
B. Protein c
C. Protein d

Answer: a (high paraprotien like multiple myloma)


False positives may occur in patients with erythrocytosis, hyperglobulinemia, extreme leukocytosis or hyperlipidemia. Coarse
flocculation may occur in these samples due to elevated levels of total serum protein. These patient samples may be washed in
normal physiologic saline and centrifuged to minimize these problems.
False positives or false negatives may occur in patients with severe anemia hematocrit ≤ 15%.
False negatives may occur in infants under six months of age due to elevated levels of hemoglobin f.
False positives or false negatives may occur in patients with a recent blood transfusion.
Positive results may occur in patients with some rare sickling hemoglobin subtypes such as hemoglobin c harlem or hemoglobin c
georgetown.
References: http://education.questdiagnostics.com/faq/faq99
Https://www.streck.com/resources%5chematology%5csickledex%5c03_product_information%5c02_paper_sickledex_-
_screening_test_guidelines.pdf

130
46. Examination and evaluation show paraspinus edema and fluid collection -ve brucella titer and tuberculin test ,, what the
cause ?
A. Brucellosis
B. Breast cancer
C. Recurrent hodgkin lymphoma

Answer: recurrent hodgkins lymphoma?

47. Patient with osteoarthritis, you found that he developed anemia which type it would be:
A. Normochromic normocytic
B. Microcytic hypochromic (if with NSAID)
C. Macrocytic hyperchromic

Answer: a

48. Which type of anemia is associated with hyposplenism?


A. Sickle cell anemia
B. Thalassemia
C. Spherocitosis

Answer: a
Autosplenectomy is the physiological loss of spleen function (hyposplenism). It is associated with sickle cell anemia (chronic damage
to the spleen results in atrophy)
Reference: master the boards

49. Patient with past history of hodgkin lymphoma, but cured completely. Presented with back pain. Examination and evaluation
show para-spinous edema and fluid collection, -ve brucella titer and tuberculin test, what is the cause?
A. Brucellosis
B. Breast cancer
C. Recurrent hodgkin lymphoma

Answer: b
Long term complication of hodgkin lymphoma is solid tumor (breast cancer metastasis to bone) due to radiotherapy

50. Patient with past hx of hodgkin lymphoma, but cured completely. Presented with back pain examination and evaluation
show para-spinus edema and fluid collection -ve brucella titer and tuberculin test, what the cause?
A. Brucellosis
B. Breast cancer
C. Recurrent hodgkin lymphoma

Answer: b
Patients with hodgkin’s lymphoma who undergo chest radiotherapy have higher risk of developing breast cancer
Reference: http://www.ncbi.nlm.nih.gov/pmc/articles/pmc2409557/

131
51. Sickle cell anemia pt came complaining of cough & shortness of breath, there is splenomegaly, lab shows (wbc= low, hb= low,
retic= .04) what is the initial management of this case
A. Fluid & analgesia
B. Blood transfusion
C. Splenectomy

Answer: a
Http://bestpractice.bmj.com/best-practice/monograph/100/treatment/details.html

52. Pt with mild jaundice, hepatomegaly, fingers & toes parasthesia. Lab show (wbc= normal, b12=low, folate= normal, tbil=27,
dbil= 6)
A. Liver cirrhosis
B. Pernicious anemia
C. Folate def

Answer: b
Http://www.ncbi.nlm.nih.gov/pmc/articles/pmc4316289/
Http://bestpractice.bmj.com/best-practice/monograph/822/diagnosis/history-and-examination.html

53. Anemia+ neurological symptoms which Vitamin ?


A. B3
B. Folic acid
C. B12

Answer: c
rd
Reference: step-up to medicine 3 edition, page 330

54. A patient came from sudan 2w ago complaints of headache , fever, vomiting. What is the appropriate investigation to
diagnose this patient:
A. Peripheral blood smear
B. Stool culture
C. Blood culture
Answer: a

This is a case of malaria. Malaria is prevalent in tropical climates, parts of africa and the middle east. Patients present with fever and
chills, myalgias, headache, nausea, vomiting, and diarrhea. To diagnose you need to identify organism on peripheral blood smear 2.
Blood smear must have giemsa stain.
Reference: step up to medicine

55. Old age with lab results: hgb: low, lymphocyte: high, flow cytometry: different types of cd what is the treatment ?
A. Cyclophosphamide -
B. Rituximab +prednisolone -
C. Rituximab
Answer: ?
**the diagnosis is chronic lymphocytic leukemia
132
Reference: http://www.cancer.org/cancer/leukemia-chroniclymphocyticcll/detailedguide/leukemia-chronic-lymphocytic-treating-
treatment-by-risk-group
Http://www.oncolink.org/types/article.cfm?C=293&id=9590

56. 16 years old boy known case of sickle cell anemia presented to with painful right hip pain for several weeks (this was the
scenario and it was for several weeks ). What is the most likely diagnosis:
A. Avascular necrosis common in sicklers
B. Still’s disease
C. Tumor.
Osteomyelitis was not in the choices
Answer: avascular necrosis

57. Women (complain is not written). Her labs: high platelets. What’s the treatment?
A. Over-the-counter, low-dose aspirin to reduce blood clotting
B. Prescription medications to lower the risk of clotting or to reduce platelet production in the bone marrow
C. Platelet pheresis
Answer: not sure

58. Microcytic normochromic result in patient with pallor


A. Iron deficiency anemia Decrease in EP due to RCC
B. Hemolytic anemia
C. Folate deficiency
Answer: ????

59. Case of pt with malignancy , dr is planning to take the malignant cells and implant it in the pt dendritic cells to enhace
immunity against tumor cells what is this called :
A. Active immunotherapy
B. Passive immunotherapy
C. Weird name cant recall !!

60. Mcv high, ast high, no megaloblast cells:


A. B12 defe
B. Folate defeci
C. Alcohol abuse
Answer: c
Explanation:

133
61. Young patient presented with ptosis miosis depressed orbit and in the other eye there’s something he also has a neck mass
what is the most likely diagnosis:
A. Hodgkins lymphoma
B. Ewing's sarcoma
C. Wilms tumor
Answer: a

62. Factor vii (7) deficiency cause prolongation of :


A. Bleeding time
B. Pt
C. Aptt

Answer: b
Inherited factor vii (fvii) deficiency is a rare autosomal recessive hemorrhagic disorder. Clinical bleeding can widely vary and does not
always correlate with the level of factor vii coagulant activity measured in plasma. Factor vii is one of the vitamin k–dependent
coagulation factors synthesized in the liver. The prothrombin time (pt) is prolonged in factor vii (fvii) deficiency and the international
normalized ratio (inr) is elevated.
Http://emedicine.medscape.com/article/960592-workup

63. Iron overdose with bleeding per rectum


A. Lavage
B. IV anti dot
C. Chelating
Answer: c. Chelating. Treatment of a substantial ingestion is usually whole-bowel irrigation and chelation therapy with IV
deferoxamine. Merck manual
Http://www.merckmanuals.com/professional/injuries-poisoning/poisoning/iron-poisoning

64. How to stop bleeding in vwd?


A. Fresh frozen plasma
B. Vitamin. K
C. Platelets transfusion
Answer: desmopressin or factor viii concentrate. Step up to medicine

134
65. False positive sickling tests:
A. Hg c
B. Hg d
C. Hyperglobulinemia
Answer: c
False positives may occur in patients with erythrocytosis, hyperglobulinemia, extreme leukocytosis or hyperlipidemia. Coarse
flocculation may occur in these samples due to elevated levels of total serum protein. These patient samples may be washed in
normal physiologic saline and centrifuged to minimize these problems.false positives or false negatives may occur in patients
with severe anemia (<15% hematocrit).false negatives may occur in infants under six months of age due to elevated levels of
hemoglobin f.false positives or false negatives may occur in patients with a recent blood transfusion.

66. Long history about dic, lab showing fragmented RBC, low platelets . Which antibodies are target:
A. Cardiolipins
B. Adamts13
C. Glycoprotein

Answer: ?

67. Long term treatment of sickle cell is:


A. Folic acid
B. B- penicillin
C. Hydroxurea
Aanswer : a (depending on question) or c
Merck manual:
For long-term management the following interventions have reduced mortality, particularly during childhood:
Pneumococcal, haemophilus influenzae, influenza (inactivated, not live), and meningococcal vaccines
Early identification and treatment of serious bacterial infections
Prophylactic antibiotics, including continuous prophylaxis with oral penicillin from age 4 mo to 6 yr use of hydroxyurea and folate
supplementation supplemental folate, 1 mg po once/day, is usually prescribed.

68. Pt known case of g6pd low Hg what will u do?


A. Blood transfusion
B. Folic acid
C. Reassure
Answer: c
Most individuals with (g6pd) deficiency do not need treatment. Hemolysis is self-limited and often resolves in 8 to 14 days. Bmj
http://bestpractice.bmj.com/best-practice/monograph/704/treatment/details.html#expsec-3

69. Methotrexate overdose, treatment?


A. Folic acid
B. Folonic acid
C. Cobalamine
Answer: a reference: http://www.medscape.com/viewarticle/588229
135
70. Which type of anemia have high hemoglobin a2:
A. Scd
B. Thalassemia
C. Spherocytosis
Answer: b
Link: http://thalassemia.com/what-is-thal-beta.aspx#gsc.tab=0

71. Pt with starry sky pattern on biopsy > burkhits. What is the mutation?
A. C-myc gene
B. Abl2
C. Bcr- abl
Answer: a

72. Pt with anemia, high bilirubin, positive direct and indirect coombs
A. Photo of smear showing spherocytosis, what is the dx:
B. A-aiha Autoimmune hemolytic anemia
C. B-spherocytosis
Answer: a
Laboratory findings in patients with warm agglutinin aiha include hemolytic anemia of varying severity, a reticulocytosis in response
to the anemia, the presence of spherocytic red cells on the peripheral blood smear, and a positive direct antiglobulin (coombs) test.
These findings are discussed in detail below. An example of the baseline characteristics of 60 patients with warm agglutinin aiha,
seen in a french tertiary-care national referral center for adult autoimmune cytopenias is shown in the table (table 1) [16].

73. Patient on high dose aspirin present with melena endoscopy shows gastritis what to give?
A. Im iron
B. Oral iron (no IV in the choices)
C. In hx there is typical feature of hand foot syndrome:
Answer: sca

74. A 60-year-old male with history of lower back pain , constipation thirst low hemoglobin , low wbc , low plt , high Caand
lower spinal x-ray pic was attached
what is the next appropriate to be ordered:
A. Dexa
B. Protein electrophoresis
C. Parathyroid hormone
Answer: b
Multiple myeloma: m protein appears as a narrow spike in the gamma, beta, or alpha2 regions. Skeletal lesions (e.g., lytic lesions,
diffuse osteopenia, vertebral compression fractures) are present in 80% of patients. Anemia, pancytopenia, hypercalcemia, and
renal disease may be present.
Http://www.aafp.org/afp/2005/0101/p105.html

75. Iron deficiency anemia in 2 year old child Hg 9 what to give?


A. Oral
136
mild anemia: 9.5-11
Moderate: 8-9.5 or symp
severe: < 7

B. Iv
C. Transfusion
Answer: a
Oral iron therapy: is initially started at a dose of 3 mg/kg of elemental iron, given once or twice daily *. Parenteral iron
therapy: parenteral iron therapy should be reserved for patients with severe, persistent anemia who have proven intolerance to oral
supplements, malabsorption, or poor compliance to oral therapy. Iron dextran is the parenteral form most commonly used
preparation for pediatric patients *.
Blood transfusion: transfusion therapy is rarely necessary for severe ida, even with hemoglobin concentrations of 4 to
5 gm/dl. Transfusions should be reserved for patients in distress (heart rate greater than 160/min, respiratory rate greater
than 30/min, lethargy, not feeding well). Transfusions should be administered with caution to such patients, giving transfusion
volumes of 5 ml/kg over three to four hours to avoid inducing heart failure *.
Https://yhdp.vn/uptodate/contents/utd.htm?38/2/38949?Source=see_link *
Severe anemia: hemoglobin below 7.0 g/dl or hematocrit below 20%
http://www.who.int/nutrition/publications/micronutrients/guidelines_for_iron_supplementation.pdf

76. Pregnant patient on 32 weeks of gestation presented to antenatal care for the first time. She has symptoms of anemia with
orthopnea and pnd. On examination she is vitally stable with grade 3 pitting edema bilaterally labs shows low hemoglobin
low iron high ferritin low tibcwhat is the diagnosis
A. A-preeclampsia
B. Anemia
C. Heart disease
Answer: c
The patient has anemia of chronic disease that eventually leads to heart disease (pitting edema).

77. Aging amyloid where to find it ?


A. Heart
B. Kidney
C. Liver
Answer: b (not sure)
Amyloidosis
http://biomedgerontology.oxfordjournals.org/content/59/4/m361.full.pdf

78. 10 years old patient presented with general fatigue and severe bone pain in hands and feet. What is the diagnosis?
A. Sickle cell anemia
B. Alpha thalassemia
C. Thalassemia major

Answer: a
Sickle cell disease (scd) usually manifests early in childhood. The most common clinical manifestation of SCD is vaso-occlusive crisis.
It may present as dactylitis (bilateral painful and swollen hands and/or feet in children.

Reference: medscape: http://emedicine.medscape.com/article/205926-clinical

79. Pt with starry sky pattern on biopsy > burkitt's


Wts the mutation:

137
A. C-myc gene ( rt answer)
B. Abl2
C. Bcr- abl
Answer: a
Http://emedicine.medscape.com/article/1447602-overview

80. Which of the following tests will you order to check iron therapy response?
A. Reticulocyte
B. Ferritin
C. Tibc

Answer: a
Response to iron therapy can be documented by an increase in reticulocytes 5-10 days after the initiation of iron therapy. The
hemoglobin concentration increases by about 1 g/dl weekly until normal values are restored. These responses are blunted in the
presence of sustained blood loss or coexistent factors that impair hemoglobin synthesis.
[http://emedicine.medscape.com/article/202333-treatment#d13]

81. Patient has been diagnosed with lymphoma in the past and has received full course of chemotherapy. Now complaining of
painless facial swelling, cough and flushing, what is the diagnosis?
A. Superior vena cava obstruction
B. Inferior vena cava obstruction
C. Some type of facial tumor

Answer: a.
Superior vena cava (svc) obstruction

Reference: davidsone 22.

82. Pt with past hx of hodgkin lymphoma but cured completely, presented with back pain. Examination and investigations
showed a paraspinal mass, edema and fluid collection, negative brucella titer and tuberculin test, what is the cause?
A. Brucellosis

138
B. Breast cancer
C. Recurrent hodgkin lymphoma

Answer: c
http://www.ncbi.nlm.nih.gov/pmc/articles/pmc3892516/

83. Female came from 18 hours flight and she feel leg pain what to give?
A. Warfarin
B. Lmwh
C. Unfractionated heparin and warfarin
Answer: c
reference: uptodate.

84. Factor vii (7) deficiency cause prolongation of


A. Bleeding time
B. Pt
C. Appt
Answer b
Medscape

85. Prostate cancer patient with recurrent dvt. Best prophylaxis is:
A. Lmwh.
B. Unfractionated heparin.
C. Lmwh short-term therapy followed by warfarin.
Answer: a

86. Increase the hbf in sicklers, give:


A. Deferoxamine
B. Penicillamine
C. Folic acid

Answer:
hydroxyurea increase hgF

87. A patient known case of anemia on medications. Later he came complaining of dark stools. What is the medication?
A. Ferrous sulphate
B. Folic acid
C. Iron dextran

Answer: A

88. A patient with decrease in factor V. The etiology is:


A. Inherited
B. Immune
139
C. Infection

Answer: A
The inheritance of factor V deficiency is autosomal recessive.
Reference: http://emedicine.medscape.com/article/209492-clinical#b5

89. Peptic ulcer patient with Anemia, what you will do regarding his anemia?
A. Oral iron supplement.
B. I.M iron.
C. Blood Transfusion

Answer: B
Reference : uptodate

90. What of the following present in in peripheral blood smear of Sickle cell disease patient?
A. Howell jolly bodies
B. Spherocytes and….
C. Acanthocyte

Answer: A. Howell-Jolly Bodies (hyposplenism : in Sickle Cell disease)


Reference: Tornoto Notes Hematology (H4)

91. 3-4 cases about IDA


Answer: iron deficiency anemia
A. Oral iron therapy: initial therapy, gastrointestinal side effects are extremely common and may result in poor adherence.
B. Parenteral iron: for those unresponsive to or intolerant of oral iron, for pt with IBD, gastric bypass surgery.
C. Blood transfusion is not recommended unless anemia is sever or the patient has cardiopulmonary disease.
(step up to medicine)

92. Sickle cell anemia came with hepatosplenomegaly and low platelets- hgb -wbc
A. Splenectomy
B. Blood transfusion

Answer: b
Hepatic or splenic sequestration is used acutely for the treatment of severe anemia that cannot be adequately compensated by
increased red cell production.
Splenectomy is often removed after a person has survived such a crisis to try and prevent another one.
References: http://www.ncbi.nlm.nih.gov/pubmedhealth/pmh0012048/
Http://emedicine.medscape.com/article/205926-overview
Http://www.bloodjournal.org/content/123/15/2302?Sso-checked=true

93. Pt has a family hx of hemochromatosis presented with abdominal pain and fatigue, what to check?
A. Ferritin .
B. Transferrin
140
Answer: b
High transferrin saturation is the earliest evidence of hemochromatosis; a value greater than 60% in men and 50% in women is
highly specific. It is more sensitive tests for detecting early hemochromatosis. Ferritin level is less sesnsitive. Genetic tests for the
c282y and h63d mutations is conducted to confirm the diagnosis or to discover asymptomatic patients.
Reference: http://emedicine.medscape.com/article/177216-workup#c7

94. Test for sickle cell anemia?


A. Hb electrophoresis
B. Bone marrow aspiration

Answer: a

95. In factor vii deficiency what will be elevated?


A. Pt
B. Aptt

Answer: a
A normal aptt and a prolonged pt in a patient with a lifelong history of a tendency for mild or severe bleeding is consistent with the
diagnosis of factor vii deficiency or the presence of an inhibitor to factor vii.
Bleeding time is usually within the reference range.
With no significant clinical bleeding but a prolonged pt and a normal aptt, the patient has either mild factor vii deficiency or is taking
oral anticoagulants.
Reference: http://emedicine.medscape.com/article/209585-workup

96. Patient on warfarin 7 mg presented with melena. Inr was very high. What will you do?
A. Give vit. K
B. Lower the dose of warfarin

Answer: stop warfarin + Vitamin k

97. Test for sickle cell anemia?


A. Hb electrophoresis
B. Bone marrow aspiration

Answer: a
The best initial test is a peripheral smear. Sickle cell trait (as disease) does not give sickled cells. The most accurate test is the
hemoglobin electrophoresis.
Reference: master the boards usmle step 2

98. Hodgkin lymphoma with no fibrosis and eosinophils, reed sternberg cells, histocytes. Which type of hl is this?
A. Mixed cellularity Subtypes:
B. Nodular sclerosis - nodular scelorosis
: most common
Answer: a
- mixed
- rich : best prgnosis
- depleted: worst 141
- unspecified
Mixed cellularity — mixed cellularity hl (MCHl) is a heterogeneous category of classic hl with a diffuse or vaguely nodular growth
pattern without band-forming sclerosis or fibrosis. Fine interstitial fibrosis may be present, and classical diagnostic reed sternberg
cells are easily identified. Red sternberg cells are large with bilobate, double or multiple nuclei, and a large, eosinophilic nucleolus.
The background infiltrate is variable, but typically consists of eosinophils, neutrophils, histocytes, and plasma cells.
Reference: uptodate

99. In factor vii deficiency what is the lab abnormality we will detect?
A. Increased pt
B. Increased aptt

Answer: a
Prothrombin time (pt) to measure the functioning of factors i, ii, v, vii, and x
Active partial prothrombin time (ptt) to measure the functioning of factors viii, ix, xi, xii, and von willebrand factors

100.A sickler with vaso-occlusive crisis, dehydrated with a hemoglobin level of 3.5. What is the next step in management?
A. PRBCs transfusion
B. Analgesia and IV fluids

Answer: b
Vaso-occlusive crisis is treated with vigorous intravenous hydration and analgesics.
Http://emedicine.medscape.com/article/205926-treatment#d9

101.Pic of peripheral blood smear showing microcytic hypochromic RBCs & the patient came with symptoms of anemia. Direct
coombs test was +ve, indirect coombs test was + ve , diagnosis?
A. Hereditary spherocytosis

142
B. Autoimmune hemolytic anemia (aiha).
Answer: b
First aid:
Hereditary spherocytosis has a negative coombs test.
Aiha has a positive direct coombs test.

102.Cause of low sickling solubility test:


A. Anemia,
B. Presence of hemoglobin c
Answer: b

103.Cause of low sickling solubility test:


C. Anemia,
D. Presence of hemoglobin c
Answer: b

104.History of infection, hb low, wbc high what is the investigation :


A. Bone marrow biopsy
B. Hb elect because high wbc is caused by the infection
Answer: b (sca)
Ref: http://emedicine.medscape.com/article/205926-workup#c1

105.What is the most helpful test in sickle cell disease?


A. Hemoglobin electrophoresis.
B. Blood film.
Answer: a
Hemoglobin electrophoresis differentiates individuals who are homozygous for hbs from those who are heterozygous. It establishes
the diagnosis of SCD by demonstrating a single band of hbs (in hbss) or hbs with another mutant hemoglobin in compound
heterozygotes.
Http://emedicine.medscape.com/article/205926-clinical

106.Young patient presented with neck mass and itching. What is the diagnosis?
A. Hodgkin lymphoma
B. Lyme disease
Answer: a
- First aid usmle step 2ck.
- Https://en.wikipedia.org/wiki/hodgkin's_lymphoma

107.A patient came with weakness. What is the most likely deficiency?
A. Vitamin b1
B. Vitamin b3
Answer is a

143
- Vitamin b1 (thiamine) deficiency causes beriberi, which is characterized by severe burning dysesthesias, weakness and wasting
(distal more than proximal), trophic changes (shiny skin, hair loss), and acrodistal sensory loss in a graded fashion typical of
dying-back polyneuropathies.
- Vitamin b3 (niacin) deficiency causes pellagra, which is characterized by the 3 ds: (1) dermatitis, i.e., hyperkeratotic skin lesions,
particularly on hands, feet, face, and neck (sun-exposed regions); (2) diarrhea; and (3) dementia.
Source: http://emedicine.medscape.com/article/1171558-overview#a5

108.A young asymptomatic male came for routine checkup. Labs showed microcytic hypochromic anemia, what is the diagnosis?
A. Thalassemia trait (i chose it)
B. Iron def. Anemia (since he is young, not female, i excluded it)
It could be thalassemia because children who have untreated thalassemia major have ineffective erythropoiesis, decreased red cell
deformability, and enhanced clearance of defective red cells by macrophages (immune system cells). The result is a very
hypermetabolic bone marrow with thrombocytosis, leukocytosis and microcytic anemia in the young child prior to the enlargement
of their spleen.

109.An HIV patient came with bowel obstruction… i think in ct there is mass then took biopsy from it & show some type of cell ( i
do not remember the name of cell). What is the diagnosis?
A. Hodgkin
B. Tuberculosis
Answer: a
Both high and intermediate grade non-hodgkin's lymphomas and hodgkin's lymphoma (hl) have an increased incidence in patients
infected with hiv.
Http://www.medscape.com/viewarticle/444346
Classic hodgkin disease (hd) accounts for about 95% of all cases of hodgkin disease in developed countries.the cancer cells in classic
hd are called reed-sternberg cells (after the 2 doctors who first described them). These cells are usually an abnormal type of b
lymphocyte. Reed-sternberg cells are much larger than normal lymphocytes and also look different from the cells of non-hodgkin
lymphomas and other cancers.
Http://www.cancer.org/cancer/hodgkindisease/detailedguide/hodgkin-disease-what-is-hodgkin-disease

110.Cervical ln enlargement, hepatosplenomegaly, circumoral edema, acute presentation..what is the diagnosis?


A. Lymphoma
B. Angioedema
Answer: ???

111.An elderly asymptomatic patient’s tests show 90% lymphoblast.. What to do for him?
A. Observation
B. Chemo
Answer: b
This is probably a case of all and the treatment uses multi-agent dose-intense chemotherapy regimens in induction,
consolidation, and maintenance phases.

112.Patient complains of fatigue and weakness. Lab results showed:


144
- Hemoglobin : decreased
- MCH : decreased
- RDW : 13
What is your dx :
A. Thalassemia
B. Iron defiency anemia
Answer: a
Probably thalassemia since the RDW is normal

113.Sca, no joint swelling complains of lower leg ?, fever


A. Osteomyelitis
B. Vaso-occlusive crisis
Answer: b due to the fact that it's a sca patient

114.Patient presented with anemia, thrombocytopenia, hypercalcemia, picture of vertebral column?, how to confirm the
diagnosis?
A. Genetic
B. Serum protein electrophoresis
Answer: b, dx = multiple myeloma, serum electrophoresis identifies m-protein in about 80 to 90% of patients.
Http://www.merckmanuals.com/professional/hematology-and-oncology/plasma-cell-disorders/multiple-myeloma

115.Pts take methotrexate for something present with tiredness with lab result high mcv
What to give?
A. Iron
B. Folic acid
Answer: b

116.Vwd lab findings ?


A. Prolonged bleeding time
B. Prolonged pt
Answer: a

117.Patient presented with heamarthrosis what is the defect:


A. Factor 8
B. Factor 7
Answer: a

118.106. Burkitt's lymphoma associated with?


A. Epstein barr virus "EBV"
B. T lymphocyte infiltration

Answer: a

145
119.Pregnant patient with anemia, mcv high, what will you give her?
A. Iron
B. Foliate
Answer: b- foliate

120.Child had constipation after few days he develop bloody urine and lower abdominal pain
A. Autoimmune hemolytic anemia
B. Uti
Answer: a

121.Case of sickle cell anemia. What is most important on long term?


A. Penicilline
B. Hydroxyurea
Answer: b

122.Known case of SCD complaining from right shoulder pain xray show head humerus necrosis what's long term therapy
A. Hydroxyurea
B. Penicillin
Answer: a
The only drug currently approved by the us food and drug administration (fda) for the treatment of SCD is hydroxyurea. For frequent
and severe pain, long-term hydroxyurea is currently the accepted treatment.
Link: http://emedicine.medscape.com/article/205926-treatment#d8

123.What increase the absorption of iron?


A. Folic acid
B. Vitamin c
Answer: b
Ascorbic acid or vitamin c occurs naturally in vegetables and fruits, especially citrus. Ascorbic acid can also be synthesized for use in
supplements. Ascorbic acid enhances the absorption of nutrients such as iron. In studies about effects of ascorbic acid on iron
absorption, 100 milligrams of ascorbic acid increased iron absorption from a specific meal by 4.14 times.
Link: http://www.irondisorders.org/diet/

124.Polycythemia vera with blurred vision and headache what the cause of these sx?
A. Hypovolemia
B. Hyperviscosity
Answer: b
Symptoms of polycythemia vera (pv) are often insidious in onset, and they are often related to blood hyperviscosity secondary to a
marked increase in the cellular elements of blood. Subsequent sludging of blood flow and thrombosis lead to poor oxygen delivery,
with symptoms that include the following:
Headache, dizziness, vertigo, tinnitus, visual disturbances, angina pectoris, intermittent claudication.
Link: http://emedicine.medscape.com/article/205114-clinical

125.Case of sca. What is the most important on long term?


A. penicilline
146
B. hydroxyurea

Answer: b
The use of hydroxyurea is a mainstay in the overall management of individuals with scd, since it reduces the incidence of acute
painful episodes and hospitalization rates, and prolongs survival.

126.4 years old SCD w/ recurrent voc what is best management for future improvement?
A. Hydroxyurea
B. Multiple blood transfusions
Answer: a
Hydroxyurea decreases crises in patients with severe sickle cell disease
Https://www.cdc.gov/ncbddd/sicklecell/recommendations.html

127.Generalized cervical lymphadenopathy + mild tenderness + low grade fever. What's the most likely diagnosis:
A. Lymphocytic lymphoma
B. Hodgkin's lymphoma

Answer: b
Non-hodgkin's lymphoma is much more common than hodgkin's disease.
And the incidence of non-hodgkin's lymphoma has been steadily increasing over the last decades unlike hodgkin’s which only
accounts for 1% of cancers in the united states and is declining in incidence.

Reference: http://www.medicinenet.com/hodgkins_and_non-hodgkins_lymphoma_comparison/article.htm
128.The most common cause of hemarthrosis
A. ITP
B. Von willibrand disease

Answer: b

Reference: step-up to medicine (p 340-341)


Hemarothosis has a lot of causes but according to the choices von willibrand disease (coagulation disease) will be the best answer.
Hemarthrosis rarely happen with ITP (platelet disorder). Patients with hemophilia they mostly will experience hemarthrosis.

129.Generalized cervical lymphadenopathy + mild tenderness + low grade fever. What's the most likely diagnosis?
A. Small lymphocytic lymphoma (presented by generalized lymphadenopathy)
B. Hodgkin’s lymphoma

Answer: b
Hodgkins lymphoma presents with asymptomatic lymphadenopathy (above the diaphragm in 80% of patients) and it moves
between lymph node groups in an orderly fashion
Http://emedicine.medscape.com/article/201886-clinical

130.In factor v11 defiency what elevated :


A. Pt
B. Ptt
147
Answer is: a
The prothrombin time (pt) is prolonged in factor vii (fvii) deficiency and the international normalized ratio (inr) is elevated. The
activated partial thromboplastin time (aptt) is within the reference range in isolated factor vii deficiency.
( http://emedicine.medscape.com/article/960592-workup )

131.Hx of infection, hb low, wbc high, what is the investigation :


A. Bone marrow biopsy
B. Hb electrophoresis
Answer: a
This may be a case of hodgkin lymphoma but there are not enough information to make the diagnosis.

132.False positive sickle test ?


A. Hb c
B. Hb d
Answer: a
some say there was (protien) in the answers, if there's so it's the choice
false-positive results can be obtained if nucleated red blood cells are present in the peripheral blood or the patient has marked
hypergammaglobulinemia (such as in multiple myeloma).

133.Post-transfusion fever how to prevent?


A. Transfusion filter depleted leukocyte
B. Premedication with antupyretics

Answer :a (uptodate)

134.Elderly patient presented with typical symptoms of multiple myeloma. X ray showed lytic lesions, positive M protein and
hypercellular bone marrow. What other findings can be found in the blood?
A. Increase peripheral blood B cell
B. Rouleaux formation

Answer: B
148
Reference: Pubmed and Medscape
Http://emedicine.medscape.com/article/204369-workup#c6

135.A patient with Vitamin B 12 deficiency. What is the type of gastric cell that will be affected?
A. Chief cell
B. Parietal cell

Answer: B
Patient with pernicious anemia is usually directed against the intrinsic factor or parietal cell themselves. Parietal cell deficiency as it’s
responsible for intrinsic factor synthesis, which is required for vit. B12 to be absorbed in terminal ileum
“Autoimmune metaplastic atrophic gastritis – A major component of PA (pernicious anemia) is chronic atrophic gastritis, which is
associated with autoantibodies directed against gastric parietal cells in approximately 90 percent of patients with PA.”
Reference: b12-and-folate
deficiency?Source=machinelearning&search=megaloblastic+anemia&selectedtitle=3~91&sectionrank=1&anchor=H2125777#H2125
777

136.Hb F is present, Hb A2 is present also, what is the diagnosis?


A. Thalassemia minor.
B. Thalassemia major.

Answer: B (if Hb A was absent)

Beta-thalassemia trait: Hb A2 (Hb A2) values of 3.5% to 9% and Hb F is usually between 5% to 15%
Beta-thalassemia major: Hb F may be 30% to 90% or even more of the total hemoglobin
Reference: http://www.mayomedicallaboatories.com/test-catalog/Clinical+and+Interpretive/83341

137.A case of low platelets, low RBCs and low wbcs. What is the diagnosis?
A. Iron def anemia
B. Aplastic anemia

Answer: B
Aplastic anemia (AA) is characterized by diminished or absent hematopoietic precursors in the bone marrow, most often
due to injury to the pluripotent stem cell (Decrease RBCs, wbcs, Platelets).
Reference: Uptodate.

138.In hemolytic anemia which enzyme will be noticed?


149
A. Erythropoietin
B. Bilirubin

Answer: B

139.A known case of sickle cell disease presented with unilateral lower limb pain since (short period:acute). Vital signs:
Tachycardia, Fever 38.3. The range of movement is intact with no signs of inflammation over the limb. What is the diagnosis?
A. Vaso-occlusive Crisis
B. Osteomylitis

Answer: A
Findings secondary to vaso-occlusion.
A. Painful crises involving bone—bone infarction causes severe pain. This is the
Most common clinical manifestation.
• Bone pain usually involves multiple sites (e.g., tibia, humerus, femur). It may
Or may not be bilateral.
• The pain is self-limiting and usually lasts 2 to 7 days.
B. Hand–foot syndrome (dactylitis).
• Painful swelling of dorsa of hands and feet seen in infancy and early childhood
(usually 4 to 6 months).
• Often the first manifestation of sickle cell disease.
• Caused by avascular necrosis of the metacarpal and metatarsal bones.
Precipitated by infection , fever , dehydration , pregnancy and alcohol
Reference : step up to medicine

140.Sickle cell disease patient with multiple gall bladder stones. What is the Best thing to do?
A. Hydroxyurea
B. Cholysystectomy

Answer: B
If the patient does not have symptoms, no treatment is usually necessary. If there is recurrent or severe pain from gallstones, the
gallbladder may need to be removed. Minimally invasive procedures (using laparoscopy) reduce possible complications
Reference: https://umm.edu/health/medical/reports/articles/sickle-cell-disease

141.In hemolytic anemia which enzyme will be noticed?


A. Erythropoietin
B. Bilirubin

Answer: B
Standard blood studies for the workup of suspected hemolytic anemia include the following:
Complete blood cell count
Peripheral blood smear
Serum lactate dehydrogenase (LDH) study
Serum haptoglobin
Indirect bilirubin
Reference : http://emedicine.medscape.com/article/201066-workup

150
142.Patient has splenomegaly and teardrop RBC on blood film, what is the diagnosis?
A. ITP
B. Myelofibrosis
Answer: b
Ref: http://www.pathologystudent.com/?P=607

143.In factor V11 defiance what elevated?


A. Pt
B. Ptt......

Answer : A
A normal aptt and a prolonged PT in a patient with a lifelong history of a tendency for mild or severe bleeding is consistent with the
diagnosis of factor VII deficiency or the presence of an inhibitor to factor VII.
Http://emedicine.medscape.com/article/209585-workup

144.Hemolytic anemia patient with schistiocyte normal apt and bt what is management?
A. Acyclovir.
B. Antibiotic if case of hus tx supportive

145.Sickle cell anemia came with hepato splenomegaly And low platelets- HGB -WBC?
A. Splenectomy
B. Blood transfusion
Answer: A

146.Pt with anemia, high billirubin, positive direct and indirect coombs, Photo of samear shoing spherocytosis, wts the dx:
A. AIHA Autoimmune hemolytic anemia (rt answer since +ve coombs)
B. Spherocytosis
Answer : A

147.Sx of hydrop fetalis or HB h


A- normal 2 beta and abnormal 4 alfa
B- opposite of the above

Answer: A

148.Pt with Vitamin b 12 deficiency what gastric cell type will be affected?
A. Chief cell
B. Parietal cell
Answer: b
Reference: USMLE step 1

149.Young patient presents with cough and chest pain. Chest x--ray showed lung infiltrates. Cbc: anemia with high reticulocytes
count (10%) and leukocytosis, what is the diagnosis?
151
A. Sickle cell anemia
Answer: a
Sickle cell anemia presenting with acute chest syndrome supported by clinical and radiological evidence

150.Sickle cell patient experienced pain in his hands (descriptions of vaso-occlusive crisis). What is the most probable
diagnosis……splenic sequestration
Answer:?? Need more details
Vasooclusive: because deoxygenated hemoglobin s (hbs) becomes semisolid, the most likely physiologic trigger of vaso-occlusive
crises is hypoxemia. This may be due to acute chest syndrome or accompany respiratory complications.
Read about sickle cell crisis
Http://emedicine.medscape.com/article/205926-clinical

151.What is the mechanism of action of heparin?


Answer: heparin binds to the enzyme inhibitor antithrombin iii (at)
Https://en.m.wikipedia.org/wiki/heparin

152.A young patient is continuously bleeding post operatively. Coagulation profile showed high aptt, otherwise normal, which
factor is deficient?
A. Factor viii
Answer: a
This patient has hemophilia a which characterized by factor viii deficiency. Those patients usually have normal pt and thrombin time,
fibrinogen, bleeding time. Some cases may have major hemorrhage after surgery or trauma
First aid usmle step 2ck.

153.A young patient complains of a sudden onset of dyspnea. Labs show decreased mcv, normal wbc, increased platelets.
Peripheral smear shows microcytic hypochromic anemia:
A. Iron deficiency anemia- thalassemia
The answer: is ida

154.Patient with multiple blood transfusion and jaundice + osmotic fragility test…….
Answer: spectrin-ankyrin binding deficiency (hereditary spherocytosis)
The osmotic fragility test (oft) is used to measure erythrocyte resistance to hemolysis while being exposed to varying levels of
dilution of a saline solution.conditions associated with increased osmotic fragility include the following:
Hereditary spherocytosis
Autoimmune spherocytosis
Poisoning
Severe burns
The following conditions are associated with decreased fragility:
Thalassemias
Iron deficiency anemia
Sickle cell anemia
Http://emedicine.medscape.com/article/2085814-overview#a2

152
155.Most common presentation of hemophilia?
Patients with hemophilia bleed into tissues (eg, hemarthrosis, muscle hematomas, retroperitoneal hemorrhage).
Http://www.msdmanuals.com/professional/hematology-and-oncology/coagulation-disorders/hemophilia

156.Mediterranean blood disease?


Thalassemia (also known as mediterranean anemia, or cooley's anemia) is a genetic blood disease.

157.Case with CBC , lab results , increaseIgG, increase bun , creatinine, diagnosis : answer : multiple myeloma

158.In vwd bleeding time will be increased while pt is normal. Aptt may be increased due to concomitant decrease in factor viii.
Http://emedicine.medscape.com/article/206996-workup#c9

159.Case simple for iron deficiency anemia: with it's parameter?


Iron deficiency anemia
http://emedicine.medscape.com/article/202333-workup#showall

160.Spoon shape nail, what type of anemia?


A. Iron deficiency
Answer: a

Http://emedicine.medscape.com/article/1447602-overview#a4

161.Pt present CBC and differential results are low


Aplastic anemia. Bmj http://bestpractice.bmj.com/best-practice/monograph/93/diagnosis/guidelines.html

162.Post-surgical dvt what is the best treatment regarding to cost and effectiveness
1. Un-fractioned heparin
Rivaroxaban represents a cost-effective choice compared with lmwh/vka treatment, regardless of the required treatment duration.
Http://www.ncbi.nlm.nih.gov/pubmed/26074735

163.Pt presented with hemoytic anmia??


A. Increase uncongigated bilirubin
Http://bestpractice.bmj.com/best-practice/monograph/98/diagnosis/tests.html

153
164.Mechanism of aspirin?
Dose dependent:
Low doses (typically 75 to 81 mg/day) are sufficient to irreversibly acetylate serine 530 of cyclooxygenase (cox)-1. This effect inhibits
platelet generation of thromboxane a2, resulting in an antithrombotic effect.
Intermediate doses (650 mg to 4 g/day) inhibit cox-1 and cox-2, blocking prostaglandin (pg) production, and have analgesic and
antipyretic effects. Uptodate. Http://www.uptodate.com/contents/aspirin-mechanism-of-action-major-toxicities-and-use-in-
rheumatic-diseases#h3

165.Sickle cell disease with hip pain:


A. AVN . Step up to medicine

166.Old female live alone change her dietary habit what is the cause of anemia?
A. Fe deficiency

167.Sca with recurrent cholecystitis?


A. Cholecystectomy
Reference: http://emedicine.medscape.com/article/205926-treatment#d17

168.Craving ice pt with anemia:


154
A. Pagophagia
Answer: a
Link: http://www.mayoclinic.org/diseases-conditions/iron-deficiency-anemia/expert-answers/chewing-ice/faq-20057982

169.Woman (complaint). Lab: high platelets. Treatment?


A. Plateletpheresis
Best initial treatment hydroxyurea.

170.Child with hand and foot swelling and pain diagnosis?


Answer: sca
Http://emedicine.medscape.com/article/205926-overview

171.Pt with large tongue, anemia, numbness in feet. What is the best treatment?
Answer: Vitamin b12

172.About hemolytic anemia dx with peripheral blood smear shows microspherocytosis


Answer: hemolytic anemia

173.Which is feature of vwd?


Answer: prolonged bleeding time

174.Pt. With sever ida hg=10 range they put (120-something) what to do first?
A. Blood transfusion
Answer: ( no iron trial in other choices )

175.Yellow pt all normal only high indirect bilirubin (unconjugated) and alkaline phosphatase? What the diagnosis?
I think the most likely cause is a hemolytic disorder associated with bone infiltration such as thalassemia

176.Crescentic cell cause?


Answer: hemolytic anemia

155
177.Old lady living alone presents with anemia. What is the most likely cause?
A. B12 deficiency
Answer: a

178.Nhl staging (ann arbor) patient with b symptoms and lymph nodes from the neck to the para aortic with spleen involvement?
A. Cotswold modification of ann arbor staging system. Medscape http://emedicine.medscape.com/article/2007081-overview

179.Patient from africa presented with symptoms of burkitt lymphoma what gene associated with:c-myc it is characterized by
the translocation and deregulation of the c-myc gene on chromosome 8.
Reference: http://emedicine.medscape.com/article/1447602-overview#a3

180.Scenario of leukaemia with auer rods ? Aml

181.Patient with low hemoglobin and low mcv, which of the following will confirm the diagnosis?
A. Iron level and tibc

Answer: a
[https://arupconsult.com/sites/default/files/anemia%20testing%20algorithm.pdf]

182.Pt with iron deficiency anemia given iron supplements + lab hb dec, mcv dec, what you will give him?

156
Reference: uptodate

183.Leukemia bleeding is characterized by:


Multiple choices about abnormal bleeding test results.
anemia, thrombocytoenia, leukocytosis.

Answer:?!

184.Sickle cell disease with hip pain?


Answer: AVN

157
Reference: step up to medicine

185.Pt with hodgkin lymphoma with auer rods no fibrosis there is eosinophils baso ..... What is the of hodgkin lymphoma?
Answer: mixed type

186.Splenectomy case, what vaccines should be given afterwards or prior to spleen removal?

Answer: against encapsulated organisms vaccinate


Influenza vaccine each year to protect against seasonal flu.
Tdap vaccine to protect against whooping cough and tetanus.
Hib vaccine to protect against haemophilus influenzae type b (hib) if you were not previously vaccinated with the vaccine.
Pneumococcal vaccines (both types) to protect against pneumonia and other pneumococcal disease.
Meningococcal vaccines (both types) to protect against meningitis.
Zoster vaccine to protect against shingles if you are 60 years and older.
Hpv vaccine series to protect against human papillomavirus if you are a man up to age 21 or woman up to age 26.
Mmr vaccine to protect against measles, mumps, and rubella.
Varicella vaccine to protect against chickenpox.

Reference: cdc, http://www.surgicalcriticalcare.net/guidelines/splenectomy_vaccines.pdf

187.The best diagnosis tool for thalassemia is:

Answer : haemoglobin electrophoresis is the gold standard


Diagnosis of thalassemia:
Evaluation for hemolytic anemia (microcytic hemolytic anemia, RDW normal, serum bilirubin, iron, and ferritin levels are
increased, RBC count is elevated relative to hb because the cells are very microcytic).
Peripheral smear (diagnositic test), (nucleated erythroblasts, target cells, punctate and diffuse basophilia)
Electrophoresis (elevation of hb a2 and hb f, hb h or bart’s fractions
DNA testing (prenatal diagnosis)
Bone marrow. (erythroid hyperplasia)
X-rays (cortical thinning skull, widened diploic space, a sun-ray appearance of the trabeculae, and a granular or ground-glass
appearance)
Long bone x-ray (cortical thinning, marrow space widening, and areas of osteoporosis)
Vertebral x-ray (the bodies may have a granular or ground-glass appearance)
The phalanges may appear rectangular or biconvex.

Reference: merck manual, uptodate

188.ITP case management:


Answer:
Corticosteroids 


158
IV immune globulin—saturates the reticuloendothelial system binding sites for 
platelet-bound self-immunoglobulin, so there is
less platelet uptake and destruction by the spleen 

Splenectomy—induces remission in 70% to 80% of the cases of chronic ITP 

Platelet transfusions 1 u/10 kg—for life-threatening and serious hemorrhagic episodes. 1 u of platelets to increase count of a
70-kg adult by 5-10,000/mm and an 18-kg child by 20,000/mm
Two new drugs, romiplastim and eltrombopag, for splenectomy- 
resistant patients. Both work as thrombopoietin receptor
agonists to increase platelet production. 

Reference: step-up of medicine, medscape

189.Patient with high aptt what mechanism of action of that drug?


" unfractionated heparin" antithrombin
Answer:
Low dose heparin : inactivates factor xa and inhibits conversion of prothrombin to thrombin
High dose heparin : inactivates factors ix , x , xi , xii and thrombin and inhibit conversion of fibrinogen to fibrin.
Antidote is protamine sulfate

190.Aspirin inhibit which product formation:


Answer: thromboaxan a2

191.Leukemia pt with 80 blast and Auer bodies in peripheral smear what type of leukemia?
Answer: Acute Myeloid Leukemia (AML).
Peripheral blood smear shows : circulating blasts with Auer rods (azurophilic granules) are pathognomonic for AML.
Bone marrow aspirate show : blast count >20%.
Reference: Toronto Notes Hematology (H38)

192.Case suggest of multiple myeloma what can you find ?


A- Bence jones protein
Answer :a
In multiple myeloma light chains are filtered at the glomerulus and appear as bence-jones proteins in the urine (monoclonal light
chains). The clinical features include:bone disease: pain (usually back), bony tenderness, pathologic fractures,anemia, infections ,n/v,
confusion, constipation, polyuria, polydipsia ,bleeding ,amyloidosis.

AML: arabinoside (AraC) and either daunorubicin or idarubicin.


193.Treatment of acute myeloid leukemia: ALL is daunorubicin, vincristine, and prednisone.
Promyelocytic leukemia is managed with the addition of the vitamin A derivative all-trans-retinoic acid (ATRA)
Answer: induction of remission by cytarabine
Reference: http://emedicine.medscape.com/article/2004793-overview

194.Elderly with high calcium and low back pain … :


A. Multiple myeloma

Incomplete question
Explanation: presenting symptoms of multiple myeloma (mm) include bone pain (most common, especially lumbar), pathologic
fractures, weakness, anemia, infection (often pneumococcal), hypercalcemia, spinal cord compression, or renal failure.

159
Reference: http://emedicine.medscape.com/article/204369-overview

195.Which type of anemia associated with chemotherapy and radiotherapy?


Answer: aplastic anemia
Http://www.mayoclinic.org/diseases-conditions/aplastic-anemia/basics/treatment/con-2001929

196.Vwb deficiency which will be affected ?


In the great majority of cases, vwd is an inherited condition. The vwf gene is located near the tip of the short arm of chromosome
12. The gene is composed of 52 exons and spans a total of 180kb of the human genome; therefore, it is similar in size to the fviii
gene. Expression of the vwf gene is restricted to megakaryocytes, endothelial cells, and, possibly, placental syncytiotrophoblasts. A
partial, nonfunctional duplication (pseudogene) is present on chromosome 22.
Vwf exists as a series of multimers varying in molecular weight between 0.5-kd (dimer) and 20 million kd (multimer). The building
blocks of multimers are dimers, which are held together by disulfide bonds located near the c-terminal end of each subunit

197.Picture of child with macular rash all over the body with hx of fever and carditis, what's the dx:
A. Kawasaki
Answer: a. Toronto:

198.A vegetation patient with lab showing microcytic hypochromic anemia what is the dx :
a. Iron deficiency

Answer: a

160
Another clue in this scenario is that she is a vegetarian
Reference: toronto notes

199.When to give both heparin and fresh frozen plasma?


Disseminated intravascular coagulation (dic): uncontrolled release of plasmin and thrombin leading to intravascular coagulation and
depletion of platelets, coagulation factors and fibrinogen this condition should be recognized early and treat underlying disorder..•
in hemorrhage: replacement of hemostatic elements with platelet transfusion, frozen plasma, cryoprecipitate• in thrombotic phase:
unfractionated heparin or lmwh
reference: toronto notes

200.2 cases of anaemia. (interpretation)


Https://www.arupconsult.com/algorithms/anemia.pdf

201.Thrombocytopenia and recurrent infection , male and uncle has the same condition ?
Answer: wiskott–aldrich syndrome

202.Purpura ( immmnue,, henoch purpura)


A. Lesion in mm

203.Infective rheumatic fever. ! Rash ,


A. Retuc count in

204.Hemolytic anemia:
A. Unconjugated bilirubin
161
205.Question about ( autoimmune hemolysis)
Destruction of red blood cells by autoantibodies may occur suddenly, or it may develop gradually. In some people, the destruction
may stop after a period of time. In other people, red blood cell destruction persists and becomes chronic. There are two main types
of autoimmune hemolytic anemia:
- warm antibody hemolytic anemia
- Cold antibody hemolytic anemia
Https://www.merckmanuals.com/home/blood-disorders/anemia/autoimmune-hemolytic-anemia

206.A patient presented with anemia and glossitis. He also complained of paresthesia
A. Pernicious anemia

Answer: a .
These features are caused by Vitamin b12 deficiency. The presentation of the patients include:confusion, delirium, dementia cranial
nerves (rare) ,optic atrophy cord (irreversible damage) ,subacute combined degeneration,decreased vibration sense, proprioception,
and 2-point discrimination,spastic weakness, hyperactive reflexes, peripheral neuropathy (variable reversibility).

207.Scenario about extremities numbness and stomatitis ?


A. Vitamin b12 deficiency
Question and choices incomplete
Explanation: subacute combined degeneration is due to vitamin b12 deficiency; affect the brain and spinal cord white matter. In
early stages, decreased position and vibratory sensation in the extremities, accompanied by mild to moderate weakness and
hyporeflexia. In later stages, spasticity, extensor plantar responses, greater loss of position and vibratory sensation in the lower
extremities, and ataxia emerge.
Reference: http://www.msdmanuals.com/professional/nutritional-disorders/vitamin-deficiency,-dependency,-and-toxicity/vitamin-
b-12

208.Long scenario about nonmegaloblastic macrocytic anemia ?


A. Alcohol consumption
Question and choices incomplete
Explanation: a history of alcohol abuse may be an important clue to the cause of the increased mean cell volume (mcv); long-term
use of alcohol may have a direct toxic effect on the bone marrow, causing macrocytosis. Alcohol may also cause anemia by causing
folate deficiency.
Reference: http://emedicine.medscape.com/article/203858-overview#a7

209.Case of iron deficiency anemia lab showed typical microcytic with high tibc , mechanism :
A. Decrease RBCs more than serum volume )
TIBC, RDW increase

210.Two cases about anemia. (interpretation)

162
211.Peripheral blood smear about leukemia forgot it

212.Treatment of acute myeloid leukemia:


Answer: induction of remission by cytarabine

213.Risk factors for thrombosis

214.Pt with pic of anemia and thrombocytopenia - blood film showed schistocytes what dx ?
Answer : DIC

163
215.Tttx of ITP
Answer:
Http://www.uptodate.com/contents/immune-thrombocytopenia-ITP-in-adults-initial-treatment-and-
prognosis?Source=search_result&search=ITP&selectedtitle=2~150

216.4 cases about IDA


Answer:
Http://www.uptodate.com/contents/causes-and-diagnosis-of-iron-deficiency-anemia-in-the-
adult?Source=search_result&search=IDA&selectedtitle=1~150

217.Case of fanconi syndrome

164
Answer:
Http://emedicine.medscape.com/article/981774-overview

218.Pt was anemic, being given blood transfusion. Developed fever, chills, burning at site of IV line, what to do:
A. Stop transfusion and give crystalloids
Answer : A
Stop transfusion as soon as reaction is suspected
Infuse normal saline via IV site using new tubing; aggressive fluid resuscitation is ordered to maximize renal perfusion.
Monitor vital signs and urine output
Examine blood bag to see if patient was the intended recipient.
Return donor blood back to the blood bank to determine whether the corrrect unit of blood was administered.
IV furosemide and low-dose dopamine may be ordered to increase renal perfusion (maintain urine output at 30-100 ml/hr).

If there is any suggestion (eg, clerical mistake, hypotension, pink plasma or urine) that an AHTR is possible, and even before
laboratory tests are available, generous fluid replacement with saline (100 to 200 ml/hour) to support a urine output above 100 to
200 ml/hour should be initiated immediately, in an attempt to prevent the development of acute oliguric renal failure. The beneficial
effect of urinary alkalinization in patients with marked hemoglobinuria is uncertain.
Vigorous supportive care is also important for the treatment of ahtrs. If there has been massive hemolysis and clinical or laboratory
signs of disseminated intravascular coagulation, cautious and early heparinization (10 units/kg per hour) for the next 12 to 24 hours
may be of value, although there are no recent studies on the effectiveness of this intervention.
A vasopressor, such as low-dose dopamine, may be required. If massive intravascular hemolysis has already occurred, hyperkalemia
is likely, and cardiac monitoring and acute hemodialysis may be required. Accordingly, the patient's renal function and coagulation
profile (eg, prothrombin time, partial thromboplastin time, fibrinogen, platelet count) should be monitored frequently.

219.Pt with large tongue, anemia, numbness in feet. What is the best treatment:
A- Vitamin B12
Answer: A

220.Patient with high Ca and low IgAIgm what is the diagnosis (this Q was already submitted by one of our colleagues but the
answer in the (gathered smle) had multiple myeloma and an explanation to a subtype of it. In the exam they offer you both
multiple myeloma and it's subtype as choices and you have to choose between the two

http://cancer.ucsf.edu/research/multiple-myeloma/

221.Multiple myeloma in the spine


(Histopath report)
Answer:
Dx of multiple myeloma requires the following:
Bone marrow aspirate or biopsy showing that at least 10% of the cells are plasma cells or the presence of plasma cell tumor (
plasmacyte ) plus at least one of the following
Evidence of the damage to the body by plasma cells ; bone damage , kidney failure , anemia or high calcium .
Defection of one of the following ; > 60% plasma cells in bone marrow, free light chain ration of 100 or more, MRI showing
involvement of more than one lesion in the bone or bone marrow
Reference: http://www.uptodate.com/contents/multiple-myeloma-symptoms-diagnosis-and-staging-beyond-the-
basics?Source=outline_link&view=text&anchor=H12#H12

165
222.Chemotherapy can case which type of anemia?
A. Aplastic anemia
Answer: A
It will cause bone marrow suppression.

223.Old lady living alone for 5 years. She has memory problem and looks pale.
A. B12 deficiency
Answer: A
This patient is “pale”, this is a hint for anemia, vit-B12 deficiency can cause memory problems plus anemia.
That’s why if you have old patient with memory problems, before you say he has dementia, you should exclude: Hypothyroidism and
vit-B12 deficiency.
Reference: Step-up to Medicine.

224.Patient on warfarin 7 mg presented with melena. INR was very high. What will you do?

Answer:?
The treatment of excessive anticoagulation is based on the level of the INR, the presence or absence of bleeding, and clinical
circumstances. Reversal of Warfarin sodium anticoagulation may be obtained by discontinuing Warfarin sodium therapy and, if
necessary, by administration of oral or parenteral vitamin K1.
Reference: http://www.drugs.com/pro/warfarin.html

Reference: Toronto Notes.

225.Mediterranean blood disease


A. Thalassemia
Answer: A
Reference: Toronto Notes

226.What is the most common symptom of factor 8 deficiency?


A. Hemarthrosis
Answer: A
In severe hemophilia A, spontaneous joint or deep-muscle bleeding is the most frequent symptom.
Reference: http://www.ncbi.nlm.nih.gov/books/NBK1404/

166
227.Osmotic fragility test.
A. Hereditary spherocytosis
Answer: A

228.What type of anemia is associated with rheumatoid arthritis?


A. Normocytic normochromic
Answer: A

229.70 years old man complaining of back pain. Calcium: High, iga,igm and igg: Low (i.e. Three types of immunoglobulins are low).
What is the diagnosis? (They didn’t give any other details)
Answer: ?
Multiple myeloma
Reference: http://www.cancernetwork.com/hematologic-malignancies/non-secretory-myeloma-clinician%E2%80%99s-
guide#sthash.nmvyh621.dpuf
Monoclonal Gammopathy of Unknown Significance (MGUS) is the least aggressive subclass of plasma cell dyscrasia. Patients have
only a small abnormal protein spike (<3.0 gm/dl and <2.0 gm/dl forIgGand IgAand igm, respectively), minimal or no bone marrow
involvement (<10% plasma cells), no bony involvement, normal blood counts and usually normal levels of unaffected antibodies. The
urine is usually free of monoclonal protein, however, it is not unusual to find small amounts of monoclonal light chains in the urine
of MGUS patients. Patients with MGUS have a 20-25% chance of developing multiple myeloma or a related lymphoproliferative
disorder. The remainder lead a normal life. Reference: https://www.clevelandclinic.org/myeloma/mm-pt.htm

230.A patient working in a new place up the hill. Recently, he had raised hemoglobin. What is the reason?
A. Due to increased production of erythropoietin.
Answer: A

231.Read string cells


A. Hodgkin lymphoma
Answer: A
The histological hallmark of HL is the presence of Reed–Sternberg cells, large malignant lymphoid cells of B cell origin. Reference:
Davidson's Principles and Practice of Medicine

232.Sickle Cell Disease (SCD) patient presents with bloody urine. What is the cause?
A. Recurrent UTI
Answer: ?
Typically, the hematuria is mild and self-limited in SCD. As a rule, the hematuria originates from the left kidney; this has been
attributed to the greater length of the left renal vein and compression of the left renal vein between the aorta and superior
mesenteric artery (ie, the nutcracker phenomenon).
Hematuria can also be secondary to papillary necrosis or renal medullary carcinoma.
It can result from other problems such as urinary infections, renal stones or glomerulonephritis
Reference: http://emedicine.medscape.com/article/1957952-overview#a7

233.Which bleeding disorder presents with prolonged BT & aptt?


Answer: ?
Von Willebrand Disease (vwd) presents with increased Bleeding Time (BT) and elevated PTT.
167
234.40 years old man, Hb: low, MCV: high, No megaloblast. What is the most likely diagnosis?
A. Alcoholic anemia
Answer: A
Non-megaloblastic ddx: Liver disease, Alcoholism, Reticulocytosis, Hypothyroidism and Myelodysplasia.
Reference: Toronto Notes.

235.Sickle cell disease and joint pain? 2 times


o Due to avascular necrosis
Answer: ?
Not clear but Pain can be mild or severe and usually develops gradually. Pain associated with avascular necrosis of the hip may be
focused in the groin, thigh or buttock. In addition to the hip, the areas likely to be affected are the shoulder, knee, hand and foot.
Some people develop avascular necrosis bilaterally — for example, in both hips or in both knees.
Reference: http://www.mayoclinic.org/diseases-conditions/avascular-necrosis/basics/symptoms/con-20025517

236.Spoon shaped nail?


Answer: iron deficiency anemia

237.Elderly came with leukocytosis... What supports the diagnosis of CML


Answer: The diagnosis of CML is suspected based on the results of a simple blood test The test may show an abnormally high white
blood cell count in blood samples examined under a microscope, less mature white blood cells, normally found only in bone
marrow, are seen
Ref: Medical council of canada exam
On CML---- increase WBC and decrease Leukocyte alkaline phosphatase
On Leukemoid reaction -----increase WBC and Increase Leukocyte alkaline phosphatase

238.Best diagnostic tool for thalassemia is ….?


Answer: electrophoresis if asked about thalassemia in general. However if asked specifically about Alpha or Beta thalassemia is
different. Beta is diagnosed best with electrophoresis, but Alpha is diagnosed best with DNA test- polymerase chain reaction (PCR ).
Reference: Toronto notes 2015 page (H20)
Http://emedicine.medscape.com/article/206490-workup#c7

168
239.What factor causes thrombosis?
Answer: The formation of blood clots in the veins is related to three abnormalities commonly known as “Virchow’s triad”. The
processes of Virchow’s triad include the following: Damage to blood vessels (endothelial injury), Excessive clotting ability
(hypercoagulability) and Pooling of blood (stasis) - disturbed flow.

Table 1. Inherited Causes of Blood Clots


Increased Levels of Procoagulants Decreased Levels of Abnormal Fibrinolysis Other Inherited Causes
Anticoagulants
Factor V Leiden mutation or Antithrombin Plasminogen Deficiency Paroxysmal Nocturnal
*
activated protein C resistance Hemoglobinemia
Prothrombin 20210 mutation Protein C Decreased Levels of Tissue
Plasminogen Activator (t-
PA)
Hyperhomocysteinemia Protein S Increased Levels of
Plasminogen Activator
Inhibitor-1 (PAI-1)
FVIII, FIX, FXI, FVII, VWF Thrombomodulin Elevated Thrombin-
Activatable Fibrinolysis
Inhibitor (TAFI)
Heparin Cofactor II
Tissue Factor Pathway
Inhibitor (TFPI)
*The Factor V Leiden mutation does not result in increased FV levels but a resistance to the anticoagulant action of
activated protein C.

Http://www.ihtc.org/patient/blood-disorders/clotting-disorders/thrombosis/

240.What medication dissolve thrombus?


Answer: Dissolution occurs when the fibrinolytic mechanisms break up the thrombus and blood flow is restored to the vessel. With
the usage of these drug there is a risk of hemorrhage. 2 categories of drugs:
• Fibrin-specific agents, which include alteplase (tpa), reteplase (recombinant plasminogen activator [r-PA]), and tenecteplase,
produce limited plasminogen conversion in the absence of fibrin.
• Non–fibrin-specific agents (eg, streptokinase) catalyze systemic fibrinolysis. Streptokinase is indicated for the treatment of AMI,
acute massive pulmonary embolism (PE), deep vein thrombosis (DVT), arterial thrombosis, and occluded arteriovenous cannulae.
• Fibrinolytic agents can be administered systematically or can be delivered directly into the area of the thrombus. Systemic delivery
is used for treatment of AMI, acute ischemic stroke (AIS), and most cases of acute massive PE. Peripheral arterial thrombi and
thrombi in the proximal deep veins of the leg are most often treated via a catheter-directed approach.
Reference: http://emedicine.medscape.com/article/811234-overview#a2
Https://en.wikipedia.org/wiki/Thrombus

241.What the scientific term of chewing ice?


Pagophagia
169
Answer: A, is a form of the disorder pica involving the compulsive consumption of ice or iced drinks.

242.Pt with high aptt, what mechanism of action of that drug?


" unfractionated heparin" Antithrombin
Answer:
Medscape: Unfractionated Heparin = “Its anticoagulation effect is mediated by the activation of antithrombin III, which then
inactivates with relatively equal potency the coagulation enzymes thrombin (factor iia) and factor Xa. Other antithrombotic effects
include inhibition of platelet aggregation and additional antithrombin III-independent mechanisms. The partial thromboplastin time
may be elevated because of inactivation of thrombin.

243.Case of Fanconi syndrome TYPE 2 RTA

Answer:
A type of RTA a disease of the proximal renal tubules in which glucose, AA, uric acid, P and bicarbonate are passed into the urine,
instead of being reabsorbed. It may be inherited, or caused by drugs or heavy metals.
Clinical features of proximal renal tubular acidosis are:
• Polyuria, polydipsia and dehydration
• Hypophosphatemic rickets (in children) and osteomalacia (in adults)
• Growth failure
• Acidosis
• Hypokalemia
• Hyperchloremia
Other features of the generalized proximal tubular dysfunction of the Fanconi syndrome are:
• Hypophosphatemia/phosphaturia
• Glycosuria
• Proteinuria/aminoaciduria
• Hyperuricosuria
- treatment :
Mainly consists of the replacement of substances lost in the urine. Mainly fluids and electrolytes.
• Dehydration due to polyuria must be prevented by allowing free access to water; treat dehydration with either oral or
parenteral solutions.
Metabolic acidosis due to the loss of bicarbonate is corrected by 3-10 mg/kg/d of sodium bicarbonate in divided doses.
Diuretic, as 1-3 mg/kg/d of hydrochlorothiazide, may be necessary to avoid volume expansion, need to augment potassium
supplementation in the form of potassium bicarbonate, citrate, or acetate.
a. Phosphate and vitamin D supplementation are necessary, 1-3 g/d of supplemental phosphate
Https://en.wikipedia.org/wiki/Fanconi_syndrome
Http://emedicine.medscape.com/article/981774-treatment#d6

244.Case of multiple myeloma


Answer:
-cancer of plasma cells in the bone marrow.
-Criteria for diagnosis:
- A bone marrow aspirate or biopsy showing that at least 10% of the cells are plasma cells or the presence of a plasma cell tumor
(called a plasmacytoma), plus at least one of the following two features:
170
- Evidence of damage to the body as a result of the plasma cell growth, as severe bone damage, kidney failure, anemia, or high
calcium in the blood, and/or
- Detection of one of the following findings: ≥60% plasma cells in the bone marrow; free light chain ratio of 100 or more (provided
involved FLC level is at least 100 mg/L); or MRI showing more than one lesion (involving bone or bone marrow).
Variety of symptoms:
-bone pain in the back or chest, or less commonly, the arms and legs. The pain is usually triggered by movement and is absent at
night, except when changing positions, The bone loss and erosions can lead to osteoporosis and fractures “pathological fracture”
-High blood calcium levels : hyperca symptoms loss of appetite, nausea, vomiting, frequent urination, increased thirst, constipation,
weakness, confusion, stupor, or coma.
-Anemia: paleness, weakness, and fatigue.
-Impaired kidney function: Occasionally, kidney failure is the first sign of MM.
-hyperviscosity syndrome The symptoms may include bleeding from the nose and mouth, blurred vision, neurologic symptoms, and
heart failure.
-Neurologic symptoms radiculopathy causing numbness or tingling, pain, or muscle weakness.
- Generalized symptoms — The generalized symptoms of MM include an increased susceptibility to infections (especially during
chemotherapy) and weight loss.
Dx: Blood and urine tests for monoclonal protein , Bone marrow examination (aspiration and biopsy)
Http://www.uptodate.com/contents/multiple-myeloma-symptoms-diagnosis-and-staging-beyond-the-basics#H3498669556

245.Patient with prolonged PT and PTT 3

Answer:
Http://www.practical-haemostasis.com/Screening%20Tests/aptt.html

246.Case of multiple myeloma


Myeloma is an abnormal proliferation of plasma cells. These plasma cells are unregulated in their production of useless
immunoglobulin that is usuallyIgGor iga.IgMis a separate disease called Waldenstrom macroglobulinemia. These immunoglobulins
do not fight infection but clog up the kidney. The most frequent presentation of MM is with bone pain caused by a fracture
occurring under normal use.. Also may present with Hyperuricemia ,anemia and renal failure .
Diagnostic Testing :
171
· Skeletal survey to detect punched out osteolytic lesions. (Osteoblastic lesions suggest metastatic prostate cancer.)
· Serum protein electrophoresis (SPEP):You are looking for elevated levels of monoclonal antibody (usually igg). 20% are iga.
· Urine protein electrophoresis (UPEP):Detects Bence-Jones protein.
· Peripheral smear:Shows “rouleaux” formation of blood cells.
-The single most specific test is the bone marrow biopsy,which detects high numbers of plasma cells (10 percent).
Treatment :
Treat with melphalan and steroids. Thalidomide, lenalidomide, or bortezomibmay be added The most effective therapy is an
autologous stem cell bone marrow transplantation.
Reference : master the boards usmle .

247.How to differentiate between hypersplenism and aplastic anemia?


Answer:
Aplastic anemia is characterized by diminished or absent hematopoietic precursors in the bone marrow “empty marrow”,
most often due to injury to the pluripotent stem cell. Major causes: idiopathic, cytotoxic drug & radiation, drug reaction
(“Anticonvulsants”: carbamazepine Antibiotics: sulfonamides, chloramphenicol, “NSAIDs” phenylbutazone, indomethacin,
“anti thyroid” methimazole, propylthiouracil), Toxic chemicals, viral infections (EBV, HIV, other herpes), Immune disorders.
Hypersplenism is a condition in which the spleen becomes increasingly active and then rapidly removes the blood cells. It
can result from any cause of splenomegaly. It is most common with splenomegaly secondary to portal hypertension and
hematological disorders. The bone marrow in case of hypersplenism is cellular unlike in aplastic anemia.
Reference: http://www.researchgate.net/publication/237841754_Hypersplenism_Review_article

248.ITP case management >> prednisolone

249.High mcv + ast which type of anemia.


Answer: could be both folic acid and vitamin b12 deficiency. Both answers were provided.
Serum total homocysteine levels are almost always elevated in patients with folate deficiency because folate is required in the
remethylation step that converts homocysteine to methionine.[16] serum methylmalonic acid and homocysteine levels are
increased early in vitamin b-12 deficiency, even before hematologic manifestations or decreases in b-12 levels are noted.[17]
http://emedicine.medscape.com/article/203858-overview

250.A male patient from guinea with a neck mass the biopsy showed starry appearance
A. Burkitt lymphoma
Answer:a
Burkitt lymphoma, or small noncleaved cell lymphoma, is a highly aggressive b-cell non-hodgkin lymphoma.under the microscope,
the hallmark of burkitt lymphoma (bl) is the presence of a "starry sky" appearance.
Reference:http://emedicine.medscape.com/article/1447602-overview#a5

251.Pt with past hx of Hodgkin lymphoma, cured completely. Presented with back pain
Answer

252.What factor cause thrombosis ?


172
answer :
Risk Factors for VTE:
Stasis
Age > 40
Immobility
CHF
Stroke
Paralysis
Spinal Cord injury
Hyperviscosity
Polycythemia
Severe COPD
Anesthesia
Obesity
Varicose Veins
Hypercoagulability
Cancer
High estrogen states
Inflammatory Bowel
Nephrotic Syndrome
Sepsis
Smoking
Pregnancy
Thrombophilia
Endothelial Damage
Surgery
Prior VTE
Central lines
Trauma
Reference: Anderson FA Jr. & Wheeler HB. Clin Chest Med 1995;16:235.

173
Neurology

174
1. Patients with epilepsy, which of the following receptors most likely is stimulated?
A. Protein g
B. Glutamate
C. Serotonin
D. Kinase
Answer: b
Glutamate is the major excitatory neurotransmitter in the brain. The release of glutamate causes an epsp in the postsynaptic neuron
by activating the glutaminergic receptors ampa/kainate and nmda and the metabotropic receptor.
Reference: http://emedicine.medscape.com/article/1184846-overview#a3

2. What is the most accurate test for carpal tunnel syndrome?

A. Tinel
B. Compression test
C. Durkan's carpal test
D. Phallens test

Answer: all the resources i found were clinical studies with various numbers.
Specific tests might be as the following order:
1-''two point discrimination <6 mm with caliper (33% sensitive, 100% specific).
Flick test (93% sensitive, 96% specific)
Durkan's test (sensitivity from 87% to 91% and its specificity from 90% to 95%)
4-tinel's sign (44-70% sensitive, 94% specific)
Carpal compression test (90% sensitive, 90% specific)
Phalen's maneuver (70-80% sensitive, 80% specific).
So, from the choices above, most likely it is durkan test or tinnel
Resource http://fpnotebook.com/ortho/wrist/crpltnlsyndrm.htm
& https://en.wikipedia.org/wiki/durkan%27s_test

3. Patients with epilepsy, which of the following receptors most likely is stimulated?
A. Protein g
B. Glutamate
C. Serotonin
D. Kinase

Answer: b
Http://www.uptodate.com/contents/pathophysiology-of-seizures-and-
epilepsy?Source=search_result&search=seizure&selectedtitle=11%7e150

4. What is the most accurate test for carpal tunnel syndrome:


A. Tinel
B. Compression test
C. Durkan's carpal test
D. Phallens test

Answer: c

175
There isn’t a single best test for diagnosing carpal tunnel syndrome, everywhere on the internet the best method to diagnose has
been mentioned to be a proper history from the patient, we can also diagnose with these methods (electrodiagnostic tests) which
includes both tinel and phallen tests. Nerve conduction testing should be done on the affected arm for diagnosis and to exclude a
more proximal neuropathy.
Reference: merck manual

5. Patient has parotitis complains of pain with eating that radiate to the ear , with nerve transmit pain with eating?
A. 8
B. 9
C. 10
D. 7
Answer: 7?

6. Patient complain of headache (worst headache in life) csf result (blood)


A. Brain abscess
B. Brain infection
C. Epidural hematoma.
D. Ruptured berry aneurism.

Answer: d. Ruptured berry aneurism. This is subarachnoid hemorrhage and it’s classically description as “the worst headache of my
life” 
and one of its causes is berry aneurism rupture. Step up to medicine

7. Patient with increase in icp, complains of vomiting, tinnitus, nausea, headache and blurred vision, the doctor orders him a ct
scan, what the cranial nerve will discover to know the diagnosis before doing imaging study?
A. Orbital
B. Facial
C. Trochlear
D. Occulomotor

Answer: ?
Uptodate: https://yhdp.vn/uptodate/contents/mobipreview.htm?32/39/33402
Clinical manifestations: signs include cn vi palsies (abducens), papilledema secondary to impaired axonal transport and congestion,
spontaneous periorbital bruising and a triad of bradycardia, respiratory depression, and hypertension (cushing's triad, sometimes
called cushing's reflex or cushing's response).

8. After a motor vehicle accident, a patient can not bring the spoon to feed himself. Which cerebral lobe is affected?
A. Temporal
B. Parietal
C. Occipital
D. Cerebellum
Answer: d
The cerebellum: is principally concerned with balance and the regulation of posture, muscle tone and muscular co-ordination.
Frontal lobe: a-the motor cortex: the primary motor area. It receives afferents from the premotor cortex, thalamus and cerebellum
and is concerned with voluntary movements. B-the premotor cortex. *clinical anatomy applied anatomy for students and junior
doctors.

176
9. A seizing mother was given phenobarbital. How will you advise her regarding her lactation?
A. Stop immediately lactation
B. Wait for 8 hours after taking the drug then lactate
C. Stop lactating gradually over 3 weeks
D. Continue lactation
Answer:

10. 32 years old male presenting with left sided headache associated with nausea, vomiting, photophobia, aura, lasting for 12
hours occurring 4-5 times per month, what's your best modality of investigation?
A. Cbc
B. Esr
C. Precise history and physical examination.
D. Mri
Answer: c

11. Which nerve carries the referred pain of the parotid to the ear?
A. Vagus
B. Facial
C. Auriculotemporal
D. Trigeminal Via
auricotemporal
Answer: d branch of CN V

12. Old male hemiplegia, ct showed ischemia, examination normal except for hemiplegia, now he is stable doing physiotherapy.
Taking hydrochlorothiazide. 2 yrs ago had a history of acute gastric ulcer...
What to give him:
A. Do nothing
B. Tpa
C. Aspirin
D. Warfarin
Answer: c
Tpa: time to therapy for acute ischemic stroke management is 3 - 4.5 hours. Contraindication to tpa …
aspirin: aha/asa guidelines recommend giving aspirin, 325 mg orally, within 24-48 hours of ischemic stroke onset. Warfarin:
currently, data are inadequate to justify the routine use of heparin or other anticoagulants in the acute management of ischemic
stroke.
Http://emedicine.medscape.com/article/1916852-treatment#showall

13. Which antiviral used in parkinsonism:


A. Ribiverin
B. Zuvedamin
C. Gancyclovir
D. Amantadine
Answer: d

14. Unwanted side effect of anticholinergics?


177
A. Diarrhea
B. Blurred vision
C. Excessive salivation
D. Urinary incontinence
Answer: b
According to medscape, remember common signs and symptoms with the mnemonic, "red as a beet, dry as a bone, blind as a bat,
mad as a hatter, hot as a hare, and full as a flask." the mnemonic refers to the symptoms of flushing, dry skin and mucous
membranes, mydriasis with loss of accommodation, altered mental status, fever, and urinary retention, respectively.
Http://emedicine.medscape.com/article/812644-clinical#showall

15. Mainstay treatment for parkinson’s disease?


A. Dopamine agonists
B. Mao inhibitors
C. Anticholinergics
D. Levodopa
Answer: d
Dopamine agonists provide symptomatic benefit that is comparable to that with levodopa/carbidopa in early disease, but these
agents lack sufficient efficacy in more advanced disease. They are commonly reserved for younger individuals (< 65-70 years) who
are cognitively intact. Mao-b inhibitors, such as selegiline and rasagiline, may be used for early symptomatic treatment of parkinson
disease. These medications provide mild symptomatic benefit & have excellent adverse effect profiles. Anticholinergic agents: are
not first-line drugs, because of their limited efficacy and the possibility of neuropsychiatric side effects. Levodopa, coupled with a
peripheral decarboxylase inhibitor such as carbidopa, remains the standard of symptomatic treatment for parkinson disease. It
provides the greatest antiparkinsonian benefit with the fewest adverse effects in the short term.

16. Which of the following drugs causes hair growth:


A. Phenytoin
B. Phenobarbital
C. Valproic acid
D. Carbamazepine
Answer: both a&d may cause hair growth.

17. Patients with epilepsy, which of the following receptors most likely is stimulated?
A. Protein g
B. Glutamate
C. Serotonin
D. Kinase
Answer: - glutamate
Https://en.wikipedia.org/wiki/glutamate_receptor#seizures

18. Adolescent with fever, headache, meninges after swimming in a river; causative organism
A. Streptococcus
B. H. Influenza
C. N. Meningitides
D. Naegleria fowelri
Answer: d
It is a rare deadly infection of the meninges by that amoeba.
178
Http://www.cdc.gov/parasites/naegleria/

19. Side effect of anticholinergic. Hot as a hare: increased body temperature


A. Blind as a bat: mydriasis (dilated pupils
B. Dry as a bone: dry mouth, dry eyes, decreased sweat
C. Red as a beet: flushed face
D. Mad as a hatter: delirium

20. Case of meningitis caused by meningococcal type b. Which of the following can decrease risk of spreading of the infection:
A. Do nothing
B. Give ceftriaxone or cefotaxime to decrease risk of spreading from nasal mucosa
C. Give meningiooccal vaccine for pt & contact
D. Isolation of all contact for 4 weeks

21. Hemiplegic patient come with something in the eye (may be nystagmus or something )where is the lesion in the brain?
A. Pons
B. Medulla oblongata
C. Internal capsule
D. Midbrain
Answer: b

22. Loss of sense of smell which lobe affected?


A. Frontal
B. Occipital
C. Parietal
D. Temporal
Answer: d?
Temporal lobe—senses of smell and sound, as well as processing of complex stimuli like faces and scenes
Frontal lobe—conscious thought; damage can result in mood changes, social differences, etc. The frontal lobes are the most
uniquely human of all the brain structures.
Parietal lobe—plays important roles in integrating sensory information from various senses, and in the manipulation of objects;
portions of the parietal lobe are involved with visuospatial processing
Occipital lobe—sense of sight; lesions can produce hallucinations
Limbic system—emotion, memory, emotional expression, resolve conflict from frustration. "odors often trigger emotional reactions
and memories" (elaine & katja, 2015).
Insula—"island inside" pain, taste, hunger, visceral functions, social emotions, time perception and awareness "connects to the
cortex and the limbic systems" (blanc et. Al., 2014).

23. A patient with tuberculosis on medication for 3 months. He developed pins and needles sensation of his lower limbs.
Deficiency of which of the following caused his symptoms?
A. Niacin
B. Folic acid
C. Iron
D. Pyridoxine (B6)

179
Answer: D
Vitamin B6 (pyridoxine) supplementation during isoniazid (INH) therapy is necessary in some patients to prevent the development of
peripheral neuropathy.
Reference: Pubmed.

24. 19 years old male can’t bring the spoon in front of himself to eat after a bike accident. Where is the site of his lesion?
A. Parietal lobe
B. Temporal lobe
C. Occipital lobe
D. Cerebellum

Answer D
It is a problem of coordination.
References:
Http://www.healthline.com/health/movement-uncoordinated#Overview1

25. A girl with migraine. She doesn’t want to take prophylactic medicine.
A. Biofeedback
B. Sumatriptan
C. Ergotamine
D. Propranolol

Answer: A

26. College student have meningitis. What to do as a prophylaxis to dorm friends next?
A. Isolate all contacts for 4 weeks
B. Immunize all contacts
C. Give antibiotic ( Penicillin and other similar antibiotics) – exact sentence was written
D. Do nothing

Answer: d
- “Close contacts” means those who have major respiratory fluid contact, such as
Household contacts, kissing, or sharing cigarettes or eating utensils.
- Routine school and work contacts are not close contacts. Sitting in class with
Someone with Neisseria infection does not make them a close contact Reference: master the board

27. Old patient presented with agitation, urinary incontinence, confusion and impaired short-term memory. Long-term memory
is intact. CT shows temporal and hippocampal atrophy. Which of the following genes might be affected?
A. 13
B. 15
C. 18
D. X

Answer: 19?
Alzheimer’s Disease:

180
Early-onset, (age 30 to 60): 3 major genes for autosomal dominant AD have been identified: Šamyloid precursor protein
(chromosome 21), presenilin 1 (chromosome 14) and presenilin 2 (chromosome 1).
Late-onset (mid-60s and later): The E4 polymorphism of apolipoprotein E is a susceptibility genotype (E2 is protective). The APOE
gene is located on the long (q) arm of chromosome 19.
Reference: Toronto Notes

28. A patient with high ICP (I think it was due to a brain tumor), which of the following nerves is most likely to be affected?
A. Trochlear
B. 6th cranial
C. Optic
D. Facial

Answer: B
Increase ICP leads to CN III and VI deficient.
Reference: First Aid USMLE 2.

29. A 72 y male disoriented and hallucinating and disorganized thinking had aorto popletial graft and symptom fluctuates in the 2
days what the cause?
A. Multi infraction demntia
B. Mania
C. Demensia
D. Delirium

Answer: D (post operative delirium)


Delirium is a common but often undiagnosed complication in the elderly following a major operation. There are multiple risk factors
for developing postoperative delirium. Risk factors can be separated into patient-specific and operation-specific risk factors.
Established patient-specific risk factors for the development of delirium following noncardiac operations include pre-existing
dementia(strongest factor), older age, functional impairment, greater co-morbidities, and psycopathological symptoms. Operation-
specific risk factors for the development of postoperative delirium are based on the degree of operative stress. For example, low
operative stress procedures such as cataract surgery result in delirium in 4% of cases in comparison to high surgical risk procedures
such as vascular operations.
Http://www.ncbi.nlm.nih.gov/pmc/articles/PMC2546478/

30. Hempligic pt come with some thing in the eye( may be nystamius or some thing )where is the lesion in the brain :
A. Pons
B. Medulla oblangta
C. Interna capsule
D. Midbrain

31. Pt with absence seizure the doctor wants to start him on sodium valproate, what test should be done before starting the tx?
A. Liver function
B. Creatinine
C. Urea

Answer: a (hepatotoxicity & teratogenicity)

181
Side effects of vpa include nausea, vomiting, hair loss, easy bruising, and tremor. Vpa is associated with weight gain, obesity, insulin
resistance, and the metabolic syndrome. Vpa can also cause thrombocytopenia and other coagulation disturbances and has also
been associated with subclinical hypothyroidism with mild to moderate elevations in thyrotropin (TSH ) levels. Vpa has also been
linked to the polycystic ovarian syndrome.
Vpa-exposure in utero is associated with major malformations and other adverse effects, including neurodevelopmental
abnormalities. Vpa should be avoided in pregnancy when possible
Approximately 5 to 10 percent of patients develop ALT elevations during long term vpa therapy; most of the time these
abnormalities are asymptomatic and can even resolve with continuation of the drug. In addition, there are more serious forms of
hepatotoxicity that can occur with vpa:
The fda recommends checking LFTs prior to initiating treatment and at frequent intervals thereafter, especially during the first six
months
Reference: uptodate, toronto notes page (n54)
Http://www.drugs.com/sfx/valproic-acid-side-effects.html
Http://www.epilepsy.com/medications/valproic-acid
Http://reference.medscape.com/drug/depakene-stavzor-valproic-acid-343024#5

32. What is the most common risk factor for stroke?


A. Htn
B. Atrial fibrillation
C. Ldl

Answer: a

33. Pt had head trauma, he has nausea, vomiting and decreased level of consciousness, icp was suspected and ct was arranged,
what cranial nerve examination can confirm the dx?
A. Optic
B. Oculomotor
C. Trochlear
Answer: b
Uptodate: “ global symptoms of elevated icp include headache, which is probably mediated via the pain fibers of cranial nerve (cn) v
in the dura and blood vessels, depressed global consciousness due to either the local effect of mass lesions or pressure on the
midbrain reticular formation, and vomiting.

34. Most common intracranial tumor in adults?


A. Hemangioblastoma
B. Ependymoma
C. Schwannoma

Answer: b
Gliomas are the most prevalent type of adult primary brain tumor, accounting for 78 percent of malignant brain tumors.
Meningiomas are the most common benign intracranial tumors.
Pituitary adenomas are the most common intracranial tumors after gliomas, meningiomas and schwannomas. So, the most common
brain tumors are (in sequence)
Glioma
Meningioma
Schwannoma
182
Pituitary adenoma
Gliomas include: astrocytoma, ependymoma, glioblastoma multiform, medulloblastoma, and oligodendroglioma.
Most common in children: medulloblastoma
Reference: american association of neurological surgeons
http://www.aans.org/patient%20information/conditions%20and%20treatments/brain%20tumors.aspx

35. Most common risk factor for stroke:


A. Htn
B. Atrial fibrillation
C. Ldl

Answer: a
Reference: step-up of medicine

36. Patient had head trauma, he has nausea, vomiting and decreased level of consciousness, icp was suspected and ct was
arranged, what cranial nerve examination can confirm the dx?
A. Optic
B. Oculomotor
C. Trochlear

Answer: a
Looking for papilloedema which is swelling of the optic disc
Reference: http://jnnp.bmj.com/content/73/suppl_1/i23.full.pdf

37. Case of migraine what next step:


A. Mri
B. Blood culture
C. More history and physical ex
Answer: c

38. Patient complain of headache band like distribution ?


A. Cluster
B. Migraine
C. Tension
Answer: c

39. Best anti conversant for treatment of trigeminal neuralgia;


A. Phynetoin
B. Carbamezabin
C. Ethuximab
Answer: b
Http://emedicine.medscape.com/article/1145144-treatment

40. Which one risky (more) to stroke?


183
A. 55 , male, HTN , obese
B. 50, male , dm, hypercholesterolemia
C. 22,female,smoker
Answer: a
Risk factors for stroke are:
The most important risk factors are age and htn. Others include smoking, 
dm, hyperlipidemia, atrial fibrillation, coronary artery
disease, family history 
of stroke, previous stroke/tia, and carotid bruits. 

In younger patients, risk factors include oral contraceptive use, hypercoagulable states (e.g., protein c and s deficiencies,
antiphospholipid antibody syndrome), vasoconstrictive drug use (e.g., cocaine, amphetamines), polycythemia vera, and sickle
cell disease. 


41. Patient with parotid gland pain. Which nerve carries the sensation from parotid?
A. Auriculotemporal nerve
B. Glossopharyngeal nerve
C. Facial nerve
Answer: a , auriculotemporal from mandible
Sensation of parotid by auriculotemporal
Autonomic innervation (salivation) by glossopharyngeal.

42. Women brought her father had dementia of recent events what will you do?
A. Refer to geriatric
B. Give him antipsycotic
C. Measure iq
Answer: a
Explanation: no reference for explanation.

43. Patient admitted to hospital with headache, nausea and vomiting (signs of increased icp ) what you find in eye examination ?
A. Papiledema
B. Central retinal artery ischemia
C. Glaucoma
Answer: a

44. Patient presented with status epilepticus, lorazepam was given, however the patient didn’t improve. What is the most
appropriate drug to be given?
A. IV phenobarbital
B. IV phenytoin
C. Oral carbamazepine

Answer: b
The first line in managing status epilepticus are benzodiazepines, second line is phenytoin and third line is phenobarbital.
Http://bestpractice.bmj.com/best-practice/monograph/464/treatment/details.html

45. Amyotrophic lateral sclerosis which horn is infected>>


A. Anterior horn
184
B. Lateral horn
C. Posterior horn

Answer: a.
Anterior horn cell. Als: a disorder affecting the anterior horn cells and corticospinal tracts at many levels. Corticobulbar involvement
is common as well. Step up to medicine.

46. Most important risk factor for stroke:


A. Htn
B. Smoking
C. Dm
Answer: a

47. Type of cerebral palsy lower paralysis more than upper?


A. Diplagia
B. Quadriplagia
C. Hemiplagia
Answer: a

48. Best strategy to decrease stroke among public:


A. Anti smoking campaign
B. Free blood lipid screening
C. Blood pressure screening
Answer: c
Hypertension is the most important modifiable risk factor for stroke and intracerebral hemorrhage (ich), and the risk of stroke
increases progressively with increasing blood pressure, independent of other factors. Regular blood pressure screening and a
combination of behavioral lifestyle modification and drug therapy are recommended to achieve these goals. Studies on the
comparative benefits of specific classes of antihypertensive agents have not shown definitive results. In patients who have
hypertension with diabetes or renal disease, the blood pressure goal is < 130/80 mm hg

49. Cavernous sinus thromboembolism. Which sinus is more common?


A. Ophthalmic
B. Maxillary
C. Sphenoid
Answer: c
Most commonly nasal furuncle then ethmoid and sphenoid sinuses.

50. Young male had a history of two seizures in the last 30 minutes, no history of diabetes no history of head trauma, at initial
presentation had, another seizure. What is the best to give him now?
A. Phenytoin
B. Diazepam
C. Phenobarbital
Answer: b
185
Benzodiazepines are the preferred drug class for the initial treatment of se. Lorazepam, when available, is thought to be the most
effective of the benzodiazepines and has a longer seizure half-life than diazepam. Phenytoin or fosphenytoin (cerebyx) is the next
drug to be administered when a second drug is needed. Failure to respond to optimal benzodiazepine and phenytoin loading
operationally defines refractory se.
Http://emedicine.medscape.com/article/1609294-overview#a11

51. Pt presented with meningitis symptoms and he swim in the river what is the organism
A. S. Pneumonia
B. H. Influenza
C. N. Meningitis
Answer: amobeic meningitis (naegleria fowleri)
It is commonly found in warm fresh water (e.g. Lakes, rivers, and hot springs) and soil. Naegleria fowleri usually infects people when
contaminated water enters the body through the nose.
Http://www.cdc.gov/parasites/naegleria/

52. A 30-year-old male presented with severe headache, never had like this headache before, photophobia, neck rigidity.
Intracranial hemorrhage was suspected. Where is the most likely site of this hemorrhage?
A. Intracerebral
B. Subarachnoid
C. Epidural
Answer: b
Symptoms of subarachnoid hemorrhage include; headache described as “worst headache of my life” (up to 97% sensitive, 12-25%
specific), n/v, photophobia, meningismus (neck pain/stiffness, positive kernig’s and brudzinski’s sign), decreased loc. Toronto notes
2015

53. Lt. Hemiplagia, MRI at t2 show hyperdense area in the rt. Side of the brain. Which of the following will worsening the pt
prognosis?
A. blood glucose < 6.5
B. Blood pressure < 140
C. Incomplete mcq

54. Most common intracranial tumor in adult ?


A. Hemangiomblastoma
B. Ependymoma
C. Shwanoma

Answer: b
Glioblastoma multiforme (gbm) , also known as gliomas is the most common primary brain tumor in adults. Ependymoma is a type
of glioma.
Http://braintumor.org/brain-tumor-information/understanding-brain-tumors/tumor-types/

55. 50 yo w painless loss of vision in one eye, with headache and pain when touching the hair on the same side. Wts next:
A. Topical steroids
B. Oral steroids
C. Brain ct
186
Answer: b
The universally accepted treatment of giant cell arteritis (gca) is high-dose corticosteroid therapy,the major justification for the use
of corticosteroids is the impending danger of blindness in untreated patients. Patients who present with visual symptoms have a 22-
fold increased chance of visual improvement if therapy is started within the first day. Damage may be irreversible if treatment is
delayed beyond 48 hours. Http://emedicine.medscape.com/article/332483-treatment
Once it started to affect the vision >> IV sterio

56. Patient with exophthalmos and swollen lids and you can feel its pulse, TFT normal. Whats ur diagnosis.
A. Hyperthyroid
B. Cellulitis
C. Cavernous sinus thrombosis
Answer c
Reference: http://emedicine.medscape.com/article/791704-clinical#showall
It can be b according to other info if provided in the q

57. Clear case of absence seizure then asked what is going to happen to this patient if given fentanyl :
A. Glutamate receptors activation
B. Seizure activity due to toxic neurotransmitters release
C. Demyelination.
Answer: b
Epidural fentanyl is local anesthetic : moa due to its high lipid solubility it rapidly binds dorsal horn receptors in the spinal cord. G-
protein coupled receptors & inhibit adenylate cyclase
Nb: its not related to absence seizure.
However, there are reports of epileptogenic EEG activity with the use of fentanyl, alfentanil and remifentanil - these drugs can be helpful in localizing the epileptic focus 71-73. Therefore, we
recommend avoiding the use of high doses or rapid administration of opioids of the phenylpiperidine group in epileptic patients

58. Teacher who suffer from headache on temporal side , ct normal blood pressure normal he was taking paracetamol but not
improve well , what can you give ?
A. Aspirin
B. Ibuprofen
C. Triptans
Answer: c (for treatment of migraine and cluster headache)
If the scenario goes with tension type; give ibuprofen. If it goes with temporal arteritis; give steroid with aspirin. If it goes with
migraine; give triptans.

59. Medical student diagnosed as meningitis, what you do for him?


A. Start antibiotics
B. Isolate him for 4 weeks
C. Give him influenza vaccine for his colleagues

Answer: a
bacterial meningitis must be the first and foremost consideration in the differential diagnosis of patients with headache, neck
stiffness, fever, and altered mental status. Acute bacterial meningitis is a medical emergency, and delays in instituting effective
antimicrobial therapy result in increased morbidity and mortality.
[http://emedicine.medscape.com/article/232915-workup]

60. Female complaining of hair growth no seizure for 7 years wants to stop the drug?
187
A. Stop the drug after 6 months (tapering)
B. Continue the drug indifinitly
C. Continue till 10 years

Answer: a

61. Gingival hypertrophy side effect


A. Carbamazepine
B. Phenytoin
C. Phenobarbital

Answer: b
Gastrointestinal side effects including gingival hyperplasia (in as many as 50% of treated patients) have been reported. The
gingival hyperplasia associated with phenytoin is occasionally severe enough to merit surgical removal.

Reference: http://www.drugs.com/sfx/phenytoin-side-effects.html

62. Reversible cause of stroke?


A. Hypertension
B. Obesity
C. Smoking
Answer: a

63. What is the most important risk factor for stroke?


A. Hypertension
B. DM
C. Smoking

Answer: A
Hypertension emerges as the single most powerful and reversible risk factor in stroke and for survival after stroke.
Impaired cardiac function is the second most important precursor of stroke.
Reference: Pubmed.

64. What is the drug of choice for absence seizure?


A. Phenytoin
B. Carbamazepine
C. Ethosuximide

Answer: C
Reference: Master the Boards.

65. How you can confirm Brain stem death?

188
A. Absence of doll eye
B. Active coughing & gag reflex
C. One inactive pupil

Answer: A
The process for brain death certification includes “3- Performance of a complete neurological examination. Components of a
complete neurological examination are: iii. Absent corneal, oculocephalic (doll's eye ), cough and gag reflexes. The corneal reflex
may be altered as a result of facial weakness.” Negative vestibule-ocular reflex "doll's eyes" would stay fixed midorbit, and having
negative "doll's eyes" is therefore a sign that a comatose patient's brainstem is functionally not intact.
Reference: http://www.ncbi.nlm.nih.gov/pmc/articles/PMC2772257/
Https://en.wikipedia.org/wiki/Vestibulo%E2%80%93ocular_reflex

66. Case of Absence seizure What is best treatment for that? " 4 times
A. Phonation
B. Carbazapin
C. Ethosuximide

Answer: C
Ref: Only 2 first-line aeds have approval from the US Food and Drug Administration (FDA) to be indicated for absence seizures:
ethosuximide (Zarontin) and valproic acid (Depakene, Depacon). Ethosuximide has efficacy for absence seizures only and valproic
acid has efficacy for absence, generalized tonic-clonic, and myoclonic seizures.
Ethosuximide (Zarontin) is effective only against absence seizures.
Reference: http://reference.medscape.com/article/1183858-overview#a9

67. 45 years old with proximal muscle weakness, tongue fasciculation, with history of recurrent aspiration pneumonia. What is
the diagnosis?
A. Mononeuropathy
B. Myasthenia gravis
C. Mysthenic syndrome

Answer: B. Myasthenia Gravis


Reference: http://m.jnnp.bmj.com/content/73/6/766.full

68. 23 y girl come e one eye blindness+past hx 2 year back headech + diplopia ...ect best investigation?
A. Mri
B. Ct
C. Cbc
Multiple sclerosis is probable and MRI is the probable answer

69. Bacterial meningitis in lp:


A. Decrease glucose and increase protein
B. Increase glucose and decrease protein

Answer: a

189
70. A women with neck pain after the pain there is tingling and shock like waves and weakness ,what is diagnosis
A. Whiplash injury
B. Brachial plexus injury
Answer:?

71. Lady with headache " worst headache in her life, neck stiffness, blood in
Csf:
A. Epidural hemorrhage
B. Ruptured aneurysm
Answer: b
Subarachnoid hemorrhage is sudden bleeding into the subarachnoid space. The most common cause of spontaneous bleeding is a
ruptured aneurysm. Symptoms include sudden, severe headache, usually with loss or impairment of consciousness. Secondary
vasospasm (causing focal brain ischemia), meningismus, and hydrocephalus (causing persistent headache and obtundation) are
common. Diagnosis is by ct or mri; if neuroimaging is normal, diagnosis is by csf analysis. Treatment is with supportive measures and
neurosurgery or endovascular measures, preferably in a comprehensive stroke center.

72. 16-year-old came with headache band like, behind eye throbbing. Stressful life
A. Migraine
B. Tension

Answer: b http://emedicine.medscape.com/article/792384-clinical

73. Patient after motor vehicle accident suspected increase icp. What cranial nerve to check for?
A. Optic
B. Trochlear
Answer: b
190
The sixth cranial nerve as at risk of injury due to stretch which might cause diplopia
Http://www.ajol.info/index.php/cme/article/download/87803/77457

74. Guillain-barre syndrome?


A. Ascending paralysis more in the legs
B. Descending asymmetrical paralysis

Answer: a
Davidson’s medicine page 1229: distal paraesthesia and limb pain (often severe) precede a rapidly ascending muscle weakness,
from lower to upper limbs, more marked proximally than distally. proximally in the limb

75. Vegetarian with numbness and socks and gloves distribution:


A. B6 deficiency
B. B12 deficiency
Answer: b

76. Most common brain tumor:


A. Epindemoma
B. Astrocytoma
Answer:
The option was glioma types : asrrosytoma and glioblastoma multiform the most common.

77. Glove nuropathy cause?


A. Vitamin b12
B. Vitamin b6
Answer: a

Presentations vary greatly among patients. The symmetric glove-and-stocking paresthesias, or tingling in the distal aspect of the
toes, numbness, coldness, a pins-and-needles feeling, and occasional feelings of swelling or constriction, are slowly progressive and
insidious. Symptoms progress up the legs, occasionally affect the fingers, and culminate in weakness and spasticity.
Link: http://emedicine.medscape.com/article/1171558-clinical

78. Gillian barre syndrome, distal progresses paralysis upper and lower limb what will you find on csf?
A. Increase protein
B. Decrease glucose
Answer: a

Lumbar puncture
Most, but not all, patients with gbs have an elevated csf protein level (>400 mg/l), with normal csf cell counts. Elevated or rising
3
protein levels on serial lumbar punctures and 10 or fewer mononuclear cells/mm strongly support the diagnosis.

79. Baby after operation has loss of gag reflex in left side, normal uveal movement. What nerve injured?
191
A. Glossopharyngeal
B. Vagus
Answer: a
The sensory limb is mediated predominantly by cn ix (glossopharyngeal nerve)
The motor limb by cn x (vagus nerve). In unilateral (one-sided) glossopharyngeal nerve (cn ix- sensory component) damage, there
will be no gag response when touching the pharyngeal wall on the same side of the damaged nerve. With one-sided vagal nerve (cn
x- motor component) damage, the soft palate will elevate and pull toward the intact side regardless of the side of the pharynx that is
touched. This is because the sensory component is intact on both sides, but only the motor nerves supplying one side of the soft
palatine and pharyngeal muscles is working, therefore the contraction of the muscles in the reflex is asymmetrical. If both cn ix and x
are damaged on one side (not uncommon), stimulation of the normal side elicits only a unilateral response, with deviation of the
soft palate to that side; no consensual response is seen. Touching the damaged side produces no response at all.

80. Old patient with recent memory loss and poor self care and social withdrawal, what to give him ?
A. Neostigmen
B. Rivastigmen

Answer: b
The treatment for mild to moderate alzheimer’s disease is central cholinesterase inhibition. We chose rivastagmine because it
crosses the blood-brain barrier while neostigmine doesn’t.
Http://emedicine.medscape.com/article/1134817-treatment#d10

81. Most common neurosurgical tumor?


A. Astrocytoma.
B. Medullablastoma.

Answer:
gliomas (principally astrocytoma) account for approximately 30% and meningioma 35% to 40% of symptomatic primary brain
tumors.

reference : pic and uptodate

82. A pt with meningitis, which combination of antibiotic should be given ?


A. Vancomycin + ampicillin
B. Amoxicillin + gentamycin
Answer: a
192
According to age

th
Reference: step-up to medicine 4 edition

83. Patient with seizure, what is the treatment


A. Phenobarbitone
B. Phenytoin
Answer: depend on the type of seizure
toronto:
antiepileptic drugs:
Generalized-onset and partial-onset seizures: felbamate, lamotrigine, levetiracetam, refinamide, topiramate, valproate,
zonisamide
Partial seizures (simple partial, complex partial, and secondarily generalized seizures): carbamazepine, gabapentin, lacosamide,
oxcarbazepine, phenobarbital, phenytoin, pregabalin, primidone, tiagabine, vigabatrin (note: these drugs may exacerbate
generalized seizures)
Absence seizures: ethosuximide

84. A patient with myasthenia gravis presents with myasthenic crisis. What is the cause?
A. Anticholinesterases
B. Pyridostigmine

Answer: ?
Numerous medications may exacerbate MG, including quinidine, procainamide, β-adrenergic antagonists, calcium channel
antagonists (verapamil, nifedipine, felodipine), magnesium, antibiotics (ampicillin, gentamicin, streptomycin, polymyxin,
ciprofloxacin, erythromycin), phenytoin, gabapentin, methimazole, α-interferon and contrast media. Reference: Pubmed

Myasthenia gravis (MG) is a disorder of muscular weakness from the production of


antibodies against acetylcholine receptors at the neuromuscular junction.
Best initial treatment of myasthenia gravis: Neostigmine or pyridostigmine
(anticholinesterase agents / acetylcholinesterase inhibitors). These are longer acting
versions of edrophonium.

193
Acute myasthenic crisis presents with severe, overwhelming disease with profound weakness or respiratory involvement. It is
treated with IVIG or plasmapheresis. Reference: Master the boards

85. A young female experiencing symptoms of optic neuritis and other neuro symptoms (case of ms) what is the best diagnostic
test:
Answer: (mri)
Multiple sclerosis (ms) is diagnosed on the basis of clinical findings and supporting evidence from ancillary tests, such as magnetic
resonance imaging (mri) of the brain and spinal cord and cerebrospinal fluid examination.
Http://emedicine.medscape.com/article/1146199-workup

86. A question about huntington disease:


You can read more about huntington disease here:
Https://yhdp.vn/uptodate/contents/mobipreview.htm?29/8/29832
Https://yhdp.vn/uptodate/contents/mobipreview.htm?38/29/39385

87. Case of migraine headache.


Migraines: http://emedicine.medscape.com/article/1142556-overview

88. What is most risk factor for stroke?


Modifiable: • high blood pressure. This is the single most important risk factor for stroke because it’s the no. 1 cause of stroke. •
tobacco use. • diabetes. • high cholesterol. • physical inactivity and obesity. • carotid or other artery disease. • transient
ischemic attacks (tias). • atrial fibrillation (afib) or other heart disease. • certain blood disorders e.g.: a high red blood cell count,
sickle cell anemia. • excessive alcohol intake. • illegal drug use.
Non-modifiable: • increasing age. • gender. • heredity and race. • prior stroke.
Https://www.heart.org/idc/groups/stroke-public/@wcm/@hcm/documents/downloadable/ucm_309713.pdf

89. Gillian barre associated with ?


A. Cranial nerve involvement
Answer: a
Gbs http://emedicine.medscape.com/article/315632-overview

90. Neurofibromatosis genome:


The nf1 gene is cytogenetically located on the long (q) arm of chromosome 17, at band 11.2 (17q11.2). Medscape
http://emedicine.medscape.com/article/950151-overview

91. Pt had hx of AFib and prior stroke, how to prevent this patient from second stroke?
A. Warfarin

92. Epilepsy stimulated by >>glutamate

194
93. 65 years old male, presenting with peripheral neuropathy which progressed to weakness (sub-acute combined
degeneration), labs shows macrocytic anemia, diagnosis?
A. Vitamin b12 deficiency

Vitamin b 12 deficiency presentation beside the anemia symptoms (megaloblastic macrocytic anemia), they present with
neurological changes — classic picture of sub-acute combined degeneration of the dorsal and lateral spinal columns. This lesion,
specific for Vitamin b12 deficiency, is due to a defect in myelin formation. The neuropathy is symmetrical and affects the legs more
than the arms. It begins with paresthesia and ataxia associated with loss of vibration and position sense, and can progress to severe
weakness, spasticity, clonus, paraplegia, and even fecal and urinary incontinence. Other symptoms include: axonal degeneration of
peripheral nerves and cns symptoms including memory loss, irritability, and dementia.

94. Old lady with forgetfulness and numbness


A. B12 deficiency

95. Wernicke's area pathology leads to:


Damage caused to wernicke's area results in receptive, fluent aphasia. This means that the person with aphasia will be able to
fluently connect words, but the phrases will lack meaning. This is unlike non-fluent aphasia, in which the person will use meaningful
words, but in a non-fluent, telegraphic manner.

96. Blindness. Numbness that comes and goes (multiple sclerosis). Diagnostic test
A. Mri
Reference: http://emedicine.medscape.com/article/1146199-workup#c10

97. Case of a lady presented with inability to blink the eye, unsteady gait, something, she had brain tumor in the ct it involve
perventrcular, part of brain i don't remember and cord. Tumor type?
Answer: missing info

98. Brain tumors


http://www.disabled-world.com/health/cancer/brain/

99. Case of old patient with signs and symptoms of parkinson disease, where is the lesion?
Answer: basal ganglia, mainly substantia nigra
The dopaminergic tract is predominantly affected in parkinson disease, and histologically, it is characterised by nigrostriatal
dopaminergic degeneration leading to neuronal loss in the substantia nigra pars compacta, most conspicuous in the ventrolateral
tier of neurons. A number of other regions including parts of the basal ganglia, brainstem, autonomic nervous system and cerebral
cortex. Link: http://radiopaedia.org/articles/parkinson-disease-1

100.Child after hx of urti developed ascending muscle weakness?


A. Guillian-barre syndrome
Answer: a
Link: http://emedicine.medscape.com/article/315632-clinical

195
101.Brest feeding mother k/c of seziure on phenytoin ask about brest feeding:
Answer: regarding to nuropedia consultant : answer either to stop feeding which better or change to another antiepileptic drug not
excreted in milk both answer right

102.Which nerve is responsible for muscle that causes tongue movement?


Answer: hypoglossal

103.On the eye type of headache:


Answer: clusters headache

104.Antidote of opiate:
A. Naloxone
Answer: a

105.Multiple sclerosis present to er. What to give ?


Medical management goals that are sometimes achievable in the emergency department are to relieve symptoms and to ameliorate
risk factors associated with an acute exacerbation. In patients with fulminant ms or disseminating acute encephalitis, management
involves the following:
- stabilize acute life-threatening conditions
- initiate supportive care and seizure precautions
- monitor for increasing intracranial pressure
consider intravenous steroids, IV immunoglobulin (ivig), or emergent plasmapheresis. One study suggested that plasmapheresis may
be superior to IV steroids in patients with acute fulminant ms, the 2011 american academy of neurology (aan) plasmapheresis
guideline update states that plasmapheresis is possibly effective and may be considered in acute fulminant demyelinating cns
disease.
Identification and control of known precipitants of ms exacerbation include the following:
aggressively treat infections with antibiotics
in patients with a fever, normalize the body temperature with antipyretics, as even small increases in temperature can strongly
affect conduction through partially demyelinated fibers
provide urinary drainage and skin care, as appropriate.
Reference: medscape

106.Treatment of cluster headache:


A. 100% oxygen
Answer: a
100% oxygen by mask is the abortive therapy for cluster headache.
Reference: master the board

196
107.Case of gbs. What you will find in lp?
Answer: high protein
Guillain-barre syndrome (diagnosis):
a. Csf analysis—elevated protein, but normal cell count
b. Electrodiagnostic studies—decreased motor nerve conduction velocity
Reference: step up to medicine

108.Cafe au lait spots, to diagnose neurofibromatosis:


A. Presence of axillary freckling
Answer: a
toronto:
Diagnosis of nf-1 requires 2 or more of:
≥6 café-au-lait spots (>5 mm if prepubertal, >1.5 cm if postpubertal)
≥2 neurofibromas of any type or one plexiform neurofibroma
≥2 lisch nodules (hamartomas of the iris)
Optic glioma
Freckling in the axillary or inguinal region
A distinctive bony lesion (e.g. Sphenoid dysplasia, cortical thinning of long bones) ƒ
First degree relative with confirmed nf-1

109.Epileptic patient with gingival bleeding and white gain which medication:
A. Phenytoin
Answer a
Http://reference.medscape.com/drug/dilantin-phenytek-phenytoin-343019#5
Http://emedicine.medscape.com/article/1076264-overview#showall

110.Patient taking carbamazepine developed generalized rash and peeling of epidermis:


Answer steven johnson syndrome
Http://emedicine.medscape.com/article/1197450-overview#a5

111.Best treatment of trigeminal neuralgia?


Toronto:

112.70 Year-old female brought to your clinic by her daughter . The daughter said her mother's memory deteriorated in the last 2
years . She can dress her self but with difficulty , she can cook for herself but sometimes leave the oven on ,,,,,,,,,what's the
management ?
A. Refer her to geriatric clinic.
197
113.Old patient with alzheimer's dementia, became agitated and have hallucinations and delusions. What is the appropriate drug
in his case ?
A. Haloperido

114.Ascending paralysis with areflexia post urti


A. Gullen barre syndrome (gbs)
Answer:
Inflammatory demyelinating polyneuropathy, that primarily affects motor nerves tract.
Usually proceeded by a viral or mycoplasma infection or upper respirator or gi tract. Common infection may include
campylobacter jejune, CMV , hepatitis, and hiv.
rd
Reference: step up to medicine 3 edition.pg 223

115.Alzheimer disease or lowes bodies ct brain changes ?


The initial criteria for ct scan diagnosis of alzheimer disease includes diffuse cerebral atrophy with enlargement of the cortical sulci
and increased size of the ventricles. A multitude of studies indicated that cerebral atrophy is significantly eater in patients with
alzheimer disease than in patients who are aging without alzheimer disease.
Ref: http://emedicine.medscape.com/article/336281-overview#a2

s an epileptic seizure of greater than five minutes or more than one seizure within a
five-minute period without the person returning to normal between them
SE is defined as a continuous seizure lasting more than 30 min, or two or more seizures without full recovery
116.Definition of status epilepticus? of consciousness between any of them
Status epilepticus (se) is a common, life-threatening neurologic disorder that is essentially an acute, prolonged epileptic crisis. In
patients with known epilepsy, the most common cause is a change in medication. Most seizures terminate spontaneously.
Http://emedicine.medscape.com/article/1164462-overview

117.Clinical dementia ?
Dementia is a pattern of mental decline caused by different diseases or conditions. Most commonly, dementia occurs when brain
nerve cells (neurons) die, and connections between neurons are interrupted. These disruptions have a variety of causes and usually
cannot be reversed. Alzheimer's disease causes over 60% of all dementias. Vascular disease, such as stroke, is the second most
common cause. In rare cases, dementia is caused by a treatable condition, and it may be partially or entirely reversed if the
condition is diagnosed and treated early.
Http://www.drugs.com/health-guide/dementia.html

118.Brain ventricles anatomy


The neural canal dilates within the prosencephalon, leading to the formation of the lateral ventricles and third ventricle. The cavity
of the mesencephalon forms the cerebral aqueduct. The dilation of the neural canal within the rhombencephalon forms the fourth
ventricle.
The lateral ventricles communicate with the third ventricle through interventricular foramens, and the third ventricle communicates
with the fourth ventricle through the cerebral aqueduct.
Http://emedicine.medscape.com/article/1923254-overview#a2

119.What is a type of headache that a patient who have stress at school had:

198
A. Tension
Answer:a
Tension headache never happens during sleep, gradual over 24 h. The site is posterior/occipital.
Increases with stressors. Treated by modifying stressor(s), local measures, NSAIDs, tricyclic antidepressants.
Reference: toronto notes

120.Most common brain cancer?


A. Astrocytoma
Explanation: astrocytoma type of glioma which is the most common brain tumors
Reference: http://www.neurosurgery.pitt.edu/centers-excellence/neurosurgical-oncology/brain-and-brain-tumors/types-brain-
tumors

121.Sign in duchene muscular dystrophy?


A. Growers sign

122.Case of nuroparlysis and numbness in all extermity .

123.Duchenne muscular dystrophy:


Answer: proximal muscle wasting

124.Ascending paralysis with areflexia ( no reflex) post URT infection;


A. Gullen Barre syndrome
Answer:
GBS is a demyelinating neuropathy with ascending weakness.

Http://emedicine.medscape.com/article/315632-overview#a2

125.Patient came with severe pain involving the forehead to nose (trigeminal distribution) what is the diagnosis?
A. Trigeminal neuralgia

Answer: A
Trigeminal neuralgia is an idiopathic disorder of the fifth cranial nerve resulting in severe, overwhelming pain in the face.
Attacks of pain can be precipitated by chewing, touching the face, or pronouncing certain words in which the tongue strikes
the back of the front teeth. Patients describe the pain as feeling as if a knife is being stuck into the face.
There is no specific diagnostic test.
Treat with oxcarbazepine or carbamazepine. Baclofen and lamotrigine have also been effective. If medications do not
control the pain, gamma knife surgery or surgical decompression can be curative.
Reference: Master the Boards

126.B6 & B12 deficiency

199
Answer: ??
➢ B6 deficiency: Overt deficiencies of vitamin B6 are probably rare. Marginal deficiencies may be more common, manifested as
nonspecific stomatitis, glossitis, cheilosis, irritability, confusion, and depression.
➢ The classic clinical picture of cobalamin deficiency (Vitamin B12), mentally sluggish, shiny tongue (atrophic glossitis), and a
shuffling broad-based gait. Hematologic changes (e.g., macrocytic anemia with oval macrocytes and increased neutrophil
lobulation) and neurologic abnormalities classic picture of subacute combined degeneration of the dorsal (posterior) and lateral
spinal columns, neuropathy which is symmetrical and affects the legs more than the arms. It begins with paresthesias and ataxia
associated with loss of vibration and position sense, and can progress to severe weakness, spasticity, clonus, paraplegia, and
even fecal and urinary incontinence.
Reference: Uptodate

127.What is the treatment for trigeminal neuralgia?


A. Carbamazepine
Answer: A
1st line: carbamazepine or oxcarbazepine.
Reference: Toronto notes

128.A patient with facial nerve involvement. Presents with loss of taste sensation of the anterior 2/3 of tongue. There is loss of
function of stapedius as well. At what point is the injury?
*options includes different points of facial nerve course
Answer: ?
Facial canal between geniculate ganglion and nerve to stapedius muscle.
Reference: http://www.ncbi.nlm.nih.gov/books/NBK385/

129.Postherpetic neuralgia treatment


A. Antiviral
Answer: TCA (Amitriptyline or Nortiptyline), Antiviral aims to shorten the clinical course, prevent complications, prevent the
development of latency and/or subsequent recurrences, decrease transmission, and eliminate established latency. (Medscape)

200
Reference: Master the board

130.What the investigation should be done before lumbar puncture?


A. Platelets
Answer: A
We recommend NOT performing an LP in patients with coagulation defects who are actively bleeding, have severe
thrombocytopenia (eg, platelet counts <50,000 to 80,000/µl), or an INR >1.4, without correcting the underlying abnormalities.
Reference: uptodate

131.In Guillain-Barré syndrome, which cell is affected?


Answer: Schwann cells
Guillain-Barré syndrome; Rapidly progressive limb weakness that ascends following GI/ upper respiratory infection.
Reference: http://www.uptodate.com/contents/pathogenesis-of-guillain-barre-syndrome

132.Clear case of absence seizure, what happens if we give fentanyl?

Answer: fentanyl-induced epileptiform activity on the electrocorticogram


Reference :http://www.epilepsy.com/information/professionals/diagnosis-treatment/procedures-epilepsy-patients/general-
anesthetics-4

133.Patient is seizure sence 35 min, he take diazepam i.v but nor effective , what will you do ? Frn
Answer:
Phenyton
➢ In the initial therapy phase, a benzodiazepine (specifically IM midazolam, IV lorazepam, or IV diazepam) is recommended as
initial therapy.
➢ In the second phase, options include IV fosphenytoin, valproic acid, or levetiracetam. If none of these is available, IV
phenobarbital is a reasonable alternative.
➢ In the third phase, if a patient experiences 40+ minutes of seizure activity, treatment considerations should include repeating
second-line therapy or anesthetic doses of thiopental, midazolam, pentobarbital, or propofol
Http://emedicine.medscape.com/article/1164462-treatment
201
Http://www.ncbi.nlm.nih.gov/pmc/articles/PMC2824929/

134.CLUSTER HEADACHE TREATED BY:


A- Oxygen
Answer: A
Pharmacologic management of cluster headache (CH) may be classified into 2 general approaches as follows:
Abortive/symptomatic (eg, oxygen, triptans, ergot alkaloids, and anesthetics)
Preventive/prophylactic (eg, calcium channel blockers, mood stabilizers, and anticonvulsants)
Reference: http://emedicine.medscape.com/article/1142459-treatment

135.Patient is seizure sence 35 mint, he take diazepam I.V but not effective , what will you do ? “ Phenytoin is not in the choices ”
Answer:
Seizures occurring continuously for at least 30 minutes, or two or more seizures occurring without full recovery of consciousness
between attacks.
■First-line therapy = benzodizepines—diazepam, lorazepam, or midazolam.
- Lorazepam has a relatively longer duration of seizure suppression.
■ Second-line therapy = phenytoin and/or phenobarbital
-Phenytoin: Rapid administration may cause hypotension and cardiac dysrhythmias this may be avoided with fosphenytoin .
- Phenobarbital: Anticipate sedation, respiratory depression and hypotension.
■ Drug-induced coma (pentobarbital, midazolam, propofol) or general anesthesia, if resistant to above
Reference : first aid of emergency

136.A pateint with difficulty in swallowing she has frontal baldness and cataract (other symptoms that i can't remember) her
mother has the same condition
Myotonic muscular dystrophy
Answer: a
((first aid for the usmle step 1 ck 2014,p89 ))

137.3rd Cranial palsy ?


Answer: Out and down

202
138.Calculate gcs?

203
Infectious disease

204
1. Asyptomatic pt. With positive HBV antigen?

A. Acute hepatitis
B. Chronic hepatitis
C. Active carrier
D. Non active carrier

Answer: inactive carrier asymptomatic, carry HBV antigen more than 6m and HBVeab

2. What is the common dz to make the pt. Retire in ksa?


A. HBV
B. Hbc
C. Hiv
D. Hep a

Answer: c

3. Ksa have implemented strong regulations regarding worker health cares, which of the following diseases if the worker had,
he can't work? Repeated in family medicine
A. HBV
B. Hcv
C. Hiv
D. All of the above

Answer: d

4. After appendectomy a patient got abdominal infection by enterococcus faecium. He is allergic to penicillin what are you
going to give him:
A. Ceftriaxone
B. Vancomycin
C. Cefotaxim
D. Tmp-smx or metronidazole (not sure which one of them was mentioned)
Answer: b

205
Fortunately, there are a number of antibiotics with broader spectrum and high efficacy that can be used as alternatives to penicillin.
The chances of cross reactivity between penicillins and antibiotics of other groups are rare. Therefore, other antibiotics can be used
safely in patients with history of penicillin allergy. However, the choice of alternative antibiotic depends upon the kind of infection
that needs to be treated.
Http://medlicker.com/880-allergy-to-penicillin-alternative-antibiotics

5. Which vaccine is contraindicated in HIV patient


A. Opv
B. Varicella
C. Mmr
D. HBV
Answer: a ** the live, attenuated oral polio vaccine (opv) is not recommended for persons with hiv. Reference :
http://hivinsite.ucsf.edu/insite?Page=kb-03-01-08

6. Methicillin-resistant, sensitive to chloramphenicol and aminoglycosides. What not to give?


A. Gentamycin
B. Azithromycin
C. Chloramphenicol
D. Vancomycin
Answer: b

7. A patient has symptoms of infectious mononucleosis. Monospote test was positive. After 8 days, he suddenly complained of
acute abdominal pain, decrease Blood pressure . What will you do?
A. Antibiotic
B. Fluid resuscitation, urgent ct abdomen
C. Urgent gasto.
D. Barium enema
Answer: b

8. Patient has uti organism grows on both antiseptic & detergent?


A. E.coli
B. Proteus
C. Pseudomonas
D. Staph areus or strep
Answer: c, pseudomonas is associated with nosocomial infections.
Http://www.soeagra.com/abr/december%202011/9.pdf

9. Methicillin-resistance, sensitive to chloramphenicol and aminoglycosides, what not to give?


A. Gentamycin
B. Azithromycin
C. Chloramphenicol
D. Vancomycin
Answer: b
mrsa is most commonly resistant to:
beta-lactams (penicillins and cephalosporins), fluoroquinolones (e.g., levofloxacin), macrolides (e.g., erythromycin, azithromycin)
206
Mrsa can usually be treated with “last-resort” antibiotics: clindamycin, vancomycin, linezolid and daptomycin
http://www.pewtrusts.org/en/research-and-analysis/issue-briefs/2012/04/03/mrsa-a-deadly-pathogen-with-fewer-and-fewer-
treatment-options

10. Immune deficient patient what vaccine could be given:


A. Meseals
B. Rubella
C. Pneumococcal
D. Varicella
Answer: c
https://www.cdc.gov/vaccines/pubs/pinkbook/downloads/appendices/a/immuno-table.pdf

11. (long scenario) report {single strand RNA)?


A. Hepatitis a
B. Hepatitis b
C. Hepatitis c
D. Hepatitis d.
Answer: scenario is missing information to decide between option a and option c
The structure of hepatitis viruses are as follow:
Hepatitis a virus (non-enveloped single-stranded RNA)
Hepatitis b virus (enveloped double-stranded dna)
Hepatitis c virus (enveloped single-stranded RNA)
Hepatitis d virus (envelop from HBV, single-stranded RNA)
Hepatitis e virus (non-enveloped single-stranded RNA)
Link: http://www.ncbi.nlm.nih.gov/books/nbk7864/

12. A patient presented with (meningitis symptoms) and history of swimming in the river, what is the organism? Repeated
multiple times
A. S. Pneumoniae
B. H. Influenzae
C. N. Meningiditis
D. Naegleria fowleri
Answer is d
Primary amebic meningoencephalitis (pam) is a very rare form of parasitic meningitis that causes a brain infection that is usually
fatal. Pam is caused by the microscopic ameba (a single-celled living organism). Naegleria fowleri when water containing the ameba
enters the body through the nose.
Source: http://www.cdc.gov/meningitis/amebic.html

13. Enteric fever resistanant to chloramphenicol, what's next ?


A. Double chloramphenicol
B. Ciprofloxacin alone
C. Add ciprofloxacin
D. Im ceftriaxone
Answer: d?

207
In general, preferred antibiotics include ceftriaxone 1 g im or IV q 12 h for 14 days and various fluoroquinolones (eg, ciprofloxacin
500 mg po bid for 10 to 14 days, levofloxacin 500 mg po or IV once/day for 14 days, moxifloxacin 400 mg po or IV once/day for 14
days). Chloramphenicol 500 mg po or IV q 6 h is still widely used, but resistance is increasing.

14. A family who ate at a restaurants come to the er with diarrhea (…sing of food poising). After 24 hours all of them improved.
Their cultures showed gram positive spore with oxidative +ve what the organism??
A. Salmonella typhi
B. Staphylococcus aureus
C. Shigella
D. Bacillus
Answer: d
Spore-forming pathogenic bacteria in ready-to-eat food. Bacillus cereus and clostridium perfringens are examples of pathogenic
bacteria that can exist in both spore and vegetative forms. The thick-walled structure of the spore is resistant to heat and it can help
the pathogenic bacteria survive the heat of cooking.
Http://www.cfs.gov.hk/english/multimedia/multimedia_pub/multimedia_pub_fsf_25_02.html

15. On malaria fast diagnostic test what will you see? ( something like that)
A. Malaria antigens
B. Malaria antibodies
C. Malaria pigments
D. Parasite sexual
Answer: a
Https://www.cdc.gov/malaria/diagnosis_treatment/rdt.html

16. Patient with ventilator associated pneumonia. Culture showed lactose non-fermenting, gram negative motile bacilli
producing greenish colony and oxidase positive. What is the organism?
A. Coli
B. Pseudomonas
C. Klebsiella
D. Proteus
Answer: b

17. Giardia diagnostic test is?


A. 3 stool parasite
B. 3 stool cultures
C. Concentration test
D. Stool immunoassay.
Answer: d
Http://www.cdc.gov/parasites/giardia/diagnosis.html

18. Patient diagnosed with rheumatic heart disease and mitral regurge. What is the treatment of choice?
A. Oral penicillin and aspirin (i choose this one)
B. Im penicillin once every month
C. Corticosteroids
D. IV penicillin
208
Answer: a
Oral (po) penicillin v remains the drug of choice for treatment of gabhs pharyngitis. Aspirin in anti-inflammatory doses effectively
reduces all manifestations of the disease except chorea, and the response is typically dramatic.

19. What is the chemoprophylaxis for vibrio cholera in pandemic area?


A. Penicillin
B. Cephalexin
C. Tetracycline
D. Erythromycin
Answer: c. Tetracycline. Cdc http://www.cdc.gov/cholera/treatment/antibiotic-treatment.html

20. What is the prevention method for food poisoning?


A. Prolong heating and re cooking
B. Antibiotic
C. Prolong mastication
D. No peeled fruit in choices
Answer: a

21. Patient with end stage liver disease, on central venous line and septic. His blood culture showed budding yeast. What anti-
fungal is appropriate at this stage?
A. Cuspofungin
B. Flucytosine,
C. Fluconazole
D. Itraconazole
Answer a
Cuspofungin is used for systemic candiasis that could be caused by indwelling devices. Flucoenazol is used for cryptococcal
meningitis, disseminated histoplasmosis and cocccidiodomycoses. Itraconazole is used for dermatophytes mucocautenous candiasis
and seborrheic dermatitis. Flucytosine is a weak agent that’s rarely used alone.

22. What is the definitive test for diagnosing tb?


A. Ppd
B. Sputum culture
C. Blood culture
D. Serum specific antigen or breathing test urease
Answer: b
Sputum studies (sputum acid-fast testing): definitive diagnosis is made by sputum culture—growth of m. Tuberculosis, must obtain
three morning sputum specimens—culture takes 4 to 8 weeks.
Cxr: classic findings are upper lobe infiltrates with cavitation. Other possible findings: pleural effusion, ghon complex and ranke
complex: evidence of healed primary tb.
-tuberculin skin test (ppd test) is a screening test to detect those who may have been exposed to tb. It is not for diagnosis of active
tb.

23. Female with uti most common cause?


A. Napkin use
B. Cab for contraception
209
C. Back to front cleaning
D. Using of pad
Answer: c

24. HIV patient ... (symptoms of intestinal obstruction) did intestinal resection. They found tumor white in color nearly encircling
the wall. What is the tumor?
A. Hodgkin’s
B. Non hodgkin’s
C. Adenocarcinoma
D. Plasmacytoma
Answer: c
All blood cancers except adenocarcinoma

25. Pneumococcal vaccine conjugate 13?


A. Active vaccine
B. Live attunuated vacine
C. Conjugated (conjugated polysaccharide)
D. Inactive vaccine
Answer: c
Pneumococcal conjugate vaccine (pcv, initially marketed as a 7-valent vaccine, pcv7 [prevnar or prevnar 7], now replaced by pcv13
[prevnar 13]) consists of capsular polysaccharides from the 13 most common types that cause disease, covalently linked to a
nontoxic protein that is nearly identical to diphtheria toxin. This covalent linking to a protein renders the polysaccharide antigenic in
infants and toddlers.

26. Shigela spices treatment:


A. Metrondozole
B. Azthromycin
C. Amoxicillin
D. Ceftrixone
Answer: d
In the setting of shigellosis that may have been acquired in the asian sub-continent, empiric antibiotic therapy may require a third-
generation cephalosporin, given widespread resistance to ciprofloxacin, trimethoprim-sulfamethoxazole, and azithromycin in that
region.

27. What transmitted infection will require relieve from work in health care:
A. HBV
B. HBV
C. HAV
D. HIV
Not sure about the answer
In general, three conditions are necessary for health-care personnel to pose a risk for bloodborne virus transmission to patients.
First, the health-care provider must be sufficiently viremic (i.e., have infectious virus circulating in the bloodstream). Second, the
health-care provider must have an injury (e.g., a puncture wound) or a condition (e.g., nonintact skin) that allows exposure to
his/her blood or other infectious body fluids. Third, the provider's blood or infectious body fluid must come in direct contact with a
patient's wound, traumatized tissue, mucous membranes, or similar portal of entry during an exposure-prone procedure. The vast

210
majority of HIV and HBV-infected health-care personnel pose no risk for patients because they do not perform activities in which
both the second and third conditions are met.

All healthcare workers have to go through standard healthcare clearance when applying for a medical post or training. This involves
being tested for tb, being offered a hepatitis b vaccination, and being offered tests for HIV and hepatitis c. The HIV and hepatitis c
tests are not compulsory. A positive HIV diagnosis, or declining an HIV test should not affect the employment or training of
healthcare workers who will not perform epps (exposure prone procedures). Healthcare workers who are applying for a post or
training which does involve epps have to go through additional healthcare clearance. This means testing negative for hiv, hepatitis b
and hepatitis c.

28. Patient presented with oropharengeal maculopapular rash plus rash in palm and soles?
A. CMV
B. EBV
C. Vaccine virus
D. Coxsackievirus virus
Answer: d
This seems to be a case of hand-foot-mouth disease, in which the patient has eruptions starting on the back of the throat and may
spread to the hands and soles. It usually affects children but it can also affect adults.
Reference: http://emedicine.medscape.com/article/218402-clinical

29. Most cases of cerebral edema were found to be associated with which of the following organisms?
A. Plasmodium malaria
B. P. Ovale
C. P. Vivax
D. P. Falciparum
Answer: d
Most observations of the pathophysiology of disease come from postmortem observations of plasmodium falciparum (pf) infections,
which are thought to account for the vast majority of cm cases, and show a common feature of vascular sequestration of infected
erythrocytes (ie) in the brain.

30. HIV attacks?


A. B cells
B. T cytotoxic cells
C. Macrophages
D. T helper cells
Answer: d
HIV produces cellular immune deficiency characterized by the depletion of helper t lymphocytes (cd4+ cells).
Http://emedicine.medscape.com/article/211316-overview#a3

31. Patient came from africa develops symptoms "i don't remember all of them but i’m sure he had arthralgia" the vitals normal
"no fever" what is the diagnosis:
A. Yellow fever
B. Ebola
C. Lassa fever
D. Chikungunya fever
Answer: d
211
All of the options come with high fever but chikungunya is characterized by an abrupt onset of fever frequently accompanied by
joint pain. The joint pain is often very debilitating, but usually lasts for a few days or may be prolonged to weeks. Most patients
recover fully, but in some cases joint pain may persist for several months, or even years. The disease occurs in africa, asia and the
indian subcontinent. Http://www.who.int/mediacentre/factsheets/fs327/en/

32. What is the chemoprophylaxis for vibrio cholera in pandemic area?


A. Penicillin
B. Cefalexin
C. Tetracyclin
D. Erythromycin
Answer: c
According to recent guidelines by cdc and who chemoprophylaxis is not indicated anymore. But most resources are directing toward
tetracycline as the first line.
Http://www.cdc.gov/cholera/treatment/antibiotic-treatment.html

33. Gram negative bacteria oxidase, non lactose fermenter which of the following the best is antimicrobial ?
1) Ceftriaxone
2) Cefipime
3) Ciprofloxacin
4) Smz –tmp (bactrem)

Answer: c

34. Patient came from africa with fever , myalgia and arthralgia ?
A. Ebola fever
B. Lassa fever
C. Yellow fever
D. Malaria

Answer: d

35. Fever, headache, with macule, papule, pustules & vesicles over the face, scalp & the trunk ; what is the causative organism ?
A. Herpes type 6 (roseola)
B. CMV
C. Herpes zoster
D. EBV
Answer: c?? Age in infancy and any missing symptoms mainly an episode of seizure might change the diagnosis to a
Http://emedicine.medscape.com/article/1132465-overview

36. What is the mechanism of prophylactic antiviral that given against flu?
A. DNA polymerase 2.
B. RNA1
C. RNA2 ??
D. RNA 3
Answer:
212
37. HIV patient come with diffuse pustule in skin an mouth, management is :
A. Topical steroid
B. Oral ab
C. Topical ab
D. Chemo & radiotherapy
Answer: a

38. Patient is presented with hand cellulitis and red streaks in the hand and tender axillary lymphadenopathy. This
condition is more likely to be associated with:
A. Malignancy
B. Pyoderma
C. Neuropathy
D. Lymphangitis
Answer : d
Lymphangitis is defined as an inflammation of the lymphatic channels that occurs as a result of infection at a site distal to the
channel
O/e
Erythematous and irregular linear streaks extend from the primary infection site toward draining regional nodes. These streaks may
be tender and warm.
Reference: http://emedicine.medscape.com/article/966003-clinical#showall

39. Patient with ventilator associated pneumonia. Culture showed lactose non-fermenting, gram negative motile bacilli
producing greenish colony + Oxidase positive. What is the organism?
A. Haemophilus influenza
B. Streptococcus pneumoniae
C. Klebsiella or other gram negative bacteria
D. Pseudomonas aeruginosa
Answer: D

Reference: http://www.columbia.edu/itc/hs/medical/pathophys/id/2008/utignr.pdf

213
40. What is the optimal duration of antibiotic treatment in strep throat?
A. 3 days
B. 5 days
C. 7 days
D. 10 days

Answer: D
Penicillin is the treatment of choice for strep throat. It is usually given in pill or liquid form, two to four times per day for 10 days.
Reference: uptodate.

41. Young adult came complaining of painless penile ulcer, what is the appropriate investigation to do?
A. Blood culture.
B. Excisional biopsy.
C. Swab culture and urinalysis.
D. Darkfield microscope.
E. CBC & ESR.

Answer: D
Primary syphilis usually begins with a single, painless, well-demarcated ulcer (chancre) with a clean base and indurated border.
Treponema pallidum is identified on darkfield microscopy or direct fluorescent antibody testing of a chancre or lymph node aspirate.
Reference: http://www.aafp.org/afp/2012/0201/p254.html

42. A patient was diagnosed with enteric fever. What is the presentation that he will have?
A. Confusion (or other CNS problems)
B. Maculopapular rash
C. Nausea, vomiting and loose stools
D. Abdominal pain, headache, fever( newly added)

Answer: D
Signs and symptoms of Salmonella typhi (aka Enteric Fever, Typhoid) include "Rose spot" rash (on anterior thorax, upper abdomen),
sustained fever 39° to 40° C (103° to 104° F), abdominal pain precedes diarrhea/ constipation, headache, loss of appetite and cough.
Toronto Notes. If not treated the patient may become delirious. Reference: Mayoclinic

43. A patient with signs of TB. What vaccination you would give to his family?
A. MMR
B. Dtap
C. BCG
D. Polio

Answer: C
Reference: kumar and clark’s

44. Swimming in river pool and got infection


A. Streptococcus
B. H influenza

214
C. N gonorhea
D. Something i have never heard of

Answer: Most common organisms acquired from swimming pools Bacteria: Shigella and E.coli / Protozoa : Cryptosporidium and
Giardia./ Viruses : Adenovirus , Echovirus , Norwalk virus, and Hepatitis A.
Reference: WHO and CDC
Http://www.who.int/water_sanitation_health/bathing/recreaii-ch3.pdf
Http://www.cdc.gov/healthywater/swimming/rwi/

45. Enteric fever best diagnosed in the first week of presentation by?
A. Blood culture
B. Stool culture
C. Multiple something?
D. Bone marrow>>not sure

Answer: A
Blood, intestinal secretions (vomitus or duodenal aspirate), and stool culture results are positive for S typhi in approximately 85%-
90% of patients with typhoid fever who present within the first week of onset.
Http://emedicine.medscape.com/article/231135-workup

46. Fever, headache, with macules, papules, pustules & vesicles over the face, scalp & the trunk ; what is the causative organism
?
A. Herpes type 6
B. CMV
C. Herpes zoster
D. EBV

47. What is the cause to use every year influenza vaccine ?


A. Resistance of antimicrobial
B. New antigen
C. Different type of transmission
D. Drift .. So could be new antigen .

48. Dm, hypothyroid, irregular menses female, present with recurrent itching & white adherent oral plaque , +ve mantux test ,
she was exposed to tb 4 years ago, immunoglobulin, wbc, RBCs all are normal ; dx ;
A. Chronic granulomatous disease
B. Chronic candidiasis
C. Digeorge syndrome
D. Hyperglobulinemia ( or hypo i don't remember )
Answer: risk factors for candida albicans are: dm,broad sprectrum antibiotic use, pregnancy, conrticosteroids, hiv, ocp use, increase
frequency of intercourse.reference: step2ck 8th edition pg 333

49. Old pt came to er 4 w of fever cough night sweat... (clear t.b symtome) immeditly do !??
A. Put pt in negtive pressure

215
B. Give anti t.b drug
C. Sputum culture
D. Chest x ray

50. HIV presents commonly with?


A. Opportunistic infection
B. Chronic diarrhea
C. Generalized lymphadenopathy

Answer: c
Early symptomatic HIV infection includes persistent generalized lymphadenopathy, often the earliest symptom of primary HIV
infection. The most common findings include fever and chills, lymphadenopathy, pharyngitis, anemic pallor, and rash. (flu-like
symptoms)
Reference: http://reference.medscape.com/article/211873-overview#a4

51. Healthy pt with no symptoms x ray is normal, has negative hx of tuberculin test, now has positive test ..?
A. Reassure
B. Give rifampicin and iso
C. Give iso for 6 months

Answer: most likely c more specific


6-month or 9-month isoniazid daily,
3-month rifapentine plus isoniazid weekly,
3 or 4-month isoniazid plus rifampicin daily,
3 or 4-month rifampicin alone daily.
References: http://www.cdc.gov/tb/topic/treatment/ltbi.htm
Http://www.who.int/tb/challenges/ltbi/en/

52. HIV patient, what test to confirm it?


A. PCR
B. Western blot
C. Elisa

Answer: b western blot is confirmatory.


Elisa is for screening. If it is negative, other tests are not usually needed.
(master the boards)

53. Ksa have implemented strong regulations regarding worker health cares, which of the following diseases if the worker had ,
he can't work ? Repeated in family medicine
A. Heb
B. Hec
C. Hiv

Answer:
216
Q is not clear and no official resources were found
Most likely HIV (though hepatitis will prevent foreigner workers from working in ksa)
''The aids test is mandatory, including for spouses and, if you’re shown to be HIV positive, you will be expelled immediately''.
Reference:
Https://www.justlanded.com/english/saudi-arabia/saudi-arabia-guide/jobs/employment-contracts

54. Which part of bone is firstly affected in hematogenous osteomyelitis?


A. Epiphysis
B. Metaphysis
C. Diaphysis

Answer:
In children: metaphysis
Reference: http://www.uptodate.com/contents/hematogenous-osteomyelitis-in-children-epidemiology-pathogenesis-and-
microbiology?Source=search_result&search=hematogenous+osteomyelitis&selectedtitle=1%7e30
In adults: diaphysis
Reference: http://www.uptodate.com/contents/hematogenous-osteomyelitis-in-
adults?Source=search_result&search=hematogenous+osteomyelitis&selectedtitle=3%7e30

55. HIV patient, what test confirm it?


A. PCR
B. Western blot
C. Elisa

Answer: b
The best initial test for HIV is the elisa test. This is confirmed with western blot testing. Infected infants are diagnosed with PCR or
viral culture.
Reference: master the boards usmle step 2

56. Patient came from sudan two weeks ago and developed fever headache and vomiting what is the best diagnostic test?
A. Blood culture
B. Stool culture
C. Peripheral blood picture.

Answer c.
Malarial infection is suspected. Individuals are generally asymptomatic for 12 to 35 days but can commence symptoms as early as 7
days (depending on parasite species) in most cases, the incubation period for p. Falciparum infection is about 12 to 14 days (range 7
to 30 days); most infections due to p. Falciparum become clinically apparent within one month after exposure. Detection of
parasites on giemsa-stained blood smears by light microscopy is the standard tool for diagnosis of malaria and remains the most
common onsite diagnostic method
references: http://www.uptodate.com/contents/clinical-manifestations-of-malaria
http://www.uptodate.com/contents/diagnosis-of-malaria

217
57. HIV presents commonly with?
A. Opportunistic infection
B. Chronic diarrhea
C. Generalized lymphadenopathy

Answer: a
The patient may present with signs and symptoms of any of the stages of HIV infection. Acute seroconversion manifests as a flulike
illness, consisting of fever, malaise, and a generalized rash. The asymptomatic phase is generally benign. Generalized
lymphadenopathy is common and may be a presenting symptom.
Reference: http://emedicine.medscape.com/article/211316-clinical

58. Parasite in soil contamination:


A. Tenia saginata
B. Ascaris
C. Bancrofti
Answer: b
Soil-transmitted helminths refer to the intestinal worms infecting humans that are transmitted through contaminated soil
("helminth" means parasitic worm): ascaris lumbricoides (ascaris), whipworm (trichuris trichiura), and hookworm (anclostoma
duodenale andnecator americanus).
Reference: cdc

59. Ppd was +ve , to prevent false +ve , what to do?!


A. Repeat it.
B. Do x-ray.
C. Do mantoux test.

Answer: b
Resource: previously written!

• Interferon gamma release assay (igra) after six weeks and repeat the mantoux test to increase the sensitivity (to reduce
false negative results).
• Since the quantiferon®-tb gold (qft-g) blood-based test does not cross-react with bcg, this test is particularly useful for
testing individuals with a history of bcg vaccination㜀• patients who have a positive tst reaction should undergo clinical
evaluation, including a chest x-ray (cxr) to rule out tb. If the ini- tial cxr is normal, repeated ones are not indicated unless
the individual develops signs or symp- toms of tb. Tst-positive individuals should be started on treatment for ltbi according
to the guidelines in
• Qft-g can yield cost savings in terms of medical staff time—both by elimination of a second patient visit for test
interpretation and by the elimination of common false-positive results, which typically involve both unnecessary follow-up
testing and treatment for ltbi.

60. What case infection by food with soil contaminated?


A. Ascaris
B. Tinae
C. Schistosoma
Answer: a

218
61. Patient presented with unilateral eye swelling with purple skin discoloration, what is the organism?
A. Staph.aures
B. B hemolytic strep
C. Haemophilus influenzae
Answer: c

62. Patient has been treated four times by sulfonamide antibiotics and there were lesions in the glans penis and scrotum each
time he was using that drugs ,what is the best explanation of this lesion ?
A. Vesicle
B. Patchy red nodule
C. Redness with ulcers & blister
Answer: c
Read about balanitis
Http://www.hopkinsmedicine.org/healthlibrary/conditions/dermatology/drug_rashes_85,p00280/
Http://emedicine.medscape.com/article/777026-clinical#b5

63. Transmitted of parasite by ingestion of undercooked meat ?


A. Schistosoma
B. Tinea saginata
C. Tenia solium
Answer: b
Tenia solium is more associated with pork
Schistosoma infection occurs when skin comes in contact with contaminated freshwater in which certain types of snails that carry
the parasite are living.

64. Fever, malaise, maculopapular rash over the body & behind the ear; causative organism
A. Rubella
B. Measles
C. Mumps
Answer:
Measles: distribution: starts at hairline and spreads downwards with sparing of palms and soles
Rubella: distribution: starts on face and spreads to neck and trunk
Roseola: distribution: starts at the neck and trunk and spreads to the face and extremities
Http://www.healthline.com/health/measles

65. Enteric fever presentation;


A. Fever, headache, abdominal pain
B. Nausea vomiting abdominal pain
C. Something with diarrhea
Answer: tyohiod or enteric fever is an infection salmonella typhi bacterium. Incubation period of 7-14, present with fever, abdominal
pain and red spot on the abdomen ( pathognomic) diarrhea is not a common symptom.
Http://emedicine.medscape.com/article/231135-clinical

219
66. Patient c/o fever ,productive cough , x-ray show rt lung opacification and oblitration of rt costophrenic angle what u will find
on examination?
A. Crepitations on both lungs
B. Absent of vesicular breathing sounds of rt side
C. Presence of bronchial breathing

Answer: c
Explanation: pneumonia may present with evidence of consolidation (dullness to percussion, bronchial breath sounds, crackles)
Reference: toronto notes 2015, page id8, infectious diseases

67. Ppd was +ve , to prevent false +ve , what to do?!


A. Repeat it
B. Do x-ray
C. Do mantoux test

68. A nurse with pneumonia they gave different type of penicillin.


A. Cloxacillin
B. Amoxicillin
C. Pepracillin i
Answer: c

Treatment of hospital-acquired pneumonia is tailored toward gram-negative rods (any of the following three are appropriate):
220
Cephalosporins with pseudomonal coverage: ceftazidime or cefepime
Carbapenems: imipenem
Piperacillin/tazobactam
reference: step up to medicine

69. Diabetic with painful back swelling has multiple discharges:


A. Cellulitis
B. Lymphoid
C. Something lymphadenitis carbuncle
Answer: c

Reference: http://www.merckmanuals.com/professional/dermatologic-disorders/bacterial-skin-infections/furuncles-and-carbuncles
Http://patient.info/doctor/boils-and-carbuncles

70. What is the most common organism in whooping cough ?


A. Bartonella
B. Pertussis
C. H.infeluanza
Answer: b

Pertussis, a respiratory illness commonly known as whooping cough, is a very contagious disease caused by a type of
bacteria called bordetella pertussis

Reference: http://emedicine.medscape.com/article/967268-overview
Http://www.cdc.gov/pertussis/about/causes-transmission.html

71. Picture of skin with chickenpox, with history of child with malaise and fatigue followed by single macule then spread all over
the body including the face, what is the treatment:
A. Antibiotics
B. Antiseptic
C. Acyclovir
Answer: c
Antiviral medications are recommended for people with chickenpox who are more likely to develop serious disease including
* otherwise healthy people older than 12 years of age
* people with chronic skin or lung disease
* people receiving steroid therapy
* some groups of pregnant women
acyclovir, an antiviral medication, is licensed for treatment of chickenpox

72. A patient with a central line developed an infection. What is the most common cause of infections?
A. Skin opening
B. Drug administration
C. Contamination from staff hand
Answer is a
The most common source of intravascular catheter related infections is colonization of the intra-cutaneous and intravascular
portions of the catheter by microorganisms from the patient's skin. A number of studies have found a strong correlation between
221
heavy skin colonization and both catheter colonization and subsequent catheter-related infection. Microorganisms gain access to
the catheter wound and migrate along the catheter-subcutaneous tract into the fibrin sheath that surrounds intravascular catheters.
Source: https://yhdp.vn/uptodate/contents/utd.htm?36/37/37464?Source=see_link

73. Cervical infection can enter to superior-mediastinum through:


A. Retropharyngeal space
B. Para-pharyngeal space
C. Carotid sheath
Answer: a
The “danger space” is a potential space that lies posterior to the retropharyngeal space, it is bound by the alar fascia anteriorly and
the prevertebral fascia posteriorly. It extends from the base of the skull and descends freely through the entire superior and
posterior mediastinum to the level of the diaphragm (t1 to t2) where the two fascial layers fuse. Thus, the danger space provides the
most important anatomic route for contiguous spread between the neck and the chest e.g., descending necrotizing mediastinitis
Source: https://yhdp.vn/uptodate/contents/mobipreview.htm?35/45/36569
Http://emedicine.medscape.com/article/784277-overview#a5

74. Known case of HIV , have several problems on iris ,including the word"necrotizing"
Wt the cause?
A. Hiv
B. Cytomegalovirus

222
C. Toxoplasma
Answer: most likely b

Explanation: epidemiologic data regarding iris are variable and depend largely on the incidence and types of infections that patients
have at the time of initiation of arv therapy. In the united states, retrospective studies have reported iris in 63% of hiv-infected
3
patients who had inactive CMV retinitis at the time of initiation of arv therapy and 30% to 34% of those with inactive
4,5
cryptococcus. similar rates have been found retrospectively in 30% and 31% of hiv-infected patients with mycobacterium
5
tuberculosisand m. Avium complex (mac), respectively. however, retrospective studies may overestimate the incidence of iris. A
6
prospective actg study, actg a5164, reported a rate of 7.6% ; however, this rate may have been low because most of the reported
opportunistic infections (ois) were pcp. Steroid treatment for pcp may have mitigated iris-related symptoms and reduced the
number of iris diagnoses in the study.
Link: http://www.hivguidelines.org/clinical-guidelines/adults/immune-reconstitution-inflammatory-syndrome-iris-in-hiv-infected-
patients/
Http://www.aidsinfonet.org/fact_sheets/view/483

75. Patient have a chronic liver disease and you found a fungal infection which drug you will use :
A. Floconazole
B. Itraconazole
C. Amphotricin b

76. An asymptomatic patient had a positive tb skin test. What will you do?
A. Reassurance
B. Isoniazid for 6 month
C. Rifampin and isoniazid 6 month
Answer: b
Patients with a clinically significant result on tuberculin skin testing or a positive interferon-gamma release assay (igra) result should
receive a course of therapy for latent tb, once active infection and disease are ruled out. Recommended regimens for latent tb
published by the us centers for disease control and prevention (cdc) are as follows:
Isoniazid 300 mg - daily for 9 months
Isoniazid 900 mg - twice weekly for 9 months (administered as dot)
Isoniazid 300 mg - daily for 6 months (should not be used in patients with fibrotic lesions on chest radiography, patients with
HIV infection, or children)
Isoniazid 900 mg - twice weekly for 6 months (administered as dot; should not be used in patients with fibrotic lesions on chest
radiography, patients with HIV infection, or children)
Rifampin 600 mg - daily for 4 months
Rifapentine 900 mg plus isoniazid 900 mg - once-weekly for 12 weeks (administered as dot)
No longer recommended - rifampin plus pyrazinamide daily for 2 months (increased liver toxicity)
Http://emedicine.medscape.com/article/230802-treatment#d13

77. A 12 year-old’s x-ray showed bilateral lower lung infiltration. What is the treatment?
A. Ciprofloxacin
B. Azithromycin
C. Pincillen
Answer: b repeated without options

78. A patient is taking amoxicillin for his pneumonia and is found not sufficient. What will you add?
223
A. Vancomycin
B. Erythromycin
C. Azithromycin
Answer is: a or c (not sure)
Vancomycin is used to treat cap in patients < 60 years with the organisms (s. Pnuemoniae, mycoplasma, chlamydia, legionella).
Macrolides (azithromycin, clarithromycin) will cover all these organisms and are first-line treatment.
Source: step-up to medicine “page 364”

79. A patient complained of neck pain and occipital headache. No history of trauma. There is a limitation in neck movement on
examination, weakness in upper shoulder? What is the diagnosis?
A. Cervical spondylosis
B. Cannot remember other choices
C. Meningitis
Answer: c

80. Patient presented with meningitis symptoms after he swam in the river, what is the organism?
A. S. Pneumonia
B. H. Influenza
C. N. Meningitis
Answer: a
Naegleria fowleri (parasite) lives in fresh water such as lakes, ponds and poorly maintained swimming pools, and can
cause meningitis
S. Pneumonia: is the most common cause of meningitis for all patients beyond neonatal period
H. Influenza: in the past it was the common cause of meningitis in children. After haemophilus type b vaccine this
organism is markedly reduced
N. Meningitis: spread by respiratory droplets, and it is the most common cause in adolescent
Kaplan usmle step2

81. Pt came for general check up, u found


+ hbsag: (insufficient data ?! )
A. Acute hepatitis
B. Acute hepatitis carrier
C. Chronic hepatitis
Answer:

82. A 25 year-old teacher is complaining of abdominal pain and fatigue. On exam, there was icting , palpable liver 1 cm. Also, 2
student complained of the same symptoms. What is the most likely diagnosis?
A. Hav
B. HBV
C. HBV.
Answer: a
HAV hepatitis has prodrome of mild flu-like symptoms (anorexia, nausea, fatigue, malaise and joint pain) preceding the
jaundice. This can progress to the icteric phase with dark urine (appears first). Pale stools (not always). Jaundice occurring in 70-85%
of adults with acute HAV infection. Abdominal pain occurring in 40% of patients. Itch or pruritus (usually with jaundice but can occur

224
without). Arthralgias and skin rash. These occur less often (lower limbs and with a vasculitic appearance).spread is normally by the
faecal-oral route although there are occasional outbreaks through food sources.
Http://patient.info/doctor/hepatitis-a-pro

83. Rota confirmatory diagnosis


A. Serum antibody
B. Stool antigen
C. Stool leukocyte
Answer: b http://www.cdc.gov/rotavirus/clinical.html
Direct immune-based assays of stool and polymerase chain reaction (PCR ) techniques have been employed most frequently to make
the diagnosis of rotavirus. Assays generally detect the rotavirus group antigen present on vp6.

84. Indonesian lady, suspected tb. Best test?


A. Sputum culture
B. PCR
C. Cxr
Answer: c
Chest x-ray is the first line diagnostic modality for tb. It is almost always abnormal in immunocompetent individuals. Typically
presents as fibronodular opacities in upper lobes with or without cavitation. Http://bestpractice.bmj.com/best-
practice/monograph/165/diagnosis/tests.html

85. Patient presented to primary health care with his annual health visit, in lab result: hbsag is (+ve) "this is the only test that
written in this question". The patient has ?
A. Acute HBV
B. Chronic HBV
C. Chronic HBV carrier
Answer: c

86. Young female came with jaundice disoriented, tremor. Husband has HBV
Lab -ve. LFTslightly elevated what lab to order next ?
A. Serum copper
B. Serum level of ceruplasmen
C. HBV core

Answer: b
Jaundice, tremors, and elevated LFTs are indicative of wilson’s disease. Wilson’s disease can present with hepatic or neurologic
symptoms or both. The next step in diagnosis should be to obtain urine copper or serum ceruplasmin which will be low. The gold
standard for diagnosis is liver biopsy. Http://bestpractice.bmj.com/best-practice/monograph/427/diagnosis/step-by-step.html

87. Giardiasis treatment


A. Metrindazole
B. Parpomycine
C. Ciprofloxacin
Answer: a

225
88. Patient in college diagnosed with meningitis proved and she receiving treatment
How to prevent the spread to the roommate?
A. Give them pencilin and other antibiotics
B. Isolate for 4 weeks
C. Do nasal swab and now the affected
Answer: a (no rafmpicin or ciprofloxacin or ceftrixon in the options)

89. Meningitis case fever, headache, nuchal rigidity and rash (pic) what is the most complication?
A. Deafness
B. Seizure
C. Focal neurological defect.

Answer: a. Deafness. Some of the most common complications associated with meningitis are: hearing loss, which may be partial or
total.recurrent seizures (epilepsy), problems with memory and concentration, learning difficulties and behavioural problems, vision
loss, which may be partial or total, loss of limbs, kidney problems.
Nhs http://www.nhs.uk/conditions/meningitis/pages/complications.aspx

90. Patient on anti tb medication complaining of numbness and paresthesia. What is the treatment?
A. Pyridoxine
B. Iron
C. Thiamine
Answer: a
Isoniazid: patients at risk of peripheral neuropathy, should additionally receive pyridoxine (b6), 10 mg daily. Where the standard of
health in the community is low, pyridoxine should be offered routinely. For established peripheral neuropathy, pyridoxine should be
given at a larger dose of 50–75 mg daily. Pyridoxine supplementation is recommended for all pregnant (or breastfeeding) women
taking isoniazid.

91. Which of the following is associated with animal bites?


A. Pasturella multicodi
B. Pseudomonas
C. Eikenella
Answer: a
Pasturella multicodi exists as a commensal in the upper respiratory tracts of many livestock, poultry, and domestic pet species,
especially cats and dogs. In fact, p multocida infection in humans is often associated with an animal bite, scratch, or lick. Eikenalla is
part of the hacek organisms which are gram-negative coccobacilli that exist asca oropharyngeal commensals associated with
infective endocarditis.
Http://emedicine.medscape.com/article/224920-overview
Http://emedicine.medscape.com/article/218158-overview

92. Hsv- 2 most appropriate treatments?


A. Acyclovir
B. Lamivudine
C. Ribavirin
Answer: a
226
Reference: http://emedicine.medscape.com/article/218580-medication#2

93. Patient has been bitten by a cat, what is the organism that has been transmitted?
A. Staphylococcus aureus
B. Pasteurella multocida
C. Streptoccus pneumonia
Answer: b
Cat bite-associated infections, are polymicrobial, with a mix of aerobes and anaerobes. Common aerobic pathogens in cat bites
include streptococcus species (including streptococcus pyogenes), staphylococcus species, especially s. Aureus and moraxella.
Pasteurella multocida, a small (0.2–2.0 µm) facultatively anaerobic, gram-negative, nonmotile, non-spore-forming, pleomorphic
coccobacillus is the most common organism isolated in cat bites. Reference: http://www.medscape.com/viewarticle/739023_5

94. Typical history of pneumonia, x-ray was done, lower lobe consolidation was found. Culture shows gram-positive cocci
arranged in clusters. Catalase and coagulase were positive. What is the most appropriate ab for this infection?
A. Oxacillin
B. Penicillin g
C. Amoxicillin
Answer: a

95. IV drug abuser has HIV +ve. Presented with oral thrush and symptoms of pneumonia. X-ray and bronchoalveolar lavage was
done. After staining with silver stain, pneumocystis jiroveci was found. What is the most likely predictor of her HIV infection?
A. Pneumocystis jiroveci infection
B. Iv drug use
C. Candida
Answer: a
Pneumocystis jiroveci pneumonia (pjp), formerly known as pneumocystis cariniipneumonia (pcp), is the most common opportunistic
infection in persons with HIV infection.

96. Patient with secondary syphilis was treated with penicillin. 2 hours following the first dose he developed fever myalgia and
malaise. What is your management?
A. Epinephrine
B. Antihistamine
C. Symptomatic management with paracetamol
Answer: c

97. Child with diarrhea. What you recommend the mother to do?
A. Oral hydration
B. Iv hydration
C. Antidiarrheal medication
Answer: a

227
nd
98. History of arthritis on (cephalexin). Culture is gram+ cocci that’s resistant to (cefzil), a 2 generation cephalosporin. What will
you do?
A. Prescribe vancomycin
B. Stop the drug
C. Continue same drug
Answer: a
Explanation: the patient most likely has mrsa infection. Septic arthritis due to mrsa should be treated with vancomycin.
Reference http://cursoenarm.net/uptodate/contents/mobipreview.htm?32/63/33776#h18

99. Pt has central line what is the most common cause of infection
A. Skin opening
B. Drug administration
C. Contamination from staff hand
Answer: a
Skin colonization is the most common source of cvc infection by microorganism from the patient’s skin and occasionally the hands of
the healthcares worker. -uptodate

100.Pt on cloxacillin for staph micro reported it is resistant to one of the cephalosporins what to do:
A. Continue cloxacillin
B. Start vancomycin
C. Stop antibiotics
Answer: b
Read about viral hemorrhagic disease http://emedicine.medscape.com/article/830594-overview

101.Child came back with his family from africa developed fever, n&v, abnormal reflexes, stiff legs, unable to raise legs and neck
when he is supine cns analysis nl what is the most likely cause?
A. Corona virus
B. Poliovirus
C. CMV
Answer: b

102.Which of the following transmitted through un cooked meat ?


A. Entameba coli
B. Entamebea.h
C. Teania

Answer: c
(important foodborne parasites: trichnella (undercooked pork), tinea saginata (undercooked beef), tinea solium (undercooked pork),
toxoplasma gondii (undercooked meat).)
Refernce: cdc

103.Male what is type of dysuria after urination bladder still full and palpable
Maculopapular rash on face and some inside buccal white?
A. Mumps
228
B. Herpes zoster
C. Measles
Answer: herpes simplex virus
Clinical features in men: herpetic vesicles appear in the glans penis, the prepuce, the shaft of the penis, and sometimes on the
scrotum, thighs, and buttocks. In dry areas, the lesions progress to pustules and then encrust. Herpetic urethritis occurs in 30%-40%
of affected men and is characterized by severe dysuria and mucoid discharge. The perianal area and rectum may be involved in
persons who engage in anal intercourse, resulting in herpetic proctitis.
Reference: medscape

104.Varicella vaccine in adult.


A. Dose once
B. Doses 4 weeks
C. 2 doses 6 weeks
Answer: b (2 doses at least 4 weeks apart)

105.Aids patient 34 w pregnant her cd count dropped to 200 what will u do:
A. Book for cs
B. Cs when spontaneous labor
C. Vaginal delivery
Answer: i think a
Http://www.uptodate.com/contents/hiv-and-pregnancy-beyond-the-basics
Https://aidsinfo.nih.gov/contentfiles/hivandpregnancy_fs_en.pdf

106.A married woman was diagnosed with uti. Urine culture revealed staphylococcus saprophyticus. What you should ask this
patient about?
A. Use of condom and spermicides.
B. Alcohol consumption.
C. Fecal incontinence.

Answer: a
honeymoon cystitis. Sexual activity increases the risk of s. Saprophyticus uti because bacteria are displaced from the normal flora of
the vagina and perineum into the urethra. Most cases occur within 24 hours of intercourse.

107.Patient is allergic to sulfa drugs and penicillin and shellfish. She has uti what antibiotic you will give?
A. Nitrofurantoin
B. Trimethoprim sulfamethoxazole
C. Amoxicillin

Answer: a

108.When does acute episode of rheumatic fever occur?


A. In case of pharyngitis
B. When bacteria invade joints
C. When bacteria coat the myocardium
229
Answer : a
[1, 2, 3, 4]
Rheumatic fever follows pharyngeal infection with rheumatogenic group a streptococci. the risk of developing rheumatic
[1]
fever after an episode of streptococcal pharyngitis has been estimated at 0.3-3%.
Http://emedicine.medscape.com/article/236582-overview#a5

109.Dog bite, what is the most common organism ?


A. Rabies
B. Yersinia pestis ( rodent - plaque )
C. Toxoplasma gondii ( cat )

Answer: a
Pasteurella ( 50% )
Strept 46% , staph 46%
Neiserria 32%
Corynebasterium 12%
Bite wound infections are usually polymicrobial, with a mix of animal oral flora, recipient skin flora
And environmental organisms. The most common pathogens in dog bites arepasteurella spp. (both
Pasteurella multocida and pasteurella canis), staphylococcus and streptococcusspp., and the
Fastidious gram-negative rod capnocytophaga canimorsus. The dog is the most common transmitter
Of rabies to humans worldwide, with greater than 95% of reported cases being due to these animals.
Reference:
Http://www.medscape.com/viewarticle/739023_4

110.HIV patient, presented with SOB and productive cough.


Lung biopsy showed soap bubble like intra-alveolar lesions with exudates, small cyst, stained by silver stain.
A. Pneumocystis jiroveci
B. Aspergillus fumigatus
C. Cryptococcus neoformans

Answer: A
Pneumocystis jiroveci (previously named P. Carinii), is the most common opportunistic infection in patients with AIDS, and it is an
increasing cause of disease in other immunosuppressed persons. It cannot be cultured and most clinical laboratories rely on
microscopic examination of stained material from the respiratory tract. The most widely used stain techniques are those that stain
the cyst wall, such as Gomori methenamine silver (GMS), toluidine blue, and certain fluorescent brighteners (FB).
Reference: American Journal of Clinical Pathology (AJCP).
One of the special stains is silver stain (fungi. Pneumocystis carinii) for HIV/ immunocompromised patients.
Reference: Step Up to Medicine.

Cryptococcus - Diagnosis - M/E SOAP BUBBLE like organism (silver stain, mucicarmine and PAS), India Ink negatie staining, LATEX
AGGLUTINATION test for antibodies against capsular polysaccharides; CD4 < 100

111.Patient developed nausea and vomiting then developed cranial nerve palsies then bilateral symmetrical progressive LL
paralysis. What is the most likely diagnosis?
A. Tetanus
B. Botulism
C. Lead poisoning

230
Answer: B
Botulism is an acute neurologic disorder that causes potentially life-threatening neuroparalysis due to a neurotoxin produced by
Clostridium botulinum. The 3 main clinical presentations: Infant botulism, Foodborne botulism and Wound botulism.
Signs and Symptoms:
➢ Occurs 6-48 h after ingestion
➢ Difficulty with convergence, ptosis, paralysis of extraocular muscles
● Dilated, poorly reactive pupils
● Other autonomic dysfunction: jaw weakness, dysarthria, dysphagia.
➢ Spreads to trunk and limbs
a. Abdominal cramps with nausea and vomiting
➢ Symmetric weakness with paralysis and absent/decreased deep tendon reflexes
➢ Anticholinergic symptoms: dry mouth, constipation, urinary retention
● Rarely respiratory distress, potentially advancing to respiratory failure
Reference: Toronto Notes and Medscape.

112.Child – parents Have TB - You read a PPD result after 48 hours. It showed 10 mm induration. What does that indicate?
A. Negative
B. Weakly positive
C. Strongly positive

Answer: C
The person's medical risk factors determine the size of induration the result is positive (5 mm, 10 mm, or 15 mm).
● Five mm or more is positive in:
● HIV-positive person
● Recent contacts of active tuberculosis cases
● Persons with nodular or fibrotic changes on Chest X-ray consistent with old healed TB
➢ Organ transplant recipients and other immunosuppressed patients who are on cytotoxic immune-suppressive agents such as
cyclophosphamide or methotrexate.
● Patients on long term systemic corticosteroid therapy (> than six weeks) and those on a dose of prednisone ≥ 15 mg/day or
equivalent.
● End stage renal disease
● Ten mm or more is positive in:
● Recent arrivals (less than five years) from high-prevalence countries
● Injectable drug users
● Residents and employees of high-risk congregate settings (e.g., prisons, nursing homes, hospitals, etc.)
● Mycobacteriology lab personnel
➢ Persons with clinical conditions that place them at high risk (e.g., diabetes, prolonged corticosteroid therapy, leukemia, end-
stage renal disease, chronic malabsorption syndromes, low body weight, etc.)
● Children less than four years of age, or children and adolescents exposed to adults in high-risk categories
Infants, children, and adolescents exposed to adults in high-risk categories
Fifteen mm or more is positive in:
Persons with no known risk factors for TB. (Reactions larger than 15 mm are unlikely to be due to previous BCG vaccination or
exposure to environmental mycobacteria).
231
Reference: Pubmed

113.Patient with oral ulcers. Culture showed herpesvirus.


A. HSV2
B. VSV
C. HSV1

Answer: C

114.Old patient with poor control of DM1, complaining of SOB and hemoptysis. X-ray shows lung consolidation. Culture show non
septa fungal hypha. What is the diagnosis?
A. Aspiragillus
B. Candida
C. Zymgomycetes

Answer: c
Opportunistic infection especially associated with diabetes; other predisposing factors are neutropenia, corticosteroid therapy, iron
overload and mucocutaneous trauma.
Large, non-septa hyphae with 90 degree angle branching and non-parallel walls, angioinvasive causing tissue necrosis and
hemorrhage

Reference: http://www.pathologyoutlines.com/topic/lungnontumormucor.html

115.20yrs old gentleman came with blocked nose and urt symptoms, followed by swelling eyes and redness what's the possible
answer :
A. Meningiococcal conjunctivitis
B. Chlamedia conjunctivitis
C. Parainfluenza virus

116.Man in acetaminophen drink 2 glass of win every week LFT(high) and bilirubin is high dx:
A. Alcoholic hepatitis ??
B. Drug induced hepatitis
C. Wipple

117.Patient have history of meningitis befour 4 week , come againe to hospital , what is most increase of his lap
A. Protien ?? "not sure"
B. Lekocytosis
C. Glocose

232
118.Patient has blood transfusion from kenya and had anal infection. What´s the most likely diagnosis?
A. HBV
B. Syphilis
C. Answer: ???

119.Action of cytotoxic?
A. Il6
B. Il10
C. Tnf gamma

120.Long case about adult come to er complaing of diarrhea , have weekness in body and fatige , suddenly he fall down ,
hypertension when he come was 120 / 80 now it is 90 / 60 , what is the cause ?
A. Extracelular voluim loss
B. Intracellular fluid loss !?
C. Intracellular glocse loss

121.Std case with culutre gram -ve diplocci , wt is organism ?


A. N . Gonnorhra
B. Chalmyida
C. E.coli

122.Enteric fever resistance to chloramphenicol, what should you do next ?


A. Double chloramphenicol
B. Clindamycin alone
C. Add clindamycin
Answer: ??
Explanation: drugs of choice for the treatment of typhoid fever in adults include : a fluoroquinolone such as ciprofloxacin or
ofloxacin (400 mg twice daily)/ the fluoroquinolones should not be used as a first-line treatment for typhoid fever in patients from
south asia or other regions with high rates of fluoroquinolone resistance unless antibiotic susceptibility data demonstrate
fluoroquinolone or nalidixic acid sensitivity or beta-lactam such as ceftriaxone or cefixime. Reference:
http://cursoenarm.net/uptodate/contents/mobipreview.htm?37/1/37904

123.4w back pt had mycoplasma pneumonia what will be very high in lp?
A. Protein
B. Wbc
C. Glucose
Mycoplasma pneumonia increase light chain protienIgMthats why those patient

124.Old lady with sharp chest pain and fever diagnosed with pericarditis what will you do to dx the case , most accurate test is :
(cardio)
A. Acid fact stain
B. Pericardial biopsy(my answer)
C. Pleural aspiration

233
125.A man came from Africa with some symptoms. Vital signs were provided. What is the diagnosis?
A. Yellow fever
B. Ebola
C. Lassa fever

Answer: ?

Reference: http://lectures.shanyar.com/3rd_Stage/Medicine/Dr._Muhammad_Shaikhani/5._Viral_Hemorrhagic_Fevers.pdf

126.Rhinorrhea, cough and conjunctivitis etiology?


A. Rhinovirus
B. Adenovirus

Answer: b
The most common cause of rhinorrhea and sinusitis is rhinovirus., but since there is also conjunctivitis, then adenovirus is more
appropriate.
Reference: http://emedicine.medscape.com/article/302460-clinical

127.Increase of which of the following prevalence cause reactivation of tb in developed countries? Repeated in family medicine
A. DM
B. Hiv

Answer: b
Who & uptodate: according to data from the united states national tb surveillance system for 1993 to 2005, 7.7 percent of tb
patients were HIV infected; greater than 80 percent of tb patients in the united states received HIV testing. HIV infection rates in tb
patients were highest among injection drug users, homeless persons, correctional facility inmates, and alcoholics (35, 22, 16, and 15
percent, respectively)
The coinfection with m tuberculosis and HIV has profound epidemiologic implications worldwide. From one perspective, tb has
reemerged in some countries, such as the united states, in association with multiple factors, of which HIV infection is one of the
most relevant.
References:

234
Http://hivinsite.ucsf.edu/insite?Page=kb-05-01-06
Http://www.ncbi.nlm.nih.gov/pmc/articles/pmc3276831/
Http://www.medscape.com/viewarticle/443137_2

128.Woman with recurrent uti, what is the cause?


A. Because she cleans herself of the from anus to vulva
B. Using of cervical cap for contraception

Answer: b
Note: cervical cap usage require spermicidal cream with it, & since spermicide is a risk for recurrent uti according to the uptodate

129. Pt in icu on ventilator, develop yeast infection what is rx?


A. Fluconazole
B. Itraconazole

Answer: fluconazole
(uptodate: “the most common antifungal agents used currently for the treatment of candidemia are fluconazole and the
echinocandins (caspofungin, micafungin, anidulafungin). Formulations of amphotericin b are given less often due to the risk of
toxicity. Both the echinocandins and the azoles are better tolerated than amphotericin b formulations.”
& oxford journals: “overall, fluconazole-susceptible candida albicans remains the most common species causing candidaemia in icu
patients.”)

130.Long scenario, bloody diarrhea and RBCs in urine after 7 days hx of food posing, rx?
A. Steriod
B. Antibiotic

Answer: conservative
This is e coli: hus

131.Tb case, what is the next appropriate step to get a definitive dx?
A. Sputum smear under microscope
B. Sputum culture

Answer: b
''Sputum cultures remain the gold standard for the diagnosis of tuberculosis''
Reference: http://www.aafp.org/afp/2000/0501/p2667.html

132. Which marker indicates chronic hepatitis b?


A. Igm
B. Hep b s antibody

Answer:IgManti-hbc
Hepatitis b surface antigen (hbs ag) indicates that a person is infected with the hep b virus. It is detectable as early as 2 weeks and is
seen in both acute and chronic infection. It persists in chronic infection regardless of the presence of symptoms.
235
Hepatitis b surface antigen antibody (anti-hbs) indicates immunity against hep b due to either vaccination on recovery from previous
infection. Not present in acute or chronic infection.
Hepatitis b core antigen antibody (anti-hbc) indicates that a person may have been exposed to the hep b virus but it does not
indicate immunity or protection. It is present in both acute and chronic infection.
So how is chronic infection diagnosed? The diagnosis of chronic HBV infection is based on persistence of hbs ag for more than six
months +IgGanti-hbc is positive, whileIgManti-hbc is negative.
References: step up to medicine + this cool cdc pdf: https://www.cdc.gov/hepatitis/HBV/pdfs/serologicchartv8.pdf

133.Patient in icu on ventilator develop yeast infection what is rx?


A. Fluconazole
B. Itraconazole

Answer: a

Reference: https://ccforum.biomedcentral.com/articles/10.1186/cc13775

134.What is the most specific investigation for tb?


A. Sputum culture
B. Ppd

Answer: a
Chest x-r
ay. Classic findings are upper lobe infiltrates with cavitations.
Sputum studies (sputum acid-fast testing). Definitive diagnosis is made by sputum culture—growth of m. Tuberculosis. 
obtain three
morning sputum specimens—culture takes 4 to 8 weeks. 

PCR . 

Finding of afb on microscopic examination, but 
this is not definitive.
Tuberculin skin test (ppd test). A screening test, not for diagnosis.
 positive if induration 15 mm in patients with no risk factors.

health care workers, nursing home residents, close contact of someone with tb, alcoholics, diabetics, 10 mm of induration is
considered positive. 
hiv, steroid users, organ transplant recipients, close contacts of those with active tb, or those with
radiographic evidence of primary tb, induration of 5 mm is positive. 


Quantiferon-tb gold test.
Reference: step-up of medicine

135.Fever and cough then facial nerve then loss of reflexes?


A. Tetanus (it comes with headache with lockjaw)
B. Botulism

Answer: b
It is mostly b, ''botulism have 3-5 of the following signs or symptoms: nausea, vomiting, dysphagia, diplopia, dilated/fixed pupils, and
an extremely dry mouth unrelieved by drinking fluids'' with cranial nerve palsy
Reference: http://emedicine.medscape.com/article/213311-clinical#b4

236
136.Treatment of trigeminal neuralgia?
A. Prednisiolone
B. Naloxone
Answer :carbamazepine and oxcarbazepine are considered first-line therapy in trigeminal neuralgia (tn)
http://emedicine.medscape.com/article/1145144-treatment#d9

137.67 years patient complain of progressive weakness for 1 year. Things are falling from his hand. On examination there is wide
gate, +ve hoffman & babinski sign?
A. Cervical canal stenosis
B. Spinal cord tumor?
Answer: b

138.Patient present with itchy foot, on examination there is linear lesions with red dots at the end of the lines, diagnosis;
A. Lintego ?
B. Scabies
Answer: b
Burrows are a pathognomonic sign and represent the intraepidermal tunnel created by the moving female mite. They appear as
serpiginous, grayish, threadlike elevations in the superficial epidermis.
Http://emedicine.medscape.com/article/1109204-clinical

139.Case of herpes type 1, what to give?


A. Oral antiviral
B. Topical steroids
Answer: a
Reference: overall, medical treatment of herpes simplex virus (hsv) infection is centered around specific antiviral treatment.
(http://emedicine.medscape.com/article/218580-treatment)

140.Which of the following organisms is seen in patients with chronic granulomatous disease?
A. Cl. Difficle
B. Staph aurues
Answer: b

141.There is tb outbreak in a region, however you tested one patient and you found him to be negative for tb, what are you going
to give him:
A. Bacillus calmette-guérin vaccine
B. Rifampin chemoprophylaxis
Answer:b

237
142.Mouth ulcer :
A. Hsv 1
B. Hsv2
Answer:a
Most often, hsv-1 causes gingivostomatitis, herpes labialis, and herpes keratitis. Hsv-2 usually causes genital lesions.
Http://www.merckmanuals.com/professional/infectious-diseases/herpesviruses/herpes-simplex-virus-hsv-infections

143.Ie most common organism:


A. S. Aureus
B. S.viridins

Reference: medscape table 1 http://emedicine.medscape.com/article/216650-overview#a4

144.Vesicles on the eye and forehead?


A. Herpes zoster ophthalmicus.
B. Herpesvirus ophthalmic.
Answer: a

145.Patient with infection. Culture and sensitivity showed methicillin sensitive organisms, which antibiotic you will select :
A. Piperacillin
B. Oxacillin
Answer: b
Serious staphylococcal infections require treatment with parenteral penicillinase-resistant penicillin (e.g. Nafcillin, oxacillin)

146.Gram positive organisms in cluster? What will be positive?


A. Coagulase
B. Oxidase
Answer: a

238
147.A breast abscess showed gram positive organisms. What will u do?
A. Coagulase
B. Oxidase
Answer: a
S. Aures is the most common organism causing breast abscess, and coagulase test is used to differentiate coagulase +ve
staphylococcus aureus from coagulase -ve staphylococcus.

148.61yo male patient with bilateral lung base infiltrate (x-ray chest) cough, diarrhea, temp. 38.7c (long scenario),what is the
microorganism?
A. Legionella pneumonia
B. Mycoplasma pneumonia
Answer: could be a or b.
Both are atypical community-acquired pneumonia, but x-ray direct more toward a.
Legionella pneumonia usually cause a patchy, localized infiltrate in the lower lobes.
Http://emedicine.medscape.com/article/234240-overview#a6
http://emedicine.medscape.com/article/363083-overview#a2

149.Patient with positive ppd, never was +ve before, no x-ray findings, what is next step?
A. Isoniazide and rifampin 6 months
B. Rifampin 3 months
Answer: b, close to one of cdc regimens
it's latent tb. Http://www.cdc.gov/tb/topic/basics/tbinfectiondisease.htm
(cdc) recommend regimens:
Isoniazid 300 mg – daily for 9 months
isoniazid 900 mg – twice weekly for 9 months
isoniazid 300 mg – daily for 6 months (should not be used in patients with fibrotic lesions on chest radiography, patients with HIV
infection, or children)
isoniazid 900 mg – twice weekly for 6 months (should not be used in patients with fibrotic lesions on chest radiography, patients
with HIV infection, or children)
rifampin 600 mg – daily for 4 months
rifapentine 900 mg plus isoniazid 900 mg – once-weekly for 12 weeks

150.Why patient will be in a risk of neisseria infection defect in which of the following:
A. Classical complement pathway
B. Final lytic complement pathway
Http://emedicine.medscape.com/article/135478-clinical

151.A patient showed antibodies to toxoplasma (IgM) .. How to confirm diagnosis ?


A. Double of igm
B. DetectIgGfor toxoplasma IgG avidity test OR Sabin-Feldman dye test OR IgM ELISA OR IgA ELISA OR IgE ELISA
Answer: ?

152.Trypanosoma cruzi act by mutation on what gene? chagas disease

239
3′UTR) of the calmodulin gene

A. Myocin
B. Tryptomyocin t
Answer: not sure

153.A patient came from a trip suffered an infection for which he took amoxicillin. After that he developed rash and
lymphadenopathy. What investigation will you ask for this patient?
A. EBV
B. Brucellosis
Answer: a, EBV infectious mononeucleosis.
EBV is transmitted via intimate contact with body secretions, primarily oropharyngeal secretions. EBV infects the b cells in the
oropharyngeal epithelium. The organism may also be shed from the uterine cervix, implicating the role of genital transmission in
some cases. On rare occasion, EBV is spread via blood transfusion.
Http://emedicine.medscape.com/article/222040-overview

154.Csf lp shows +IgGand irregular band under the gel?


A. Ms
B. Spinal tumor
Answer: a
igg is usually increased and correlate with disease severity. OligoclonalIgGbands can usually be detected by electrophoresis of csf.
Http://www.merckmanuals.com/professional/neurologic-disorders/demyelinating-disorders/multiple-sclerosis-ms#v1045134

155.Patient diagnosed as mg came to er with weakness & severe fatigability she is on pryditostigmine what initial step you do?
A. Add other drug
B. Plasmophresis
Answer: b, plasmapheresis has been found to be an effective short-term treatment of acute exacerbations of myasthenia gravis.
Http://emedicine.medscape.com/article/793136-overview#a10

156.Painless genital ulcer + lymph nodes enlargement


A. Syphilis
B. Secondary syphilis

Answer: a

240
157.Recurrent mouth and genital ulcers with arthralgia:
A. Systemic herpes infection
B. Syphilis
Answer: b
Rheumatic syndromes, including arthralgia, inflammatory arthritis, and neuropathic arthritis, may occur during any stage of
congenital or acquired syphilis. Also, the patient acquired syphilis through oral sex, a syphilitic chancre may be noted in the oral
cavity on the buccal mucosa, tongue, or lips. These syphilitic lesions can be misdiagnosed initially as herpetic or aphthous ulcers.
Http://www.ncbi.nlm.nih.gov/pubmed/2246956
Http://cursoenarm.net/uptodate/contents/mobipreview.htm?12/44/12993#h12

158.What is the best treatment for chlamydia ?


A. Doxycyclin
B. Azithromycin
Answer: ???
Treatment of genitourinary chlamydial infection is clearly indicated when the infection is diagnosed or suspected. Chlamydiae are
susceptible to antibiotics that interfere with DNA and protein synthesis, including tetracyclines, macrolides, and quinolones.[47] cdc
recommends azithromycin and doxycycline as first-line drugs for the treatment of chlamydial infection.
Http://emedicine.medscape.com/article/214823-treatment#d8

RF+CARDITIS+ HEART RESIDUAL = 10 YEARS UNTIL AGE 40 OR FOR LIFE


159.What is the treatment of acute active rheumatic fever? RF+ CARDITIS= 10 YEARS UNTIL AGE 25 ( WHICH EVER LONGER)
A. Monthly im pencilin RF = 5 YEARS UNTIL AGE 21
B. Amoxicillin q 6 hours
Answer: b

160.Patient with history of infective endocarditis planned for dental procedure:


A. Amoxicillin 1 hour before procedure.
B. Amoxicillin after procedure
Answer: a
Amoxicillin single dose 30-60 min prior, clindamycin if penicillin allergy

161.Patient with fever, night sweat, weight loss and unilateral supraclavicular ln which was firm and 3cm:
A. Ct
B. Biopsy
Answer: a
162.Diabetic patients presents with dark color nasal discharge. What is the most likely diagnosis?
A. Cryptococcosis
B. Zygomycosis

Answer: b
Mucormycosis is a group of mould infections caused by fungi in the class previously known as zygomycetes, now re-named
glomeromycetes.

241
163.Pt asymptomatic come to you with lab result: hbsag +ve (only this data they give me) your dx:
A. Acute infection
B. Acute chronic infection
Answer: most likely a

164.How to treat water from entamoeba histolytica?


A. Boiling
B. Chloride
Answer: a

165.Organism that grow in antiseptic area?


A. Pseudomonas
B. Proteus
Answer: a
Gram-negative bacteria that show a high level of resistance to many antiseptics and disinfectants include p. Aeruginosa,
burkholderia cepacia, proteus spp., and providencia stuartii. The outer membrane of p. Aeruginosa is responsible for its high
resistance.

166.Fever with spot in molar tooth?


A. Measles
B. Rubella
Answer: a
Koplik spots (also koplik's sign) are a prodromic viral enanthem of measles manifesting two to three days before the measles rash
itself. They are characterized as clustered, white lesions on the buccal mucosa (opposite the upper 1st & 2nd molars) and are
pathognomonic for measles. Reference: http://emedicine.medscape.com/article/966220-clinical

167.Trichomonas vaginalis usually presents with.


A. Lack of irritation
B. Greenish foul smelling discharge
Answer: b
The diagnosis is made by observation of the following features (table i):
A foul-smelling frothy discharge (present in 35% of cases
Vaginal ph >4.5 (70% of cases)
Punctate cervical microhemorrhages (25%)
Motile trichomonads on wet mount (50%-75%)
Link: http://www.medscape.com/viewarticle/719240_4

168.Mother with HBV ag gave birth what to give for child


A. Give vaccin and igg.
B. GiveIgGonly
Answer: a

242
Link: http://www.aafp.org/afp/2010/1115/p1225.html

169.19-year-old boy after unprotected sex he developed generalized rash involving hand and feet
A. Chlamydia
B. Syphilis
Answer: b
Link: http://www.acog.org/patients/faqs/gonorrhea-chlamydia-and-syphilis

170.Married male has history of unprotected sex, days later he had purulent discharge with gram negative intracellular
diplococci?
A. Non gonococcal urethritis
B. Gonococcal urethritis
Answer: b

171.Pt with ulcer on penis?


A. Chrocoid
B. Syphilis
See: infectious agents affecting the male genitalia http://emedicine.medscape.com/article/455021-overview#a5

172.What is the organism gram -ve bacilli, non-lactose fermenter, oxidase +ve
A. A-pseudomonas
B. B-proteus
Answer: a

173.Staph saprifitucus vaginal infection, what’s a risk factor for it:


A-septicides in condoms
B-douching habbits

243
Not sure about this but staphylococcus saprophyticus is normal flora in the vagina therefore it doesn’t usually cause vaginitis,
however it may commonly cause cystitis. S. Saprophyticus causes 10–20% of urinary tract infections (utis). In females 17–27 years
[5]
old, it is the second-most common cause of community-acquired utis, after escherichia coli. sexual activity increases the risk of s.
[3]
Saprophyticus utis because bacteria are displaced from the normal flora of the vagina and perineum into the urethra. most cases
[3]
occur within 24 hours of sex, earning this infection the nickname "honeymoon cystitis".

174.Child with itchy scalp and scales, other classmates affected, dx:
A. A-tinea capitus
B. B-scabies
Answer: a
Clinical manifestations — tinea capitis may present with a variety of clinical features. There are five major clinical variants of tinea
capitis as well as an asymptomatic carrier state. The causative organism and the host immune response to infection influence the
clinical presentation.
Clinical variants — the most common clinical manifestations of tinea capitis are the development of scaly patches with alopecia and
patches of alopecia with visible black dots. Diffuse scalp scaling with subtle hair loss, kerion, and favus are additional clinical variants.
Pruritus is a common associated symptom in the various presentations of tinea capitis.

175.Pt with +ve ppd for the first time and -ve cxr. No signs or symptoms of tb, wt to do:
1. A-reassure
2. B-inz for 6 mns
Answer: b

Adults
Isoniazid* Standard regimen:
300 mg po daily for nine months
Alternate regimens:
300 mg po daily for six months
900 mg po twice weekly for nine months
900 mg po twice weekly for six months
Isoniazid and rifapentine◊ Isoniazid (orally once weekly for 12 doses, given by direct observation)
15 mg/kg, rounded up to the nearest 50 or 100 mg; 900 mg maximum
Rifapentine (orally once weekly for three months, given by direct observation)
Rifampin 600 mg po daily for four months
Isoniazid and rifampin§ Isoniazid 300 mg po daily for three months
Rifampin 600 mg po daily for three months

176.Patient in icu with central and peripheral line developed fungal infection what to give:
A. Fluconazole
B. Caspofungin
Answer: b
177.Mycobacterium tuberculosis test :
A. Ifn-γ
B. Ifn- γ slow release assay (igra)
Answer: b
244
Explanation: igra is a blood test equal in significance to ppd to exclude ppd exposure.
Reference master the board step 2 ck

178.How to diagnose giardia lambila?


A. Three consecutive stool analysis
B. Three separate stool analysis
Answer: b

179.35 Year-old female, used ampicillin 1week ago for uti, now presented with hx of fever, hypotension and tachycardia:
A. Pseudomemparenous colitis
B. Toxic mega colon
Answer: i think it's sign of septic shock (intern’s answer)

180.Fever and cough then facial nerve then loss of reflexes?


A. Tetanus
B. Botulism

Answer: b
If guillain-barre syndrome was one of the choices it would be the answer.
Signs and symptoms of botulism:
Occur 6-48 h after ingestion of clostridium botulinum from soil
Difficulty with convergence, ptosis, paralysis of extraocular muscles.
Dilated, poorly reactive pupils 

Jaw weakness, dysarthria, dysphagia.
Spreads to trunk and limbs
Abdominal cramps with n/v.
Symmetric weakness with paralysis and absent/decreased deep tendon reflexes
Anticholinergic symptoms: dry mouth, constipation, urinary retention
Rarely respiratory distress.
Pattern of paresis often starts with gi symptoms (constipation, early satiety), then paresis of extraocular muscles, then dysphagia,
then limbs/respiratory involvement; all associated with dry mouth
Reference: toronto note.

181.Patient with fever, pre-auricular swelling, (description of the swelling was provided), what is the diagnosis?
A. Mumps
B. Pre-auricular lymphadenopathy

Answer: b
Mumps parotitis is usually bilateral but may be unilateral. Pain while chewing or swallowing, especially while swallowing acidic
liquids such as vinegar or citrus juice. Its swelling beyond the parotid in front of and below the ear. The skin over the glands may
become tense and shiny.
Reference: merck manual.

245
182.Which of the following is major criteria of rheumatic fever :
A- fever
B- subcutaneous nodule
Answer:b

183.Old man with dm, has redness in calf area, raised and painful, tender:
A. Cellulitis
B. Diabetic neuropathy
Answer: a
Cellulitis is acute bacterial infection of the skin and subcutaneous tissue most often caused by streptococci or staphylococci.
Symptoms and signs are pain, rapidly spreading erythema, and edema; fever may occur, and regional lymph nodes may
enlarge. Diagnosis is by appearance; cultures are sometimes helpful, but awaiting these results should not delay empiric
therapy. Treatment is with antibiotics. Prognosis is excellent with timely treatment.
Reference: merck manual

184.A farmer presented with 2 weeks history of fever, headache and one more symptom. What is the most likely diagnosis?
A. Brucellosis
B. Meningitis

Answer: A
The trick in the duration, bacterial meningitis usually present over several hours ( Cryptococcal meningitis may be present for several
weeks - but it's not one of the options). In the other hand, brucellosis may show up anytime from a few days to a few months after
the person get infected.
Brucellosis is an infectious disease. People can get the disease when they are in contact with infected animals or animal products
contaminated with the bacteria. Animals that are most commonly infected include sheep, cattle, goats, pigs, and dogs, among
others. Signs and symptoms include fever, headache, sweats, malaise, anorexia pain in muscles, joint, and/or back, fatigue, …
Reference: Centers for Disease Control and Prevention

185.Patient with honeymoon cystitis. Which of following could be the organism?


A. Staph. Saprophticous
B. Coli

Answer: A
Honeymoon cystitis is a bladder infection that results from sexual activity. It occurs when the bacteria travels in the upward
direction from the urethra to the bladder.
Reference http://www.m.webmd.com/a-to-z-guides/tc/understanding-bladder-infections-basic-information?Page=2

246
Http://www.urineinfection.net/honeymoon-cystitis-symptoms-and-relief/

186.Patient with honeymoon cystitis. Which of following could be the organism?


A. Staph. Saprophticous
B. Coli
Answer: A
Honeymoon cystitis is a bladder infection that results from sexual activity. It occurs when the bacteria travels in the upward
direction from the urethra to the bladder.
Reference http://www.m.webmd.com/a-to-z-guides/tc/understanding-bladder-infections-basic-information?Page=2
Http://www.urineinfection.net/honeymoon-cystitis-symptoms-and-relief/

187.Increase of which of the following prevalence cause reactivation of TB in developed countries?


A. DM
B. HIV

Answer: B
WHO & uptodate: According to data from the United States National TB Surveillance System for 1993 to 2005, 7.7 percent of TB
patients were HIV infected; greater than 80 percent of TB patients in the United States received HIV testing. HIV infection rates in TB
patients were highest among injection drug users, homeless persons, correctional facility inmates, and alcoholics (35, 22, 16, and 15
percent, respectively)
The coinfection with M tuberculosis and HIV has profound epidemiologic implications worldwide. From one perspective, TB has
reemerged in some countries, such as the United States, in association with multiple factors, of which HIV infection is one of the
most relevant.
Http://hivinsite.ucsf.edu/insite?Page=kb-05-01-06
Http://www.ncbi.nlm.nih.gov/pmc/articles/PMC3276831/
Http://www.medscape.com/viewarticle/443137_2

188.Positive culture of budding yeast in urine what is the management?


A. Flucanazole
B. Caspofungin

Answer: A
Vulvovaginal candidiasis (VVC) can be managed with either topical antifungal agents or a single dose of oral fluconazole (150 mg).
Candida cystitis in noncatheterized patients should be treated with fluconazole at 200 mg/d orally for at least 10-14 days. For fungi
resistant to fluconazole, amphotericin B is recommended at dose of 0.3 to 0.6 mg/kg IV once/ day for 2 wk for cystitis and 0.5 to 0.7
mg/kg IV once/day for 2 wk for pyelonephritis.
Http://www.merckmanuals.com/professional/genitourinary-disorders/urinary-tract-infections-uti/fungal-urinary-tract-infections
Http://emedicine.medscape.com/article/213853-treatment

189.Pt in ICU on ventilator develop yeast infection what is the treatment?


A. Fluconazole
B. Itraconazole

Answer:A
247
Antifungal drugs for the treatment of candidiasis include polyenes (amphotericin B and triazoles-fluconazole, itraconazole). The
main adverse events of Amphotericin B include nephrotoxicity, hepatotoxicity, anemia, and thrombocytopenia while triazoles have
many drug interaction. For Aspergillosis although amphotericin B, azoles and echinocandins may be used for the treatment of
Aspergillosis, voriconazole is considered as the first line treatment.
Http://www.ncbi.nlm.nih.gov/pmc/articles/PMC4419464/
Http://emedicine.medscape.com/article/213853-treatment

190.Patient with hemoptysis, at first it was blood tinged then it appeared bright red blood what should the next investigation be?
A. Chest x-ray
B. Ppd

191.There is tb outbreak in a region, however you tested one patient and you found him to be negative for tb, what are you going
to give him:
A. Bacillus calmette-guérin vaccine
B. Rifampin chemoprophylaxis
Answer:a
Http://www.cdc.gov/tb/publications/factsheets/prevention/bcg.htm

192.Scenario of diarrhea. Choices about which type of clostridium:


A. Difficile
B. Tetani

193.Treatment of enteric fever :


A. Ciprofloxacin
B. Chloramphenicol

194.Obese, acid reflux symptoms, 2week diagnosed with rf, he is on asprin rx??
A. Metlozam
B. Cinitidine

195.Patient came from africa:


A. Ebola
B. Hiv
Answer: ??

196.Trypanoma cruzi, act by mutaion on what gen.?


A. Myocin
B. Tryptomyocin t

197.Patient transfers blood from kenya and had anal infection…?

248
A. HBV
B. Syphilis
Most probably is syphilis (their original answer)
Http://www.uptodate.com/contents/blood-donor-screening-laboratory-testing?Source=machinelearning&search=transfusion-
transmitted+infection&selectedtitle=2~34&sectionrank=1&anchor=h8#h22112077
http://www.uptodate.com/contents/pathogenesis-clinical-manifestations-and-treatment-of-early-
syphilis?Source=machinelearning&search=syphilis&selectedtitle=1~150&sectionrank=1&anchor=h10#h10

198.78 year old painless genital ulcer and lymph nodes enlargement: primary: painless chancre
A. Primary syphilis Secondary: Rash, flu symp, MM lesions
Tertiary: organ damage: heart , CNS
B. Secondary syphilis Latent stage: symp disappear for 1-20 y
Answer: not sure relapses.
Explanation:
Primary syphilis: chancer that begins as single painless papule as site of inoculation 2-3 weeks after initial exposure. May be
accompanied with bilateral painless rubbery adenopathy
Secondary syphilis: rash may be macular, maculopapular or pustular and may involve whole body or palms or sole. May be
associated with flu-like symptoms, generalized lymphadenopathy and temporary patchy alopecia
Reference: the johns hopkins internal medicine board reviews book

199.Boy with sever abdominal pain, guarding, CT show, gas in the wall of intestine
Undercooked meet infected with:
A. Entero Coli
B. Entamiba histolitica
Answer: ?
Organisms that can be found in raw meat:
➢ Beaf: E. Coli O157:H7, Salmonella, Shigella, Staphylococcus aureus and Listeria monocytogenes
➢ Poultry: Salmonella and Campylobacter
➢ Shellfish: Vibrio gastroenteritis, Salmonellas, Plesiomonas shigelloides, Staphylococcus and Bacillus cereus

200.Human papilloma virus cause cervical cancer: (Infectious – Medicine )


A. 16,
B. 18
Answer: both A&B
HPV genotypes can be broadly split into “high-risk” (16, 18, 31, 33, 35, 39, 45, 51, 52, 56, 58, 59 and 68) and “low-risk” (6, 11, 40, 42,
43, 44, 53, 54, 61, 72, 73 and 81) based upon their phenotypic association with the development of cervical cancer. Types 16 and 18
are the most commonly isolated HPV types in cervical cancer with type 16 found in approximately 50 percent of patients.
Reference : https://yhdp.vn/uptodate/contents/mobipreview.htm?25/53/26455

201.A LONG scenario about a patient with high temperature, high WBC, low RBC
And low PH. What are you going to do?
A. Bone marrow aspiration
B. Cultures
Answer : B

249
202.A case of a patient diagnosied to have cutenous lachmenia or bghlabar/ bhagdad type which type of lachmenia?
A. Kalazar
B. Donavan
And 2 more type of lachmenia

Answer: TROPICA
There are 3 main forms of leishmaniases – visceral (also known as kala-azar and the most serious form of the disease), cutaneous
(the most common), and mucocutaneous. Species that cause cutaneous are L donovani (can cause visceral leishmaniases), L tropica ,
L aethiopica , L major , L infantum. So, L.tropica could be most accurate.
Http://www.cdc.gov/parasites/leishmaniasis/health_professionals/index.html#vl
Http://emedicine.medscape.com/article/220298-overview#a4

203.Long scenario about pt coming from Africa 3 wks ago with fever no other +ve points, What is the Dx?
A. Ebola
B. Yellow fever

Answer:
Yellow fever incubation period 3-6 days and in this scenario 3 weeks so we exclude it
Ebola from 2 to 21 days so it could be
If malaria present most likely 6–30 days (98% onset within 3 months of travel)

DISEASE USUAL INCUBATION PERIOD (RANGE) DISTRIBUTION

Incubation <14 days

Chikungunya 2–4 days (1–14 days) Tropics, subtropics

Dengue 4–8 days (3–14 days) Topics, subtropics

Encephalitis, arboviral (Japanese encephalitis, 3–14 days (1–20 days) Specific agents vary by region
tickborne encephalitis, West Nile virus, other)

Enteric fever 7–18 days (3–60 days) Especially in Indian subcontinent

Acute HIV 10–28 days (10 days to 6 weeks) Worldwide

Influenza 1–3 days Worldwide, can also be acquired


while traveling

Legionellosis 5–6 days (2–10 days) Widespread

Leptospirosis 7–12 days (2–26 days) Widespread, most common in


tropical areas

Malaria, Plasmodium falciparum 6–30 days (98% onset within 3 months Tropics, subtropics
of travel)

250
Malaria, P. Vivax 8 days to 12 months (almost half have Widespread in tropics and
onset >30 days after completion of subtropics
travel)

Spotted-fever rickettsiae Few days to 2–3 weeks Causative species vary by region

Incubation 14 Days to 6 Weeks

Encephalitis, arboviral; enteric fever; acute HIV; See above incubation periods for See above distribution for
leptospirosis; malaria relevant diseases relevant diseases

Amebic liver abscess Weeks to months Most common in developing


countries

Hepatitis A 28–30 days (15–50 days) Most common in developing


countries

Hepatitis E 26–42 days (2–9 weeks) Widespread

Acute schistosomiasis (Katayama syndrome) 4–8 weeks Most common in sub-Saharan


Africa
Table 5-03. Common infections, by incubation period

204.A case of a patient diagnosed to have cutaneous leishmania or baghdad boil which type of leishmania ?
A. Kalazar
B. Donavan

Answers is

Cutaneous leishmaniasis (also known as oriental sore, baghdad boil)i s the most common form of leishmaniasis affecting humans
caused by (e.g., L. Major and L.tropica)

251
Visceral leishmaniasis (VL), also known as kala-azar caused by (e.g., L. Infantum and L. Donovani)

205.Case scenario, then they prescribe organism,,, gram -ve (non lactose fermenter) oxidase test -ve , urease test,,,, then they
asked about the antibiotic:
Diagnosis of one of these bacteria:
Samonella typhi
Shigella dysenteriae
Proteus mirabilis
Yersinia pestis
Shigella, yersinia: fluoroquinolone
Salmonella typhi: empiric treatment with ceftriaxone or azithromycin, fluoroquinolone resistance is increasing

206.A scenario of a boy who developed aplastic anemia after receiving an antibiotic. What is the antibiotic?
A. Tetracycline

207.Bacteria grow in antiseptic and detergent


A. E.coli
Answer: klebsiella
All the bacterial contaminants of these antimicrobial substances were klebsiellaspecies of which all k. Pneumonia species were
multidrug
Http://pubs.sciepub.com/ajnr/2/3/3/

208.Young pt wt recurrent bacterial infections (I believe encapsulated organisms), hx of arthritis treated with IVIG, lymph node
biopsy (central, depletion) In invx ) Immunegloulin within normal range , low lymphocytes on CBC ) >>> what is the disease (
not the Dx the pathophysiology of the disease adenosine …… ( SCID )
Answer:

209.Treatment of type of bacteria " fragilis"

252
Bacteroides species are anaerobic bacteria that are predominant components of the bacterial florae of mucous membrane. These
bacteria are resistant to penicillins, mostly through the production of beta-lactamase. They are part of the normal gi florae and
predominate in intra-abdominal infections and infections that originate from those florae (eg, perirectal abscesses, decubitus
ulcers). In general, b. Fragilis is susceptible to metronidazole, carbapenems, tigecycline, beta-lactam/, and certain antimicrobials of
the cephamycin class, including cefoxitin. The bacteria have inherent high-level resistance to penicillin. Production of beta
lactamase appears to be the main mechanism of antibiotic resistance in b. Fragilis. Clindamycin is no longer recommended as the
first-line agent for b. Fragilis due to emerging high-level resistance.
Https://en.wikipedia.org/wiki/bacteroides_fragilis#cite_note-mandell-12
Http://emedicine.medscape.com/article/233339-treatment

210.Red lesion on the back of the hand with previous prick sensation and fever. What is the diagnosis?
A. Cellulitis
Answer: ??

211.Infectious mononucleosis test….


Answer: monospot.
Monospot® test: horse red blood cells agglutinate on exposure to heterophile antibodies. The monospot® uses this in conjunction
with the principle of the davidsohn differential test. Sensitivity and specificity for monospot® are 70-90% and 100% respectivel
Http://patient.info/doctor/infectious-mononucleosis
Http://emedicine.medscape.com/article/222040-workup

infusion reaction
nephro and oto
thrompcytpenia, neutropenia, agranulocytosis
212.Side effect of vancomycin? anaphylaxid , SJS
Nausea or stomach upset may occur. If any of these effects persist or worsen, tell your doctor or pharmacist promptly.
Remember that your doctor has prescribed this medication because he or she has judged that the benefit to you is greater than the
risk of side effects. Many people using this medication do not have serious side effects.
Tell your doctor right away if any of these rare but very serious side effects occur: dizziness, difficulty hearing (e.g., ringing in the
ears, hearing loss), signs of kidneyproblems (such as change in the amount of urine), fever, chills, persistent diarrhea.
A very serious allergic reaction to this drug is rare. However, seek immediate medical attention if you notice any of the following
symptoms of a serious allergic reaction: rash, itching/swelling (especially of the face/tongue/throat), severe dizziness, trouble
breathing.
Link: http://www.webmd.com/drugs/2/drug-8858/vancomycin-oral/details#side-effects

213.A farmer was injured with a flower for which he was treated. What is the side effect of his treatment?
Answer: no choices
- Farmers and gardeners are commonly affected by a fungal infection called sporotrichosis. The organism causing this
infection is sporothrix schenckii, which is naturally found in soil, hay, and plants.
- Clinical experience has shown that itraconazole (200 mg/day orally) is the treatment of choice for patients with most
localized forms of sporotrichosis, and amphotericin b (3 to 5 mg/day iv) is the preferred treatment for patients who are
severely ill
- Ses of itraconazole are: hearing loss, elevated liver enzymes, hepatotoxicity, and heart failure.
Source: https://yhdp.vn/uptodate/contents/mobipreview.htm?34/0/34822

253
214.An end stage renal disease patient is admitted to the icu. He develop fever. Blood culture showed yeast. What is the
treatment?
Answer: prompt administration of azoles (e.g., fluconazole) or echinocandins (e.g., anidulafungin, caspofungin), depending on the
patient’s clinical picture, co-morbidities and blood culture. The most appropriate drug is fluconazole. Amphotericin b is not used
due to nephrotoxicity.
Review of antifungal medications: http://www.medscape.com/viewarticle/775172_4

215.What is the definitive test for visceral leishmania?


Answer: bone marrow aspiration
Leishmaniasis is a disease caused by an intracellular protozoan parasite (genus leishmania) transmitted by the bite of a female
phlebotomine sandfly.
Laboratory diagnosis of leishmaniasis can include the following:
Isolation, visualization, and culturing of the parasite from infected tissue
Serologic detection of antibodies to recombinant k39 antigen
Polymerase chain reaction (PCR ) assay for sensitive, rapid diagnosis of leishmania species
Other tests that may be considered include the following:
CBC count, coagulation studies, liver function tests, peripheral blood smear
Measurements of lipase, amylase, gamma globulin, and albumin
Leishmanin (montenegro) skin testing (lst) (not fda approved in the united states)
Http://emedicine.medscape.com/article/220298-overview

216.Parasite found in soil?


A. Ascaris
Soil-transmitted helminths refer to the intestinal worms infecting humans that are transmitted through contaminated soil: ascaris
lumbricoides (sometimes called just "ascaris"), whipworm (trichuris trichiura), and hookworm (anclostoma duodenale and necator
americanus).
Http://www.cdc.gov/parasites/sth/

217.Typical presentation of coxsackie b virus.


Symptoms of infection with viruses in the coxsackie b grouping include fever, headache, sore throat, gastrointestinal distress, as
well as chest and muscle pain. This presentation is known as pleurodynia or bornholm disease in many areas. In some cases,
viruses in the coxsackie b family progress to myocarditis or pericarditis, which can result in permanent heart damage or death.

218.Antiviral inhaled how it work (DNA polymerase-nurnd....-prots...)


Inhibits viral neuraminidases; stops release of virus from cells & prevents virus from crossing mucous lining of respiratory tract
Http://reference.medscape.com/drug/relenza-zanamivir-342636
Zanamivir (rx)relenza:
Http://reference.medscape.com/drug/relenza-zanamivir-342636#10

219.A patient is having sore throat, gingivitis and papule and pustule with erythematous base what is diagnosis?
254
Answer: no choices
Possible answer: hsv-1 infection
Hsv is the most common viral infection of the oral/facial area. It has two subtypes: type 1, which affects the oral cavity; and type 2,
which affects the genitals. Primary herpetic gingivo-stomatitis is most commonly observed in children from 7 months to 4 years of
age but can also be found in adolescents or young adults. The primary infection may be asymptomatic but can manifest as severe
gingivo-stomatitis, in which the gingiva are painful, inflamed and ulcerated.
Read more about the differentials of gingivitis in the following link: http://cdn.intechopen.com/pdfs/20292/intech-
etiology_of_gingivitis.pdf

220.What is the most specific test for tb?


Acid-fast bacilli (afb) smear and culture using sputum obtained from the patient: absence of a positive smear result does not exclude
active tb infection; afb culture is the most specific test for tb
Http://emedicine.medscape.com/article/230802-overview

221.An asymptomatic man results showed a positive ppd skin test. What will you do next?
Answer: chest x ray if negative inh for 9 m
Chest radiographs may show a patchy or nodular infiltrate. Tb may be found in any part of the lung, but upper lobe involvement
is most common.
Http://emedicine.medscape.com/article/230802-workup#c12

222. An x-ray of a patient showed upper lung lobe lesion with calcium stone. What is the most likely organism:
A. Proteus
Answer: a
- It is anaerobic gram-negative bacteria from enterobacteriaceae family
- It has the ability to produce high levels of urease, which hydrolyzes urea to ammonia (nh3), which makes the urine
more alkaline
- Increased alkalinity can lead to the formation of crystals of struvite, calcium carbonate, and/orapatite, which can result
in kidney stones
- Proteus species can also cause wound infections, septicemia, and pneumonia, mostly in hospitalized patients
- Cxr can shows large, thick walled upper lobe abscess with an air fluid level, and associated with infiltrates
(proteus pneumonia)
- In general treatment is through antibiotics such as b-lactams.

223.What is type of pneumococcal vaccine pcv?

255
224.A tb patient is on anti-tb drugs. He developed needles sensation in his limbs. Deficiency of which of the following cause his
symptom?
Paresthesia is a sensation of tingling, tickling, pricking, or burning of a person's skin with no apparent physical cause.
The manifestation of a paresthesia may be transient or chronic. The most familiar kind of paresthesia is the sensation
known as "pins and needles" or of a limb "falling asleep". A less well-known and uncommon but important paresthesia
is formication, the sensation of bugs crawling underneath the skin. Https://en.wikipedia.org/wiki/paresthesia
Segmental involvement of the distal ileum, such as in tuberculosis, can cause vitamin b12 deficiency.
Http://jmedicalcasereports.biomedcentral.com/articles/10.1186/1752-1947-2-90
Isoniazid can sometimes cause damage to peripheral nerves. Http://patient.info/medicine/isoniazid-for-tuberculosis

225.What is the treatment of gonorrhea in general ?


Recommended regimen: ceftriaxone 250 mg im in a single dose plus azithromycin 1g orally in a single dose – cdc

226.Which virus cause valvular cancer ?


Data suggest that human papillomavirus (hpv) may be a cause of some vulvar malignancies
Http://emedicine.medscape.com/article/264898-overview

227.A 19 year-old presented with palm and sole papule after 8 weeks of having unprotected sex?
Answer: secondary syphilis
Secondary syphilis present as rash on (palms and soles) (master the boards)

228.A patient with trichomonas is completely asymptomatic. When to treat?


Answer: treat the patient and partner even if they are asymptomatic..
Treatment should be instituted immediately and, whenever possible, in conjunction with all sexual partners. 5-nitroimidazole drugs
are used for the treatment of trichomoniasis, single-dose therapy of metronidazole.
256
Http://emedicine.medscape.com/article/230617-overview

229.Which antibiotic give for shigella gastroenteritis in pediatrics?


A. Ampicillin
Antibiotic treatment may shorten illness duration and shedding but does not prevent complications. Most mild infections will
recover without antibiotics. Moderate to severe cases should be treated with antibiotics. Ampicillin is preferred for drug-sensitive
strains. For ampicillin-resistant strains or in cases of penicillin allergy, trimethoprim-sulfamethoxazole is the drug of choice, although
resistance does occur. Fluoroquinolones may be considered in patients with highly resistant organisms.
Http://emedicine.medscape.com/article/176400-medication

230.Food poisoning > shigella -- what ttt


Source of shigella is food/water; associated with overcrowding. Symptoms are fever, nausea, vomiting, severe bloody diarrhea,
abdominal pain (risk of hus). Treatment is hydration; self-limited; ciprofloxacin, tmp-smx in severe cases. (step up to step 2 ck)

231.Salmonella gastroenteritis which ab??


Salmonella gastroenteritis is usually a self-limiting disease. Fluid and electrolyte replacement may be indicated in severe cases.
Because antibiotics do not appear to shorten the duration of symptoms and may actually prolong the duration of convalescent
carriage, they are not routinely used to treat uncomplicated nontyphoidal salmonella gastroenteritis. Current recommendations are
that antibiotics be reserved for patients with severe disease or patients who are at a high risk for invasive disease
Azithromycin is likely to be the preferred empirical treatment, often given together with ceftriaxone, in developed countries where
chloramphenicol is usually reserved for life-threatening situations, for which no alternatives are available, and physicians are
reluctant to use fluoroquinolones in children and lack easy access to gatifloxacin.
Http://emedicine.medscape.com/article/228174-treatment

232.Another same q what is the mechanism ??

233.Picture of parasite or fungal ( not sure ) and patient complain of diarrhea ( blood and mucous):
A. Metronidazole

234.Best test to diagnose syphilis:


A. Fluorescent treponemal antibody-absorption
Answer: a

235.Tb case; best test:


A. Sputum cx
Answer: a

257
236.Treatment of enteric fever?
A. Ciprofloxacin
Answer: a

237.Male patient has diarrhea after he came back from indonesia and ate from their food, what is the organism?
A. Answer: traveler’s diarrhea e. Coli.

238.Most common organism of infective endocarditis?


(no choices listed)
Answer: (varies between references). Uptodate, medscape, goldman cecil medicine:
StaphylococcalIEis substantially more common in health care associated ie. Streptococci remain common causes of community
acquired ie. In a cohort study: s. Aureus 31% - viridans 17%
Native valve:
Community dwelling patients: streptococcus viridans. Patients with health care contact or injection drug use: s.aureus
Prosthetic valve:
Early within 2m: s. Aureus and coagulase negative staphylococci. Followed by gram neg bacilli and fungi
Late > 2m: streptococci and s. Aureus. Followed by coagulase negative staphylococci and enterococci.
Overall, s. Aureus is the most common cause of ie, including prosthetic valve endocarditis, acute ie, and iv drug abuse ie.
Http://emedicine.medscape.com/article/216650-overview#a4

239.Duke criteria?
(no choices listed)
Answer:

258
259
240.Patient with flushing face & neck, which antibiotic she's using:
A. Answer: vancomycin (called "red neck" or "red man syndrome") (rxlist) http://www.rxlist.com/vancomycin-injection-side-
effects-drug-center.htm

260
241.Case of infectious mononucleosis ( diagnosis )bmj http://bestpractice.bmj.com/best-
practice/monograph/123/diagnosis/tests.html

242.Diagnosis of pertussis:

PCR or culture of a nasopharyngeal aspirate or swab from the posterior nasopharynx. Bmj http://bestpractice.bmj.com/best-
practice/monograph/682/diagnosis/tests.html

243.Indole positive bacteria:


Aeromonas hydrophila, aeromonas punctata, bacillus alvei,edwardsiella sp., escherichia coli, flavobacterium sp., haemophilus
influenzae, klebsiella oxytoca, proteus sp. (not p. Mirabilis and p. Penneri), plesiomonas shigelloides,pasteurella multocida,
pasteurella pneumotropica, enterococcus faecalis, and vibrio sp. Microbiologyinfo http://www.microbiologyinfo.com/indole-test-
principle-reagents-procedure-result-interpretation-and-limitations/

244.Patient in icu, esrd develops fever culture show yeast on blood what is the?
Answer: it is candidemia

245.Best method to prevent food poisoning


Answer: thoroughly cooked food and rewarm
The four c’s of prevention: cleaning, cooking, chilling & cross contamination (avoiding it) http://www.nhs.uk/conditions/food-
poisoning/pages/prevention.aspx

246.Painless genital ulcer. ?


A. Syphilis
Primary syphilis: single, painless, well-demarcated ulcer (chancre) with a clean base and indurated border. Mild or minimally tender
inguinal lymphadenopathy.
Reference: http://www.aafp.org/afp/2012/0201/p254.html

261
247.Patient with diarrhea and cx-ray showing bilateral infiltrates (pneumonia). Which organism responsible?
Answer: legionella : the patient is having atypical pneumonia.
Reference: https://www.nlm.nih.gov/medlineplus/ency/article/000079.htm

248.Recurrent viral & fungal infection cell deficient?


1. T cell
Answer: a, link : http://www.antimicrobe.org/e33.asp

249.Child came with rhinorrhea, cough, respiratory distress, which vaccine can prevent this disease?
A. Haemophilus influenzae serotype b (hib)

250.Know about hsv1and hsv2? What is the difference?


Herpes simplex viruses are ubiquitous, host-adapted pathogens that cause a wide variety of disease states. Two types exist:
herpes simplex virus type 1 (hsv-1) and type 2 (hsv-2). Both are closely related but differ in epidemiology. Hsv-1 is traditionally
associated with orofacial disease (see the image below), while hsv-2 is traditionally associated with genital disease; however,
lesion location is not necessarily indicative of viral type.

251.Patient eat mushroom came with nausea vomiting .abdominal pain what the mechanism of this?
[2]
Possible toxins: alpha-amanitin: α-amanitin is an inhibitor of RNApolymerase ii and iii. this mechanism makes it a deadly
toxin. For 6–12 hours, there are no symptoms. This is followed by a period of gastrointestinal upset (vomiting and profuse,
[22]
watery diarrhea). This stage is caused primarily by the phallotoxins and typically lasts 24 hours. At the end of this second
stage is when severe liver damage begins. The damage may continue for another 2–3 days. Kidney damage can also occur. Some
[23]
patients will require a liver transplant. amatoxins are found in some mushrooms in the genus amanita, but are also found in
[10]
some species of galerina and lepiota. overall, mortality is between 10 and 15 percent.
Muscarine: muscarine stimulates the muscarinic receptors of the nerves and muscles. Symptoms include sweating, salivation,
[29]
tears, blurred vision, palpitations, and, in high doses, respiratory failure. muscarine is found in mushrooms of the genus
omphalotus, notably the jack o' lantern mushrooms. It is also found in a. Muscaria, although it is now known that the main effect
of this mushroom is caused by ibotenic acid.

252.Pt with red points in palms and hands, buttocks, fever. Turned into vesicles
A. -herpes simplex

253.Pt travelled, came back with large watery diarrhea, dx:


262
A. The more likely answer is traveler’s diarrhea caused by e. Coli

anerobic gm-ve bacteria


254.Bacteroids in gunshot wound abdomen, what antibiotics:
Penicillin g is still the drug of choice against most non–beta-lactamase–producing agnb. However, in addition to the b fragilis group,
which is resistant to penicillin, other agnb show increased resistance.

The combination of beta-lactamase inhibitors (eg, clavulanic acid, sulbactam, tazobactam) with a beta-lactam antibiotic (eg,
ampicillin, amoxicillin, ticarcillin, piperacillin) can overcome these beta-lactamase–producing agnb.
In high concentrations, ticarcillin, piperacillin, and mezlocillin have good activity against gram-negative enteric bacilli and most
anaerobes cefoxitin is the most effective cephalosporin against the b fragilis group, although 5-15% may be resistant.
Carbapenems these agents, including imipenem, meropenem, doripenem, and ertapenem have excellent activity against a broad
spectrum of aerobic and anaerobic bacteria.

255.N. Meningitis meningococcemia, prophylaxis for family:


Answer: rifampin

256.Question about syphilis ulcer. How will you diagnose?


Answer: dark field microscopy
Explanation: primary syphilis produce painless that is positive under darkfield microscopy for spirochete
Reference kaplan ob/gyne usmle step2

257.Most common organism of infective endocarditis?


A. Staphylococcus aureus
Answer: overall, s. Aureus is the most common causative organism in infective endocarditisreference toronto notes 2015

258.College guy with meningococcemia how to prevent spread:

Isolation for 4 weeks . Chemoprophylaxis seems to be most appropriate especially since classically the case arises in a college
student living in a dorm. Rifampin is commonly used for meningococcal prophylaxis of household contacts in the united states,
where one third of the prevalent strains are sulfadiazine resistant. A 2-day course of rifampin is recommended. A single dose of
ciprofloxacin has been found to provide an effective alternative to rifampin for the eradication of meningococcal carriage in adults.

259.Why we give new influenza vaccine every year?


The rational of giving the vaccine yearly is related to the high mutagenicity of the influenza viruses. Influenza virus is remarkable for
its high rate of mutation, compromising the ability of the immune system to protect against new variants. As a consequence, new
263
vaccines are produced each year to match circulating viruses. Currently, vaccine production takes, on average, six months from the
selection of seed strains to the final vaccine product. The decision of which influenza antigens to include in the vaccines is made in
advance of the influenza season and is based upon global surveillance of influenza viruses circulating at the end of the prior
influenza season. As a result, rarely there are mismatches between the vaccine strain and the circulating strain that result in reduced
efficacy of the vaccine.

260.What the agar used for mycobacterium?


Culture of mycobacteria:
There are three types of traditional culture media: egg based (lowenstein-jensen), agar based (middlebrook 7h10 or 7h11), and
liquid (middlebrook 7h12 and other commercially available broths). Growth in liquid media is faster (one to three weeks) than
growth on solid media (three to eight weeks). Growth tends to be slightly better on egg-based medium, but growth is more rapid on
agar medium. Agar medium permits examination of colony morphology and detection of mixed cultures.

261.New born with meningitis organism + cocci??


Etiology of neonatal meningitis:
Group b streptococci (gbs) are the most commonly identified causes of bacterial meningitis, implicated in roughly 50% of all cases.
Escherichia coli accounts for another 20%. Thus, identification and treatment of maternal genitourinary infections is an important
[5]
prevention strategy. listeria monocytogenes is the third most common pathogen, accounting for 5-10% of cases; it is unique in that
it exhibits transplacental transmission.

262.Diagnosis of giardiasis:
Diagnosis — tools for diagnosis of giardiasis include antigen detection assays and stool examination. In areas where available,
antigen detection tests are preferred over stool examination.
Antigen detection assays — a number of immunoassays using antibodies against cyst or trophozoite antigens have been developed
for stool analysis. Available kits include direct immunofluorescent assays (dfa) that use fluorescein-tagged monoclonal antibodies,
immunochromatographic assays, and enzyme-linked immunosorbent assays (elisas). In general, these methods have greater
sensitivity and faster turn-around time than conventional stool microscopy methods.

263.Wet cutaneous leishmania transmitted by?


Answer: sand fly
Leishmaniasis is caused by infection with leishmania parasites, which are spread by the bite of phlebotomine sand flies. There are
several different forms of leishmaniasis in people. The most common forms are cutaneous leishmaniasis, which causes skin sores.
Http://www.cdc.gov/parasites/leishmaniasis/

264.Brucellosis prevention?
A. Pastrization.

Answer: a
The best way to prevent brucellosis infection is to be sure you do not consume:
• undercooked meat
• unpasteurized dairy products
264
Http://www.cdc.gov/brucellosis/prevention/index.html

265.DM with uti, culture shown, antiseptic ? Psedoumonas.


True. Pseudomonas is known to persist in hospital solutions and antiseptics.

266.Prophylactic of rheumatic disease normal pt ?


Answer: pencillin

267.Q about pt having some infection and having allergy from penicillin. What other medications will you give?
Answer: ??
1. What about other types of antibiotics (alternatives)?
Tetracyclines (e.g. Doxycycline), quinolones (e.g. Ciprofloxacin), macrolides (e.g. Clarithromycin), aminoglycosides (e.g.
Gentamicin) and glycopeptides (e.g. Vancomycin) are all unrelated to penicillins and are safe to use in the penicillin allergic
patient.
Reference: http://www.nhstaysideadtc.scot.nhs.uk/antibiotic%20site/penhypers.htm

268.Syphilis case, what do you give?


Answer: penicillin benzathine (aka penicillin g)
Benzathine penicillin g (one dose im) is the preferred agent. If the patient is allergic to penicillin, give oral antibiotics
(doxycycline, tetracycline) for 2 weeks.

Reference: step up to medicine

269. Which of the following antibiotics is DNA gyrase and it works on what organism?
Answer: fluroquinolon = pseudomonus

265
folic acid metabolism
direct DNA damage

DNA gyrase

270.Fever, flu, then developed rashes, which virus ? Measles

271.A patient diagnosed with n. Gonorrhea what other infection you should look for ?
Answer : chlamydia
(master the board)
Also: http://emedicine.medscape.com/article/218059-overview

272.A patient given yellow fever vaccine, started vomiting and sob.. Can’t remember the rest of q.
Answer: http://www.nhs.uk/conditions/yellow-fever/pages/prevention.aspx

273.A scenario about a patient who deals with flowers and got pricked by rose thorn, presented with redness at the prick site.
Which of the following is the cause:
Answer: sporothrix schenckii . Http://botit.botany.wisc.edu/toms_fungi/feb2003.html

274.Mechanism of action of inhaled antivirus?


Answer: block the ion channel function of the m2 protein of influenza a virus , thus interfering with corresponding specific steps in
the viral life cycle. The neuraminidase inhibitors are novel drugs, designed on the basis of the three-dimensional structure of the

266
influenza a and b neuraminidase the mechanism by which antiviral drugs interrupt the replicative cycle of influenza is illustrated. M2
inhibitors prevent the m2-mediated acidification of the interior of the virus while it resides in endosomes and the subsequent
uncoating of the viral genome, thus inhibiting viral replication. Neuraminidase inhibitors (nais) prevent cleavage of sialic acid
residues and thus newly formed virus cannot be released from the cell surface to infect adjacent cells; also, virus particles remain
associated to one another.

275.Painless genital ulcer + lymph nodes enlargement?


A. Syphilis (chancre)
Answer: a

276.Female complain of malaise rash all over the body sparing the face:
A. Syphilis.
Answer: a. Secondary syphilis (can resolve spontaneously) ƒ 2-6 mo after initial infection ƒ nonspecific symptoms: malaise, anorexia,
headache, diffuse lymphadenopathy ƒ generalized maculopapular rash: palms, soles, trunk, limbs ƒ condylomata lata: anogenital,
broad-based fleshy gray lesions ƒ serological tests usually positive except the face, this sx after unprotected sex. Dx?!

277.2 cases about EBV, one was about a diagnosis (which was infectious mononucleosis) the other one was about the causative
organism.

278.Female with jaundice. Her husband is HBV +ve , they did for her hbsag –ve, HAV –ve, HBV-ve. What to do next?
A. Hbcag
Answer: a

279.Scenario about herpes zoster ,, they asked about treatment.


Toronto:
Herpes zoster management:
Compress with normal saline, burow’s, or betadine solution.
Analgesics (NSAIDs, amitriptyline)
Famciclovir or valacyclovir or acyclovir for 7 d; must initiate within 72 h to be of benefit; iv acyclovir for ophthalmic or disseminated
involvement.
Gabapentin 300-600 mg po tid for post-herpetic neuralgia.

280.Abscess in which area in neck will extend to mediastinum


A. Retropharyngeal

281.Parasite transmitted by meat?


A. Trichinosis

282.Most common cause of hepatitis a?


267
Hepatitis a is usually spread when the hepatitis a virus is taken in by mouth from contact with objects, food, or drinks contaminated
by the feces. Http://www.cdc.gov/hepatitis/hav/afaq.htm

283.Most common infectious disease in pre-employment ex?


A. HBV

284.Known vulvar damage, undergoing tooth extraction, what is the organism :


A. Strep viridians
Explanation: most subacute disease caused by s viridians infection is related to dental disease. Most cases are not caused by dental
procedures but by transient bacteremias secondary to gingivitis. In 85% of patients, symptoms of endocarditis appear within 2
weeks of dental or other procedures.
Reference: http://emedicine.medscape.com/article/216650-clinical

285.Traveller diarrhea treatment !


Answer: rehydration therapy and antibiotic, like : fluoroquinolone antibiotics are the drugs of choice. Trimethoprim-
sulfamethoxazole and doxycycline are no longer recommended because of high levels of resistance to these agent
Http://www.merckmanuals.com/professional/gastrointestinal-disorders/gastroenteritis/traveler’s-diarrhea

general, close contacts are defined as household or day-care members who sleep or eat in the same dwelling as the index patient. Therefore, health care workers do not require
chemoprophylaxis unless close contact with the patient's secretions occurs, as in mouth-to-mouth resuscitation
286.Student had a meningitis what should you do for other students?
RIFAMPINE FOR 4 DAYS( H.influenza) and 2 days ( N.meningitidis)
A. Antibiotics prophylaxis or ciprofloxacin
Answer:a if pregnant ( I/M ceftriaxone)
Http://www.medscape.com/viewarticle/451596_4

287.A scenario about a patient who deals with flowers and got pricked by rose thorn, presented with redness at the prick site.
Which of the following is the cause:
A. Sporothrix schenckii .
Answer is a
Sporotrichosis (also known as "rose gardener's disease"[1]) is a disease caused by the infection of the fungus sporothrix schenckii
Https://en.wikipedia.org/wiki/sporotrichosis

288.Hepatitis prevented by vaccine:


Answer: hepatitis b.
Plasma-derived and recombinant HBV vaccines use hbsag to stimulate the production of anti-hbs in noninfected individuals. The
vaccines are highly effective, with a greater than 95% rate of seroconversion. Vaccine administration is recommended for all infants
as part of the usual immunization schedule, as well as for adults at high risk of infection (eg, those receiving dialysis and healthcare
workers).
Http://emedicine.medscape.com/article/775507-treatment#d15

289.Viral gastroenteritis prevented by which vaccine

268
Answer: rota vaccine . Viruses are responsible for a significant percentage of gastroenteritis cases affecting patients of all ages. Viral
gastroenteritis ranges from a self-limited watery diarrheal illness (usually < 1 wk) associated with symptoms of nausea, vomiting,
anorexia, malaise, or fever, to severe dehydration resulting in hospitalization or even death.
Http://emedicine.medscape.com/article/176515-followup

290.Women not oriented and confused her husband have HBV ig titer is -ve direct bilirobin is high (55) what will you do to
confirm dx:
A. Igm
Answer: a
Hepatitis b surface antigen (hbsag) and hepatitis b e antigen (hbeag) (marker of infectivity) are the first markers that can be
identified in the serum in acute disease. Hepatitis b core antibody (anti-hbc) immunoglobulin m (igm) follows.
Reference: http://emedicine.medscape.com/article/177632-workup#c7

291.True fever vs factitious fever :


Refers to a condition in which the patient has an elevated temperature (fever) but despite investigations by a physician no
explanation has been found( it should for a week inpatient and a 3 wks. Outpatients).
Https://en.wikipedia.org/wiki/fever_of_unknown_origin

292.Long case most probably leishmania disease. What is the treatment for this case
Answer: pentamidine

293.Why we give influenza vaccine yearly?


Answer: because antigenic drift

drift: small changes in the genes of influenza viruses that happen continually over time ( cross-protection)
and the body can identify them
294.Differentiate between antigenic “drift” and “shift” Shift: abrupt, major change in the influenza A viruses, resulting in new hemagglutinin ( H1N1)
Http://www.cdc.gov/flu/about/viruses/change.htm
While influenza viruses are changing by antigenic drift all the time, antigenic shift happens only occasionally.
Type A viruses undergo both kinds of changes; influenza type B viruses change only by the more gradual
process of antigenic drift

295.HIV patient has velvety skin rash :


A. Kaposi sarcoma
Explanation: kaposi sarcoma is a multicentric vascular tumor caused by herpesvirus type 8. It is the most common aids associated
cancer, and may be the first aids presentation. Presents as cutaneous lesions are asymptomatic purple, pink, or red macules that
may coalesce into blue-violet to black plaques and nodules.
Reference: http://www.msdmanuals.com/professional/dermatologic-disorders/cancers-of-the-skin/kaposi-sarcoma

296.Fever, diarrhea bloody and stool microscopic showed ( pic ) based on the organism above which drug supposed to be given ?
Answer: metronidazole
Clostridium difficile pseudomembranous colitis results from a disturbance of the normal bacterial flora of the colon, colonization by
c difficile, and the release of toxins that cause mucosal inflammation and damage. Antibiotic therapy is the key factor that alters the
269
colonic flora. C difficile infection (cdi) occurs primarily in hospitalized patients. Http://emedicine.medscape.com/article/186458-
overview

297.After appendectomy he has lung abscess which antibiotic you will choose:
Most abscesses develop secondary to aspiration and are caused by anaerobes. Clindamycin covers anaerobes.

298.Best way to prevent human brucellosis?


A. Pasteurization of milk products
Refrence: cdc
Http://www.cdc.gov/brucellosis/prevention/index.html

299.Tb medication causing red urine :


A. Rifampin
Answer; a, reference ; http://aippg.net/threads/a-patient-with-tuberculosis-develops-bright-orange-red-urine.82781/

300.Scenario of a child after urti there is a history of bleeding after brushing his tooth lab :low platelets. Bone marrow:
megakaryocyte what is the treatment?
Prednisolone i couldn't remember the other choices
Recommended general approach for children with acute immune thrombocytopenic purpurafor initial (induction) treatment, in
patients with a platelet count of 20-30 × 109/l [20-30 × 103/μl] and/or mucocutaneous bleeding), one regimen is prednisone 4-8
mg/kg/d with the intent of a rapid and complete taper after 7-10 days or when the platelet count reaches 50 × 109/l (50 × 103/μl),
Reference : for diagnosis http://emedicine.medscape.com/article/202158-clinical
For treatment http://emedicine.medscape.com/article/202158-medication

301.Reiter's syndrome
** the classic triad of arthritis, urethritis and conjunctivitis
Reference : http://www.aafp.org/afp/1999/0801/p499.html
Http://www.fpnotebook.com/rheum/spondylitis/rtrssyndrm.htm

302.Patient with history urethral discharge culture (negative )and severe unilateral knee pain what is the diagnosis ?

303.Croup case what will you hear by pulmonary auscultation inspiratory stridor,
Http://www.msdmanuals.com/professional/pediatrics/respiratory-disorders-in-young-children/croup

304.Review conjunctivitis

270
305. Taenia saginata
Taeniasis in humans is a parasitic infection caused by the tapeworm species taenia saginata (beef tapeworm), taenia solium (pork
tapeworm), and taenia asiatica (asian tapeworm). Humans can become infected with these tapeworms by eating raw or
undercooked beef (t. Saginata) or pork (t. Solium and t. Asiatica). People with taeniasis may not know they have a tapeworm
infection because symptoms are usually mild or nonexistent.
Rx: treated with praziquantel
Ref. Cdc

306.Elderly with back bone pain, bone biopsy positive acid fast bacilli ,the organism?
Answer: mycobacterium tuberculosis

307.Where do latent herpes virus stay?


A. Macrophages. in sensory ganglia ( cell body)
Answer: sensory neuron?

308.How can prevent dust mite?


Http://www.aafa.org/page/dust-mite-allergy.aspx

309.Side effect of the medication used to treat infection (you saw donovan body):
A. Answer: no choices.
Donovan bodies or klebsiella granulomatis are intracellular gram-ve rod organisms seen on ulcer smear in individuals with the
infection: granuloma inguinale.
Azithromycin (1st line tx) se: diarrhea, n/v, abdominal pain
Source: http://reference.medscape.com/drug/zithromax-zmax-azithromycin-342523#4
Donovan bodies (or leishman-donovan bodies) can also refer to the organisms leishmania donovani seen on blood or bone marrow
smear. It is found in patients with visceral leishmaniasis.
Liposomal amphotericin b (1st line tx in vl) se: fever, chills, n/v, hypokalemia, hypomagnesemia, nephrotoxicity
Source: http://reference.medscape.com/drug/ambisome-amphotericin-b-liposomal-999576#4

310.What the organism for someone sky with diver ?

311.Type produce inside cell( DNA , RNA...)

312.Clear question about ( infectious mononucleosis)

271
Most patients with epstein-barr virus (EBV) infectious mononucleosis are asymptomatic and, therefore, have few if any symptoms.
Most adults (approximately 90%) show serological evidence of previous EBV infection.
Http://emedicine.medscape.com/article/222040-overview

313.Which antibiotic is used in the treatment of salmonella?


A. Empiric treatment with ceftriaxone or azithromycin. Fluoroquinolone resistance is increasing
Reference: toronto notes

314.A patient had repeated attacks of reddish rash and plaques in mouth. What is the diagnosis?
A. Answer: candidiasis?

Pseudomembranous candidiasis. The typical adherent white plaques may be removed by wiping firmly with a tongue blade or gauze

315.Pt had constpation +tender abd +wbc high +fever+ s.s of uti....rx?
D. Ceftro (antibotic) ..perdonsine(stroid )

316.A scenario about a patient who deals with flowers and got pricked by rose thorn, presented with redness at the prick site.
Which of the following is the cause?
A. Sporothrix schenckii .
Answer is a
Sporotrichosis (also known as "rose gardener's disease"[1]) is a disease caused by the infection of the fungus sporothrix schenckii
Reference: https://en.wikipedia.org/wiki/sporotrichosis

317.Viral gastroenteritis prevented by which vaccine :


a. Rota vaccine
Answer: A
Rotavirus vaccine. A vaccine that helps protect children against gastroenteritis caused by the rotavirus is now part of the routine
childhood vaccination schedule. This vaccine is given as a liquid that is dropped into a baby's mouth. It is given in two doses, with the
first given at two months and another at three months.
Two possible side effects of the vaccine are diarrhoea and irritability, but these are usually mild and short-lived.
Http://www.nhs.uk/Conditions/Rotavirus-gastroenteritis/Pages/Prevention.aspx

318.Hypopigmentation and loss of sensation in forearm with ulnar nerve thickness?

272
A. Leprosy
Answer: Leprosy is a chronic infectious disease caused by Mycobacterium leprae, an acid-fast, rod-shaped bacillus. The disease
mainly affects the skin, the peripheral nerves, mucosa of the upper respiratory tract, and the eyes.

319.Hepatitis prevented by vaccine:


A. Hepatitis B.

320.N. Meningitis meningococcemia, prophylaxis for family:


A. Rifampin
Answer: Deterrence and prevention of meningococcal meningitis can be achieved by either immunoprophylaxis or
chemoprophylaxis. Rifampin, quinolones, and ceftriaxone are the antimicrobials that are used to eradicate meningococci from the
nasopharynx.
(http://emedicine.medscape.com/article/1165557-treatment#d11)

321.Condition which increasing the incidence of TB recurrence?


A. HIV.

322.What does it mean when the patient has positive Hbs Ag?
Answer: Antigen found on surface of HBV; indicates hepatitis B infection (acute or chronic).
Reference: USMLE step 1
(medicine)

323.Honeymoon cystitis which organism

Answer:
Honeymoon cystitis (or "honeymoon disease") is cystitis caused by sexual activity.
The microbial spectrum of uncomplicated cystitis and pyelonephritis in women consists mainly of Escherichia coli (75 to 95 percent),
Reference: http://www.uptodate.com/contents/acute-uncomplicated-cystitis-and-pyelonephritis-in-
women?Source=search_result&search=cystitis&selectedtitle=1~80

324.What are the findings in CSF analysis that determines “bacterial” cause of meningitis?

Answer:
• Appearance: Clear, cloudy, or purulent
• Opening pressure: Elevated (>25 cm H 2 O)
• WBC count: >100 cells/µl (>90% PMN)
• Glucose level: Low (< 40% of serum glucose)
• Protein level: Elevated (>50 mg/dl)

273
325.Patient with watery diarrhea.

Answer: NOT SURE


A. Acute diarrhea most probably is due to GI infection.
B. Most common cause of acute watery diarrhea (gastroenteritis) worldwide is viral (rotavirus)
Reference: http://www.medicinenet.com/rotavirus/article.htm

Currently, pegylated interferon alfa (PEG-IFN-a), entecavir (ETV), and tenofovir disoproxil fumarate (TDF)
are the first-line agents in the treatment of hepatitis B disease.
326.Treatment of HBV? Lamivudine (3TC), telbivudine, and adefovir are of historical interest. These agents are currently considered second- or
Answer: ? third-line therapy, or “nonpreferred” treatmen
Interferone + ribavirin (Kumar)
Patients with acute hepatitis C virus (HBV) infection appear to have an excellent chance of responding to 6 months of standard
therapy with interferon (IFN). Reference:http://emedicine.medscape.com/article/177792-treatment

327.Schistosoma Antibiotics?
A. Praziquantel

Answer: A
Praziquantel.
Add glucocorticoid if acute schistosomiasis or neurologic complications develop. Reference: Toronto notes

328.Patient on central cath developed infection. Blood gram stain shows budding yeast.
A. Fluconazole

274
Answer: A
Reference: Toronto notes

329.The most specific/ best diagnostic test for syphilis?


a. Fluorescent treponemal antibody absorption (FTA-ABS)

Answer: A
• Nontreponemal testing is commonly used for serologic screening for syphilis, which include VDRL, RPR, and TRUST. (These
tests must be confirmed with specific treponemal testing since false positive nontreponemal tests can occur.
Nontreponemal testing is performed initially due to its low cost.)
• Specific treponemal tests include FTA-ABS, MHA-TP, TP-PA, and TP-EIA. Reference: uptodate

330.Signs and symptoms of 1st syphilis?


A. Painless genital ulcer

Answer: A
Reference: http://www.cdc.gov/std/syphilis/stdfact-syphilis-detailed.htm

331.Patient with needle sensation after TB drug.


A. Isoniazid

Answer: A

332.Bacterial meningitis in LP
A. Decrease glucose and increase protein

Answer: A
Glucose level: Low (< 40% of serum glucose) and Protein level: Elevated (>50 mg/dl)
Reference: http://emedicine.medscape.com/article/2172226-overview

333.How is giardia diagnosed?


A. 3 different stool sample

Answer: A
Because Giardia cysts can be excreted intermittently, multiple stool collections (i.e., three stool specimens collected on separate
days) increase test sensitivity. Reference: Centers for Disease Control and Prevention

275
334.40 years old man presents with persistent lesion on the forearm that started 3 months
ago. He came from a deserted area that has sandfly infestation. Giemsa stain showed
Donovan bodies inside and outside monocytes. What will you use to treat him?
A. Miltefosine

Answer: A?
This patient has leishmaniasis.
• Uncomplicated cutaneous leishmaniasis: Topical application of paromomycin or
Intralesional antimony (Sodium stibogluconate, Meglumine antimoniate).
• Complicated cutaneous leishmaniasis: Oral systemic therapy; include azoles and miltefosine
• Agents with activity against visceral leishmaniasis (VL) include amphotericin B, pentavalent antimonial drugs, Paromomycin (a
parenteral aminoglycoside), Miltefosine (the first oral drug for treatment of VL)
Reference: http://www.uptodate.com/contents/treatment-of-cutaneous-leishmaniasis
Additional information ; Topical paromomycin has been shown to be effective against cutaneous leishmaniasis caused by L major ( in
Iraq ) and L mexicana.
While Oral miltefosine is used for Treatment of New World cutaneous leishmaniasis.
Http://emedicine.medscape.com/article/220298-treatment#d9

335.What is the treatment for Chlamydia?


A. Azithromycin

Answer: A
Doxycycline is an alternative to Azithromycin.
Reference: http://www.cdc.gov/std/tg2015/chlamydia.htm

336.(Long scenario) Patient with fever, lymphadenopathy and hepatosplenomegaly…


Answer: ? (Case of Mononucleosis)
See: http://emedicine.medscape.com/article/222040-overview

337.Chlamydia ttt?
Answer: Azithromycin (oral one dose) or doxycycline (oral for 7 days ) + treat all sexuall parners
Reference : step up to medicine

internal and external bleeding (e.g. oozing from the gums, blood in the stools).
338.Ebola very clear ? Laboratory findings include low white blood cell and platelet counts and elevated liver enzymes.
Answer:
Reference: http://www.who.int/mediacentre/factsheets/fs103/en/

339.Parasite infection in under cooked food


Answer: Trichinellosis/Trichinosis
Infection is initiated by ingestion of viable larvae in raw or undercooked meat. Digestive action liberates the larvae.
Reference: http://emedicine.medscape.com/article/787591-overview#a5

276
340.Cholera vibro antibiotic
A. Doxycycline
Answer: Doxycycline inhibits protein synthesis and, thus, bacterial growth by binding to 30S and possibly 50S ribosomal subunits of
susceptible bacteria.
In areas of known tetracycline resistance, therapeutic options include ciprofloxacin and erythromycin. Strains resistant to
ciprofloxacin have been reported from Calcutta, India.
Pharmacotherapy plays a secondary role in the management of cholera; fluid replacement is primary.
Reference http://emedicine.medscape.com/article/962643-medication#2

341.Female pt diagnosed with IBD on ceftriaxon with no benefit wt is the organism


A. Herpes
Can't recall others
Answer: Pseudomonas aeruginosa
Ceftriaxone does not have useful activity against Pseudomonas aeruginosa.It is generally not active against Enterobacter species,
and its use should be avoided in the treatment of Enterobacter infections even if the isolate appears susceptible because of the
emergence of resistance.Some organisms, such as Citrobacter, Providencia, and Serratia, have the ability to become resistant
through the development of cephalosporinases (these enzymes hydrolyze cephalosporins and render them inactive)

342.Monospot test +ve what is the diagnosis?


A. Infectous mononeclosis

Answer: A
Epstein-Barr virus (EBV) infectious mononucleosis has been recognized as a clinical syndrome consisting of fever, pharyngitis, and
adenopathy. Heterophile test antibodies are sensitive and specific for EBV heterophile antibodies, they are present in peak levels 2-6
weeks after primary EBV infection, and they may remain positive in low levels for up to a year.The latex agglutination assay, which is
the basis of the Monospot test using horse RBCs, is highly specific. Sensitivity is 85%, and specificity is 100%
Http://emedicine.medscape.com/article/222040-workup
Https://www.nlm.nih.gov/medlineplus/ency/article/003454.htm

343.Definitive diagnosis of TB.

Answer: culture is done on all initial samples to confirm the diagnosis. (However, a positive culture is not always necessary to begin
or continue treatment for TB.) A positive culture for M. Tuberculosis confirms the diagnosis of TB disease. Skin test (PPD\Mantoux
test) diagnose prior exposure or infection of TB, it doesn’t tell if the person has current or progress infection. Sputum culture (acid-
fast bacilli smear) is needed to confirm active TB.
Reference: http://www.cdc.gov/tb/publications/factsheets/testing/diagnosis.htm, Toronto notes 2015 page (ID 28)

344.How to dx hepatitis B?
Answer: Hbsag, hepatitis B core antibody (anti-hbc), and hepatitis B surface antibody (anti-hbs)

277
Http://62.210.217.55/humhub-1.0.1/space/page/hepatitis-b-blood-test-results-interpretation
Http://www.hepatitisaustralia.com/hepatitis-b-facts/
Https://labtestsonline.org/understanding/analytes/hepatitis-b/tab/test/

345.Pt with heart valve lesion developed endocarditis, what is the organism?

Answer: Streptococcus viridans


Native valve: Streptococci account for 50%–80% of IE cases ( Streptococcus viridans).
Prosthetic valve:
• Early-onset (within 2 months after surgery): Staphylococcus aureus and Staphylococcus epidermidis are the most common.
• Late-onset (more than 2 months of surgery): streptococci is more common.
Staphylococcus aureus is the most common in intravenous drug abusers (ivdas) (most commonly affecting tricuspid valve).
Http://emedicine.medscape.com/article/216650-overview#a4
Http://www.ncbi.nlm.nih.gov/books/NBK2208/
Step up to medicine book (page 47)
278
346.Infection in venous lines? With needle insertion. (Something like that)

Answer: catheter related blood stream infection (CRBSI), central line associated blood stream infection (CLABSI)
Gram-positive organisms currently account for ~ 50–60% of nosocomial bacteremic events. Staphylococcus epidermidis( coagulase
negative staphylococci) is the most common gram-positive organism isolated from blood (~ 30% of isolates) and accounts for the
majority of infections that are associated with an intravascular catheter. Staphylococcus aureus also causes a significant number of
bloodstream and intravascular catheter. Enterococci have become a significant cause of bacteremia in surgical patients and have
been isolated increasingly from patients with burns or multiple injuries
Gram-negative bacteria account for ~ 30% of all episodes of bacteremia at most institutions.
The mortality associated with gram-negative bacteremia in normal individuals is ~ 10% and may exceed 50% in
immunocompromised patients. The most common causative microbes include Escherichia coli, Klebsiella pneumoniae, Pseudomonas
aeruginosa, Enterobacter aerogenes and cloacae, although a vast array of organisms can be responsible.
Candida most common in yeast category.
In pathogenesis: there are 4 routes for contamination, the most common route of infection for short-term catheters is migration of
skin organisms at the insertion site into the cutaneous catheter tract and along the surface of the catheter with colonization of the
catheter tip.
Http://www.ncbi.nlm.nih.gov/books/NBK7008/
Http://www.cdc.gov/hicpac/BSI/04-bsi-background-info-2011.html
Http://www.ncbi.nlm.nih.gov/pmc/articles/PMC3805442/
Http://lifeinthefastlane.com/ccc/central-line-infections/

347.Patient with lymphadenopathy , splenomegaly and fever, what is the diagnosis?


A. Infectious mononucleosis
Answer:A
Infectious mononucleosis by EBV , early sign is a fever, lymphadenopathy. Later finding include hepatomegaly and splenomegaly.
Reference: medscape

348.Gastoentritis ddx
Answer:
DIFFERENTIAL DIAGNOSIS — The differential diagnosis of acute viral gastroenteritis includes other causes (infectious and non-
infectious) of acute diarrhea. Diarrhea that lasts over a week in an individual with a history of travel, hiking, or oral-anal sexual
activity should prompt evaluation for protozoa such as Giardia and cryptosporidium. Recent antibiotic use or hospitalization should
prompt consideration of Clostridium difficile infection. Common foodborne illnesses (eg, Staphylococcus aureus) need to be
considered, particularly when the incubation period is shorter than is typical for viral illness (ie, within 8 to 16 hours). The presence
of alarm symptoms or signs should prompt further investigation for an alternate diagnosis.
Causes of chronic diarrhea that may less commonly masquerade as acute viral gastroenteritis include: colorectal cancer, irritable
bowel syndrome, inflammatory bowel disease, microscopic colitis, malabsorption syndromes, post-cholecystectomy related
diarrhea, medication-induced diarrhea, laxative abuse, and chronic infections. Patients with acute viral gastroenteritis may also
present with isolated vomiting without prominent diarrhea. Clinicians should consider adverse effects of medications and acute
vestibular disorders in the differential diagnosis of these patients.
Uptodate

279
349.Type of lachmenia
Answer :

350.Splenctomy what vaccine he should took ?


Answer: pneumococcus, Haemophilus influenza type B (Hib), and meningococcus
''People without a functional spleen have certain vaccinations to reduce the risk of sepsis. Although these vaccines do not prevent all
infections with pneumococcus, Haemophilus influenzaetype B (Hib), and meningococcus, they can protect most people and reduce
the severity of infection in others''
Reference:
Http://www.uptodate.com/contents/preventing-severe-infection-after-splenectomy-beyond-the-basics

351.Mycobacterium tuberculosis , that is the best culture media ?


A. Blood agar
Answer: löwenstein Jensen agar
I think the blood agar is not the correct answer
Löwenstein–Jensen medium is a growth medium especially used for culture of Mycobacterium, notably Mycobacterium
tuberculosis.
Also egg based media was used so I don’t know the correct answer without the choices.
References: http://www.microbiologyinpictures.com/mycobacterium-tuberculosis.php

352.What is the best Tx for traveler diarrhea ?


A. Ciprofloxacin
Answer: ciprofloxacin (you might take anti diarrhea with it like loperamide)
Reference: http://www.aafp.org/afp/2005/0601/p2095.html

353.Brucellosis prevention?
A. Pastrization
Answer: A

280
Rheumatology

281
1. How are you going to manage a HTN patient with acute gout?
A. NSAID
B. Allopurinol
C. Colchicine
D. Intra articular steroid
Answer: d
The treatment of acute gout is NSAIDs: high dose, then taper as symptoms improve but
corticosteroids (ia, oral, or intra-muscular) if renal, cardiovascular, or gi disease and/or if NSAIDs contraindicated or failed.
Reference:toronto notes

2. High density bone in dexamethasone with scheduled total hip replacement


A. Osteoporosis
B. Osteomalacia
C. Osteoarthritis
D. Paget disease
Answer: d
It could be paget disease. However, steroids have no role in its treatment except in case of developing pseudomalignancies. .
Another possibility is osteopetrosis (marble bone disease) which also has increased bone density & steroids are used to enhance
bone resorption and improve the anemia. Both commonly affect pelvis and increase its risk of fracture. So it could’ve been in the
answers.
Http://www.aafp.org/afp/2002/0515/p2069.html
Http://emedicine.medscape.com/article/123968-medication

3. Most specific sign of osteropreosis?


A. Increase alp
B. Increase calcium
C. Calcification
D. Decrease bone matrix
Answer: d
4. Bechet disease is more common in:
A. Children
B. Young men
C. Old women
D. Equal in men & women
Answer:
The sex prevalence varies by country. In the middle east, behçet disease is more common among males, with male-to-female ratios
of 3.8:1 (israel), 5.3:1 (egypt), and 3.4:1 (turkey). In germany, japan, and brazil, the disease is slightly more common in females. In
the united states, behçet disease is more common in females (5:1 female-to-male ratio)
Link: http://emedicine.medscape.com/article/329099-overview#a7

5. Female patient with skin thickening over the forearm, with raynaud’s phenomenon. [case of scleroderma] which of the
following would help you the most?
A. Antiscleroderma - scl 70
B. Anti-centromere
C. Ss-a
D. Ss-b

282
Answer: b
This case is showing limited disease. Symptoms of limited disease include (skin sclerosis is restricted to the hands, the distal forearm,
face and neck. And generally have prominent vascular manifestations, including severe raynaud phenomenon and cutaneous
telangiectasia). So in limited disease anti centromere would help the most. - uptodate

6. Female with arthritis and butterfly rash on the face. Ana is positive. What other marker you would order?
A. Anti double stranded DNA
B. Anti centromere
C. Ss-a
D. Ss-b
Answer: a

7. 61 y female known case of osteoarthritis, came for regular checkup, not taking ca supplements nor high ca diet, she is a high
risk of osteoporosis what is the best initial thing before deciding the appropriate mx ?
A. Dexa scan
B. Oral ca, Vitamin d, biaphpsphonate
C. TSH , ca, Vitamin d
D. Intranasal calcitonin, ca, Vitamin d
Answer: a

8. The first initial treatment of osteoarthritis in middle-aged patient is:


A. Intra articular steroids.
B. Oral steroids.
C. Stairs exercise
283
D. Muscle strengthening.

Answer: d
Optimally, patients should receive a combination of non-pharmacologic and pharmacologic treatment.
Non-pharmacologic: patient education, heat and cold, weight loss, exercise, physical therapy, occupational therapy,
unloading in certain joints (eg, knee, hip).
Pharmacologic: acetaminophen/NSAIDs.
Reference: medscape: http://emedicine.medscape.com/article/330487-treatment#d1

9. Long scenario about a patient who presented with gout. Inhibition of which enzyme will treat this disease?
A. PRPP synthase
B. Adenosine deaminase
C. Xanthine oxidase
D. Orotate phosphoribosyltransferase

Answer: C
The enzyme xanthine oxidase catalyses the oxidation of hypoxanthine to xanthine and then to uric acid, which plays a crucial role in
gout. Reference: Pubmed.
That’s why we give Xanthine Oxidase inhibitors such as Allopurinol.

10. Which of the following is the most specific for Rheumatoid arthritis?
A. HLA-DR4.
B. Rheumatoid factor.
C. CRP.
D. Anti cyclic citrullinated peptide. (Anti-CCP)

Answer: D
Anti CCP is the most specific for RA
Reference: http://qjmed.oxfordjournals.org/content/100/4/193

11. Pt with 1st metatarsal joint pain, redness and erythema. High temperatures? What is the cause?
A. Staph aureus
B. Na monourate crystal
C. Ca pyrophosphate crystal

Answer: b
Came with gout
This joint ,podagra, is typical for gout(sodium monourate crystals) though it is clinically indistinguishable from pseudogout (this is
why it is pseudo)
I think b is correct. Acute gout can come with high grade fever
Reference: http://worldwidescience.org/topicpages/a/acute+gouty+arthritis.html

12. Patient with 1st metatarsal joint pain, redness and erythema. High temperatures. What is the cause?

284
A. Staph aureus
B. Na monourate crystal
C. Ca pyrophosphate crystal

Answer: b
Monosodium urate monohydrate crystals cause gout.
Calcium pyrophosphate crystals cause psuedogout.
The scenario describes podagra, which is spontaneous onset of excruciating pain, edema, and inflammation in the metatarsal-
phalangeal joint of the great toe
Podagra is the initial joint manifestation in 50% of gout cases; eventually, it is involved in 90% of cases.
Psuedogout usually manifests in large joints like the knee.
Reference: Medscape

13. How are you going to manage a patient with a long history of big toe pain with –ve bifrengt [?] Crystals what is the
management
A. Colchicine
B. Allopurinol
C. NSAID

Answer: c

14. Which of the following anti osteoporotic agent cause osteoclast purse [?]?
A. Alendronate
B. Risedronate
C. Ibandronate

Answer:?
These medications are bisphosphonates. They inhibits osteoclastic bone resorption and promotes renal excretion of calcium. There
side effects include gi,msk pain, headache, osteonecrosis of the jaw and atypical femoral shaft fractures .
Reference: toronto notes

15. Dermatomyositis associated with :


A. A-generlized morbiliform eruption
B. B-distal muscle weakness
C. C-malignancy
Answer: c
Reference: http://www.mayoclinic.org/diseases-conditions/dermatomyositis/basics/complications/con-20020727

16. Man with osteoarthritis initial management


A. Intra-articular corticosteroid injections
B. Exercise to increase the strength of thigh muscle
C. Opioid
Answer: b
285
Local analgesics (e.g., capsaicin, methylsalicylate cream, or topical NSAIDs) should be used as first-line therapy. In addition to
exercise programmes, physiotherapy and occupational therapy and quadriceps-strengthening exercises
Http://bestpractice.bmj.com/best-practice/monograph/192/treatment/details.html

17. When diagnosing patient with sle, what's most important:


A. Age
B. Sex
C. Joint affection
Answer: c
Although more than 90% of cases of sle occur in women, gender and age are not as important as “joint affection” because it is part
of the diagnostic criteria of sle
Sle criteria: http://www.rheumtutor.com/2012-slicc-sle-criteria/

18. A patient complains of unilateral knee swelling and pain. Knee tap shows: cloudy yellow color, mucoid, wbc - pmn 15 (normal
less than 200), lymphocytes 80%. What is the most likely diagnosis?
A. Gout
B. Septic arthritis,
C. Rheumatoid arthritis
Answer:

19. Something about prognosis of sle?


A. Sex
B. Arthritis
C. Nephritis
Answer: c

20. 50 yo w painless loss of vision in one eye, with headache and pain when touching the hair on the same side. What is next?
A. A-topical steroids
B. B-oral steroids
C. C-brain ct
Answer: b

286
This appears to be a presentation of giant cell arteritis. Symptoms include:
Headache — also common in the presentation of gca is headache, which occurs in more than two-thirds of patients [15,19]. The
quality of headache in gca, apart from the occasional specific complaint of tenderness of the scalp to touch, has no defining
characteristics.
Jaw claudication — nearly one-half of gca patients experience jaw claudication. In some cases, a trismus-like symptom occurs rather
than fatigue of the muscles of mastication. Two striking features of jaw claudication are its rapid onset after the start of chewing and
the ensuing severity of pain. Patients seldom recognize the significance of symptoms of jaw claudication and must be questioned
directly about this symptom
Transient visual loss (amaurosis fugax) — transient monocular (and, rarely, binocular) impairment of vision can be an early
manifestation of gca. With transient monocular visual loss (tmvl), affected patients typically note an abrupt partial field defect or
temporary curtain effect in the field of vision of one eye.
Treatment:
Glucocorticoid treatment should be instituted promptly once the diagnosis of gca is suspected strongly, often even before it is
confirmed. For patients in whom the diagnostic suspicion of gca is high, especially those with recent or threatened vascular
complications such as visual loss, therapy should be started immediately. A temporal artery biopsy should be obtained as soon as
possible, but treatment should not be withheld while awaiting the performance or the results of the biopsy.
If gca is not complicated by symptoms or signs of ischemic organ damage (eg, visual loss), we suggest an initial dose of glucocorticoid
equivalent to 40 to 60 mg of prednisone in a single dose.

21. Case of gout the q is what case the crystal in the joint:
A. Purine
B. Amino acid
C. Fatty acid
Answer: a
Http://ccdalmatians.com.au/product/urate-crystals-stones/
Uric acid, the final product of purine metabolism, is a weak acid that circulates as the deprotonated urate anion under physiologic
conditions, and combines with sodium ions to form monosodium urate (msu). Msu crystals are known to have a triclinic structure; in
which stacked sheets of purine rings form the needle-shaped crystals that are observed
microscopically. Http://www.ncbi.nlm.nih.gov/pmc/articles/pmc3975080/
Many enzymes are involved in the conversion of two purine nucleic acids, adenine and guanine, to uric acid.
Http://www.sciencedirect.com/science/article/pii/s0167527315303429

22. A patient devolved gout. Which of the following could possibly cause it?
A. Thiazide
B. Hypertension
C. Congestive heart failure
Answer: a
Explanation: gout is caused by defect in urate metabolism which could be due to:
Over-production: idiopathic, increased turnover of cells (cancer, hemolysis, psoriasis, chemotherapy) and enzyme deficiencies (lesch-
nyhan syndrome, glycogen storage disease).
Under-excretion: renal insufficiency, keto/lactic acidosis and thiazide & aspirin
Reference master the board step 2 ck

23. Patient with 1st metatarsal joint pain, redness and erythema .. High temperatures? What is the cause?
A. Staph aureus
B. Na monourate crystal
287
C. Ca pyrophosphate crystal

Answer: b
This question seems to point to gout.
Http://emedicine.medscape.com/article/329958-overview

24. Old man with multiple fractures during the last 2 years, he is k/c of osteoporosis. What should you give him at this stage?
A- alendronate.
B- vitamin d.
C- estrogen.
Answer: a- alendronate.

25. A patient with gout. What drug should be avoided?


A. Thiazide
B. Furosemide
C. ACEI

Answer: A
Hyperuricemia is a relatively common finding in patients treated with a loop or thiazide diuretic and may, over a period of time, lead
to gouty arthritis. Furosemide can elevate uric acid levels as well. Reference: uptodate.

26. 60 years old man with a fractured thoracic vertebrae. T Score = -2.6. What is the diagnosis?
A. Osteoporosis
B. Established osteoporosis
C. Osteopenia

Answer: B
Dexa scan interpretation:
T-score Diagnosis

>–1.0 Normal

<–1.0, >–2.5 Osteopenia

<–2.5 Osteoporosis

<–2.5 plus fragility fractures Severe/ Established osteoporosis

288
27. 70 years old smoker woman with low vit-D and osteoporosis. Which of the following has the highest risk for osteoporosis?
A. Smoking
B. Age
C. Vitamin D

Answer: B
Increasing age is an important risk factor in both men and women, since bones become less dense and weaker with age.
Reference: http://www.medscape.com/viewarticle/410461_3

28. (long scenario) adult with right toe tenderness for 1 week and he have ear pain also (investigation included). What is the best
treatment for him?
A. Cortisone.
B. NSAIDs.
C. Allopurinol.

Answer: B
(For acute attack. Allopurinol can be given after attack resolution)
Treatment of acute gout attacks does not differ substantially in patients with or without clinically apparent tophi, although the
presence of tophi is an indication for the initiation of long-term urate-lowering therapy after attack resolution to prevent or reverse
chronic gouty arthropathy.
Reference:
Http://emedicine.medscape.com/article/329958-treatment#showall
Http://www.uptodate.com/contents/treatment-of-acute-gout

29. Rheumatoid arthritis with the loss of bones in joint? What is the cause?
A. Substance released by synovial cell
B. Synovial fluid pressure
C. Something prostaglandin?

Answer: A
The bone loss is most pronounced in areas immediately surrounding the affected joints. Bone is composed primarily of type I
collagen, bony destruction is a characteristic of RA. This process is primarily driven by the activation of osteoclasts. Osteoclasts
differentiate under the influence of cytokines especially the interaction of RANK with its ligand. The expression of these are driven
by cytokines including TNF and IL1, as well as other cytokines including IL-17. There may also be a contribution to bony destruction
from mediators derived from activated synovial cells.
Http://www.hopkinsarthritis.org/arthritis-info/rheumatoid-arthritis/ra-pathophysiology-2/
Http://arthritis-research.biomedcentral.com/articles/10.1186/ar2337

30. Case of arthritis in first metatarsal joint ?Dx‫ ؟‬sumptoms were pointing to gout
A. Gout ..
B. Ra..
C. Osteoarthritis ..

31. Giant cell arteritis sequelae:


A. Complete loss of vision
B. Destructive arthritis

289
Answer: a
If left untreated may lead to permanent vision loss
Reference: kaplan internal medicine

32. Dermatomyositis associated with :


A. Generlized morbiliform eruption
B. Distal muscle weakness
C. Malignancy
Answer: c
http://www.myositis.org/learn-about-myositis/complications/cancer

33. What is the treatment of temporal arteritis?


A. Oral steroids
B. Topical steroids

Answer: a
The universally accepted treatment of giant cell arteritis (gca) is high-dose corticosteroid therapy. The major justification for the use
of corticosteroids is the impending danger of blindness in untreated patients.
Few studies exist regarding dosing protocols for corticosteroids in gca. It is generally agreed that most patients with suspected gca
should be started on oral prednisone 40-60 mg/ or 1 mg/kg/d with a temporal artery biopsy performed within 1 week.. Higher doses
of 80 to 100 mg/d are suggested for patients with visual or neurological symptoms of gca.
Reference: http://emedicine.medscape.com/article/332483-treatment
Http://www.ncbi.nlm.nih.gov/pmc/articles/pmc3014829/

34. Adolescent male with swelled parotid and salivary gland with dry eye and dry mouth, labs hla, ana and rf are positive, which
of the following is appropriate treatment?
A. Physostegmine
B. Artificial eye and saliva drops

Answer: b

35. Rheumatoid arthritis with the loss of bones in joint? What is the cause?
A. Substance released by synovial cell
B. Synovial fluid pressure

Answer: a

290
Http://www.hopkinsarthritis.org/arthritis-info/rheumatoid-arthritis/ra-pathophysiology-2/

36. Temporal arteritis, confirmatory dx?


A. Biopsy temporal muscle
B. Biopsy temporal artery
Answer b
Superficial temporal artery biopsy (tab) is the criterion standard for making a diagnosis of temporal arteritis. Tab should be obtained
almost without exception in patients in whom gca is suspected clinically.

291
37. 60 years old female with distal phalangeal joint swelling and shoulder pain and knee pain x ray showed narrow joint space
and osteophytes:
A. Rheumatoid arthritis
B. Osteoarthritis
Answer : b toronto notes

38. A clear case of septic arthritis - high wbc high esr?


A. Aspiration
B. Aso
Answer: a definitive diagnosis uptodate

39. Patient with rheumatic fever on asprin developed heart burn what you will add
A. Misoprostol
B. Durg from ( h2 blocker )
Answer: a
Treatment of NSAID induced ulcer: combine NSAID with PPIor misoprostol ( torrento 2015)

40. Patient is diagnosed with rheumatoid arthritis and is on aspirin he developed symptoms of heartburn which is relieved with
antacids what will you give him:
A. Mesoprostol
B. H2 blocker
Answer: a
Ongoing medication for peptic ulcer specifically indicated for non-steroidal anti-inflammatory drug (NSAID)-associated ulcers only
Http://bestpractice.bmj.com/best-practice/monograph/80/treatment/details.html

41. Patient presented with knee swelling and pain they did x ray and aspiration and found negative birefringence needlelike
what are you going to discharge the patient with:
A. Allopurinol
B. NSAID
Answer: b

42. Morning stiffness , pain and weakness in proximal muscles , difficulty in getting out of setting position :
A. Polymyalgia reuhmatica
B. Polymyositis

Answer: b
Explanation: polymositis is progressive symmetrical proximal muscle weakness (shoulder and hip) developing over weeks to months
(difficulty lifting head off pillow, arising from chair, climbing stairs). It is a systemic disease" symptoms may include the following:
morning stiffness - fatigue - anorexia - fever - weight loss
292
rd
Reference: step up to medicine – 3 editon, page 259
Http://emedicine.medscape.com/article/335925-clinical

43. Headache when combing hair, tender vessel on temporal area, sudden loss of vision or clouding in one eye. What next
action?
A. Oral prednisolone for 3 months
B. Immediate cortisone eye drops\
Answer: a
Treatment with steroid should be started as soon as the diagnosis of temporal arteritis (ta) is suspected even before confirmation. A
biopsy of temporal artery should be obtained immediately. However, if the biopsy did not show evidence of ta despite high clinical
suspicion, steroid therapy should be continued anyways.
Http://cursoenarm.net/uptodate/contents/mobipreview.htm?20/11/20656v

44. Polymyalgia rheumatica. Aid dx ?


A. Proximal muscle weaknesses
B. Proximal muscle tenderness
Answer: b

45. Old man with joint pain that worsens on movement. X-ray of wrist shows narrow joints in small joints. What's the dx?
A. Osteoarthritis oa
B. Rheumatoid arthritis ra
Answer: a
Radiographs help distinguish rheumatoid arthritis from osteoarthritis. OAis characterized by narrowing of the joint space due to
cartilage loss and osteophytes due to bone remodeling, but not erosions or cysts which are characteristics of ra.
Http://cursoenarm.net/uptodate/contents/mobipreview.htm?20/22/20833#h8594869

46. A case of t score of -3.5 what is the diagnosis:


A. Osteopenia
B. Osteoporosis

Answer: b
-1 and above: normal.
Between -1 and -2.5: osteopenia
-2.5 and below: osteoporosis.

47. What’s the management of septic arthritis?


A. Aspiration and antibiotic
B. Iv antibiotic
Answer: a
Reference: http://cursoenarm.net/uptodate/contents/mobipreview.htm?32/63/33776#h21

293
48. A patient who is medically free presented with swollen knee, no tenderness or redness. What is the best next step?
A. Mri
B. X-ray
Answer: b
Explanation: the use of plain radiographs is often necessary to assess a swollen knee. In an acute injury, selecting the appropriate
radiographic series is critical. The most common views are the anteroposterior (ap), lateral and axial patellar images to assess for
fracture, dislocation and effusion.
Reference: http://www.aafp.org/afp/2000/0415/p2391.html

49. In gout what substance would be high?


A. Prpp
B. Xanthine
Answer: a
There are at least three different inherited defects that lead to early development of severe hyperuricemia and gout: glucose-6-
phosphatase (gene symbol: g6pt) deficiency; severe and partial hypoxanthine-guanine phosphoribosyltransferase (hgprt, gene
symbol: hprt) deficiency; and elevated 5'-phosphoribosyl-1'-pyrophosphate synthetase (prpp synthetase) activity
Http://themedicalbiochemistrypage.org/gout.php
- xanthine oxidase inhibitor allopurinol is used in the treatment of gout.

50. Old lady with osteoarthritis and risk for osteoporosis , what you will give?
A. Calcium, TSH , dihydroxy Vitamin d.
B. Bisphosphonate, Vitamin d , calcium.

Answer: b
Guidelines from the american association of clinical endocrinologists (aace), published in 2010, include the following
[11]
recommendations for choosing drugs to treat osteoporosis :
First-line agents: alendronate, risedronate, zoledronic acid, denosumab
Second-line agent: ibandronate
Second- or third-line agent: raloxifene
Last-line agent: calcitonin
[http://emedicine.medscape.com/article/330598-treatment#d8]

51. Bilateral shoulder and hip stiffness and pain what is dx :


A- polymyalgia rheumatica
B- OA

Answer: a
Polymyalgia rheumatica (pmr) is characterized by aching and morning stiffness in the shoulders, hip girdle, neck, and torso in
patients over the age of 50
Reference: uptodate http://goo.gl/zzpjli

52. Pt with pic of osteoarthritis on examination there is nodule in the tips of the finger the name of the finding?
A. Heberden
B. Boutchared
294
Answer: a
note:
OApip nodules- boutchard (the p/b sound )
OAdip nodules: heberden.
Reference: uptodate

53. Patient with OAwhich type of exercise is the best :


A- high repitition and .........
B- low repition and .......
Answer: a? Not enough info

54. A patient presented with hip and shoulder pain. ESR: high (Polymyalgia Rheumatica case). In addition to these symptoms,
what else can be there in the history?
A. Proximal muscle weakness.
B. Proximal muscle Tenderness.

Answer: B
Muscle weakness is not a feature of PMR.
Reference: http://emedicine.medscape.com/article/330815-clinical

55. A patient with right knee osteoarthritis, presented with swelling of the right knee. On examination the right knee is swollen
with no change in temperature. What will you do?
A. Aspiration of the knee fluid
B. Bilateral Knee X-Ray and Ibuprofen.

Answer: B
Because there is no change in temperature.
It’s normal for osteophyte to cause swelling of the knee in OA

56. How would you manage aggressive ra:


A. Methotrexate
B. Cyclosporine
Answer:a
Methotrexate is the gold standard and is first-line unless contraindicated. The management of ra should starts with disease
modifying anti-rheumatic drugs (dmards) as soon as possible.including: hydroxychloroquine, sulfasalazine,methotrexate,
leflunomide. Note that cyclosporine is one of dmards but it is not commonly used. Surgical therapy is indicated for structural joint
damage.
Reference: toronto notes

57. Treatment of acute gout :


A. Indomethacin
B. Indomethacin is commonly used if patient has contraindication to it or other nsid colchcen could be use and at the end if
there is no response steroid is the final path

295
58. Pt with rheumatic fever on aspirin developed heart burn what you will add
A. Misoprostol
B. Drug from ( h2 blocker )

59. Pseudogout What type of calcium crystals?


A. Carbonate
B. Pyrophosphate
Answer : B
Https://yhdp.vn/uptodate/contents/mobipreview.htm?17/45/18134
Http://www.mayoclinic.org/diseases-conditions/pseudogout/basics/definition/con-20028152

60. Behcet's disease:


A. Vasculitis << ulcer ...test
Answer:a
Behçet disease is a rare vasculitic disorder that is characterized by a triple-symptom complex of recurrent oral aphthous ulcers,
genital ulcers, and uveitis.
Reference: http://emedicine.medscape.com/article/329099-overview#showall

61. Long scenario about ra and he give lab result he q was what is the type of anemia in ra patient ?
Answer: normocytic normochromic ( ddx: iron deficiecy anemia in 50 to 70 ٪ )
Uptodate:
Http://www.uptodate.com/contents/hematologic-manifestations-of-rheumatoid-
arthritis?Source=outline_link&view=text&anchor=h3#h3

62. Patient known case of behcet's disease came with arthritis resistant to steroid what to do:
A. Colchicine
Answer: a
- Lesions resistant to local measures may require systemic treatment with colchicine, or an oral corticosteroid or other
immunosuppresant agent
- Tnf-alpha inhibitors: helpful in mild behcet's syndrome resistant to corticosteroids and oral immunosuppressants
- Http://bestpractice.bmj.com/best-practice/monograph/376/treatment/details.html

63. A patient is a known case of behcet came with arthritis resistant to steroid, what to do:
A. Colchicine
Answer: a

64. Patient with behçet syndrome not responding to steroid wash what to give?
A. Azathiiprine
Answer: a

296
65. Man with features of temporal arteritis, biopsy showed giant cells. What is the most concern problem?
A. Loss of vision
Reference: http://emedicine.medscape.com/article/332483-overview

66. Patient with ankylosing spondylitis associated with ?


A. Uveitis
Answer: a
Acute anterior uveitis is the most common extra-articular feature of as, occurring in 25%-40% of patients at some time in the course
of their disease.
Link: http://www.medscape.org/viewarticle/544971_2

67. Headache unilateral. Sample taken from temporal artery showed.. This patient at risk of....
Answer: vision loss due to temporal arteritis.

68. Polymyalgia rhumatica question, what supports the dx


There is no pathognomonic test or established diagnostic criteria for polymyalgia rheumatica (pmr). We use the presence of all
of the following empirically formulated criteria for the clinical diagnosis of pmr, in whom another disease to explain the findings
is not present [20,45-47]:
●age 50 years or older at disease onset.
●proximally and bilaterally distributed aching and morning stiffness (lasting at least 30 minutes or more) persisting for at least
two weeks. The stiffness should involve at least two of the following three areas: neck or torso, shoulders or proximal regions of
the arms, and hips or proximal aspects of the thighs.
●erythrocyte sedimentation rate (esr) (westergren) ≥40 mm/h.
●rapid resolution of symptoms with low-dose glucocorticoids. Symptoms are generally 50 to 70 percent better within three days
in patients with pmr started on prednisone at a dose of 10 to 20 mg/day. The lack of response to initial therapy strongly
suggests an alternative diagnosis.

69. A patient has osteoporosis on ca and Vitamin d but no improvement why?


A. Hypomagnesemia
Answer: a. Very low mg prevents release of pth and causes unresponsive hypocalcemia but not related to osteoporosis

70. Pt comes with oral and genital ulcer ...dx?


A) bechet disease
Answer: a

71. Case of giant cell arteritis what is the treatment


Oral steroid. No visual or neurological symptoms or signs give prednisolone, if visual or neurological symptoms or signs iv
methylprednisolone, bmj http://bestpractice.bmj.com/best-practice/monograph/177/treatment/details.html

297
72. Case of early osteoporosis. How you'll manage?
Bisphosphonates inhibit bone resorption and are first-line treatment.
• They decrease osteoclastic activity (via binding to hydroxyapatite) and decrease the risk of fractures.
• Oral bisphosphonates (alendronate, risedronate) are preferred in most patients
• Side effects include reflux, esophageal irritation, and ulceration.
• • if patient cannot tolerate oral bisphosphonates, iv bisphosphonates (iv zoledronic acid)
Reference: step up to medicine

73. Pt does not complain of anything ,, has sudden knee swelling ? What is the best thing to do ?
Answer: xray

74. Patient does not complain of anything, has sudden knee swelling? What is the best thing to do?

Answer:
Treatment will depend on the cause of the swollen knee, but the most common ways to reduce the swelling are:
• Ice: to slow down the blood flow and therefore reduce swelling and pain.
• Compression: tubigrip and knee braces can be used to provide compression to the knee which helps reduce swelling.
• Medication: non-steroidal anti-inflammatories e.g. Ibuprofen may be prescribed to reduce the knee swelling and pain
• Knee aspiration
• Steroid injections: reducing inflammation and pain
• Elevation: higher than the level of the heart can help treat a swollen by gravity.
• Rest

75. Young boy with pain in his knee, aspiration of fluid reveal yellowish and turbid appearance, diagnosis?
A. Septic arthritis
Answer: infected joint fluid is typically yellow-green due to elevated levels of nucleated cells, and the cell count is usually markedly
elevated, demonstrating a predominance of polymorphonuclear leukocytes.
(http://emedicine.medscape.com/article/236299-workup#c8)

76. Treatment of rheumatic fever?


A. Penicillin and high dose of aspirin
Answer:
A regiment of antibiotic and anti-inflammatory. Penicillin and aspirin or prednisone is severe cases. Erythromycin or sulfadiazine
may be used in patients who are allergic to penicillin.
Http://emedicine.medscape.com/article/236582-medication#2

298
77. Ttt of polymyalgia rheumatica?
A. Answer: corticosteroid
Oral corticosteroids are the first line of treatment
Reference: http://emedicine.medscape.com/article/330815-medication

78. Patient symptoms of rheumatic fever with heart involvement and history of sore throat was not treated.. Treatment ?
A. Aspirin and steroid

79. Treatment for rheumatoid arthritis uses (something about macrophages and tnf and il) what can recur in patients using the
treatment mechanism?

80. Rheumatoid arthritis

81. Pt numbness in rt arm +feel electric shock dx ?


B. Cervical disc prolapse or spondylitis

82. Patient with symptoms of gout , which medication will help :


A. Inhibit the xanthien oxidase
Explanation: allopurinol (a xanthine oxidase inhibitor, decreases uric acid synthesis)—if the 24 hour urine uric acid is >800 mg/day,
this indicates overproduction. Never give this for acute gout; it makes it worse. Drugs used for acute gout include NSAIDs and
colchicine.
Reference: step up to medicine 3rd edition, page 256,257

83. Management of septic arthritis mortality risk?

84. Definition of leiomyoma?


Answer : is a benign smooth muscle neoplasm that very rarely becomes cancer.
Https://en.wikipedia.org/wiki/leiomyoma

85. Major jones criteria of rheumatic fever?


Major diagnostic criteria:
Carditis, polyarthritis, chorea, erythema marginatum, subcutaneous nodules
Http://emedicine.medscape.com/article/236582-clinical
Http://reference.medscape.com/calculator/jones-criteria-diagnosis-rheumatic

299
86. Patient with arthritis and rash on the face. ANA is positive. What should you do next?
A. Anti DNA

Answer: A
Reference: http://www.hopkinslupus.org/lupus-tests/lupus-blood-tests/

87. A patient with SLE with rash on her cheeks, etc. What will you advise her?
A. Avoid sun exposure as much as she can
Answer: A
Reference: Medical diagnosis and Management by Mohammad Inam Danish

88. Characteristic finding in Behçet disease?


Answer: ?
Leukocytoclastic vasculitis, multi-system disorder presenting with ocular involvement (uveitis), recurrent oral and genital ulceration,
venous thrombosis, skin and joint involvement, more common in Mediterranean and Asia, average age 30s, M>F
Reference: Toronto Notes

89. Raynaud phenomenon. Which antibody will be positive?


Anti-SCL-70
Answer: A
If it’s scleroderma case the answer will be A
ANA is ordered for Raynaud phenomenon, although it's not specific. The anti-centromere antibodies are ordered for CREST
syndrome. (R for Raynaud)
Reference: http://emedicine.medscape.com/article/1064663-workup

90. Henoch schonlein purpura:


IgA Vasculitis
Answer: A

91. What is the treatment of choice for Kawasaki?


Answer: ?
Intravenous Immunoglobulins (IVIG) is mainstay of treatment + Aspirin
Reference: canadaqbank

92. Patient 3 weeks after URTI develop rash, knee pain, and hematuria. What' the Dx?
A- Henoch-Schönlein purpura ( specific type of hypersensitivity vasculitis)
Answer: A
Henoch-Schönlein purpura “IgAvasculitis” characterized by a tetrad of clinical manifestations:
●Palpable purpura in patients with neither thrombocytopenia nor coagulopathy 95-100%
●Arthritis/arthralgia “especially involving the knees and ankles” 60-84%
●Abdominal pain “usually diffuse, with acute-onset” 35-85%

300
●Renal disease “proteinuria, hematuria”
In one half to two thirds of children, an upper respiratory tract infection (URTI) precedes the clinical onset of HSP by 1-3 weeks.
Http://www.uptodate.com/contents/henoch-schonlein-purpura-immunoglobulin-a-vasculitis-clinical-manifestations-and-diagnosis

93. Polymyalgia Rheumatica question, what supports the diagnosis?


Pain and stiffness in shoulders and hips, often with fever, malaise, weight loss. Does not cause muscular weakness. More common in
women > 50 years old; associated with temporal (giant cell) arteritis.
Answer: The symptoms include pain and stiffness of the shoulder and hip girdle. Stiffness after periods of rest (gel phenomenon) as
well as morning stiffness of more than 1 hour typically occurs. Low-grade fever and weight loss. Malaise, fatigue, and
depression.Muscle weakness is not a feature of PMR
FINDINGS :high• ESR, • high CRP.
(http://emedicine.medscape.com/article/330815-clinical)

94. Typical symptoms of RA, wts the dx.


A. RA
Answer: Rheumatoid arthritis is a chronic inflammatory disorder autoimmune disorder that typically affects the small joints in your
hands and feet on both sides of your body.
Symptoms: tender swelling joint, morning stiffness last for hours, rhomatoid nodules, fever .
Complication: osteoporosis ,carpal tunel syndrom, heart proplem,lung diseas.
Blood test: high CRP, anti-cyclic citrullinate d peptide (anti-CCP) antibodies.
Medication:NSAIDs. Side effects ringing in your ears, stomach irritation, heart problems, and liver and kidney damage.
Steroids. Side effects may include thinning of bones, weight gain and diabetes.
Disease-modifying antirheumatic drugs (dmards).. Include methotrexate (Trexall), leflunomide (Arava), hydroxychloroquine
(Plaquenil) and sulfasalazine (Azulfidine).
Side effects vary but may include liver damage, bone marrow suppression and severe lung infections.

95. Pt with PIP, DIP swollen and painful. Tests show high uric acid. Wts the treatment:
A-allopuranol
Answer: A

301
96. A patient with symmetrical joint involvement, complaining of morning stiffness that is relieved with movement. MCP and PIP
are involved as well. What is the diagnosis?
RA
Answer: A

97. Symmetric joint pain and swelling worse at morning. What is the Dx? 2 times
Answer: RA
Signs and symptoms of rheumatoid arthritis may include the following:
Persistent symmetric polyarthritis (synovitis) of hands and feet (hallmark feature)
Http://www.mayoclinic.org/diseases-/rheumatoid-arthritis/basics/symptoms/con-20014868

98. Gout metabolism of


A)purine
Answer: Overproduction accounts for only a minority of patients presenting with hyperuricemia. The causes for hyperuricemia in
overproducers may be either exogenous (diet rich in purines) or endogenous (increased purine nucleotide breakdown). A small
percentage of overproducers have enzymatic defects that account for their hyperuricemia. These include a complete deficiency of
hypoxanthine guanine phosphoribosyltransferase (HGPRT) as in Lesch-Nyhan syndrome, partial deficiency of HGPRT (Kelley-
Seegmiller syndrome), and increased production of 5-phospho-alpha-d-ribosyl pyrophosphate (PRPP) activity. Accelerated purine
degradation can result from rapid cell proliferation and turnover (blast crisis of leukemias) or from cell death (rhabdomyolysis,
cytotoxic therapy). Glycogenoses types III, IV, and VII can result in hyperuricemia from excessive degradation of skeletal muscle ATP.
Reference: http://emedicine.medscape.com/article/241767-overview#a5

99. Symmetric joint pain and swelling worse at morning. Dx?


Answer: rheumatoid artheritis
Reference: http://www.webmd.com/rheumatoid-arthritis/guide/diagnosing-ra

100.Acute management of gout ?

Answer: INITIAL TREATMENT CHOICES — The choice of medications depends upon the comorbidities that are present, the
effectiveness of past treatments, patient preferences for use, and the experience of the clinician with joint injection. Despite wide
use in the treatment of acute gout attacks, the various antiinflammatory agents have only infrequently been compared with placebo
or with each other in randomized trials. We take the following approach to the initial treatment of acute attacks of gout (algorithm
1):
●We treat most patients able to take an oral medication with a nonsteroidal antiinflammatory drug (NSAID). (See 'NSAID therapy'
below.)
●An oral low-dose colchicine regimen may be used in patients who are able to take an oral medication but who have
contraindications to NSAIDs (eg, moderate or more severe chronic kidney disease [CKD], active peptic ulcer disease, or a history of
NSAID-intolerance). (See 'Colchicine therapy' below.)
●In patients with contraindications to the use of both NSAIDs and colchicine, we prefer intraarticular, oral, or parenteral
glucocorticoids, depending upon the number of involved joints, the experience of the clinician with joint injection techniques, and
the need, if present, for parenteral rather than orally administered therapy. (See 'Glucocorticoids' below.)
Reference: http://www.uptodate.com/contents/treatment-of-acute-gout
302
101.Chronic Gout >allopuriol
Answer: Allopurinol inhibition of uric acid production is in large part due to inhibition of xanthine oxidase (xanthine dehydrogenase)
by both the native drug and the active metabolite oxypurinol. Allopurinol and oxypurinol are pyrazolo-pyrimidine analogs of the
purine bases hypoxanthine and xanthine, respectively
Ref :uptodate

102.Pyseodogout >pyrophysphate crystel


Answer: PSEUDOGOUT DIAGNOSIS — A healthcare provider can confirm or rule out a diagnosis of pseudogout by performing an
examination and tests. In many patients, a sample of joint fluid is obtained in order to determine whether calcium pyrophosphate
dihydrate (CPP) crystals are present and to exclude arthritis due to other causes, such as gout or joint infection.
Ref: uptodate

103.Dx of RA?
Answer: Anti-CCP Anti-citrullinated peptide/protein antibody
- more specific than rheumatoid factor, positive very early in the course of disease.
Http://www.uptodate.com/contents/rheumatoid-arthritis-symptoms-and-diagnosis-beyond-the-basics#H19 (Medscape)

104.HTN patient develops gout, Which drug side effects can cause that?
Answer: thiazide
Thiazides reduce the clearance of uric acid since they compete for the same transporter, and therefore raise the levels of uric acid in
the blood. Hence they are prescribed with caution in patients with gout or hyperuricemia

105.Pt does not complain of anything, has sudden knee swelling? What is the best thing to do?
Answer: Aspirate
''The most common causes of this type of fluid accumulation are due to infection or gout.''
Reference: https://www.verywell.com/knee-swelling-2549489

106.2 questions has the same idea with the same options about polymyalgia rheumatica (they mention that ESR was high)
Answer: PMR
Go and read: http://emedicine.medscape.com/article/330815-overview

107.Case of rheumatoid arthritis.

Answer: ??
• Rheumatoid arthritis is a chronic inflammatory condition. Its symptoms develop gradually and may include joint pain,
stiffness, and swelling. The condition can affect many tissues throughout the body, but the joints are usually most severely
affected. The cause of rheumatoid arthritis is unknown.
303
• A person with well-established rheumatoid arthritis typically has or has had at least several of the following:
o Morning stiffness that lasts at least one hour (> 30 minutes) and that has been present for at least six weeks
o Swelling of three or more joints for at least six weeks
o Swelling of the wrist, hand, or finger joints for at least six weeks
o Swelling of the same joints on both sides of the body
o Changes in hand x-rays that are characteristic of rheumatoid arthritis
o Rheumatoid nodules of the skin
o Blood test positive for rheumatoid factor and/or anti-citrullinated peptide/protein antibodies (ACPA)
• Not all of these features are present in people with early RA, and these problems may be present in some people with other
rheumatic conditions.
• In some cases, it may be necessary to monitor the condition over time before a diagnosis of rheumatoid arthritis can be
made with certainty.
• Laboratory tests — Laboratory tests help to confirm the presence of rheumatoid arthritis, to differentiate it from other
conditions, and to predict the likely course of the condition and its response to treatment.
o Rheumatoid factor — An antibody called rheumatoid factor is present in the blood of 70 to 80 percent of people
with rheumatoid arthritis. However, rheumatoid factor is also found in people with other types of rheumatic
disease and in a small number of healthy individuals.
o Anti-citrullinated peptide/protein antibody test — Blood tests for ACPA are more specific than rheumatoid factor
for diagnosing rheumatoid arthritis. Anti-ACPA antibody tests may be positive very early in the course of disease.
The test is positive in most patients with rheumatoid arthritis.
Reference: Uptodate

108.Case of juvenile rheumatoid/idiopathic arthritis treatment.

Answer: ??
• Oligoarticular juvenile idiopathic arthritis (JIA) is usually responsive to intra-articular glucocorticoids.
• Methotrexate and other immunosuppressive drugs are recommended for children with disease that extends to involve five
or more joints or require repeat injections.
• Biologic agents are typically reserved for patients with uveitis and are also used in some patients with extended
oligoarticular JIA.
MORE DETAILS ABOUT TREATMENT IN DIFFERENT SUBTYPES HERE: http://emedicine.medscape.com/article/1007276-
treatment#showall

109.a patient came to you complaining of morning stiffness of the pip and dip that decreases and goes away with activity. On
x-ray you observed a bone growth. What is the name of that growth ?
A. Heberden node
Answer: ? (depends on the x-ray given)
It is a case of arthritis.

304
Osteoarthritis hand
● Cartilage loss with narrowing of interphalangeal joints
● B: bouchard nodes (osteophytes proximal interphalangeal joints)
● H: heberden nodes (osteophytes distal interphalangeal joints)

110.boutonniere deformity:
A. Flexion of pip joint & hyperextension of dip
Answer: a
swan neck deformity :DIP flexion with PIP hyperextension

Endocrinology
305
DDP-IV INHIBITORS
INCREASE GLP-1
STIMULATE INSULIN SECRETION
INHIBIT GLUCAON RELEASE
LOWERING OF BLOOD GLUCOSE

SLOW GASTRIC EMPTYING

1. Incretin mechanism of action:


A. Increase insulin sensitivity
B. Increase insulin secretion
C. Increase gastric emptying
D. Decrease hepatic gluconeogenesis
Answer: b
A hormone that stimulates insulin secretion in response to meals. The two most important incretin hormones are called glucagon-
like peptide-1 (glp-1) and glucose-dependent insulin tropic polypeptide (gip). Understanding how these hormones work is helping to
yield new treatments for type 1 and type 2 diabetes.
http://www.diabetesselfmanagement.com/diabetes-resources/definitions/incretin-hormone/

2. Q pt k/c of DM with uncontrolled blood sugar with figure shows high at the 6am wt to do?
A. ↑ long acting night insulin
B. ↑ short acting night insulin
C. ↑ long acting morning in...
D. ↑ short acting morning in…

Answer: a
Reference: by senior medical resident

306
3. Hypothyroidism patient on dose of thyroxin 75 mg. Missed the dose 2 days because he does not have the drug, lab result
show high TSH and normal t4. What dose should be taken?
A. 25
B. 50
C. 75
D. 100

Answer: c

4. Patient k/c of DM with uncontrolled blood sugar, with figure shows high at the 6am what to do?
A. ↑ long acting night insulin
B. ↑ short acting night insulin
C. ↑ long acting morning insulin
D. ↑ short acting morning insulin

Answer: a
We should increase the long acting night insulin as the patient is sleeping for hours and it wouldn’t be ideal to wake him/her up daily
in the middle of the night to take medications.
Cortisol among other counter-regulatory hormones such as epiniphrine, glucagon, or growth hormone are anti-insulin in the sense
that they elevate blood glucose, cortisol will work mainly during the early hours of the morning peaking especially around 6 am.
Adjusting the long acting insulin night dose should be able to adjust this physiologic hyperglycemia

5. Patient with dka he start to breath rapidly to buffer his acidosis "to get rid of" :
A. Oxygen
B. Carbon monoxide
C. Carbon dioxide
D. Nitrogen

Answer: c

6. Patient dx with dm1 which will confirm the dx:


A. Dr3
B. Dr4 ??
C. Dr7
D. Dr5
According to best practice and medscape, both dr3/dr4 are considered as risk factors (approximately 95% of patients with type 1 DM
have either hla-dr3 or hla-dr4) and are not used in diagnosing type 1 dm, but instead, insulin and c-peptide levels and
immune markers (eg, glutamic acid decarboxylase [gad] autoantibodies) are used.
References:
Http://emedicine.medscape.com/article/117739-overview#showall
Http://bestpractice.bmj.com/best-practice/monograph/25/diagnosis/tests.html
Medicine

7. Early sign of puberty in males?


A. Hair in the face

307
B. Hair in the gentiles
C. Hoarseness of voice
D. Enlarged penis
Answer is b
The first manifestation of male puberty is gonadarche or testicular enlargement. It’s not mentioned in the choices so the next event
in male puberty would be growth of pubic hair.
Source: crash course endocrinology

8. Difference between type 1 and type 2 dm?


A. CPR
B. Level of insulin
C. Insulin growth factor
D. Islet of b cell
Answer: b
DM 1: beta cells in pancreas are attacked by the body's own immune system, therefore reducing insulin production, leading to
elevated blood glucose. Insulin is not produced or is produced in insufficient amounts
DM 2: persistently high intakes of dietary sugars lead to excess demands on insulin production, which leads to insulin resistance over
time. Receptor cells that have become less sensitive (resistant) to insulin are unable to remove glucose from the blood, leading to
higher blood glucose and greater demands on insulin production.

9. Pt for annual checkup everything is normal. Normal blood glucose. What should be the next checkup?
A. 3 month
B. 6 month
C. 12 month
D. 36 month
Answer: d
The american diabetes association (ada) recommends testing at three-year intervals for diabetes or prediabetes in all adults with
bmi ≥25 kg/m2 and one or more additional risk factors for diabetes using either a1c, fasting plasma glucose, or 2-hour ogtt [2]. In
individuals without risk factors, the ada recommends that testing begin at age 45 years.

10. A 30-year-old male, diabetic otherwise healthy with persistent one hour post prandial hyperglycemia after lunch and dinner:
A. Milk
B. Meat
C. Diet cola
D. Potato
Answer: d
the glycemic index of a food refers to the speed with which the food raises blood glucose level.
Http://static.diabetesselfmanagement.com/pdfs/pdf_2102.pdf

11. Diabetic and hypertensive, was given a drug then experienced cough, improved after changing the drug to
hydrochlorothiazide what to add:
A. Atenolol
B. Amlodipine
C. Losartan
D. ACEI (i forgot the name)
308
Answer: c

12. Myasthenia gravis on neostigmine and case worsen?


A. Add more neostigmine
B. Add revistagmine
C. Add azthropitine
D. Plasmapheresis or hemodialysis
Answer: a
Mg
https://books.google.com.sa/books?Id=bj-
dbgaaqbaj&pg=pa255&lpg=pa255&dq=myasthenia+gravis+on+neostigmine+and+case+worsed&source=bl&ots=0xj9v0x7zh&sig=fbt
b0zxw89qgf_a7fcim8u_yq0a&hl=en&sa=x&redir_esc=y#v=onepage&q=myasthenia%20gravis%20on%20neostigmine%20and%20cas
e%20worsed&f=false

13. What is maximum normal 2 h postprandial blood glucose?


A. 8
B. 9
C. 10
D. 11
Answer: a

14. Cushing case which skin manifestation is associated with it


A. Vitiligo
B. Telangiectasia
C. Acropathy
D. Something derma

Answer: b
Http://emedicine.medscape.com/article/2233083-clinical#b3

15. Elderly patient presented with chest pain palpitation and SVT with normal vital signs including o2 sat. Air entry was bilateral
which of the following is most helpful in establishing the diagnosis:
A. Pulmonary arteriography
B. Chest CT
C. TSH
D. Brain mri

Answer: c

16. Patient with hard palpable nodule in left thyroid. Fna taken and show amorphous intercellular substance & by special stain
there is amyloid deposition.
A. NHL.
B. Follicular cancer.
C. Papillary cancer
309
D. Medullary carcinoma
Answer: d

17. Case of hyperthyroidism clear but with bradycardia


A. Hypothyroid
B. Hyperthyroidism
C. Goiter
D. Neoplasm
Answer: b

18. Patient with dka he starts to breath rapidly to buffer his acidosis through:
A. Oxygen
B. Carbon monoxide
C. Carbon dioxide
D. Nitrogen

Answer: c

19. Hypothyroidism pt on dose of thyroxine 75 .. Missed the dose 2 days because he does not have the drug, lab result show high
TSH and normal t4 .. What dose should be taken?
A. 25
B. 50
C. 75
D. 100
Answer: c
Normal t4 is the desired effect.

20. Patient with dka he start to breath rapidly to buffer his acidosis through
A. Oxygen
B. Carbon monoxide
C. Carbon dioxide
D. Nitrogen
Answer c:
I found only wikipedia references

21. What is the preferable imaging to diagnose Pituitary Microadenoma?


A. Contrast with enhanced brain CT
B. Brain CT
C. Contrast with enhanced brain MRI
D. Unenhanced brain MRI

Answer: C
Reference: uptodate , Usmle 2 first aid

310
22. Yrs old girl her parent have dyslipedimia and she denies having dyslipidemia. Lpid profile
showing high triglyc, cholestrol, high LDL, low HDL what is the best to check next ?
A. GH
B. TSH
C. FSH
D. ACTH

Answer: c
TSH to rule out hypothyroidism
Reference: http://www.uptodate.com/contents/lipid-abnormalities-in-thyroid-disease

23. Patient k/c of DM with uncontrolled blood sugar with figure shows high at the 6 am what to
do?
A. ↑ long acting night insulin
B. ↑ short acting night insulin
C. ↑ long acting morning in…
D. ↑ short acting morning in…

Answer: A

24. Case of hyperthyroidism clear but with bradycardia


A. Hypothyroidism
B. Hyperthyroidism
C. Goiter
D. Neoplasm

25. Se female irritable + loss of wt+ bradycardia?


A. Hypothyroid
B. Hyperthyroid
C. Nodule
D. Thyroid cancer ( i chose this because hyperthyroidism comes with afib) < their original answer
Reference: toronto notes

26. Diabetic patient who is allergic to sulfa drugs, on metformin but it’s not controlled. What will you add to control his diabetes?
A. Chlorpropamide
B. Glyburide
C. Rosiglitazone

Answer: c or gliptin?
Gliptin (dpp-4 inhibitor) is a better choice (answered by an endocrine consultant).
Chlorpropamide (sulfonylurea)
Glyburide (sulfonylurea)
Rosiglitazone (thiazolidinedione)

311
27. One small thyroid nodule, we do investigation and its iodine uptake increases, what is the best treatment?
A. Conservative
B. Antithyroid drug
C. Iodine radiotherapy

Answer: a if it is (hot + small and the patient is asymptomatic).


If the hot nodule is large in size or causing clinical hyperthyroidism, then treatment is started. Treatment may be by surgery
(especially when there is compressive symptoms) or radioactive ablation
Reference: http://www.mythyroid.com/hotnodule.html

28. Cushing case, which skin manifestation is associated with it?


A. Vitiligo
B. Telenagectasia
C. Acropathy

Answer: b telenagectasia
Skin manifestations of cushing:
Easy bruising , purple striae
Telangiectatic cheeks
Fragile skin and poor wound healing
Acne and hirsutism
Reference: http://www.dermnetnz.org/systemic/cushings.html

29. 70 years old smoker woman with low vit-d and osteoporosis. Which of the following has the highest risk for osteoporosis?
A. Smoking
B. Age
C. Vitamin d

Answer: b

30. Patient k/c of DM on glipizide want to go for elective surgery. You want to control his blood sugar during the surgery, what to
add?
A. Insulin
B. Metformin
C. Sulfonylurea

Answer: a
Patients with type 2 diabetes that take oral hypoglycemic drugs or non-insulin injectable are advised to hold their oral hypoglycemic
and noninsulin injectable drugs on the morning of surgery. For patients who develop hyperglycemia, supplemental short or rapid-
acting insulin may be administered subcutaneously (typically every six hours), based on frequently (every one to two hours)
measured glucose levels, which are often obtained on capillary "fingerstick" samples.
Reference: http://www.uptodate.com/contents/perioperative-management-of-blood-glucose-in-adults-with-diabetes-
mellitus?Source=preview&search=%2fcontents%2fsearch&anchor=h20#h20

312
31. Single small thyroid nodule, investigations revealed an increase in iodine uptake, what is the best treatment?
A. Conservative before 45: hemityrodictomy
B. Antithyroid drug after 45 : radiotherapy
C. Iodine radiotherapy

Answer: c
Those in whom the nodule causes hyperthyroidism should be treated with radioiodine or surgery, possibly after a period of
antithyroid drug therapy
Reference: http://cursoenarm.net/uptodate/contents/mobipreview.htm?21/2/21537#h25

32. Patient with 3 reading of high Blood pressure . All investigations are normal except high na. What’s the diagnosis?
A. Primary hyperaldosteronism
B. Essential hypertension
C. Secondary hypertension

Answer: a
Mild serum hypernatremia in the 143-147 meq/l range and mild hypomagnesemia from renal magnesium wasting are other
associated biochemical findings in established primary aldosteronism.
Reference: http://emedicine.medscape.com/article/127080-workup#c8

33. DM pt . Which of the following increase risk of retinopathy :


A. HTN + obesity
B. HTN + smoking
C. Dylipidemia + obesity
Answer: a
HTN & obesity are high risk factors; dyslipidemia is low risk while smoking is not a risk factor.
Http://bestpractice.bmj.com/best-practice/monograph/532/diagnosis/history-and-examination.html
Http://www.ncbi.nlm.nih.gov/pubmed/2060413
Http://www.ncbi.nlm.nih.gov/pubmed/21482643

34. Which of the following drug may presenting with hypoglycemic attack?
A. Metformin
B. Glitazone
C. Glyburide

Answer: c
Http://www.webmd.com/diabetes/guide/diabetes-hypoglycemia#2

35. Patient on metformin, lab values given with low ph, normal urine osmolality, no ketones ;
A. DKA
B. Hyperismolar hyperglycemia
C. Lactic acidosis
Answer: c

313
Metformin side effects: lactic acidosis, it is rare but dengerous. Http://reference.medscape.com/drug/glucophage-metformin-
342717#4
Long term treatment to avoid dependence

36. Best treatment for hot thyroid nodule ?


A. Excision
B. Antithyroid medication
C. Radio-iodine
Answer: ?? I think a
Http://www.uptodate.com/contents/thyroid-nodules-beyond-the-basics?Source=outline_link&view=text&anchor=h1#h1

37. Hyperthyroidism patient present with thyroid nodule high radioiodine tx??
A. Excision
B. Radioiodine ablation.
C. Anti-thyroid medication
Answer: for patients with toxic adenoma or mng, we suggest treatment with either surgery or radioiodine rather than prolonged
thionamide therapy (grade 2b). The individual patient's fears regarding radiation exposure, general anesthesia, or surgical
complications often influence the decision, and some patients refuse both radioiodine and surgery. For them, prolonged thionamide
therapy is acceptable as long as it is tolerated and the hyperthyroidism is controlled. Percutaneous ethanol injection or laser
therapy, where available, are alternatives to prolonged thionamide therapy.

38. Controlled DM what make an abnormal post prandial glucose


A. Diet cola
B. Meat
C. Potato
Answer: c

39. What is the optimal treatment of osteoporosis?


A. Calcium, vitamin d and bisphosphonate
B. Vitamin d and calcium
C. Calcium and bisphosphonate
Answer: a
The goals of treatment are to prevent fractures, decrease pain, and maintain function which is achieved by this combination that
mainly helps preserving and building bone mass.
Http://www.merckmanuals.com/professional/musculoskeletal-and-connective-tissue-
disorders/osteoporosis/osteoporosis#v906887

40. Patient with 3 reading of high Blood pressure did not start on meds yet
Showing labs all normal excepatient low k what is the most likely cause
A. Essential hypertention
B. Hyperaldestronism
314
C. Phenochromocytoma
Answer: b
Hyperaldosteronism is characterized by excessive secretion of aldosterone, which causes increases in sodium reabsorption and loss
of potassium and hydrogen ions.

41. Patient heard about the new anti-DM medication “incretin” on tv, asking about the mechanism of action of the new drug?
A. Inhibit liver gluconeogenesis
B. Increases the insulin secretion
C. Decreases insulin resistance
Answer: b
The incretins are peptide hormones secreted from the gut in response to food. They increase the secretion of insulin. The incretin
response is reduced in patients with type 2 diabetes, so drugs acting on incretins may improve glycaemic control.

42. 46 Year-old came to the clinic with her 1w old baby, complaining of (symptoms of hyperthyroidism recently developed):
A. Hypothyroidism
B. Postpartum thyroiditis
C. Thyroid tumor (i can't remember the other choices)
Answer: b
Postpartum thyroiditis (ppt) reportedly affects 4-10% of women. Ppatient is an autoimmune thyroid disease that occurs during the
first year after delivery. Women with ppatient present with transient thyrotoxicosis, hypothyroidism, or transient thyrotoxicosis
followed by hypothyroidism.
Link: http://emedicine.medscape.com/article/261913-overview

43. 10 or 12 yo girl diabetic for 3 years. Which of the following should be checked annually?
A. Ophthalmoscopy
B. Celiac test
C. Kidney ultrasound
Answer: a

44. Patient has DM & HTN (i think on captopril which induce dry cough) present with mild ankle edema and positive protein in
urine. What will you give instead of ace inhibitors? (not sure about this question)
A. ARB
B. Thiazide
C. CCB
Answer: a

45. 30yo with high cholesterol what’s the most common gene defect.
A. Apo cii
B. Apo b100
C. Ldl receptor (rt)
Answer: c
Familial hypercholesterolemia (fh) is a genetically modulated clinical syndrome in which the phenotype is characterized by a high
low density lipoprotein cholesterol (ldl-c) level from birth, a propensity to tendon xanthomata, and early onset coronary heart
315
disease (chd). In its most common form, fh is a monogenic, autosomal dominant disorder caused by defects in the gene that encode
for the apo b/e (ldl) receptor.

46. HTN came for follow up a1c 6.9 when to follow up again
A. 1 months
B. 3 months
C. 12 months
Answer: b
An a1c level ≥6.5 percent be used to diagnose diabetes. - uptodate

47. Diabetes mellitus type 1 patients express what mutation:


A. HLA-DR4
B. HLA-DR 6
C. HLA-DR 7
Answer: a
Reference: http://www.merckmanuals.com/professional/endocrine-and-metabolic-disorders/diabetes-mellitus-and-disorders-of-
carbohydrate-metabolism/diabetes-mellitus-dm

48. One small thyroid nodule, we do investigation and its increase in iodine uptake, what is the best treatment?
A. Conservative
B. Antithyroid drug
C. Iodine radiotherapy

Answer: c (functioning hot nodules are treated with iodine, another option is surgery after stabilizing with drugs)
Reference: http://emedicine.medscape.com/article/120497-treatment

49. Diabetic with painful back swelling has multiple discharges:


A. Cellulitis
316
B. Lymphoid
C. Something lymphadenitis carbuncle
Answer: a
Patients with uncontrolled diabetes are predisposed to multiple bacterial infections like abscesses, carbuncles, chronic balanitis, and
diabetic foot

50. Patient with swelling in the thyroid, she is euthyroid what will you do ?
A. Thyroid lobectomy
B. Fna biopsy
C. Excisional biopsy
Answer: b

51. Patient with hba1c within pre diabetic range .when to repeat the test ? –
A. 3months
B. 6 months
C. 1 year
Answer: c.
Reference: american diabetes association guidelines: at least annual monitoring for the development of diabetes in those with
prediabetes is suggested. E

52. Hypertension, high k in urine, high serum na, low serum k.


A. Cushing
B. Conn's syndrome
C. Hypothyroidism

Answer: b

Primary aldosteronism (conn's syndrome) is aldosteronism caused by autonomous production of aldosterone by the adrenal cortex
(due to hyperplasia, adenoma, or carcinoma). Symptoms and signs include hypernatremia and hypervolemia episodic weakness,
elevated Blood pressure , and hypokalemia.
Reference: merck manual: http://www.merckmanuals.com/professional/endocrine-and-metabolic-disorders/adrenal-
disorders/primary-aldosteronism

53. In dka which ketone is predominantly found in urine?


A. Acetone
B. Acetoacetate
C. Beta-hydroxybutyrate

Answer: b, dipstick detect acetoacetate but not beta-hydroxybutyrate


References
Http://fitsweb.uchc.edu/student/selectives/timurgraham/ketoacidosis_dka.html
Http://patient.info/doctor/urine-dipstick-analysis

317
54. Female presents with weight loss …. What clinical finding you will see?
A. Buffalo hump
B. Skin hyperpigmentations
C. Cutaneous stria
Answer : b (addison disease)

55. Why DM decrease wound healing


A. Decrease immunity
B. Increase bacteria due to increase glucose
C. Decrease phagocytosis
Answer: c i think the best answer is a.

Uptodate: over 100 known cytologic factors contribute to impaired wound healing in patients with diabetes.these include decreased
or impaired growth factor production, angiogenic response, macrophage function, collagen accumulation, epidermal barrier
function, quantity of granulation tissue, keratinocyte and fibroblast migration and proliferation, number of epidermal nerves, bone
healing, and abnormal balance between the accumulation of extracellular matrix components and their remodeling by matrix
metalloproteinases.

56. Case a guy has high phosphate question organ phosphate elevated which organ?
A. Liver
B. Kidney
vascular system, bones, skin, joint, heart
C. Lung
Answer:

57. What causes polyuria in DM?


A. Increase glucose in urine
B. Increase glucose in serum
C. Increase ketones in serum

Answer: A

58. 52 years old woman recently diagnosed to have DM, came with high ketones and
hyperglycemia. She was treated for it but she developed DKA again. The doctor is confused
whether she has type 1 or 2 DM. What test should be ordered?
A. Insulin
B. Hba1c
C. C-peptide

318
Answer: C
C-peptide is commonly used in preference to insulin measurement when assessing β-cell function in clinical practice. In patients on
insulin, C-peptide measurement must be used as exogenous insulin will be detected by insulin assays.
Reference: http://www.ncbi.nlm.nih.gov/pmc/articles/PMC3748788/

59. What is the best anti-diabetic regime that resembles the normal physiology?
A. Lispro & glargine
B. NPH & glargine
C. Lispro & NPH

Answer: A
Glargine is better than NPH

Single injection of insulin glargine leads to a smooth 24-hour time–action profile with no undesirable pronounced peaks of activity.
In clinical trials, this profile has been associated with at least equivalent, if not better, glycemic control than other traditional basal
insulins and a significantly lower rate of overall and nocturnal hypoglycemia.
Reference: http://www.ncbi.nlm.nih.gov/pmc/articles/PMC1993975/

60. A patient with signs and symptoms of Cushing syndrome. What is the best next investigation?
A. Brain MRI
B. Adrenal scan
C. Adrenal MRI

Answer: A
In adults, 80% of CS is due to ACTH-dependent causes and 20% due to adrenal causes. Since the majority of patients with ACTH-
secreting tumors have a pituitary lesion (often very small), a MRI of the pituitary gland with gadolinium enhancement is always the
initial approach.
References:
Www.ncbi.nlm.nih.gov/pubmed/18209870
Https://csrf.net/understanding-cushings/diagnostic-testing/

61. Patient with DM and HTN controlled on ace , frusemide, spironolactone ,, elctrolyte balance is normal what is the action
should do:
A. Stop frusemide
B. Stop spironolactone
C. Add digoxin.

62. 25 Year-oldwoman came to the clinic with her 6 weeks old baby, complaining of irritability, weight loss, and inability to
sleep?
A. Post partum thyroditis *
B. Hyperthyroidism
C. Hashimoto thyroditis
Reference: toronto notes

319
63. Response glycolysis?
A. Hyxokinase
B. Phsophphrktokinase
C. Pyrovate kienzyme nase

64. Acetone + high glucose?


A. Dka
B. Hyperosmolar

Answer: a

320
Diabetic ketoacidosis is typically characterized by hyperglycemia over 250 mg/dl, a bicarbonate level less than 18 meq/l, and a ph
less than 7.30, with ketonemia and ketonuria.
Reference: http://emedicine.medscape.com/article/118361-workup

65. 70 years old female patient with osteoporosis what is the treatment?
A. Estrogen
B. Biophosphate

Answer: b
Diagnosed by dexa. Bisphosphonates are first-line drug therapy.
References: 1st aid page (320) +
Http://www.msdmanuals.com/professional/musculoskeletal-and-connective-tissue-disorders/osteoporosis/osteoporosis

66. 70 years old female patient with osteoporosis what is the treatment?
A. Estrogen
B. Biophosphate

Answer: b
Bisphosphonates (e.g. Alendronate) šis first line
-hrt: second-line treatment (unless for vasomotor instability as well)
reference: toronto notes 2015 p 516

67. Female patient with high prolactin, what to exclude?


A. Thyroid disease
B. Pituitary tumors

Answer: b
Causes of hyperprolactinemia:
- Prolactinoma
o Most common cause of hyperprolactinemia
o Most common type of pituitary adenoma (up to 40%)
- Medications (e.g., psychiatric medications, h2 blockers, metoclopramide, verapamil, Estrogen)
- Pregnancy
- Renal failure
- Hypothyroidism
Reference: step up to medicine

68. 50 years old patient with third reading of persistent hypertension wasn't started on medication yet, lab shows high na of 147
and low k of 3 other parameters were normal, what's most likely the diagnosis?
A. Essential hypertension
B. Hyperaldorensism

Answer: b
321
69. Patient is concerned about osteoporosis as her mother had it, what you will do?
A. Give Vitamin d, calcium
B. Give estrogen postmenopausal

Answer: a

70. 70 years old female patient with osteoporosis what is the treatment?
A. Estrogen
B. Biophosphate

Answer: b
For the treatment of osteoporosis in postmenopausal women, we suggest oral bisphosphonates as first-line therapy. We prefer oral
bisphosphonates as initial therapy because of their efficacy
Reference: uptodate

71. Female patient with high prolactin, what to exclude?


A. Thyroid disease
B. Pituitary tumors

Answer: b
Prolactinoma
(most common cause of hyperprolactinemia
and most common type of pituitary adenoma up to 40%) 

Reference: step-up of

72. Patient with HTN 3 readings n; 146, k:3:


322
A. Primary hyperaldosteronism.
B. Primary htn
Answer a
Http://emedicine.medscape.com/article/127080-workup#showall

73. Pateint with DM taking 2 oral hypoglycemic agents heard about incretin, he wants to know more about it. What's the
mechanism of action?!
A. Increase insulin sensitivity.
B. Increase insulin secretion.
Answer: b
First aid

74. DM patient with dka what to give?


A. 1l of normal saline.
B. Dextrose 50% followed by insulin.
Answer: a
First aid:
Fluids, potassium, continuous insulin (caution: will lower potassium), phosphorus. Bicarbonate rarely used (only if ph is
<6.9)

75. Diabetic man with uncontrolled blood sugar he is on metformin and pramlintide, what drug you will add or change ( i don't
remember the exact question )
A. Repaglinide
B. Glitazone
Answer:

76. Thyroid move with swallowing because:


A. Pretracheal fscia
B. Carotid sheath
Answer:a
Condensation of the pretracheal fascia called the posterior suspensory ligament of berry is
Responsible for the upward movement of the thyroid gland on swallowing (sba mcqs for the mrcs part a)

77. Patient presented with acne , upper body obesity and hypertension ,what is the test to diagnose this patient ?
A. Anabolic stress test
B. Urine metanephrines
Answer: b
Not complete question: might be a case of phaeochromocytoma with cushing's syndrome and first test to do is 24hrs urine
metanephrines

78. Dka management?


A. Fluids.
323
B. Insulin infusion.
Answer: a
Explanation: managing diabetic ketoacidosis (dka) in an intensive care unit during the first 24-48 hours always is advisable. When
treating patients with dka, the following points must be considered and closely monitored:
Correction of fluid loss with intravenous fluids
Correction of hyperglycemia with insulin
Correction of electrolyte disturbances, particularly potassium loss
Correction of acid-base balance
Treatment of concurrent infection, if present
Reference: http://emedicine.medscape.com/article/118361-treatment

79. What’s the difference between type 1 and type 2 dm?


A. Endogenous insulin
B. Weight
Answer: a

80. A young female patient complains of heat intolerance and excessive sweating. On examination, neck swelling is felt. TSH
normal and fna done but not conclusive. What is your next step :
A. Repeat fna
B. Thyroidectomy
Answer: a or b
An inconclusive diagnosis is one for which there is no certainty about the nature of your nodule; it could be either benign or
malignant. This means that it is not possible to determine the nature of your lump. Either the fna has to be repeated or the
possibility of surgery should be discussed with your doctor.

81. 27 old lady with osteoporosis best prevention is:


A. Wight bearing exercise

324
B. Daily vitamin d supplements
Answer: b
Prevention measure for osteoporosis include:
- Diet: elemental calcium and vitamin d
- Exercise: 30 minutes of weight bearing exercise 3 times per week.
- Smoking cessation & reducing caffeine
Http://cursoenarm.net/uptodate/contents/mobipreview.htm?38/60/39873#h3

82. Female dm, father has dyslipidemia she came to check lipid shows hyperlipidemia what you will do?
A. LFT
B. TSH
Answer: b

83. Female DM on metformin and sulfonylurea can’t tolerate them what you will do?
A. Insulin
B. Glitazone
Answer: a

84. A history of missed insulin was approved from the pt, what do you suspect to find in urine?
A. Ketone
B. Protein
Answer: a
Http://emedicine.medscape.com/article/118361-workup#c10

85. Patient presented with severe vomiting. His labs showed hypocalcaemia. What is your management?
A. Furosemide
B. Hydration
Answer:
If hypo! Iv ca gluconate
If hyper iv fluid

86. Hx and investigation all normal exce hyponatremia and crepitation on ex. What would you give?
A. Iv ns
B. Furosemide
Appears to be siadh due to pulmonary disease. So the most appropriate answer is probably treat the pneumonia and fluid
restriction. Antidiuretic hormone (adh, arginine vasopressin) secretion results in a concentrated urine and therefore a reduced urine
volume. The higher the plasma adh, the more concentrated the urine. In most patients with the syndrome of inappropriate secretion
of antidiuretic hormone (siadh), ingestion of water does not adequately suppress adh, and the urine remains concentrated. This
leads to water retention, which increases tbw. This increase in tbw lowers the plasma sodium concentration by dilution (see above
equation) [1]. In addition, the increase in tbw transiently expands the extracellular fluid volume and thereby triggers increased
urinary sodium excretion, which both returns the extracellular fluid volume toward normal and further lowers the plasma sodium
concentration.
Treat the underlying disease — a variety of causes of siadh can be effectively treated, leading to resolution of the hyponatremia.
These include:

325
- Hormone replacement in adrenal insufficiency (which can lead to overly rapid correction of the hyponatremia) or
hypothyroidism
- Treatment of infections such as meningitis, pneumonia, or tuberculosis
- Cessation of offending drugs
Fluid restriction — fluid restriction is a mainstay of therapy in most patients with siadh, with a suggested goal intake of less than 800
ml/day [4]. The associated negative water balance initially raises the serum sodium concentration toward normal and, with
maintenance therapy in chronic siadh, prevents a further reduction in serum sodium. It can also lead to volume depletion due to
unmasking of the sodium deficit described above unless sodium intake is also adequate.

87. Consultation .patient with hypercholsterima. What best food to avoid?


A. Organ meats
B. Egg white (protein)
Answer: a
Eating too much saturated fat increases cholesterol levels. This is why it is best to limit the amount of foods we eat that are high in
saturated fats such as:
- Butter
- Ghee
- Hard margarines
- Lard, dripping and goose fat
- Fatty meat and meat products such as sausages
- Full fat cheese, milk, cream and yogurt
- Coconut and palm oils and coconut cream
Additionally, many foods such as milk chocolate, toffee, cakes, puddings, pastries, pies and rich biscuits, which are made from the
list above can also increase cholesterol.

88. A patient has increased radioactive iodine uptake on scan. What is the treatment?
A. Anti-thyroid hormone
B. Radioactive iodine
Answer: b
Explanation: the patient most likely has graves disease treatment of choice for which is radioactive iodine
Reference master the board step 2ck

89. Male is a k/c hyperthyroidism on medication, has infection. His labs: low neutrophil.
What drug has these se?
A. Methimazole
B. Iodate
Answer: a
Http://www.uptodate.com/contents/antithyroid-drugs-beyond-the-basics?Source=outline_link&view=text

90. Diabetic patient went for surgery; he was given insulin and dextrose. Then developed neurological symptoms (low na). What
is the mechanism?
A. Water overload.
B. Siadh.

326
Answer: incomplete q the two pictures explain the possible answers

Reference: pics and toronto note

91. Female patient with high prolactin what to exclude


A. Thyroid disease
B. Pituitary tumors

Answer:b
most common cause of female hyperprolactinemia is pitutary tumor (microprolactinoma )
reference : up to date
http://www.uptodate.com/contents/causes-of-hyperprolactinemia?Source=outline_link&view=text&anchor=h1#h1

92. DM on metformin and gliclazide .. She is not well controlled .. Which drug u will add ?
A. Acarbose
B. Pioglitazone
Answer: b

93. A patient presented with high parathyroid hormone and high calcium what is the diagnosis?
A. Primary parathyroid
B. Secondary parathyroid

Answer: A
High ca + high PTH = primary hyperparathyroid
Reference: medlineplus medical encyclopedia

94. Patient with type 1 DM, what's the diagnostic antibody?


A. Glutamic Acid Decarboxylase

327
B. Others antibodies not related to DM1

Answer: A
Reference: Islet-cell (IA2), anti-GAD65, and anti-insulin autoantibodies can be present in early type 1 but not type 2 DM.
Measurements of IA2 autoantibodies within 6 months of diagnosis can help differentiate between type 1 and type 2 DM. These
titers decrease after 6 months. Anti-GAD65 antibodies can be present at diagnosis of type 1 DM and are persistently positive over
time
Http://emedicine.medscape.com/article/117739-workup#c8

95. Patient with dyslipedemia , which hormone you will screen?


A. TSH
B. Testosterone

Answer: a
Patient with hypercholestrolemia should be screened for hypothyroidism before being given any specific lipid-lowering drug
therapy.
Reference http://www.uptodate.com/contents/lipid-abnormalities-in-thyroid-
disease?Source=outline_link&view=text&anchor=H6582291#H6582291

96. Which dietary supply prevent cancer


A. Fibers
B. Vitamin d
Vitamin d may reduce the risk of colorectal cancer. Http://cursoenarm.net/uptodate/contents/mobipreview.htm?40/8/41089#h39

97. Bond between t-RNAand mrna.


A. Answer: hydrogen
Http://depts.washington.edu/hhmibio/translationstudyguide.pdf

98. Symptoms of hyperthyroid wh present ?


A. Retraction led
B. Prietabial myxedema

99. X-ray of lower back , inv shows increase calicium , what to do :


A. Dexa
B. Parathyroid
Answer: b
Explanation: clinical syndrome of primary hyperparathyroidism can be easily remembered as bones, renal stones, abdominal groans,
and psychic moans. Hypercalcemia should be documented on more than one occasion before a diagnostic workup is undertaken.
Testing of the intact parathyroid hormone level is the core of the diagnosis. Reference:
http://emedicine.medscape.com/article/127351-overview#a1
328
100.Sore throat and pancytopenia in women taking antithyroid , which one ?
A. Methimazole
B. Propranolol
Answer: Methimazole .

101.Patient has osteoporosis on ca supplement and vitamin d, but has no improvement why?
A. Hypomagnesemia
Answer: a
- Magnesium deficiency contributes to osteoporosis directly by acting on crystal formation and on bone cells and indirectly by
impacting on the secretion and the activity of parathyroid hormone and by promoting low grade inflammation
- Http://www.mdpi.com/2072-6643/5/8/3022/htm

102.Pharmacodynamics of metformin, stimulation or inhibition of which enzyme?


Answer: metformin activate ampk and inhibit glycerophosphate dehydrogenase
Metformin decreases blood glucose levels by 3 mechanisms:
- Decreasing hepatic glucose production (gluconeogenesis).
- Decreasing intestinal absorption of glucose.
- Improving insulin sensitivity by increasing peripheral glucose uptake and utilization.
Metformin works by activating an enzyme called amp-activated protein kinase (ampk), that in turn signals the liver to produce
less glucose (inhibits gluconeogenesis) and encourages cells to store glucose from the bloodstream thus lowering blood
glucose levels.
Also, metformin non-competitively inhibits the redox shuttle enzyme mitochondrial glycerophosphate dehydrogenase, resulting in
an altered hepatocellular redox state, reduced conversion of lactate and glycerol to glucose, and decreased hepatic
gluconeogenesis.

329
103.Mechanism of metformin in fasting blood glucose?
Metformin is an anti-hyperglycemic agent which improves glucose tolerance in patients with type 2 diabetes, lowering both basal
and postprandial plasma glucose. Its pharmacologic mechanisms of action are different from other classes of oral anti-hyperglycemic
agents. Metformin decreases hepatic glucose production, decreases intestinal absorption of glucose, and improves
insulin sensitivity by increasing peripheral glucose uptake and utilization.
Http://www.rxlist.com/glucophage-drug/clinical-pharmacology.htm

104.Why does the thyroid move with swallowing?


The thyroid gland is invested in a sheath derived from the pretracheal fascia. This holds the gland onto the larynx and the trachea, so
the thyroid follows the movements of the larynx during swallowing. The university of michigan- com :
http://www.med.umich.edu/lrc/coursepages/m1/anatomy2010/html/nervous_system/antneck_case.html

330
105.DM patient who is allergic to sulfonylureas what is the alternative treatment option:
Meglitinides. Bmj http://bestpractice.bmj.com/best-practice/monograph/24/treatment/details.html#expsec-4

106.The molecular mechanism of metformin is incompletely understood:


Inhibition of the mitochondrial respiratory chain (complex i), activation of amp-activated protein kinase (ampk), inhibition of
glucagon-induced elevation of cyclic adenosine monophosphate (camp), and consequent activation of protein kinase a (pka),
inhibition of mitochondrial glycerophosphate dehydrogenase, and an effect on gut microbiota have been proposed as potential
mechanisms.

107.In dka what is found in urine:


Glucose and ketones and acetoacetate. Medscape. Http://emedicine.medscape.com/article/118361-workup

108.The cause of fracture in osteoporosis is due to:


Answer: vitamin d deficiency
Vitamin deficiency is not a direct cause rather it is a risk factor that predisposes to fractures due to its effect on bone mineralization
and its neuromuscular support preventing falls.
Http://cursoenarm.net/uptodate/contents/mobipreview.htm?32/55/33649?Source=see_link
Http://emedicine.medscape.com/article/330598-overview#a3

109.Male got sting and ignored, develop sob + itching when he went to hospital, collapsed at the taxi, what to give?
Answer: epinephrine
Reference: http://emedicine.medscape.com/article/769067-treatment

110.Osteoporosis patient had bone density scan dxa (-2)?


T score — this number shows the amount of bone you have compared with a young adult of the same gender with peak bone
mass. A score above -1 is considered normal. A score between -1 and -2.5 is classified as osteopenia (low bone mass). A score
below -2.5 is defined as osteoporosis. The t score is used to estimate your risk of developing a fracture. Reference:
http://www.radiologyinfo.org/en/info.cfm?Pg=dexa

111.Rapid treatment of hyperthyroid


A. Beta blockers
Answer:a
• β-blockers often necessary for symptomatic treatment prior to definitive therapy (tornto note)

112.DM patient presented with hx of itching and rash like lesions with white center in inguinal region but with sparing of the
folds (picture) what is the cause??
A. Candida
Answer: a

331
Candidiasis or dermatophytosis may underlie pruritus in diabetic patients. Anogenital pruritus is often caused by candidiasis in
diabetic patients
Link: http://www.idb.hr/diabetologia/02no3-2.pdf

113.Why decrease wound healing in dm


Diabetes is a particularly important risk factor for the development of chronic wounds because it is associated with vasculopathy,
neuropathy and immunopathy [17].
Diabetes is frequently associated with peripheral artery disease with atherosclerosis developing at a younger age and affecting more
distal arteries (eg, popliteal, tibial arteries). Peripheral artery disease in combination with diabetic neuropathy contributes to higher
rates of non-healing ulcers and limb loss in diabetic patients compared with those without diabetes [18,19]. Approximately 25
percent of patients with diabetes in the united states will develop a foot ulcer [20,21]. Peripheral artery obstruction is present in
about 20 percent of these patients, and diabetic neuropathy in about 50 percent with about 30 percent having both [22].
Neuropathy alone can be responsible for the development of diabetic foot ulcers. Neuropathy associated with diabetes affects
sensory, motor, and autonomic nerves. Sensory neuropathy diminishes the perception of pain that is protective when tissue injury
has occurred [23]. Patients with diabetes may not be aware of the injury, particularly if the injured region cannot be seen or if the
patient has a visual impairment. The motor nerves to the intrinsic muscles of the foot are affected in approximately 50 percent of
patients with diabetes resulting in claw deformities in the digits that transfer pressure to the plantar metatarsal heads. Increased
local tissue pressure on the plantar surface or in other regions where bony deformities contact the shoe may lead to skin erosion
and ulceration that may go unnoticed in patients with sensory deficits. In addition, autonomic neuropathy causes the skin to become
dry and susceptible to skin fissures, tearing and infection due to a loss of sweat and oil gland function. Loss of vascular tone may lead
to foot edema

114.Man his wife diagnosed with osteoporosis, he has history of recurrent renal stone, labs showed can’t remember?!! What is
the diagnosis?
The most common clinical presentation of primary hyperparathyroidism (phpt) is asymptomatic phpt. Atypical presentations include
normocalcemic phand parathyroid crisis. The classical manifestations of ph("bones, stones, abdominal moans, and psychic groans")
are uncommon in the united states but are still prevalent in developing countries.
Biochemical screening tests that include measurements of serum calcium currently account for the identification of at least 80
percent of patients with phin western countries [1]. These patients are usually asymptomatic and have mild and sometimes only
intermittent hypercalcemia [2,3]. In most asymptomatic patients, the mean serum calcium concentration is less than 1.0 mg/dl (0.25
mmol/l) above the upper limit of the normal range

115.Obese patient diagnosed with diabetes type 2, in addition to lifestyle modification which drug you want to add?
Answer: metformin

116.Patient with hx of dyslipidemia on statins lipid profile is normal which drug would you add to the medications?
They mentioned the other medications of dyslipidemia
Not sure i understand this

117.Drug cause hypoglacemia?


And they mentioned some types of antideibetics, i think sulfynelaurea.
332
118.Incretins mechanism of action?
Incretins are a group of metabolic hormones that stimulate a decrease in blood glucose levels. Incretins do so by causing an increase
in the amount of insulin released from pancreatic beta cells of the islets of langerhans after eating, before blood glucose levels
become elevated.
They also slow the rate of absorption of nutrients into the blood stream by reducing gastric emptying and may directly reduce food
intake. They also inhibit glucagon release from the alpha cells of the islets of langerhans. The two main candidate molecules that
fulfill criteria for an incretin are the intestinal peptides glucagon-like peptide-1 (glp-1) and gastric inhibitory peptide (also known as:
glucose-dependent insulinotropic polypeptide or gip). Both glp-1 and gip are rapidly inactivated by the enzyme dipeptidyl peptidase-
4 (dpp-4); both glp-1 and gip are members of the glucagon peptide superfamily

119.Patient with high cholesterol on treatment, has muscles aches what was she given:
Answer: statins

120.Patient diabetic for years and was just dx as htn, what to give:
Answer: ACEI best
Thus, our overall approach in diabetic patients who require antihypertensive therapy is as follows:

In patients with severely increased albuminuria (formerly called "macroalbuminuria"), we treat with an ace inhibitor or an arb. We
also use these drugs in patients with moderately increased albuminuria (formerly called "microalbuminuria"), even though the
benefits of angiotensin inhibition on kidney disease progression in such patients are unproven.

In patients without increased albuminuria, initial monotherapy can consist of an ace inhibitor, arb, thiazide diuretic, or calcium
channel blocker. However, because angiotensin inhibitors can prevent albuminuria, and because thiazide diuretics have the
disadvantage of an adverse effect on glucose metabolism, many experts will choose an ace inhibitor or arb even in patients without
albuminuria.

In patients who require more than one drug to control their blood pressure, we treat with a combination of an ace inhibitor (or arb if
unable to take an ace inhibitor) and a dihydropyridine calcium channel blocker

121.Calculation of bmi and what degree pf obesity


Classification of bmi — bmi classifications are based upon risk of cvd [16]. The recommended classifications for bmi adopted by the
national institute of health (nih) and world health organization (who) [19,20] for caucasian, hispanic, and black individuals are:
Underweight – bmi <18.5 kg/m2.
Normal weight – bmi ≥18.5 to 24.9 kg/m2.
Overweight – bmi ≥25.0 to 29.9 kg/m2.
Obesity – bmi ≥30 kg/m2.
Obesity class i – bmi of 30.0 to 34.9 kg/m2.
Obesity class ii – bmi of 35.0 to 39.9 kg/m2.
Obesity class iii – bmi ≥40 kg/m2. This type of obesity is also referred to as severe, extreme, or massive obesity.
Source: http://www.uptodate.com/contents/obesity-in-adults-prevalence-screening-and-
evaluation?Source=search_result&search=bmi&selectedtitle=1%7e150#h7

122.Patient diagnosed as HTN and started meds came back with high glucose. Wt was he given:
Answer: thiazides
333
Thiazide diuretics
Mode of action
Inhibition of sodium and chloride reabsorption in the distal tubule of the kidney, resulting in increased urinary excretion of sodium
and water. Direct arteriolar vasodilation.
Contraindications
Known hypersensitivity to thiazides or other sulfonamides. Anuria.
Adverse effects
Electrolyte abnormalities, including hypokalemia, hypomagnesemia, hyponatremia, and hypercalcemia may occur. Elevated blood
glucose levels have also been reported. Hyperuricemia is possible, therefore use with caution in patients who suffer from gout.
Arrhythmias may be precipitated secondary to electrolyte abnormalities. Hyperlipidemia (increase in total cholesterol, triglycerides,
and ldl cholesterol) has occurred. Dermatologic side effects include photosensitivity and an sle-like syndrome.
Source: http://www.medscape.com/viewarticle/421426_1

123.Patient diagnosed DM and compliant on medication but he complained of hypoglycemia more than 3 times. Cause?
Answer: honeymoon phase
Http://www.diabetes.co.uk/blood-glucose/honeymoon-phase.html

124.Primary hyperparathyroidism what will be the lab results:


a. High pth, hypercalcemia, hypophosphatemia, hypercalciuria, and urine camp is elevated.
b. Chloride/phosphorus ratio of >33 is diagnostic of primary hyperparathyroidism (33-to-1 rule). Chloride is high secondary to
renal bicarbonate wasting (direct effect of pth).
Reference: step up to medicine

125.Effect of metformin on DM ?
A. Decrease hepatic gluconeogenesis
Answer:a
Action of metformin :
- 1.decrease insulin resistance.
- 2.decrease appetite.
- 3.increase anerobic glycolysis
(davidsons principles & practice of medicine 22nd edtion)

126.Papillary thyroid ca mostly associated with which of the following ?


A. Hürthle cell

Answer: a
most common thyroid malignancy (80%)
radiation exposure
lymphatic spread to cervical nodes
prognosis: excellent- slow growth. 20 years survival

334
tx: surgery ressection or radiotherapy with to i131 if mts.
Histology: papillary pattern, psammoma bodies, clear nuclei “orphan annie eyes nuclei”, intranuclear cytoplasmatic inclusions.

Medscape:
another subclassification of hürthle cell neoplasms has been proposed, namely the papillary variant of hürthle cell cancer (ie, hürthle
cell papillary thyroid carcinoma), in addition to hürthle cell cancer and adenoma. Clinically, tumors in this group tend to behave like
papillary thyroid carcinoma; however, they are more indolent, with a propensity for lymph node metastasis rather than
hematogenous spread. Hürthle cell carcinoma is a variant of papillary cell carcinoma.

127.Patient with constipation increase weight,thinning of hair ?


A. Hypothyroidism
Answer: a

128.Diabetic patient, what type of carbohydrates is recommended?

Answer: polysaccharide (complex)


Reference: dietitian

129.Earliest sign of hypophosphatemia


Symptoms of hypophosphatemia are nonspecific and highly dependent on cause, duration, and severity. Mild
hypophosphatemia (ie, 2-2.5 mg/dl), whether acute or chronic, is generally asymptomatic. Patients with severe and/or
chronic hypophosphatemia are more likely to be symptomatic. Weakness, bone pain, rhabdomyolysis, and altered mental
status are the most common presenting features of persons with symptomatic hypophosphatemia.
Reference: medscape: http://emedicine.medscape.com/article/242280-clinical

130.Which type of thyroid cancer will responds to radioactive therapy?

Both papillary (as adjuvant) and anaplastic carcinoma respond to radioactive therapy.

Reference: step up to medicine.

131.A lot of q about the best type of insulin to give/side effects of each of them:
Know the anti-diabetic medications well.

132.Diabetic on metformi+glib , uncontrolled , we add: "no insulin in choices "

133.Old with osteoporosis risk scenario, has spine compressive fracture, what to give her to protect?
A. Bisphosphonate, calcium, Vitamin d other
Answer:
335
Http://www.m.webmd.com/a-to-z-guides/spinal-compression-fractures-preventing

134.Metformin work in which enzymes?


A. Ampk

135.DM with high Blood pressure and deteriorating renal what to give?
A. ACEI (lisinopril)
Answer: scientists have made great progress in developing methods that slow the onset and progression of kidney disease in people
with diabetes. Drugs used to lower blood pressure can slow the progression of kidney disease significantly. Two types of drugs,
angiotensin-converting enzyme (ace) inhibitors and angiotensin receptor blockers (ARBs), have proven effective in slowing the
progression of kidney disease. Many people require two or more drugs to control their blood pressure. In addition to an ace
inhibitor or an ARB, a diuretic can also be useful. Beta blockers, calcium channel blockers, and other blood pressure drugs may also
be needed.
An example of an effective ace inhibitor is lisinopril (prinivil, zestril), which doctors commonly prescribe for treating kidney disease of
diabetes. The benefits of lisinopril extend beyond its ability to lower blood pressure: it may directly protect the kidneys' glomeruli.
Ace inhibitors have lowered proteinuria and slowed deterioration even in people with diabetes who did not have high blood
pressure.
An example of an effective ARB is losartan (cozaar), which has also been shown to protect kidney function and lower the risk of
cardiovascular events.

136.Follow up of blood lipids and follow up of blood sugar.


Answer:
Http://www.e-mercy.com/images/cholesterol-metabolism/research/2/guidelines-to-interpret-results-from-health-screening.pdf

137.High triglyceride what to add to statins?


Answer:
Hmg coa reductase inhibitor medications (statins) are most effective in lowering ldl cholesterol, mildly effective in increasing hdl
cholesterol, and mildly effective in lowering triglycerides.
Fibric acid derivative medications such as gemfibrozil (lopid) and fenofibrate (tricor) are most effective in lowering triglycerides,
effective in increasing hdl, and minimally effective in lowering ldl levels.
Nicotinic acid (niacin), known by the names niacin, niaspan, or slo-niacin, is most effective in increasing hdl, effective in lowering
triglycerides, and mildly to moderately effective in lowering ldl levels
Http://www.emedicinehealth.com/script/main/mobileart-emh.asp?Articlekey=114550&page=7

138.Dyslipidemia on tx, developed muscle weakness, what’s the drug he’s taking?
A. Statins

139.Patient with fracture, constipation and abdominal pain, high serum ca:
A. Hyperparathyroidism
Answer: a
336
Source: master the boards usmle step 2 ck

140.Primary hyperparathyroidism what will be the lab results:


Answer: calcium and pth both high
Reference : uptodate

141.Patient use anti-hyperlipidemia, liver enzymes normal, creatin kinase(ck) high:


A. Statins
One of the side effects of statins is myositis which raise ck, but patietns are not followed for that. They who use it are followed
regularly with LFTto detect any liver damage early which is a common se of the drug. (case files family medicine hyperlipidemia
chapter)

142.Drug used for hypertriglycerdemia in addition to statins:


A. Fibrates
(case files family medicine hyperlipidemia chapter)

143.Which of the following drug from biphosphonate work as atp analog to suppers osteoclasts? All the medication given end
with dronate ...........
(answer: clodronate + etidronate+ tiludronate )

144.Patient with high level of cholesterol, taking medication, complain of muscle pain , what he is taking?
A. Simvastatin.
Answer: a
Toronto:
Statins side effects are: myalgia, rhabdomyolysis, abdominal pain.

145.DM patient on medication present with weakness & dizziness, what the drug he is taking?!
A. Sulfonylurea.
Answer: a
Medscape

146.Hormone that helps in increase body sensitivity to insulin:


A. Leptin.
Answer: a

147.Cushing syndrome with fracture, because of :


A. Osteoporosis
Explanation: signs of cushing : central obesity, round face, supraclavicular and dorsal fat pads, facial plethora, proximal muscle
wasting, purple abdominal striae, skin atrophy, acanthosis nigricans, htn, hyperglycemia, osteoporosis, pathologic fractures
Reference: toronto notes 2015, page e33, endocrinology
337
148.An asthmatic patient with no hx of DM in the family, now his bs is within normal range when to check again:
A. 3 years if the patient did not have any risk factor

Reference: http://guidelines.diabetes.ca/browse/chapter4

149.A patient presented with multiple stones and high ca:


A. Hyperparathyroidism
Answer:a
hyperparathyroidism is caused by one or more glands produce inappropriately high amounts of pth. It is the most common cause of
hypercalcemia .patients may present with nephrolithiasis,bone aches and pains ,muscle pain and weakness, pancreatitis, peptic
ulcer disease,gout ,constipation, polydipsia, polyuria, htn, depression, fatigue, anorexia, sleep disturbances, anxiety and or lethargy.
Reference: step up to medicine

150.Drug used for hypertriglyceridemia in addition to statins:


A. Fibrates
Answer: the fda approved a new fenofibrate formulation known as fenofibric acid (trilipix) with a specific indication for use with a
statin in patients with mixed dyslipidemia
Reference: http://emedicine.medscape.com/article/126568-medication
338
151.Case about a diabetic who takes metformin which resulted in correction of ha1c question metformin lowers glucose by?
A. Answer; moa: it lowers glucose production by liver, increases numbers of insulin receptors on muscles and fat cells.
Reference: http://www.diabetesnet.com/about-diabetes/diabetes-medications/metformin

152.Diabetic on metformi+glib , uncontrolled , we add: "no insulin in choices "

153.Vitamine c deficiency which affected in stages of wound healing ?


A. Collagen synthesis
Vitamin c is important in the wound repair process, facilitating the building of collagen in the wound, which forms the framework for
the building of new tissue .
Answer:a
Reference: http://www.surgerysupplements.com/the-role-of-vitamin-c-in-wound-healing/
Reference : http://emedicine.medscape.com/article/1173204-clinical

154.Administering pyridoxine and supportive care?


Pyridoxine used as vitamin b6 dietary supplement. The classic clinical syndrome for vitamin b6 deficiency is a seborrhoeic dermatitis-
like eruption, atrophic glossitis with ulceration, angular cheilitis, conjunctivitis,intertrigo, and neurologic symptoms of somnolence,
confusion, and neuropathy. The elderly and alcoholics have an increased risk of vitamin b6 deficiency, as well as other micronutrient
deficiencies.

155.Hypoglycemia is a side effect of which medication?


Answer is: no choices + no sulfonylurea in options
Su is the most common oral anti-diabetic drug to cause hypoglycemia.
Metformin has very low-risk se of hypoglycemia.
Any 2 oral anti-diabetic agents combination can cause hypoglycemia.
Source: http://www.medscape.com/viewarticle/722513_3

156.Case of hypothyroidism
A. Hypothyroidism: http://emedicine.medscape.com/article/122393-overview

157.Most significant risk factor for op?


A. Advanced age

158.Ostoprosis prevention..
A. Biphosphat +Vitamin d +ca

159.Pheochromocytoma what is the investigation :


339
A. Catecholamine in urine
Answer: a
It is are catecholamine secreting tumor derived from chromaffin cells of the sympathetic system.it has a classic triad
(pheochromocytoma )palpitations headache episodic sweating. Urine catecholamines will show increased catecholamine
metabolites (metanephrines) and free catecholamine. Plasma metanephrines if available is the most sensitive test. Reference:
toronto notes

160.Vitamin b3 deficiency.
Pellagra is a vitamin deficiency disease most frequently caused by a chronic lack of niacin (vitamin b3) in the diet. Pellagra is
classically described by "the three ds": diarrhea, dermatitis, and dementia.

161.Mechanism of antidiabetic medication?

162.Mechanism of action of propthyrocil (ttt of hyperthyroidsm):


Answer: inhibity thyroperoxsidase enzyme centrally and inhibit conversion of t4 to active t3 peripherally. Medcine. Kaplan
medicine endocrine chapter

163.Young patient has hypertension, high Na and low K. What is the treatment?
Spironolactone

Answer: A
This patient has hyperaldosteronism.
Spironolactone is a potassium sparing diuretic.

164.Case of polyuria, polydipsia and weight loss. Na: ?. What is the diagnosis?
DI

Answer: A (Depends on the case)


Diabetes Insipidus (DI) clinical features: Passage of large volumes of dilute urine, polydipsia, dehydration; hypernatremia can
develop with lack of access to water or impaired thirst mechanism.
Reference: Toronto Notes.

165.Female patient complaining of constipation, weight gain and fatigue. What is the most likely diagnosis?
A. Hypothyroidism

Answer: A

166.T1DM whats the antigen he has? DR 4, DR 7?


Answer: HLA-DR 3 & 4
Ref: 1ST AID USMLE STEP 2
340
167.Pheochromocytoma ..what is the initial tx ?
➢Alpha antagonist.
Answer: A
Alpha adrenergic blocker to control hypertension follow by beta blocker to control tachycardia never give BB first ( unopposed
action lead to refractory hypertension.
Reference: USMLE step 2 first aid +step up to medicine

168.Young boy presented with abdominal pain and vomiting

Answer: DKA?
➢Very short question, not enough info, can’t decide but the answer depends on the clinical scenario.
➢Common causes of abdominal pain include constipation, gastrointestinal (GI) infections, infections outside of the GI tract, and
colic.
➢Less common GI conditions (i.e., inflammatory bowel disease, pancreatitis, cholecystitis, intra-abdominal abscess, dietary milk
protein allergy, malabsorption, and Meckel's diverticulum)
➢Causes of abdominal pain outside GI tract include: diabetic ketoacidosis, painful crisis with sickle syndromes, Henoch Schönlein
purpura (IgAvasculitis), tumors, urolithiasis, ovarian torsion, testicular torsion, and some toxic ingestions.
➢The answer “DKA” was written by the person who provided the question, so it’s possibly the right answer.
Reference: http://www.uptodate.com/contents/causes-of-acute-abdominal-pain-in-children-and-
adolescents?Source=preview&search=%2Fcontents%2Fsearch&anchor=H30#H30
Please read here for more specific classification according to age:
Http://www.uptodate.com/contents/image?Imagekey=EM/65488&topickey=EM%2F6454&source=outline_link&search=%2
Fcontents%2Fsearch&utdpopup=true

341
Gastroenterology

342
1. Patient c/o liver cirrhosis and ascites now he c/o weight loss, what should u do ?
A. cea
B. ca125,
C. abdominal us
D. alpha-fetoprotien

Answer: d
Explanation: cirrhosis is a risk factor for hepatocellular carcinoma. When elevated, the afp is 75-91% specific, and values greater than
400 ng/ml are generally considered diagnostic of hcc in the proper clinical context. Ultrasonographic identification of hcc can be
difficult in the background of regenerative nodules in the cirrhotic liver. Cea is mainly for colon cancer. Ca125 for ovarian cancer.
Reference: http://emedicine.medscape.com/article/197319-workup#c1

2. Patient with history of gastric ulcer. Which of the following are used for pain control does not cause gastric irritation?
A. Aspirin
B. Ibuprofen
C. Celecoxib
D. Indomethacin

Answer: c
Reference: http://cursoenarm.net/uptodate/contents/mobipreview.htm?29/13/29917?Source=see_link

3. Migratory thrombophlebitis is associated with which of the following:


A. Chronic pancreatitis
B. Acute pancreatitis
C. Pancreatic malignancy
D. Pancreatic cyst

Answer: c
Http://emedicine.medscape.com/article/463256-clinical#b1

4. Hbsag +ve
A. Acute hepatitis
B. Chronic hepatitis
C. Acute carrier
D. Chronic carrier

Answer: a or b (question incomplete)


Explanation:

343
Reference: toronto notes 2015, page g30, gastroenterology

5. What hepatitis can be prevented by vaccination?


A. A
B. B
C. D
D. E
Answer: b !!

6. Diagnostic peritoneal lavage indicated in :


A. All patient with rta
B. Patient with hypotension and distended abdomen
C. Patient with abdominal pain
D. Patient with sever head trauma
Answer: b

7. A patient complains from dysphagia and feel of foreign body sensation. What is the most likely diagnosis?
A. Achalasia
B. Laryngeal cancer
C. Globus pharyngis
D. Esophageal cancer
Answer: d
Explanation:
Esophageal cancer: dysphagia is most common presenting symptom; it is progressive first to solids then liquids. Second most
common presenting symptom is weight loss. Patients also complain of foreign body sensation
Achalasia: dysphagia equal to solids and liquids. Patients tends to eat slowly and drink a lot of water to wash down food.
344
Globus: persistent or intermittent non-painful sensation of a lump or foreign body in the throat. However, it frequently
improves with eating and is generally unaccompanied by dysphagia or odynophagia.
Step up to medicine
Http://www.hopkinsmedicine.org/gastroenterology_hepatology/_pdfs/esophagus_stomach/esophageal_cancer.pdf
Http://www.ncbi.nlm.nih.gov/pmc/articles/pmc3360444/

8. Patient was screened for hemochromatosis by ALT & ast what another test can be used also:
A. Glucose
B. Ferritin
C. Creatinine level
D. Ceruplasmin
Answer: b
Reference: http://emedicine.medscape.com/article/177216-workup#c1

9. Scenario, best site to insert the needle for liver biopsy?


A. 6th intercostal space
B. 7th intercostal space
th
C. 10 intercostal space
D. Subcostal space.

Answer: b
The biopsy site is usually located in the seventh or eighth intercostal space in the mid axillary line. Reference:
http://emedicine.medscape.com/article/149684-technique

10. Young male complaining of epigastric discomfort, he tried over-the-counter (otc) medication to relieve this discomfort, he
noticed some improvement but experienced constipation.
Which otc drug most likely he tried ?
A. Calcium carbonate
B. Sodium carbonate
C. Aluminum hydroxide
D. Something aluminum
Answer: c

11. A 40-year-old male, 2 years history of difficulty swallowing and lump sensation in throat, excessive salivation, intermittent
hoarseness, weight loss:
A. Achalasia
B. Scleroderma
C. Diffuse esophageal spasm
D. Cricopharyngeal dysfunction
Answer: a
Achalasia: dysphagia for solids and liquids is the primary clinical feature of achalasia. Weight loss, regurgitation, chest pain,
heartburn, globus sensation (a lump in the throat) & hiccups. Https://yhdp.vn/uptodate/contents/mobipreview.htm?14/26/14759

345
Scleroderma:gi findings in systemic sclerosis include the following: gastroesophageal reflux (may lead to hoarseness, dysphagia
and aspiration pneumonia), dyspepsia, early satiety, malnutrition from inadequate caloric intake.
Http://emedicine.medscape.com/article/331864-clinical - showall
Esophageal spasm: noncardiac chest, globus sensation, dysphagia, regurgitation &
heartburn http://emedicine.medscape.com/article/174975-clinical - showall
Cricopharyngeal dysfunction: progressive dysphagia, coughing, choking, drooling, and regurgitation when swallowing liquids or solid
food
Https://yhdp.vn/uptodate/contents/utd.htm?1/63/2037?Source=see_link#h9

12. Man present with epigastric pain for 3 months worse after eat diagnosed as hpylori triple treatment given to him what is the
best indicator for his improvement
A. A-endoscope
B. Ph
C. Blood test for h pylori
D. Clinical improvement
Answer: d or urea breath test or fecal antigen test

13. Patient with abdominal pain, nausea, vomiting, wt loss. On examination: palpable mass. What is your action?
A. PPI
B. Follow up
C. Urgent referral
D. Ultrasound

Answer: c
This may be a case of gastric cancer which needs to be confirmed by upper gi endoscopy
Http://emedicine.medscape.com/article/278744-workup

14. Asymptomatic patient with positive HBV antigen?


A. Acute hepatitis
B. Chronic hepatitis
C. Active carrier
D. Non active carrier

Answer: d
If the hbsag is positive and the liver enzymes are normal it can be either hbeag negative chronic hepatitis of inactive carrier status.
The only way to differentiate is by follow up.

346
Reference: https://www.cdc.gov/hepatitis/HBV/pdfs/serologicchartv8.pdf
Http://www.bjmp.org/content/hbsag-carriers-normal-alt-levels-healthy-carriers-or-true-patients

15. What is the common disease to make the patient retire in ksa?
A. HBV
B. HBC
C. HIV
D. HAV

Answer: a

16. Lady come to the clinic to regular check up with everything is normal with table of direct and indirect bilirubin (within the
normal range) what is your diagnosis:
A. Rotor syndrome
B. Crigler najjar
C. Dubin johnson
D. Gilbert syndrome
Answer:

17. A patient with heartburn taking antacids. She had rheumatic fever 1 week ago and was started on aspirin. What side effect
she can develop?
A. Constipation
B. Diarrhea
C. Dry mouth
D. Galactorrhea
Meaning of q is se of which drug!! Antacid or aspirin??
Answer: aspirin se: conditions of excess stomach acid secretion, nausea, vomiting, heartburn, irritation of the stomach (cramps)
347
Antacid se: cause nausea, constipation, diarrhea, or headache. Diarrhea is more common with this product than constipation
http://www.webmd.com/drugs/2/drug-76860-769/antacid-oral/aluminum-magnesiumantacid-simethicone-oral/details#side-effects

18. What is the percentage of complete recovery from HBV?


A. 20%
B. 40%
C. 60%
D. 80%

Answer: d (80%)

19. A patient was diagnosed with enteric fever. What is the presentation that he will have?
A. Confusion (or other cns problems)
B. Maculopapular rash
C. Nausea, vomiting and loose stools
D. Abdominal pain

Answer: d

Over the course of the first week of illness, the notorious gastrointestinal manifestations of the disease develop. These include
diffuse abdominal pain and tenderness and, in some cases, fierce colicky right upper quadrant pain. Monocytic infiltration inflames
peyer patches and narrows the bowel lumen, causing constipation that lasts the duration of the illness. The individual then develops
a dry cough, dull frontal headache, delirium, and an increasingly stuporous malaise.

Reference: medscape: http://emedicine.medscape.com/article/231135-clinical

20. Liver biopsy taken at level of intercostal space:


A. Fifth
B. Seventh
C. Ninth.
D. Eleventh

Answer: b

Proper identification of the biopsy site is of paramount importance. To choose the site, start with percussion over the right upper
quadrant. The biopsy site is usually located in the seventh or eighth intercostal space in the midaxillary line. The site can be further
confirmed with either routine ultrasonography or a bedside portable ultrasound machine.

Reference: medscape: http://emedicine.medscape.com/article/149684-technique

21. Most common hepatitis is:


A. HBV
B. HBV
348
C. HDV
D. HEV

Answer: a
reference: medscape

22. Best prophylactic against traveller's diarrhea: repeated in family medicine


A. Fresh fruit and vegetables
B. Peeled fruit
C. Daily antibiotic
D. Drinks with rice
Answer: b
Reference: uptodate

23. 40 years old patient complaining of weight loss, nausea, lethargy and jaundice. When he was asked about similar attack in
the past, he mentioned 4 episodes during the past two years. What is the most likely diagnosis?
A. Acute pancreatitis.
B. Cancer head of pancreas.
C. Peptic ulcer disease.
D. Chronic pancreatitis

Answer: B
The most characteristic sign of pancreatic carcinoma of the head of the pancreas is painless obstructive jaundice.
Reference:http://emedicine.medscape.com/article/280605-clinical

24. 27 years old smoker who was studying in a foreign country for two years and lived in a student housing. He returned home
two months ago. The patient complains of 4 day mid epigastric pain, what is the most likely diagnosis?
A. Viral Hepatitis.
B. H. Pylori infection.
C. Acute Pancreatitis.
D. Myocardial infarction.

Answer: B
Epigastric pain is the most common symptom of both gastric and duodenal ulcers. It is characterized by a gnawing or burning
sensation and occurs after meals—classically, shortly after meals with gastric ulcer and 2-3 hours afterward with duodenal ulcer.
Pain radiates to back with pancreatitis.
Reference: http://emedicine.medscape.com/article/181753-overview

25. Which disease involves antibiotic in treatment regimen?


A. Crohn’s
B. Ulcerative colitis
C. Celiac
D. Whipple

Answer: D
Reference: http://emedicine.medscape.com/article/183350-treatment
349
26. What is the best treatement for traveler diarrhea? 3 times
A. Ciprofloxacin
B. Cipro
C. Amoxycillin
D. Metronodazole

Answer: fluoroquinolones, such as ciprofloxacin or levofloxacin


Http://www.uptodate.com/contents/travelers-diarrhea-clinical-manifestations-diagnosis-and-treatment

27. H.pylori infection cause antral gastritis :h.pylori stimulate which cell?
A. Chief cell
A. Gastrin
B. Parietal
C. Mucous

28. Lady come to the clinc to regural check up with every thing is normal with table of direct and indirect bliurbin (within the
normal range ) what is your diagnosis :
A. Rotor syndrome
B. Crigler najjar
C. Dubin johnson
D. Gilbert syndrome
Not clear every thing is normal.

29. Young boy presented with diarrhea which is sometime bloody, weight loss, arthritis, anemia. The most likely diagnosis is?
A. Crohn’s disease
B. Uc
C. Celiac disease
Answer: a or b
Both crohn & uc cause the same symptoms (bleeding is more in uc)
Celiac will cause steatorrhea (step up)

30. Patient has whitish elevated patchy lesion over the dorsal surface of the tongue, it does not remove after scrubbing the
lesion , what is the most likely diagnosis ?
A. Dysplasia
B. Neurofibroma
C. Foreign body
Answer:

31. Oral ulcers with intermittent chronic bloody diarrhea...


350
A. Celiac disease
B. Uc
C. Crohn's disease

Answer: c
Http://emedicine.medscape.com/article/172940-clinical

32. Alcoholic patient presented with ruq pain, jaundice , spleenomegaly , ascites ,and caput medusa
A. Pht
B. Hepatitis
C. Budd-chiari syndrome
Answer: c

The clinical presentation of budd-chiari syndrome (resemble those of cirrhosis)—hepatomegaly, ascites, abdominal pain (ruq),
jaundice, variceal bleeding. This disease is caused by occlusion of hepatic venous out ow, which leads to hepatic congestion and
subsequent microvascular ischemia.
Note: if cirrhosis or alcoholic liver disease was mentioned in the choices it is more correct than budd-chiari syndrome
Reference: step up to medicine

33. A young boy presented with diarrhea that sometimes becomes bloody, wight loss, arthritis, and anemia. What is the
diagnosis is:
A. Crohn's disease
B. Ulcerative colitis
C. Celiac

351
Answer: b

34. Origin from cadal part of foregut and cephalic of midgut


A. Esophagus
B. Duodenum
C. Ileum
Answer: b
the duodenum develops from the caudal portion of the foregut and cranial portion of the midgut.
Http://www.chronolab.com/atlas/embryo/duodenum.htm

35. Patient complains of acute sub mandibular pain which comes after food. He has history of similar episodes with secretions.
What is the most likely diagnosis?
A. Calaculs in submandibular gland duct (wharton duct)
B. Parotid cancer
C. Parotitis
Answer: a
Salivary duct stones present as pain and swelling in the involved area. It is usually aggravated by eating or anticipation of food.
Parotid cancer’s most common presentation is painless swelling. Parotitis presents as progressive painful swelling with fever.
Http://cursoenarm.net/uptodate/contents/mobipreview.htm?7/20/7488

352
Http://emedicine.medscape.com/article/882461-clinical
Http://emedicine.medscape.com/article/1289616-overview#a2

36. 19 years old female, presenting with abdominal pain, diarrhea, bloating, improved with defecation, diagnosis?
A. Ibd
B. Ibs
C. Celiac
Answer: b
Irritable bowel syndrome (ibs) is a functional gi disorder characterized by abdominal pain that commonly relived by defecation,
abdominal distension and altered bowel habits in the absence of a specific and unique organic pathology.

37. When does the jaundice occurs in gilbert syndrome?


A. Pregnancy
B. Inter current illness
C. Others (no stress in options.)

Answer: jaundice may appear under conditions of exertion, stress, fasting, and infections, but the condition is otherwise usually
asymptomatic.

38. Patient on NSAID with signs of ulcer what you will do?
A. Triple therapy
B. Urea breath test
C. Immediate endoscope

Answer: b
Reference: http://www.aafp.org/afp/2007/1001/p1005.html
Algorithm for the treatment of peptic ulcer disease:
*—alarm symptoms include evidence of bleeding (e.g., anemia, heme-positive stool, melena), perforation
(e.g., severe pain), obstruction (e.g., vomiting), and malignancy (e.g., weight loss, anorexia).

353
(egd = esophagogastroduodenoscopy; PPI= proton pump inhibitor; NSAID = nonsteroidal anti-inflammatory drug.)

39. Hemoangioma with high heart output. What other site will be affected?
A- Lung.
B- Spleen
C- Kidney.
Not sure i think the best answer would be liver

Giant cutaneous hemangiomas — giant cutaneous hemangiomas can also promote the development of high-output failure.
Hemangiomas are the most common tumors of infancy, and seldom cause more than a cosmetic problem. Fifty percent of
cutaneous lesions are present at birth; the remainder usually surface by two months of age. In rare cases, high-flow arteriovenous
shunting in giant or multiple cutaneous hemangiomas can lead to the development of high-output failure

Multifocal hepatic hemangiomas most commonly occur in the presence of multiple skin hemangiomas and are probably most often
asymptomatic, in which case they can be managed with observation with or without serial imaging.
Rarely, multifocal hepatic hemangiomas can have large vessel shunts that result in heart failure, which can be treated with
pharmacotherapy or embolization if necessary. Diffuse liver hemangiomas, which may occur in the absence of skin hemangiomas,
cause massive hepatomegaly with abdominal compartment syndrome, impaired ventilation, impaired venous return, and renal vein
compression, and are associated with a high mortality rate

40. 30 years with chronic diarrhea and ataxia and abnormal movements. Jejunal biopsy showed: t. Wheppli infection
management?
A- short term antibiotics
B- steroids
C- long term antibiotics.
Answer: c

354
Spectrum of disease — the spectrum of clinical findings due to t. Whipplei infection is wide.
Classic whipple’s disease is a multisystem process characterized by joint symptoms, chronic diarrhea, malabsorption, and weight
loss; many other organ systems can also be affected. The disease presents over time, with joint symptoms preceding the others by
many years, so not all symptoms may be manifest at the time of presentation in affected individuals. Isolated involvement of other
organs, most prominently the central nervous system (cns) and heart valves, can also occur in the absence of the classic findings of
whipple’s disease.
Treatment rationale:
Initial management — whipple's disease was uniformly fatal prior to the availability of antibiotics but can be successfully treated
with them. The optimal regimen is uncertain. For patients with chronic whipple's disease (either classic or localized chronic
infection), we suggest an initial phase of an intravenous antibiotic that is active against t. Whipplei and is known to penetrate the
blood-brain barrier, followed by 12 months of oral maintenance therapy with trimethoprim-sulfamethoxazole (tmp-smx)

41. Dietary supplementation prevent colon cancer:


A. Folic acid
B. Vitamin d
C. Vitamin c
Answer: b
Cohort studies and meta-analyses provide evidence on the benefits of circulating, diet-derived and supplemented vitamin d and
calcium. On the basis of current evidence one could suggest intake of vitamin d.
Http://www.ncbi.nlm.nih.gov/pmc/articles/pmc3819783/
- a review of nutrition and cancer by the world cancer research fund and the american institute of cancer research concluded that:
“there is enough evidence to conclude that milk may protect against colorectal cancer.”
- folic acid: http://www.nature.com/articles/srep12044

42. A 30-year-old female, history of constipation, strains when she passes stool.
What muscle should be relaxed?
A. Coccygeus
B. Pubococcygeus
C. Puborectalis
Answer: c
the levator ani is a broad sheet of muscle. It is composed of three separate paired muscles, called
the pubococcygeus, puborectalis and iliococcygeus.
Puborectalis main function is to maintain fecal continence – during defecation this muscle relaxes.
Http://teachmeanatomy.info/pelvis/muscles/pelvic-floor/

43. 25 years' teacher, complaining of abdominal pain, fatigue, on exam there was icting, palpable liver 1cm, also 2 student
complaining from same complain?
A. Hav
B. HBV
C. HBV
Answer: a

44. Patient c/o liver cirrhosis and ascites now he c/o weight loss, what should you do?
A. Cea
B. Ca125
355
C. Abdominal ultrasound
Answer: not clear but with only these choices maybe c

45. What vitamin increases metabolism or absorption of iron?


A. Vitamin c
B. Vitamin b
C. Vitamin d
Answer: a
Explanation: try to take iron supplement with vitamin c (for example, a glass of orange juice) to increase absorption.
Reference https://my.clevelandclinic.org/health/diseases_conditions/hic_anemia/hic_oral_iron_supplementation

46. 70 years old male presented with recent tiredness and dizziness, he gives history of change of his bowel habit since awhile, in
that he change his diet to gaurge (something i don't know type of food) on lap he had hypochromic microcytic anemia and
iron deficiency. What could be the cause:
A. Change in his bowel habit
B. Change in diet
C. His age
Answer: a

47. Patient with hba after 3 weeks we take biopsy, what is show?
A. Normal architecture
B. Fibrosis something
C. Another something

Answer: b
May be the question mean HBV…
"ground-glass" cells seen in approximately 50-75% of livers affected by chronic HBV infection which may progress to fibrosis in
severe disease.
Reference: http://emedicine.medscape.com/article/177632-workup#c9

48. Man came with loose stools, history of loose stools before, its watery with mucus not containing blood (forget the other
details ) what is your diagnosis:
A. Ibs
B. Crohn's disease
C. Ulcerative colitis

Answer : a
Unlike ibd, ibs does not cause inflammation, ulcers or other damage to the bowel which can cause bleeding. Symptoms of ibs may
include crampy pain, bloating, gas, mucus in the stool, diarrhea and constipation.
Reference: http://www.webmd.com/ibs/ibd-versus-ibs

49. Patient came with history of alternating bowel habits, sometimes the stool is loose and sometimes suffer from constipation
for weeks, what’s the diagnosis :
A. Ibs
B. Crohn’s disease
356
C. Ulcerative colitis

Answer: a
Unlike ibd, ibs does not cause inflammation, ulcers or other damage to the bowel which can cause bleeding. Symptoms of ibs may
include crampy pain, bloating, gas, mucus in the stool, diarrhea and constipation.
Reference: http://www.webmd.com/ibs/ibd-versus-ibs
reference: http://www.mayoclinic.org/diseases-conditions/irritable-bowel-syndrome/basics/tests-diagnosis/con-20024578

50. What is most likely diagnosis in patient came for regular check and found to have hbsag ? No more details
A. Acute
B. Chronic
C. Carrier inactive

Answer: a
Both acute and chronic could be asymptomatic.
Hepatitis b surface antigen (hbsag)
Present in acute or chronic infection
Detectable as early as 1 to 2 weeks after infection
• it persists in chronic hepatitis regardless of whether symptoms are present. If virus is cleared, then hbsag is undetectable.
Reference: step up to medicine

51. Abdominal pain and fever, then constipation then diarrhea. Culture showed gram –ve rod, non-lactose fermenting, oxidase -
ve organism that produces hydrous sulphate. Most appropriate treatment?
(salmonella typhi)
A. 50s subunit
B. DNA gyrase inhibitor
C. Transpeptidase
Answer: b fluoroquinolones is the drug of choice for treating typhoid fever.
Reference
Peer review
Medscape but not very clear there is a lot of details regarding resistance.

52. Which type of hepatitis have available vaccine


A. Hep b
B. Hep c
C. Hep d

Answer: a
Hepatitis a and b

53. Mountain climber who has hypoxia, which of the following liver zones is most affected by hypoxia
A. Central of acini zone ii
B. Peripheral of acini zone ii
C. Sinusoidal
357
Answer : zone 1 encircles the portal tracts where the oxygenated blood from hepatic arteries enters. Zone 3 is located around
central veins, where oxygenation is poor. Zone 2 is located in between.

54. Which common feature of ibs:


A. Diarrhea
B. Constipation
C. Vomiting

Answer: a

Reference: http://www.ncbi.nlm.nih.gov/m/pubmed/21929652/

55. Best prophylactic against traveller's diarrhea? Repeated in family medicine


A. Fresh fruit and vegetable daily antibiotic
B. Peeled fruit
C. Drinks with rice ....

Answer: b
Uptodate: “basic advice for travelers to moderate or high-risk regions for travelers' diarrhea includes eating only food that has been
thoroughly cooked and served hot, fruits that the traveler peels just prior to eating, and pasteurized dairy products. Beverages
should be bottled or disinfected. Bottled drinks should be requested without ice and should be drunk from the bottle with a straw
rather than from a glass. Hot tea and coffee are usually safe alternatives to boiled water.”
“although antibiotics and other agents (namely bismuth salicylate) are effective in reducing the rate of travelers' diarrhea for
individuals traveling from resource-rich to resource-poor areas, we do not routinely recommend chemoprophylaxis. Use of daily
antibiotics is expensive, has potential side effects, can wipe out normal gastrointestinal flora that may be beneficial, and can
promote bacterial resistance.”
“however, chemoprophylaxis may be a reasonable approach in the setting of an underlying medical condition that would increase
the risk of complications from diarrhea or would be severely exacerbated by dehydration from diarrhea such that the benefits of
using antibiotic prophylaxis outweigh its risks. Such situations include known severe inflammatory bowel disease that could be
exacerbated by an episode of infectious diarrhea; severe vascular, cardiac, or renal disease that would be seriously compromised by
dehydration; or a severe immunocompromised state, such as advanced HIV disease or after a complicated organ transplant.
If prophylaxis is administered, the options include:
●ciprofloxacin — 500 mg once daily
●norfloxacin (not available in the us) — 400 mg once daily
●rifaximin — 200 mg once or twice daily
●bismuth subsalicylate — two tablets chewed four times daily

56. Which common feature of ibs?


A. Diarrhea

358
B. Constipation
C. Vomiting
Answer: a
Ibs can present with both diarrhea and constipation

Reference: mayo clinic, davidsons 22, toronto notes.

57. Patient with Crohn's disease, what is the most relevant and associated with CD?
A. Positive Family History
B. Smoking
C. Alcohol

Answer: A
Positive family history is the largest independent risk factor for CD and UC
Reference: (Kumar)

58. What is the treatment of choice for Traveler's diarrhea?


A. Ciprofloxacin.
B. Amoxicillin.
C. Metronidazole.
Answer: A
The most common organism is enterotoxigenic E. Coli and it is treated with fluoroquinolones (e.g. Ciprofloxacin).
Reference: Uptodate.
Initial treatment: rehydration, the Abx only will decrease the duration of symptoms;
1st line: antibiotics include fluoroquinolones, such as ciprofloxacin or levofloxacin or ofloxacin
2nd line: azithromycin. 3rd line: Rifaximin.

59. Women coming with elevated indirect bilirubin:


A. Rotor syndrome
B. Crigler Najjar

359
C. Dubin Johnson

Answer: B
Differential diagnosis:
- Unconjugated (indirect) hyperbilirubinemia: Hemolytic, physiologic (newborns), Crigler-Najjar, Gilbert syndrome
- Conjugated (direct) hyperbilirubinemia:
1- Biliary tract obstruction: gallstones, cholangiocarcinoma, pancreatic or liver cancer, liver fluke.
2- Biliary tract disease:ƒ 1° sclerosing cholangitis and 1° biliary cirrhosis
3- Excretion defect: Dubin-Johnson syndrome, Rotor syndrome.
- Mixed (direct and indirect) hyperbilirubinemia: Hepatitis, cirrhosis.

60. Which of the following is a common feature of IBS?


A. Diarrhea
B. Constipation
C. Vomiting

Answer: A or Abdominal Pain relived by defecation


IBS is the most commonly diagnosed GI condition and also the most common reason for referral to gastroenterology clinics. IBS is
more frequent in women than in men. Typical clinical manifestations in IBS are discomfort or abdominal pain relieved by defecation,
associated with a change in stool form e.g diarrhea or constipation.
Http://www.ncbi.nlm.nih.gov/pmc/articles/PMC4161800

61. First sign of portal HTN?


A. Spleenomegaly
B. Hepatomegaly
C. Ascites

Answer : A
Non of the sources mention any of the above options as being the first sign. Ascites is always mentioned first before splenomegaly in
signs and symptoms section and step up books state that portal hypertension can’t happen without ascites. However, some sources
stated that splenomegaly is the most cardinal sign of portal hypertension.
Old sources: Davidson’s Principles and Practice of Medicine.
New sources:
Https://dundeemedstudentnotes.wordpress.com/2014/06/13/portal-hypertension-and-oesophageal-varices/
Http://www.score95.com/blog/blog/usmle-portal-hypertension/

62. Young boy presented with diarrhea some time bloody , wight loos , arthritis , anemia the diagnosis is :
A. Chrons
B. Uc
C. Celic
A clear picture of chronic diarrhea in pediatric age groub you have to read about but in this q the picture of ibd is more clear and
supported by arthritis and bloody diarrhea which present in all types of ibd but its more common in crohn. Also, crohn is common in
young age and the absence of uc features like tenesmus and urgency make crohn more probable

360
63. Asymptomatic 40 y female patient, direct bilirubin 5, indirect 9, what is your diagnosis?
A. Gilbert disease
B. Rotor syndrome
C. Dubin-johnson syndrome
Answer: a, patient is asymptomatic and the indirect (unconjegated) bilirubin is higher than direct (conjugated). Hereditary or
inborn metabolic disorders may cause unconjugated or conjugated hyperbilirubinemia.
Unconjugated: gilbert syndrome, crigler-najjar syndrome, and primary shunt hyperbilirubinemia. Conjugated: dubin-johnson
syndrome and rotor syndrome
http://www.clevelandclinicmeded.com/medicalpubs/diseasemanagement/hepatology/guide-to-common-liver-tests/
Http://www.merckmanuals.com/professional/hepatic-and-biliary-disorders/approach-to-the-patient-with-liver-disease/inborn-
metabolic-disorders-causing-hyperbilirubinemia#v897839

64. Which dietary supply prevent cancer;


A. Fibers
B. Vitamin d
Answer: no enough data

65. Patient with mild to moderate inflammatory bowel disease, what is the mainstay of treatment?
A. Surgery
B. Antibiotics
Answer:
The first step in medication therapy for ibd is usually aminosalicylates with greater efficacy ulcerative colitis than for crohn disease.
For crohn disease, metronidazole or ciprofloxacin is occasionally used.
Second step is often corticosteroids (oral prednisone), which tend to provide rapid relief of symptoms and a significant decrease in
inflammation.
The immune-modifying agents are step iii drugs and are used if corticosteroids fail or are required for prolonged periods. Anti-tnf
monoclonal antibody therapies are also step iii drugs that are effective in both crohn disease and ulcerative colitis.
Consider surgical intervention for ulcerative colitis patients in whom medical therapy fails.

Http://emedicine.medscape.com/article/179037-treatment

361
66. Child ate a number of iron tablets presented with severe symptoms including constipation and bloody stool nausea and
vomiting and drowsiness how would you treat him:
A. Iv deferoxamie.
B. Dialysis
Answer: a
Chelation is the mainstay of therapy. It is indicated for serum iron levels >350 mcg/dl with evidence of toxicity or >500 mcg/dl
regardless of signs or symptoms.
Http://emedicine.medscape.com/article/815213-treatment

67. A patient has migratory thrombophlebitis this suggests:


A. Chronic pancreatitis
B. Carcinoma of the pancreas
Answer: b

Migratory thrombophlebitis also known as trousseau’s sign of malignancy, migratory thrombophlebitis is described as
thrombophlebitis that travels, often from one leg to the other. It has a strong association with adenocarcinoma of the pancreas and
lung; therefore, the history should be directed toward finding malignancy.

Reference: kumar and clark’s clinical medicine

68. Obese, acid reflux symptoms ,2week diagnosed with rf ,he is on asprin rx??
A. Metlozam
B. Cinitidine
Reference: http://emedicine.medscape.com/article/236582-overview

69. A patient presented with bilateral lower limb edema, distended abdomen and palmer erythema. On examination, small
vessels appear on abdomen. What’s the diagnosis?
A. Liver cirrhosis
B. Heart failure
Answer is a
Lower limb edema and distended abdomen (ascites) are physical signs of portal hypertension that can happen in both right-sided hr
and liver cirrhosis, while palmar erythema and small vessels on the abdomen (spider telangiectasia) are some of the stigmata of liver
disease.
Source: http://www.turner-white.com/pdf/hp_jul03_stigmata.pdf

70. 12 -years with hepatic failure admitted to icu, his skin was yellow . Now become green in color , what does this indicates :(
not mentioned obstructive jaundice in the choices!)
A. Oxidation of bilirubin.
B. Impending death
Answer: a
Bilirubin is created by the activity of biliverdin reductase on biliverdin, a green tetrapyrrolic bile pigment that is also a product of
heme catabolism. Bilirubin, when oxidized, reverts to become biliverdin once again.
362
71. Patient with right lower quadrant pain and swelling and loss of weight. Colonoscopy done showed mass in right lower
quadrant. What is the diagnosis?
A. Appendic tumor
B. Cecum tumor
Answer: b

72. Case about ibd which cell type responsible about ulceration in intestine?
A. B cell
B. T cell
Answer: b
Cytokines differentiate lymphocytes into different types of t cells. Helper t cells, type 1 (th-1), are associated principally with crohn
disease, whereas th-2 cells are associated principally with ulcerative colitis. The immune response disrupts the intestinal mucosa and
leads to a chronic inflammatory process.
Link: http://emedicine.medscape.com/article/179037-overview#a3

73. Which of the following lead to dilatation of aorta?


A) achalasia
B) barrett
Answer??
I suspect this is dilation on the esophagus not the aorta
The diagnosis of achalasia is established by the presence of aperistalsis in the distal two-thirds of the esophagus and incomplete
lower esophageal sphincter (les) relaxation on manometry. In patients with typical achalasia symptoms (dysphagia to solids and
liquids and regurgitation of bland undigested food or saliva) and equivocal manometric findings, the diagnosis is supported by
aperistalsis, dilation of the esophagus, bird beak appearance, and poor emptying on barium esophagram.
Achalasia should be suspected in patients with dysphagia to solids and liquids and in those with regurgitation unresponsive
to a trial of proton pump inhibitor (ppi) therapy for four weeks. Manometry is required to establish the diagnosis of achalasia. In
patients with equivocal motility testing, barium esophagram should be performed to assess esophageal emptying and
esophagogastric junction morphology. Endoscopic evaluation should be performed in patients with suspected achalasia to exclude a
malignancy at the esophagogastric junction that can mimic primary achalasia

74. Tenia coli which layer


A) adventitia
B) musculosa
Answer: muscularis externa

75. Female itching high alkaline phosphatase +ve antimitochondrial antibody


A) primary biliary cirrhosis
B) sclerosing cholangitis
Answer: a

76. Best treatment for primary biliary cirrhosis is:


363
A. Azathioprine
B. Urdox

Answer: b
Ursodeoxycholic acid (udca) is the major medication used to slow the progression of the disease.
Http://emedicine.medscape.com/article/171117-medication#showall

77. What type of cell is decreased in uc?


A. Enterocyte
B. Goblet cell
Answer: b
Uptodate: the biopsy features suggestive of ulcerative colitis include crypt abscesses, crybranching, shortening and disarray, and
cryatrophy. Epithelial cell abnormalities including mucin depletion and paneth cell metaplasia may be seen.
Https://yhdp.vn/uptodate/contents/mobipreview.htm?11/7/11386#h596557081
Medscape, biopsy and microscopic features: http://emedicine.medscape.com/article/2005396-overview#a8

78. Long scenario, bloody diarrhea and RBC in urine after 7 days hx of food poisoning, rx?

A. Steroid
B. Antibiotic
Answer: b
Hus etiology:
1. Diarrhea positive hus: 90% of pediatric hus from e. Coli o157:h7, shIgAtoxin, or verotoxin.
2. Diarrhea negative hus: other bacteria, viruses, familial, drugs, familial/genetic.

364
Reference: toronto note, step-up of medicine

79. Kwashiorkor disease is due to which kind of diet?


A. High carb, low protein diet
B. Low carb, high protein

Answer: a

Kwashiorkor is a severe form of malnutrition, caused by a deficiency in dietary protein

Reference: wikipedia: https://en.wikipedia.org/wiki/kwashiorkor

80. Hereditary chronic pancreatitis is:


A. Autosomal dominant
B. Polygene
Answer: a
(uptodate) and (pubmed) thereafter, hereditary chronic pancreatitis (hcp) was defined as an autosomal dominant disease with a
penetrance of approximately 80%.

365
81. If the patient has hbsag +ve andIgM+ve what do you treat the patient with?
A. Interferon
B. Lamivudine
Answer: b,
toronto: treatment options: interferon, tenofovir, entacavir, lamivudine, adefovir
medscape: interferon-a and etacavir ~> 1st line agents

82. Pt. With hba after 3 weeks we take biopsy, what is show?
A. Normal architecture
B. Fibrosis something
Answer: a

83. What is the oncogene for pancreatic cancer?


A. Ras
B. Myc

Answer: A
Genetic alterations can be detected at different levels. These alterations include oncogene mutations (most commonly, K-ras
mutations, which occur in 75% to more than 95% of pancreatic cancer tissues), tumour suppressor genes alterations (mainly, p53,
p16, DCC, etc.), overexpression of growth factors (such as EGF, TGF alpha, TGF beta 1-3, afgf, btgf, etc.) And their receptors (i.e., EGF
receptor, TGF beta receptor I-III, etc.).
Reference:http://www.ncbi.nlm.nih.gov/pubmed/1964102 http://www.ncbi.nlm.nih.gov/pubmed/10660490

84. During colonoscopy of patient who has fragile thin surface of the colon with multiple blood dots , there was no previous
chronic disease , what is the most likely diagnosis ?
A. Ulcerative colitis
B. Mesentric ischemia
Answer:
Aphthous ulcer:
Ibd
It is one of the main manifestations of crohns disease usually in addition to prolonged diarrhea with abdominal pain, weight loss,
anal skin tags, and fistulae. Http://emedicine.medscape.com/article/172940-clinical#b3
First aid:

366
85. 40 y male 6m hx of diarrhea mix e blood +wt loss+ no family hx of inflammatory dis then patient had intermittent fever ..in
sigmoid scope see fragile mucosa of rectum and spot blood ..dx? Bacterial dysntria
A. Uc
B. Ischmic colitis

86. 16 years old female . Fever and Chronic diarrhea for 10 months Post meal para umbilical pain Sometimes blood mixed with
stool
A-crohn
B- chronic pancreatitis
Answer: A

87. Kayser flescher ring what treatment?


A. Penicillamine
B. Deferoxamine
Answer: a
Treatment of wilson disease
4 drugs are available:
Penicillamine chelates copper, poorly tolerated
Trientine chelates copper zinc impairs copper excretion in stool/decreases copper absorption from gut tetrathiomolybdate
preferred if neurological involvement
Liver transplant in severe cases

88. Old man started a year ago on medication for peptic ulcer developed gynecomastia what is the drug:
A. Cimetidine
Answer: a
- Https://en.wikipedia.org/wiki/cimetidine
- Http://reference.medscape.com/drug/tagamet-cimetidine-341984#4

89. Patient 24 hours with diarrhea and vomiting. Blood pressure supine 120/80, Blood pressure . 80/40.what is the cause?
Answer: decrease intravascular
367
Dehydration (low fluid volume in the body) causes orthostatic hypotension other causes:
http://my.clevelandclinic.org/health/diseases_conditions/hic_orthostatic_hypotension

90. Case of acetaminophen toxicity, what’s the sequels of liver damage?


Answer: see image

91. Patient with history of gerd, required aspirin for the treatment of rheumatoid
Arthritis, what medication you’re going to add next?
Answer: misoprostol

92. Diarrhea in a kid what's the mechanism ( decreased absorption , membranous coating )
Answer: ???

93. Treatment of plummer-vinson syndrome?


The association of postcricoid dysphagia, upper esophageal webs, and iron deficiency anemia is known as plummer-vinson syndrome
(pvs). Iron replacement to correct the anemia it may improve the dysphagia. Those with significant and long-standing dysphagia
usually require mechanical dilation e.g. Endoscopy.

94. Peptic ulcer disease rx the kid is on PPIand metronidazole what to add?
o Quadruple therapy
Ppi
And --
Bismuth subsalicylate: 525 mg orally four times daily
And --
Metronidazole: 500 mg orally four times daily
And --
Tetracycline: 500 mg orally four times daily
Or
o Sequential therapy
Ppi

368
-- and --
Amoxicillin: 1000 mg orally twice daily on days 1-5
-- and --
Clarithromycin: 500 mg orally twice daily on days 6-10
Bmj http://bestpractice.bmj.com/best-practice/monograph/80/treatment.html

95. Antacid on of its side effects is constipation


A. Aluminum hydroxide.
Http://www.rxlist.com/antacids-page5/drugs-condition.htm

96. The cause of crohn’s disease:


A. Idiopathic
Answer: a

97. Patient ast very high, the cause?


Answer: alcohol

98. Treatment of acute hepatitis c?


A. Interferon
Answer: a
Uptodate: distinguishing acute from chronic infection has important treatment implications since patients with acute HCV who do
not spontaneously clear the virus should receive treatment with an interferon-based regimen.

99. Treatment of plummer vinson syndrome?


A. Iron
Answer: a.
Syndrome is treated by correcting nutritional deficiency (iron supplementation) plus esophageal dilatation ((step-up to medicine,
4e,p152))

100.Pt with dysphagia decrease manometry in pharynx and upper esophagus what is the diagnosis?
No options for this one.
Possible causes: mysthenia graves, stoke, or dermatomyositis, parkinson…etc

101.Child with jaundice and kieser ring in iris.


Answer: wilson disease.

102.Mode of inheritance for wilson's disease answer: autosomal recessive

369
103.Most common infectious disease among medical staff in saudi arabia
answer: HBV

104.Oral leukoplakia that can’t be swiped off. Most likely to be?


A. Dysplasia

Answer : ?? It’s hyperplasia of squamous cells >> dysplasia >> carcinoma in situ

105.What antacid causes constipation?


Answer: aluminum hydroxide
Reference: merck manual: http://reference.medscape.com/drug/alternagel-amphojel-aluminum-hydroxide-341981#4

106.Enterococcus faecalis antibiotic and the patient is allergic to ampicillin, what to give?
A. Vancomycin
Answer: a
Reference: uptodate.

107.Patient came with cough and she takes an anti-cholesterol medication (statins), she started it 3 weeks ago, what should the
doctor monitor?
A. Liver function test
Answer: a
LFT should be carried out before and within 4-6 weeks of starting statin therapy. Thereafter at intervals of 6 months to 1 yr.
If satisfactory lipid control and no evidence of adverse effects then review again at 4-6 months, then 6-12 monthly.
If unsatisfactory lipid control then measurements should be repeated 6 weeks after dosage adjustments are made until the
desired lipid concentrations are achieved.
Nice state that LFTs only need to be measured on three occasions:
Baseline liver enzymes.
Within 3 months of starting treatment and at 12 months.
Treatment should be discontinued if serum transaminase concentrations rise to, and persist at, 3x normal range.

108.What is the best treatment for traveler’s diarrhea?


A. Ciprofloxacin
Answer: a
Traveler's diarrhea self resolved, stay hydrated.
Antibiotics used for the treatment of traveler’s diarrhea:
Ciprofloxacin 500 mg twice daily for one to three days.
Other quinolones (e.g., ofloxacin, norfloxacin and levofloxacin).
370
Rifaximin 200 mg three times daily for three days
Azithromycin 500 mg daily for one to three days or 1,000 mg in a single dose.
Treat symptoms: antidiarrheal agents (e.g. Bismuth salicylate, loperamide).
Source: http://www.aafp.org/afp/2005/0601/p2095.htmlm, davidsone 22

109.Triple therapy for gastric ulcer in 10 yo boy?


Answer: triple therapy regimens for h pylori consist of a ppi, amoxicillin, and clarithromycin for 7-14 days.

110.Pt with bilateral flank swelling dull move with shifting


A. Ascitis

111.Prevents stomach ulcers caused by nonsteroidal anti-inflammatory drugs (NSAIDs).


A. Source: https://en.wikipedia.org/wiki/misoprostol

112.Clear history about gerd, they want diagnosis.


First aid:
Primarily a clinical diagnosis. History of:
- heartburn that commonly occurs 30-90 minutes after a meal, worsens with reclining, and often improves with antacids, sitting, or
standing. Also, sour taste, a sensation of a lump in the throat, unexplained cough, morning hoarseness, and chest pain mimicking
cad.
A TRIAL OF PPI .. IF NO IMOROVMENT DO 24H PH MONITORING
- know how to diagnose it and treat it.

113.Patient came complain of heartburn after taking med. He is takin- is. What the med?!
The answer is one of bisphonate (ends with -onate)

114.Pt diagnosed to have panacinar emphysema, splenomegaly & liver disease. What is the underlying cause :
Answer: alpha 1 antitrypsin deficiency

115.Female using NSAID for her dysmenorrheal developed epigastric pain , most likely dx :
A. Gastritis
Explanation: drugs like NSAIDs are known to cause gastritis. The major mechanism of injury is the reduction in prostaglandin
synthesis. Prostaglandins are chemicals responsible for maintaining mechanisms that result in the protection of the mucosa from the
injurious effects of the gastric acid.
Reference: http://emedicine.medscape.com/article/175909-overview#a4

116.Case of hepatic failure on medication for treatment what the cause of change jaundice from yellow to green yellow ??

117.Patient treated for duodenal ulcer. Now complains of breast enlargement and decrease sexual desire. Which drug?
Answer :

371
Ranitidine (h2 blocker)
Ref./ http://www.drugs.com/sfx/ranitidine-side-effects.html

118.Young girl with jaundice ( and i think liver failure) her color turned from yellow to green, why?
A. Hypoxia
I don't remember the rest

119.I had 3 qs about ibd manifestation::


Http://emedicine.medscape.com/article/179037-overview
Inflammatory bowel disease (ibd) is an idiopathic disease caused by a dysregulated immune response to host intestinal microflora.
The two major types of inflammatory bowel disease are ulcerative colitis (uc), which is limited to the colon, and crohn disease (cd),
which can affect any segment of the gastrointestinal tract from the mouth to the anus, involves "skip lesions," and is transmural.
There is a genetic predisposition for ibd, and patients with this condition are more prone to the development of malignancy.

120.Case of painless jaundice & the answer is “gilbert’s syndrome”.


Gilbert syndrome (also called familial non-hemolytic jaundice) has no signs or symptoms other than mild jaundice and is discovered
only when routine blood tests reveal elevated unconjugated bilirubin levels. You can read more about gilbert’s syndrome here:
https://yhdp.vn/uptodate/contents/mobipreview.htm?41/23/42359

121.Pt take asprin ..what give e it to protact git ?


A. Cimatidine ......!!!

122.Antacid mechanism of action_


Antacids aluminum salts and magnesium antacids are contact; they decrease the acidity of gastric secretion, and by their buffer by
direct neutralization of the hydrochloric acid present in the stomach.
Http://www.rayur.com/antacids-definition-mechanism-objectives-indications-adverse-precautions-and-risk.html

123.Patient with epigastric pain that decrease by food intake what is the organism?
A. Hilcobacter pylori
Answer:a
This is a case of duodenal ulcers(epigastric pain; may localize to tip of xiphoid,burning,develops 1-3 h after meals and relieved by
eating and antacids). The most common causes of ulcer are(h.pylori infection,idiopathic, NSAIDs, stress induced and ze
syndrome)
reference: toronto notes.

124.Long scenario about patient with peptic ulcer disease you give him PPI+ amoxicillin , what can you add to them ?
A. Clarithromycin
Answer: a
To eradicate h.pylori there are 2 regimens of therapy:
372
Triple therapy for 7-14 d (hp-pac®): PPIbid (e.g. Lansoprazole 30 mg bid) + amoxicillin 1 g bid + clarithromycin 500 mg bid (the
regimen in the question)
Quadruple therapy for 10-14 d: PPIbid + bismuth 525 mg qid + tetracycline 500 mg qid + metronidazole 250 mg qid
Reference: toronto notes

125.Which of the following is contraindicated to do gastric lavage


A. Drain cleaning solution
Answer: a (their original answer)

126.Patient treated for duodenal ulcer. Now complains of breast enlargement and decrease sexual desire. Which drug?
Answer: gynecomastia and impotence occur with cimetidine in a dose- and time-dependent fashion
Reference: http://cursoenarm.net/uptodate/contents/mobipreview.htm?36/55/37744?Source=related_link

127.College student complains of generalized pain relieved by defecation. No blood or mucus. Diagnosis?
A. Ibs.
Answer: a
Diagnostic criteria: 3d in 3m of episodic abdominal discomfort that is two or more of 1- relieved by defecation, 2-change in stool
frequency or consistency 3- change in stool appearance?

128.70 Year-old female brought to ur clinic by her daughter . The daughter said her mother's memory deteriorated in the last 2
years . She can dress her self but e difficulty , she can cook for herself but sometimes leave the oven on ,,,,,,,,,what's the
management ?!
A. Refer her to geriatric clinic

What gene is related to Celiac Disease?

Answer: ?
HLA-DQ2 (chromosome 6) found in 80-90% of patients compared with 20% in general
population; also associated with HLA-DQ8.
Reference: Toronto Notes and http://emedicine.medscape.com/article/171805-overview#a6

129.Which one of the following can cause liver cirrhosis?


Answer:?
➢Fatty liver (alcohol, metabolic syndrome)
➢Chronic viral hepatitis (B, B+D, C; not A or E)
➢Autoimmune hepatitis
➢Hemochromatosis
➢Α1-antitrypsin deficiency
➢Primary biliary cirrhosis
➢Chronic hepatic congestion
- Cardiac cirrhosis (chronic right heart failure, constrictive pericarditis)

373
- Hepatic vein thrombosis (Budd-Chiari),
- Idiopathic
- Rare: Wilson’s disease, Gaucher’s disease . Reference: Toronto Notes.

130.A woman with jaundice and high liver enzymes. Her husband has +ve Hep B surface antigen but she does not have +ve
markers for Hep A, B, C. What will you do now?
A. Check for anti Hep B core antibody (igm)
Answer: A
Because it will become +ve early.

131.Celiac disease affects which of the following locations?


A. Distal SB
Answer:?
Affect the upper small bowel (Duodenum and proximal jejunum) more than lower small bowel.
Reference: webmed website and digestive disease center

132.A patient with chronic retrosternal pain, cough and metallic taste in mouth. What is the most likely diagnosis?
A. GERD

Answer: A
“Heartburn”(pyrosis) and acid regurgitation (together are 80% sensitive and specific for reflux) ± sensation of a lump in the throat
(bolus sensation), coughing, chest pain, and wheezing. Usually a clinical diagnosis based on symptom history and relief following a
trial of pharmacotherapy (proton pump inhibitor (PPI): symptom relief 80% sensitive for reflux).
NB: Ambulatory 24-hour ph monitoring is the criterion standard in establishing a diagnosis of GERD, with a sensitivity of 96% and a
specificity of 95%. Ppis are the most effective therapy.
Reference: Toronto Notes and http://emedicine.medscape.com/article/176595-clinical

133.Pyoderma gangrenosum is associated with which one of these diseases?


A. Ulcerative colitis.
Answer: A
PG is more common in UC (5–12%) than CD (1–2%).
Reference: http://www.ncbi.nlm.nih.gov/pmc/articles/PMC3273725/table/T1/

134.Scenario pt. Came with melena and bleeding take NSAID for 3 weeks, what is the cause ?
Answer: NSAID use is associated with an increased risk of gastric or duodenal ulcer by inhibition of prostaglandins that leads to
symptomatic chronic ulceration ,also inhibit platelet function, and their use has been associated with GI bleeding from throughout
the GI tract.
Reference: http://www.uwgi.org/guidelines/ch_07/ch07txt.htm

374
135.Triple therapy for gastric ulcer in 10 yo boy? PPI+ metro + ?
Answer: treatment of PU is the same for adults and children
PPI-based triple therapy regimens for H pylori consist of a PPI, amoxicillin, and clarithromycin for 7-14 days
Reference: http://emedicine.medscape.com/article/181753-treatment#d8

136.Celiac pt what is safe for him?


A.Rice
Answer : A. Rice is safe for celiac patients and corn
Http://wheat.pw.usda.gov/ggpages/topics/Celiac.vs.grains.html
Https://www.nlm.nih.gov/medlineplus/ency/article/002443.htm

137.Primary biliary cirrhosis options were about pathophysiology.


Answer:
Intrahepatic - T lymphocyte mediated attack on small intralobular duct
Go and read: http://emedicine.medscape.com/article/171117-overview#a6

138.Zone of liver affected by hypoxia


Answer : zone 3
Functionally, the liver can be divided into three zones, based upon oxygen supply. Zone 1 encircles the portal tracts where the
oxygenated blood from hepatic arteries enters. Zone 3 is located around central veins, where oxygenation is poor. Zone 2 is located
in between.

375
Immunology &
Allergy

376
1. Least harmful vaccine in immunocompromised?
A. Bcg
B. Mumps
C. Measles
D. Pneumococcal
Answer: d
A,b,c are all live vaccines. Pneumococcal vaccines is a subunit vaccine.
Http://www.historyofvaccines.org/content/articles/different-types-vaccines

2. Dm, hypothyroid, irregular menses female, present withe recurrent itching & white adherent oral plaque, +ve mantux test,
she was exposed to tb 4 years ago, immunoglobulin, wbc, RBCs all are normal; diagnosis;
A. Chronic granulomatous disease
B. Chronic mucocotanous candidiasis
C. Digeorge syndrome
D. Hyperglobulinemia ( or hypo i don't remember )
Answer: b
Chronic granulomatous: leuckosytosis <charestersitc>, levels of the 3 major classes of immunoglobulins, immunoglobulin g,
immunoglobulin m, and immunoglobulin a, are increased. Immunoglobulin e levels are increased or in the reference range.

3. Patient got rapid swelling response after a bee sting what type of hypersensitivity ?
A. 1
B. 2
C. 3
D. 4

Answer: a

Pathology is similar to other immunoglobulin e (ige)–mediated allergic reactions.

Reference: medscape: http://emedicine.medscape.com/article/768764-overview#a5

4. Which vaccine you will give to immunocompromised?


A. Recom. HBV
B. Sabin
C. Salk
D. Bcg

5. What type of vaccine is the pneumococcal conjugate vaccine (pcv13)?


A. Live

377
B. Conjugate
C. Inactive
Answer: b reference: http://vk.ovg.ox.ac.uk/pcv

6. Most common cause of central line infection


A. During the insertion
B. Migration of bacteria from other site
C. While giving the medication through the line
Answer: a.
Reference: http://www.uptodate.com/contents/epidemiology-pathogenesis-and-microbiology-of-intravascular-catheter-infections

7. Like mump senario :"can't remember "


A. Sjögren
B. Hypersensitvity vasculitis
C. Autoimmune....
Answer: a
Sjögren syndrome is a systemic chronic inflammatory disorder characterized by lymphocytic infiltrates in exocrine organs. Recurrent
swelling of the parotid glands which is similar to mumps. Http://emedicine.medscape.com/article/332125-clinical#b3

8. Bee sting since 18 hrs. With swelling and redness, what will you do?
A. Antihistamines
B. Steroids
C. Observe

Answer: a
• Local reactions can be life threatening if swelling occludes the airway. Initiate invasive measures to secure the airway if this
occurs. Otherwise, the following local care measures suffice:
• Provide supplemental oxygen
• Diphenhydramine limits the size of the local reaction.
• Clean the wound and remove the stinger if present.
• Apply ice or cool packs.
• Elevate the extremity to limit edema.

• Treatment should include an initial intravenous (iv) bolus of 10-20 ml/kg isotonic crystalloids in addition to
diphenhydramine and epinephrine.
If the patient has not removed the stinger, it should be removed as soon as possible by the first caregiver on the scene.
Delay increases venom load, so the fastest removal technique is the best. Pinching and traction is an acceptable technique.
Reference: medscape: http://emedicine.medscape.com/article/768764-treatment

9. Clear presentation of sjogren syndrome, asking about the complication:


A. Pulmonary fibrosis
B. Malabsorption
C. Lymphocytic tissue infiltration
Answer: c
Morbidity associated with sjögren syndrome is mainly associated with the gradually decreased function of exocrine organs, which
become infiltrated with lymphocytes.
378
Reference: http://emedicine.medscape.com/article/332125-overview#a7

10. Man got a bee sting then his wife trying look for the epinephrine what it gonna inhibit?
A. Leukotriene release from macrophages
B. Cross reactivity with the cardiac..
C. Inhibit immunocomplex formation

Answer: (inhibit histamine and tryptase, missed option)


Epinephrine is the drug of choice for anaphylaxis. It stimulates both the beta-and alpha-adrenergic receptors and inhibits further
mediator release from mast cells and basophils. It inhibits histamine, the proteases tryptase and chymase.
References: https://quizlet.com/8362963/immuno-block-4-practice-exam-flash-cards/
Http://www.worldallergy.org/professional/allergic_diseases_center/anaphylaxis/anaphylaxissynopsis.php
Http://www.ncbi.nlm.nih.gov/pmc/articles/pmc3343118/

11. Patient can't take bcg vaccine because he deficiency in?


A. Il
B. Tnf gama
C. Ifnγ

Answer: c
Complete il-12p40 and il-12rβ1 deficiencies and partial ifnγr1 and ifnγr2 deficiencies generally predispose the patient to curable
infections. Complete ifnγr1 and ifnγr2 deficiencies predispose the patient to overwhelming infection

12. BCG vaccine is contraindicated in which of the following deficiencies?


A. IL
B. TNF
C. INF

Answer: C
Reference:https://books.google.com.sa/books?Id=mrta5qwtc7ac&pg=PA459&lpg=PA459&dq#v=onepage&q=BCG%20contraindicat
ed&f=false P: 459

13. Which of the following organisms is seen in patients with chronic granulomatous disease?
A. Cl. Difficle
B. Staph aureus
Answer: b
Cgd is a primary immunodeficiency that affects phagocytes of the innate immune system and leads to recurrent or persistent
intracellular bacterial and fungal infections
Most infections in cgd are caused by staphylococcus aureus.
Reference: http://emedicine.medscape.com/article/1116022-clinical#showall

14. Pt with allergy to penicillin. When he receive the medication he develop sob, urtecaria &….. . What does this called?
A. Asthma
B. Anaphylactic
379
Answer: b

15. Long scenario where the investigation shows highIgG:


A. Monoclonal something
B. Mutiple myeloma .
Answer: b (question incomplete)
Explanation: multiple myeloma diagnosis: protein electrophoresis: monoclonal spike due to a malignant clone of plasma cells
synthesizing a single ig (usually igg) called a monoclonal protein (m-protein). The 1st choice is likely monoclonal gammopathy of
unknown significance, diagnosed by the presence of a serum monoclonal protein (m protein) at a concentration <3.0 g/dl. And <10%
plasma cells in bone marrow
References: step up to medicine 3rd edition, page 351
Toronto notes 2015, page h51
Reference: http://emedicine.medscape.com/article/966003-overview

16. What vaccine is contraindicated in immunocompromised (HIV or chemotherapy) patients?


Live attenuated vaccines( measles, mumps, dpt..)
Reference: http://www.cdc.gov/vaccines/schedules/hcp/imz/adult-contraindications-shell.html

17. Which vaccine is least harmful to immunocompromised patients?


A. Pneumococcal vaccine
Reference: https://www.cdc.gov/mmwr/preview/mmwrhtml/00023141.htm

18. Allergy to peanut butter what is the mechanism ?


(not written from the original writer, but it is type i ige mediated hypersensitivity, initial sensitization with the ag passing through the
gi tract)

19. Pt with recurrent oral and genital ulcers and arthritis when the pt taken some type of im vaccine develop sterile abscess at
the site of injection. What is the most likely dx.?

Answer: behcet disease

Behçet disease is characterized by a triple-symptom complex of recurrent oral aphthous ulcers, genital ulcers, and uveitis.

Reference: medscape: http://emedicine.medscape.com/article/329099-overview

20. -x-linked a gamma globulinemia cd 19 and 20 cd40 mutation

21. Case of lactose intolerance



Answer: http://www.medicinenet.com/script/main/mobileart.asp?Articlekey=7809

380
22. Peanut allergy mechanism of action

23. Case of multiple myeloma.


Answer: http://emedicine.medscape.com/article/204369-overview#a1

24. A child present with s&s of leukemia with calla +ve? Dx?
A. All

25. What of the following is poor prognosis? _ high ig m -low ig a -high ca, should be beta 2 micro globin

26. Case of guillain barre syndrome ..clear. (hx of diarrhea 3w past then devolp asending symetrical ll parlysis... )

27. Case of sle .. Rash type: * case of ?

28. Scenario about a patient with bone lytic lesions and lab result what is the disease
Paget’s disease
Answer:a
It is a metabolic disease characterized by excessive bone destruction and repair. Patients usually are asymptomatic. Severe bone
pain is one of the presenting complaint and skeletal deformities.in cases of skull involvement: headaches, increased hat size,
deafness. Increased warmth over involved bones. Patients will have high levels of alp and urinary hydroxyproline. Ca and po are
normal(ca could be elevated).
Reference: toronto notes.

381
Bullet points

382
*1. minutes of Xray radiation? (answer not known)
A - 25
B - 15
C-5
D-0
Answer: not clear
Evidence: <15 minutes to do the x-ray, one site mentions that mammogram is equal to 15 minutes of natural background
radiation
Reference: http://www.xrayrisk.com/faq.php

Read about epstien bar virus


Read about kawasaki disease
Read about mononucleosis

Corona virus
Http://www.uptodate.com/contents/middle-east-respiratory-syndrome-coronavirus
Ebola virus
Http://www.uptodate.com/contents/clinical-manifestations-and-diagnosis-of-ebola-virus-disease
Alport disease
Hereditary nephritis (alport’s disease): x-linked nephritis often associated with sensorineural hearing loss; proteinuria <2
g/d
Dic
Addison disease.
Cardiomyopathy
Iron deficiency with koilonychia.
Types of bacteria and antibiotics
Siadh
Osteoarthritis
Guillain barre syndrome
Liver cirrhosis
HBV
Juvenile rheumatoid arthritis
Chronic pain
Bilateral pneumonia
Hyperthyroidism
Read about a thalassemia.
Q about cohort study
Child fell from 10 meter height with bleeding from his ears, where is the injury?
Gcs of a 10 yrs child, continuously crying and screaming not responding to his name, doesn't move but respond to pain by
...'i forgot the scenario'
11 b. 10 c. 9 d. 8
Pt took her anti osteoporosis drug then had a sever retrosternal pain.
Don’t remember exact drugs, but belongs to bisphosphonate.
383
Goodposture syndrome: types of gn
2nd index finger pain with numbness of hand: tendon injury
Oral hyperglycemic drugs case hypoglycemia
Lipid lowering agent can cause sever muscle weakness
Best muscle relaxing agent
Pt on hypoglycemic agent with poor control blood sugar have sulpha drug allergy <<< which medication you give
The most common disorders difrantional sex in boys?
Case of tonsillitis <<< antibiotic
Most common stain blood nipple discharge
Drug maintenance in bipolar
Drugs in several pre-eclampsia to prevent eclampsia seizure
Picture of ectotropion
Picture of senlle chart how many meters pt stand >>>>6 m
Direction of im injection related to sciatic nerve? Safest to use the upper outer quadrant.
Presentation of posterior hip dislocation? The limb is shortened and internally rotated
Treatment for severe depression?
Sensrion , valgus test positive which ligament will be affected ? Collateral ligament
Child with meningitis treated with antibiotics see report what is the cause
Agrranulocytoma >>> carmphicanol treatment
Cavernous sinus thrombosis which vein drainage? Maxillary, ophthalmic
Farmer with lesion protruding keratin? Keratosanthoma.
Highest risk of stroke? Htn.
Child with gradual developing of cyanosis and ejection systolic murmur of left upper sternal border? Tof.
Bacteria associated with ventilator- pneumonia? Pseudomonas
Child with strabismus, treatment? Glasses
Depigmented lesion? Vitiligo
Child with morning stiffness and joint pain of wrist and ankle? Juvenile rheumatoid arthritis
Investigation for bone density? Dxa scan.
Retired man used to work in factory with sensorineural hearing loss? Noise related
Best lab for patient with hemochromatosis? Ferriten
Older patient with memory loss and change of behavior? Alzheimer
Patient with pinpoint pupil antidote? Nalxone.
Patient with dilated pupil tachycardia cause? Sympathomimetic
Girl with premenstrual syndrome treatment? Ssri.
Patient with anemia, thrombocytopenia, splenomegaly and hepatomegaly investigation? Bone marrow biopsy
Pregnant third trimester with bright red blood? Placenta previa.
Patient history of travel c/o bloody diarrhea? Amebiasis
Sickle cell patient with hip pain? Avascular necrosis.
Patient with foot pain? Planter fasciits
Patient involved in trauma with signs of increase icp which nerve you should examine? Optic
Older patient lifting heavy object with mass not reaching the scrotum? Direct inguinal hernia
Pediatric boy 14 years with follow up asymptomatic by ex arrhythmia and (pic of ecg which show irregular rate and t
elevation!! What's dx:
Rx of alzheimer disease?
Mass in upper part of kidney what's the gene
Muscles of knee
Embryology >> origin of portion smooth muscle in right atrium or ventricle ..
Patient complain of pain swelling of big toe what's the dx > gout
384
HTN with Blood pressure h what's the drug of htn
Gene in pt breast Ca> brca
What's the anion gap na= 138
Which ligament when you do epidural anaesthesia
Women fully dilated 0 station, for breech delivery with head extension what's the management > >
Migraine
Metabolic acidosis
Campaign to prevent stroke: Blood pressure control
2 qs about croup
Gout: xanthine oxidase
Tb. Numbness: pyridoxine
Numbness. What drug: isoniazid
Q about the muscles of the mandible.
Pt with urti symptoms when he coughed he see something i forget: retina pressure
Pic of dendriatic herpatitis:
Best description of case-control study.
Relative risk equation
Pt with depigmentation and problems in her eyes (i can't recall them) what can be associated with it? Renal
Pregnant lady twins a ciphalic b breach

Pregnant bleeding from every where after delivery >>> dic


Patient presented with pneumonia symptoms for 2 weeks i think the gram stain negative: mycoplasma pneumonia
Case about rh –ve women married a rh +ve man, their first child was rh +ve like the father, now the wife is pregnant for
the second time,
Not sure about the question, or what they were asking about, may about the frequency of monitoring, and the titers.
11 yrs old female with jaundice and splenomegaly?
(sca, spherositosis) both present with splenomegaly and jaundice!
Child with left sided abdominal mass, and other features, dx? Wilms tumor.
Small fluid discharge <<<< ‫اسم تيست بيبر تحول لونها ازرق‬
Embryologic origin of follicle cells in the ovaries
Pt with diarrhea and mild dehydration shows very enlarge clitoris? She received ors what else? Hydrocortisone
Which pathology does echoccus granulosus?
Child came with swelling in the scalp with loss of hair?
Something sebum
G1p0 accidently discovered fibroid on us what best advice?
If it is asymptomatic have no effect
Gene in DM 2 <<<<
Characteristic of sca in peripheral blood smear
What best treatment for mania with advanced liver disease? May be lithium
What breast cancer present bilateral
14y girl had sexual contact in mid-cycle and she concerned about pregnancy what are you tell her:
Pregnancy rate 5%
Douching
Neck pain radiating inter-scapular and numbness
Vertebral collapse (my answer)
Polymylagia rheumatic
Pt with hyperthyroidism on treatment. C/o recurrent infections. What is the drug
o Methamizole
385
Pt present with sx of brochictasis what is the best advice?
o Stop smoking
Pt present with rash and had hx of infection >ITP or hsp
Pt has thyroid nodule we did TSH the result eutheroid>>fna
Constrictor patient complain>>lateral epicondilitis
29-case of opacification in lense >>cataract (sure)
31-athelete after exercise presented with hypopegminted lesion tx >>(tinea versicolor) >>selinum
Topical antifungal medications containing selenium sulfide are often recommended
Pt i didnt remember the case but culture result gram negative ,motile urease and oxidase negative what is the best
treatment ?? Salmonella tx(bed rest if u don't find it choose ceftraixone i think!!
Pt in pain, difficult to communicate with him how to asses his pain ?
o Numerical scale
o Face scale( my ans )
Increase effect of analgesia? Metoclopramide (my ans)
Question about tt of alzahimer
Another question about alzahimer drug that is hepatotoxic
A drug to treat DM named incritin what's the moa of it
Glipizid moa
Right peritonsilar abscess and fever
o Acute quincy ++
o Retropharyngeal abscess( complication)
Patient c/o decrees vision and another hx in the eye also had a mass in the neck
o Neuroblastoma
o Wilm's
Mammogram can detect breast cancer how many years prior to detection by self-examination?
o 1 - 2 -3 -4
A patient took a drug that cause pinpoint pupil
o Opioids
2 question about the anti dot of the previos answer
Naloxone++✅
There were many questions about kidney changes in DM and moa of DM drugs
Silver crust at hair line and nasal what is ttt ?
HTN patient with renal failure ? Name ttt of htn
Cyanotic heart disease admitted to er with convulsion what medication induce ? Digoxin(rare symptom of toxicity-
medscape)
HTN patient with renal what mechanism in kidney with HTN ?
Witch of them consider ( produce cyanosis )
All age in options 10 year .. 11 year one options :18 month with VSD muscular !! Not confirm
Read about embryology problem about cleft palate ?
Bee sting lead erythema - swelling what ttt : 2 q with diffrent options
Http://www.mayoclinic.org/diseases-conditions/bee-stings/basics/treatment/con-20034120
Women with stress pick of hair ? What is disorder .. !!
Another about disorder ? Read !!
Read the algorithm of bacteria
Read type of pregnancy ?
DM 1 loss conscious in school ? Best management ? Admitted to hospital !! No option for glucagon
Read sexual infections !! May 6 q
Read ttt of hurshpring disease and see picture for x-ray 2 q about ttt

386
Neonate with duodenal obstruction ? What the sign ? Bubble !!! Not confirm
Read about anterior ant posterior dislocation ( shoulder and hip )
What medication lead to convulsion ?Ssri - anti depressant read about
What medication lead to HTN crisis ?Ssri - anti depressant read about !!
Read about glucoma ttt .. And congenital
Read about antibiotic and mechanism of action 2 q
Related hernia to cord ? Anatomy
Emergency treatment for mi ?
Elevated of direct bilirubin with ast and ALT ? Hepatitis
Elevated direct and indirect with ast ?
Elevated total bilirubin with alp and ast or ALT not cnfirm ? Diagnosis ( no obs jaundice or any liver disease ) .. Gb stone
in option !!
Tear eye - red conjuntiva ? Diagnosis ( no infection in option ) keratitis - uveitis .. Others
Pregnant lady developed edema and hypertension:
o Mgso4
Limping child , painful left hip ( x ray given )
o Slipped capital femoral epiphysis ( 10 y and above)
o Pethe's disease .(4-8 y)
Easy question about dka diagnosis .

Diagnosis of migraine ( straightforward )


Same weight for 6 months , amenorrhea :eating disorder ?
Diagnosis of penile painless ulcer :
o Darkfield microscopy
Itching in lower limbs , otherwise normal :
o Tinea ?Scabies ?
Well circumscribed lesion on erythamatous base , arthritis :
o Rhumatological disease
Vesicles highly suspected round worms:
o Ascaris
o Tenea saginata
Aids, retinitis :
o Cytomegalovirus
Case of uti methicillin sensitive : cloxacillin ?
Question about splenic sequestration ( with lobar infiltrate in lung )
New classification of lung ca : adenomatous mass less than 2 cm ?✅
Case of lymphoma , treatment :
o Chop r
o Abvd
Hemolytic anemia, coombs’ positive, what type of hypersensitivity: type 2
Lung infection :iv ceftriaxone ?
EBV pharyngitis what is ttt :iv acyclovir ?
About latent sle
Mother close to delivery developed respiratory symptoms + fetal distress :
o Amniotic fluid embolism
Thyroid disease associated with papillary ca :hashimoto ?✅ reidle thyroditis ?
Soldier walks 1000miles developed pain on foot :spring ligament
How poliovirus vaccine works ?

387
Kid with bilateral abscess of inguinal nodes what causes this condition ?Chronic granulmoatous disease
DM pregnant with hx of fetal death before delivery now she's 32 weeks pregnant with a new baby what to do? Deliver
her immediately wait until 36 weeks, weekly biophysical profile or fetal heart rate testing can be combined with
maternal kick counts in the third trimester. For patients who have experienced earlier loss, frequent ultrasound is
reassuring
Scalp lesion (derma) what is the treatment?

Man with low back pain and lytic lesion biopsy shows schistocytes and giant cells what is the diagnosis ?A- giant cell
tumor of bone
Boy goes camping when he's back he came with constitutional symptoms lymph nodes enlargement and took penicillin
developed rash what does he havei chose mononucleosis i think it's right
3 years old with a father known to have pulmonary tb his ppd 10mm what does he have ?
o Strong +ve
o Week +ve
Hf patient what to order next ?
Thyroid nodule rx in the right side onl
Anemia in kids what's the management ?
Growth chart for a kid with
o Low weight and length what does he have ?
o Familial short stature or normal or what
Research questions very difficult relative risk, prevalence
o Most accurate test etc
Mers cov > 66:1
Lady with a mass in left upper quadrant of the breast, aspiration was yellow fluid without masses, dx?
o Phyllodem, normal tissue variant, ..."no simple cyst"
1000-20, 5000-6, or? 2.0
Ionizing radiation effect on DNA? Pyrmedine ..., deamination, deputination (no oxidization)
Inflammatory bowel disease
Gram -ve pneumonia

388

Das könnte Ihnen auch gefallen